Sei sulla pagina 1di 312

lOOlsoLvEo PROBLEMs

IN
-

ENGINEERING MATHEMATICS
_____
.....

.._.....,.,.

Second Edition

JAIME R. TIONG
1

BSCE,.UV 1985 (Summa Cum Laude)


College of Engineering First Summa Cum Laude
,
1st Placer, PICE National Students' Quiz, 1985
..
Awardee, Outstanding Alumni, UV
. ' Prestaent, Excel First Review and Training Center, Inc.

Past President, Rotary Club of Cebu North
Pr~sident, UV Engineering & Arch. Alumi Association
Del~ate, Rotary International Convention, Chicago, USA
r(~mt(l~,. Ramon Aboitiz Foundation Inc. Triennial Awards
' . . ' Former Plant Engineer, University of the Visayas
Fo~mE!r Reviewer, Besaviiia Engineering Review Center
f,ormer Reviewer, Salazar Institute of Technology

Former Faculty, UV College of .Engineering


Author, Various Engineering Reviewers

.,... UY

ROMEO A. ROJAS Jl-i.

"'BSEE, CIT 1991 (Cum Laude), BSECE, OT 1996


1st Placer, RME Licensure Examinations, Oc;:tober 1997
8th Placer, REE Licensure Examinations, April 1999
Former Faculty, Cebu Institute of Technology
Former Technical Assistant, CIT Automation Center
Reviewer, Excel Review Center
Author, Various Electrical Engineering Rexiewers

IMPORTANT: Any copy of this book not bearing the signature ofany one
ofthe authors or ofthe publisher on this page shall be considered as
comingfrom an illegal source.

AUTHOR I PUBLISHER

TABLE OF CONTENTS

ndamenta~~~

Preface to the First Edition


Preface to the Second Edition
Dedication

DAY

of Numbers
Conversion

THEORY:
Number
1
Types of Numbers
1
Numerals
1
Digits
2
Real Numbers
2
Imaginary number
2
Complex number
3
System of Numbers
3
Fractions
3
Types of Fractions
3
Composite numbers
4
Prime numbers
4
Fundamental Theorem of
Arithmetic
4
Types of Prime Numbers
5
Perfect Number
5
Abundant Number
6
Deficient Number
6
Perfect Number
6
Amicable Number
6
Friendly Number
6
Factorial
6
Significant Figures & Digits
7
Forms of Approximation
7
Conversion
7
Celsius Scale
7
Fahrenheit Scale
7
Kelvin Scale
8
Rankine Scale
8
Degrees, Radians, Grads & Mils 9
Trivia
9
Quote
9
T"EST (50 Problems for 2 hours)
SOLUTIONS

15

Noles

20

10

Algebra

THEORY:
Properties of Integers
Properties of Addition
Properties of Multiplication
Additive Identity
Additive inverse
Multiplicative Identity
Multiplicative Inverse
Properties of Equality
Properties of Zero
Exponents
Radical
Surd
Types of Surds
Special Products
Proportion
Properties of Proportion
Least Common Denominator
Least Common Multiple
Greatest Common Factor
Remainder Theorem
Factor Theorem
Trivia
Quote

23
23
23
23
23
24
24
24
24
24
25
25
25
26
26
26
26
27

27
27
27
27
27

TEST (50 Problems for 3.75 hours)


SOLUTIONS
Notes

~@tJc
mfal
If

DAY
Equation,

Theorem &
Logarithms

THEORY:
Quadratic Equation
Nature of Roots
5
Properties of Roots
Binomial Theorem
Properties of Expansion
Pascal's Triangle
Coefficient of any term
Formula for rth term

28
34
43

3
45
45
46
46
46
46
47
47

Stjm of Coefficients
47
Sum of Exponents
47
Degree of Polynomial/Equation 47
logarithm
47
Common & Natural Logarithms 48
Euler's Number
48
Binary Logarithm
48
Properties of Logarithms
48
Trivia
48
48
Quote
TEST (40 Problems for 3 hours)
SOLUTIONS
Notes

49
54
60

DAY4

Mixture,
Digit, Motion
Problems
THEORY:
Age Problems
Work Problems
Mixture Problems
Digit Problems
Motion Problems
Coin Problems
Trivia
Quote

TEST (65 Problems for4.5 hours)


SOLUTIONS
Notes
63
63
64

64
64
64

65
65

TEST (40 Problems for 4 hours)


SOLUTIONS
Notes

66
73
84

'~'f

DAY

II

,,'
'

l'i.!'' '
I.

Variation,
Problems &
Progression
THEORY:
Clock Problems
Variation Problems
Diophantine Equations
Sequence
Series
Progression
Arithmetic Progression
Geometric Progression

Infinite Geometric Progression


Harmonic Progression
Other related sequences
Fibonacci Numbers
Lucas Numbers
Figurate Numbers
Triangular numbers
Square numbers
Gnomons
Oblong numbers
Pentagonal numbers
Cubic numbers
Tetrahedral numbers
Cubic numl;>ers
Square pyramidal numbers
Supertetrahedral numbers
Trivia
Quote

5
87

88
88

88
88

89
89

89

DAY
n Diagram,
trmutation,
Combination &
Probability
THEORY:
Venn Diagram
Combinatorics
Fundamental Principle of
Counting
Permutation
Inversion
Cyclic Permutation
Permutation with Identical
Elements
Assortment
Combination
Relation between Permutation
And Combination
Probability
Principles of Probability
Mutually Exclusive Events
Independent Events
Binomial Distribution
Odd

89
89
90
90
90
90
90
90
90
90
90
90
90
90
90
90
9t
91
92
100
114

6
117
118
118

118
118 .
119
119 '
119

119
119
120
120
120
121

121
121

Odd For
Odd Against
Mathematical Expectation
Card Games
Probability with Dice
Trivia
Quote

122
122
122
122
123
124
124

TEST (50 Problems for 4 hours)


SOLUTIONS
Notes

125
131
141

DAY
Geometry

THEORY:
143
Definition of Geometry
143
Branches of Geometry
144
Basic Postulates of Euclid
Basic Geometry Elements and
144
Figures
144
Types of Angles
145
Bisector
146
Units of Angles
Polygons
146
147
Triangles
147
Other Types of Triangles
148
Quadrilaterals
148
Types of Quadrilaterals
150
Bramaguptha's Theorem
150
Ptolemy's Theorem
Areas and Perimeters of
150
Regular Polygons
151
Perimeter
151
Circles
Useful Theorems involving
. 152
Circles
153
Ellipses
154
Trivia
154
Quote

TEST (50 Problems for 3.75 hours) 155


161
SOLUTIONS
Notes
176

DAY
Geometry
THEORY:
Polyhedrons
Five Regular Polyhedrons
Platonic Solids
Prisms
Cylinders
Pyramids
Cones
Frustum of Pyramid
Frustum of Cone
Prismatoid
Prismoidal Formula
Sphere
Zone
Spherical Segment
Spherical Sector
Spherical Pyramid
Spherical Wedge
Torus
Ellipsoid
Spheroid
Trivia
Quote
TEST (30 Problems for 2.5 hours)
SOLUTIONS
Notes

DAY
THEORY:
Definition of Trigonometry
Branches of Trigonometry
Classification of Triangles
Solution to Right Triangles
Pythagorean Theorem
Solution to Oblique Triangl(!s
Law of Sines
Law of Cosines
Law of Tangents
Trigonometric Identities

8
179
179
179
181
182
182
182
183
183
183
183
184

184
184
184
184

185
185
185
185
186

186
187
191
200

9
203
203
203
204
204
204
204
205
205
205

Exponential
Form
of the
.
Fundamental
Tngonometnc
Function
Amb1guous Case
Other Parts of Triangle
Radius of Inscribed Circle and
Circumscribing Circle
Plane Areas (Triangles)
Plane Areas (Quadrilaterals)
Ptolemy's Theorem
Important Properties of Triangles
Important Points in Triangles
Conditions for Congruency
Conditions for Similarity
Tnv1a
Quote
TEST (50 Problems for 4 hours)
SOLUTIONS
Notes

DAY

Sphencal
onometry

=EST (15 Problems


~OLUTION
206
206
207
207
208
209
209
209
209
210
210
210
210
211
217
230

THEORY:

~
'"'

Definition of Spherical
Tngonometry
Great Circle
Small Circle
Pole
Polar Distance

233
233
234
234
234

Spherical Wedge
Spherical Triangle

234
234

Propositions
Tnangle of Spherical
Solutions to Right Triangle
Nap1er's Rules
Quadrantal Spherical Triangle
Solution to Oblique Triangles
Area of Spherical Triangle
Terrestrial Sphere
Pnme Meridian
International Date Line
Greenwich Mean Time
Coordinated Universal Time
Latitude i':lnd Longitude
Terrestrial Sphere Constants
Trivia
Quote

235
235
235
236
236
237
237
237
237
238
238
238
239
239
239

Notes
.
~~,:

'fi:;;

for 1.5 hours)

..

DAY

Geometry
~;~~ \~0 ' Points . Lines &
' c I
Ire es

240
242
247

11

/'''i,X;AJtalytic

THEORY
Rectangular Coordinates System
Distance Formula
Distance Between Two Points
in space
Slope of a Line
Angle Between Two Lines
Distance Between a Point
and a line
Distance Between Two Lines
Division of Line Segment
Area by coordinates
Lines
Conic sections
General Equation of Conics
Circles
Trivia
Quote
TEST (50 Problems for 4 .hours)
SOLUTIONS
Notes

~
~:'{t:

DAY

249
250
250
250
251
251
251
252
252
252
253
253
254
255
255
256
261
275

12

1\a.lytic
Geometry
'" '"
.
.
Parabola, Ellipse
& Hyperbola
THEORY:
Parabola
Ellipse
Hyperbola
Polar coordinates
Trivia
Quote
TEST (55 Problems for 4 hours)
SOLUTIONS
Notes

277
279
281
284
285
285
286
292
:l(fT

.j,': .
r'

DAY
.

Dlff~re_ntlal

':

13

ulus .(l1m1ts &


D f
)
enva IVeS
THEORY:
Definition of Calculus
Limits
Theorems of Limits
One-Sided Limits
Continuity
Special Limits
Derivatives
Algebraic Functions
Exponential Functions
Logarithmic Functions
Trigonometric Functions
Inverse Trigonometric Functions
Hyperbolic Functions
Inverse Hyperbolic Functions
Trivia
Quote

309
309
309
310
311
311
311
311
312
312
312
312
31.2
312
313
313

TEST (40 Problems for 4 hours)


SOLUTIONS
Notes

314
319
329

.~;;

:.*.
1t<~..

DAY

14

0~,.al Calculus
rna/Minima &

Max~

DAY

a~~

338
343
359

15

THEORY:
Definition of Integral Calculus
Definite and Indefinite Integrals
Fundamental Theorem of
Calculus
Basic Integrals
Exponential & Logarithmic
Functions
Trigonometric Functions
Inverse Trigonometric Functions
Hyperbolic Functions
Trigonometric Substitution
Integration by Parts
Plane Areas
Centroid
Length of Arc
First Proposition of Pappus
Volume
Second Proposition of Pappus
Work
Hooke's Law
Moment of inertia
Ml:lltiple Integrals
Trivia
Quote

361
361
361
362
362
362
362
362
363
363
363
364
364
364
365
365
366
366
366
366
366
366

mum
Minimum Values 331
Max1ma
I M1mma
332
Time :Rates
.
332
Relat1on between the vanables &
maxima I minima values
332
Trivia
337
Quote
337
TEST (35 Problems for 3 hours)
SOLUTIONS
Notes

u.ral Calculus
0':

TEST (50 Problems for 4 hours)


SOLUTIONS
Notes

Time Rates)
THEORY:

. .

.~ :~

. .{... ... . . . .

DAY

1.)ifferential .

367
373
391

16

>iEouations

THEORY:
Types of DE
Order of DE
Degree of DE
Types of Solutions of DE
Applications of DE
Trivia
Quote

393
394
394
394
395
396
396

TEST (30 Problems for 2.5 hours)


SOLUTIONS
Notes

DAY

397
401
410

11

THEORY:
Complex Numbers
413
Different Forms of Complex
Numbers
413
Mathematical Operation of
Complex Numbers
414
Matrices
415
Sum 6( two matrices
416
Difference of two matrices
416
Product of two matrices
416
Division of matrices
417
Transpose matrix
417
Cofactor of an entry of a matrix 417
Cofactor matrix .
417
Inverse matrix
417
Determinants
418
Properties of Determinants
418
Laplace transform
419
Laplace transforms of elementary
functions
419
Trivia
420

r"'f"''t"' ~

'

'II

TEST (50 Problems for 4 hours)


SOLUTIONS
Notes

421
428
438

DAY

18

-\{
'!:"''',

~it.'.

Physics

THEORY:
Vector & Scalar Quantities
Classifications of Vectors
Speed and Velocity
Distance and Displacement
Acceleration
Laws of Motion
Force
Frictional Force

441
441
442
442
442
442
442
443

Centripetal Force
Law of Universal Gravitation
Work
Energy
Law of Conservation of Energy
Power
Momentum
Law of Conservation of
Momentum
Impulse
Types of Collisions
Coefficient of Restitution
Gas laws
Properties of Fluids
Archimedes Principle
Trivia
Quote
TEST (40 Problems for 3 hours)
SOLUTIONS
Notes

'"t'W

~-1-B~%\,

DAY

!~lJtingineering
. h~rdcs (Statics)

443
443
444
444
445
445
445
445
445
445
445
446
446
446
447
447
448
453
462

19

THEORY:
Definition of Terms
Branches of Mechanics
Conditions for Equilibrium
Friction
Parabolic Cable
Catenary
Moment of inertia
Mass moment of inertia
Trivia
Quote

465
465
465
466
466
467
467
468
469
469

TEST (35 Problems for 3.5 hours)


SOLUTIONS
Notes

470
475
484

;,.:. 5 '~ ',.,


~'3',,

-"

DAY

., "::<'

~~~t~," E~~~~=~ii~~
(Dynamics)

20

THEORY:
Types of Rectilinear Translation
Horizontal Translation
Vertical Translation
Free Falling Body
Curvilinear Translation
Projectile or Trajectory
Rotation
D'Aiembert's Principle
Centrifugal force
Banking of Highway Curve
Trivia
Quote

487
487
488
488
489
489
490
490
491
491
492
492

TEST (45 Problems for 4 hours)


SOLUTIONS
Notes

493
499
511

DAY

21

Strength of
Materials
THEORY:
Definition of Terms
Simple St~ess
Types of Normal Stress
Simple Strain
Hooke's Law
Stress-Strain Diagram
Thermal Stress
Thin-Walled Cylinder
Torsion
Helical springs
Trivia
Quote

513
513
514
514
514
515
515
516
516
517
517
517

TEST (30 Problems for 2.5 hours)


SOLUTIONS
Notes

518
522
528

~~~>
,,;~;;:r:>,;: ..
~~~>l: 'tf;~.:~: :.
;~r1:

DAY

< >;

Engineering
mfi!fiebhomy (Simple &
Compound Interest

22

THEORY:
Definition of Terms
Consumers & Producers
Goods and Serv1ces
Necessity and Luxury
Market Situations
Demand
Supply
Law of Supply and Demand
Interest
Simple Interest
Discount
Compound Interest
Continuous Compounding
Nominal & effective rates of
interest
Trivia
Quote

531
531
.532
533
534
534
535
535
536
536'
537

TEST (40 Problems for 3 hours)


SOLUTIONS
Notes

539
545
551

'iJ'''l[''''''

DAY

,#z~!;<
~-il;;\
'

1Z. .'

::~'~l:t Engmeermg

"foilomy (Annuity,
Depreciation, Bonds,
Breakeven analysis,
etc.
THEORY:
Annuity
Capitalized Cost
Annual Cost
Bonds.
Depreciation
Break Even Analysis
Legal Forms of Business
Organizations
Trivia
Quote

531

538
538
538

23
553
555
555
555
557
558
558
559
559

TEST (51 Problems for 4 hours)


SOLUTIONS
Notes

I :RJ;~i;t~t

A.
B.

c.

D.
E.

F.
G.

560
568
580

1ces

GLOSSARY
UNITS & CONVERSION
PHYSICAL CONSTANTS
NUMERATION
MATH NOTATION
GREEK ALPHABETS
DIVISIBILITY RULES

583
625
633
634
634
635
636

)",,.._

;,

k.

' ..v

rr
.)

Pl:R~ON~F\Lef(opt:: \<..11'

Cf .

~b~

~ ~ -Ft' f'ffl'ff.c:5
f&t ir. ~~fl..lc.tl- Efolt;; '<;; _.l>WCL

Topics

~
Mon

D
D
Tue

_j
D ,-Cl LJ
D [_]
Theory

Wed

Problems

Thu

Solut1ons

Fri

Notes

Sat

What is a number?
A number is an item that describes a
magnitude or a position.
~-'

What are the types of numbers?


Numbers are classified into two types,
namely cardinal numbers and ordinal
numbers.

il
~'

,,./

Cardinal numbers are numbers which


allow us to count the objects or ideas in a
given collection. Example, 1 ,2,3 ... , 1000,
100000 while ordinal numbers state the
position of the individual objects in a
sequence. Example, First, second, third ..

IIYJ:l.!lt are

numerals?

Numerals CJre symbols, or combination of


;ymb<)ls wt1ich describe i1 number.

Cardinal and Ordinal Numbers


Numerals and Digits
System of Numbers
-Natural numbers, Integers,
Rational numbers, lrrationC!I
numbers & imaginary numbers
Complex numbers
Types of Fractions
Composite Numbers
Prime Numbers
Defective and Abundant Numbers
Amicable Numbers
Significant Figures and Digits
Forms of Approximation
Conversion

The most widely used numerals are the


Arabic numerals and the Roman
numerals.
Arabic numerals were simply the
modification of the Hindu-Arabic number
signs and are written in Arabic digits.
Taken singly, 0, 1, 2, 3, 4, 5, 6, 7, 8, 9 and
in combination 20, 21, 22, ... 1999, ...
The Roman numerals are numbers which
are written in Latin alphabet. Example
MCMXCIV.
The following are Roman numerals and
their equivalent Arabic numbers:
I
1
C = 100
v =5
0=500
X = 10
M "' 1000
L :: 50

The Romuns used the following to indicate


large nurnbers:

2 l 00 l s21v~d Problems in Engineering Mathematics (2"d Edition) by Tiong & Rojas


1.

Bracket - to (11ultiply it by 100


times.

Examples: - 4, -1, 0, 3, 8

3. Rational numbers - are numbers

lVI = 500
2.

which can be expressed as


a quotient (ratio) of two
integers. The term "rational"
comes from the word
"ratio".

Vinculum (bar above the


humber) - to multiply the number
1000 times.

Examples: 0.5,

v=
3.

5,000

Doorframe- to multiply the


number by 1000000 times

fVl = 5, 000,000
What i2J! digit?
A digit is a specific symbol or symbols
used alone or in combination to denote a
number.
For example, the number 21 has two
digits, namely 2 and 1. In Roman
numerals, the number 9 is denoted as IX.
So the digits I and X were used together to
denote one number and that is the number
9.
In mathematical computations or in some
engineering applications, a system of
numbers using cardinal numbers was
established and widely used.
~re

real numbers?

The number system is divided into two


categories namely, real numbers and
imaginary number.
Real numbers are classified as follows:
1.

, -3, 0.333 ...

In the above example, 0.5 can be

What

Natural numbers - numbers which


are considered as the
"counting numbers".
Examples: 1 ,2, 3 ...

2. Integers- are all the natural number,


the negative of the natural
numbers and the number
zero.

1
expressed as - and -3 can be
2

-6

Day l - Systems of Numbers and Conversion 3


Imaginary number and its equivalent:.

i =~
i2 = -1

The absolute value of a real number is


the numerical value of the number
neglecting the sign.

i =-i =- ~
i4 = 1
3

For example, the absolute value of- 5 is 5

What is a complex number?

while of -x is x. The absolute value

A complex number is an expression of


both real and imaginary number combined.
It takes the form of a + bi, where "a" and
"b" are real numbers.

examples are rational numbers.

If a = 0, then pure imaginary number is


produced while real number is obtained
when b = 0.

The number 0.333 ... can also be

What is a system of numbers?

expressed as -,.hence the two

expressed as __!_ and therefore a

A system of numbers is a diagram or chart

rational number.

which shows the two sub-classifications of


the two basic classifications of numbers,
namely real numbers and imaginary
numbers.

The number 0.333 ... is a repeating and


non-terminating decimal. As a rule, a
non-terminating but repeating (or
periodic) decimal is always a rational
number. Also, ajl integers are rational
numbers.
4. Irrational numbers - are numbers
which cannot be expressed
as a quotient of two
integers.
Examples:

System of
Numbers

/Real
Numbers

Imaginary
Number

The numbers in the examples above


can never be expressed exactly as a
quotient of two integers. They are in
fact, a non-terminating number with
non-terminating decimal.

Ia Iis

either positive or zero but can never be


negative.
What are fractions?
Fractions are numbers which are in the
form of

~or a/b,

where a is called!he

numerator which may be any integer while


b is called the denominator which may be
any integer greater than zero. Fraction is
also defined as a part of a whole.
What are the types of fractions?
1. Simple fraction - a fraction in which
the numerator and
denominator are both
integers. This is also known
as a common fraction.
Examples:

..J2, n, e, ...

6
7

2. Proper fraction - is one where the


numerator is smaller thai!
the denominator.

Examples:

7' 3

3. Improper fraction - is one where the


numerator is greater than
the denominator.

What is an imaginary number?


An imaginary number is denoted as "i"
which is equal to the square root of
negative one. In some other areas in
mathematical computation, especially in
electronics and electrical engineering it is
denoted as "j".

What is an absolute value?

'

Examples:

Pi~l<J~::jtn showing the riumbe~ system

12

2' 7

4. Unit fraction - is a fraction with unity


for its numerator and
positive integer for its
denominator.

Day 1 - Systems of Numbers and Conversion 5

!_lt:'O 1 Solved Problems in Engineering Mathematics (2"ct Edition) by Tiong & Rojas

11. Undefined fraction- a fraction with a

Examples:

25

5. Simplified fraction - a fraction whose


numerator and denominator
are 1ntegers and their
greatest common factor is 1.

Examples:

"

denom1nator of zero. The


example below means that
8 is divided by 0, which is an
impossibility because
nothing can be divided by
zero.

8
0

Examples: -

2' -11

6. An lnteJer Represented as fraction


- a fraction in which the
denominator is 1.
2
3
E xamples: -1 , -1
7. Reciprocal- a fraction that results
from interchanging the
numerator and der.1ominator.

12. Indeterminate fraction - a fraction


which has no quantitative
meaning.
Examples:

13. Mixed number- a number that is a


combination of an integer
and a proper fraction.

Examples: 4 is the
4
8. Complex fraction -a fraction in which
the numerator or
denominator, or both are
fractions.

Examples: __4_

7'

r""''"''

II

1
2

8
11

Examples: 5-. 9-

-~

reciprocal of

-~1' -13
4

9. Similar fractions -two or more simple


fraction that have the same
denominator.

What is a composite number?


Composite numbers are positive integers
that have more than two positive whole
number factors. It can be written as
product of two or more integers, each
greater than 1. It is observed that most
integers are composite numbers. The
number 6 is a composite number because
its factors are 1, 2, 3 and 6. The number 1
is the only natural nu"'lber ti1at is neither
composite nor prime.
What is a prime nu111ber?

Examples:

g- g-

!_

10. Zero fraction - a fraction in which the


numerator is zero. A zero
fraction is equal to zero.

Examples:

2 11

A prime number is an integer qreater than


1 that is divisible only by 1 and it~elf.

2,3, 5, 7, 11, 13, 17, 19,23,29,31, 37,


41,43,47,53, 59,61,67, 71, 73, 79,83,
89, 97,101, 103,107,109, 113,127,131,
137. 139, 149, 151, 157, 163, 167, 173,
179, 181, 191, 193, 197, 199,211,223,
227,229,233,239,241,251,257,263,
269,271,277,281, 283, 293,307, 311,
3.13, 317,331, 337, 347, 349,353, 359,
367,373,379,383,389,397,401,409,
419,421,431,433,439,443,449,457,
461,463,467,479,487,491,499,503,
509,521,523,541,547,557,563,569,
571,577,587,593,599,601,607,613,
617,619,631,641,643,647,653,659,
661,673,677,683,691,701,709,719,
727,733, 739, 743, 751, 757, 761, 769,
773,787,797,809,811,821,823,827,
829,839,853,857,859,863,877,881,
883,887,907,911,919,929,937,941,
947,953,967,971,977,983,991,997,
The number 2 is the only prime number
which is an even number.
What are the types of prime numbers?
Na.tural prime numbers are those that have
only two factors; 1 and the number. Twin
primes are a set of two consecutive odd
primes, which differ by two. The following
are twin primes less than 100.

11' 13
17, 19

Symmetric primes are a pair of prime


numbers that are the same distance from
a given number in a number line.
Symmetric primes are also called Euler
primes. The following are symmetric
primes for the number 1 through 25.
Number

According to the fundamental theorem of


arithmetic, " Every positive integer greater
!h.an 1 is a prime or can be expressed as a
unique product of primes and powers of
primes".
The following 1s a list of the prime numbers
less than 1,000.

29, 31
41,43
59, 61
71, 73

3, 5
5, 7

2
3
4
5

6
7
8

Symmetric prime
None
None
None

3,5
3,
5,
3,
5,

7
7

11
11; "3, 13

9
10
11
12
13
14
15
16
17
18
19
20
21
22
23
24

7,11; 5,13
1. 1a a 11

5, 17; 3, 19
11, 13; 7, 15; 5,19
7, 19; 3, 23
11,17.5 ;;_ ..
13, 17, ''i' 9; 7, 23
15, 17; '!,., 19; 3, 29
11,23;6.2~ 3,31
17, 19; 1.l, 23; 7, 29; 5, 31
9,29; 7, 31
17, 23; 11, 29; 3, 37
19, 23; 13, 29; 11,31; 5, 37
13, 31; 7, 37; 3,41
17,29; 13, 33; 5,41; 3,43
19, 29; 17, 31; 11, 37;
7,41; 5, 43
19, 31; 13, 37, 7,43; 3,47

25

Emirp (prime spelled backwards} is a


prime number that remains a prime when
its digits are reversed.
The following are emirps less than 500:

11, 13, 17, 31, 71, 73, 79, 97, 101, 107,
113,131, 149,151, 157, 167, 179,181,
191,199,311,313,337,347,353,359,
373,383,389
Relatively prime numbers are numbers
whose greatest common factor is 1.
Unique product of power of primes is a
number whose factors are prime numbers
raised to a certain power.
Example of unique product of pow~r of
primes:

360

=23 . 32 . 51

What is a perfect number?


A perfect number is an integer that is ~
equal to the sL m of all its possible divisors,
except the number itself.
Example:

6, 28, 496 ...

In the case of 6, the factors or diviscrs .-e


1, 2 and 3. When the factors are added the
sum is ~qual to the number itself and
shown in the following equation.

1+2+3=6

6 1001 Solved Problems in Engineering Mathematics (2"d Edition) by Tidng & Rojas
What are an abundant numbers and
deficient numbers?

2P-1 (2P

If the sum of the possible divisors is


greater than the number, it is referred to as
abundant number.

where:

A defective number is an integer'with the


sum of all its possible divisor is less than
the number itself. It is also called deficient
number.

p and ( 2P

Number

1
2
3
4
5
6
7
8
9
10
11
12
13
14
15
16
17
18
19
20
21
22
23
24
25

Factors Excluding
Itself

1
1
1, 2
1
1, 2, 3
1
1, 2, 4
1, 3
1,2, 5
1
1,2,3,4,6
1
1, 2, 7
1,3,5
1, 2, 4, 8
1
1,2,3,6,9

1
1,2,4,5, 10
1, 3, 7
1' 2, 11
1
1,2,3,4,6,8, 12
1, 5

Example: n! = n(n- 1) ... 3, 2, 1

-1) are prime numbers

Tvoe

0
1
1
3
1
6
1
7
4
8
1
16
1
10

D
D
D
D
D

9
15
1
21
1
22
11
14
1
36
6

Perfect

D
D
D
D
D
A
D
D
D
D
D
A
D
A
D
D
D
A
D

What is a perfect number?


Perfect number is a number that is equal
to the sum of its factors excluding itself.
They are mathematical rarities that have
no practical use. The formula to find a
perfect number is a follows:

(0 !)(0 + 1) = (0 + 1)!

i (2 -1)

2 (2 -1)

28_

2 (2 -1)

496

The factorial symbol ( ! ) was introduced


by Christian Kramp in 1808.

8128

What are significant figures or digits?

26 (2 -1)

What are amicable numbers?


Sum

(n!)(n+1) = (n+1)!

Perfect number

integer number, fewer value-carrying (non"


zero) digits.

If n = 0, by definition:

Formula

The following is a list of the first 25


numbers with its corresponding type, D for
deficient and A for abundant.

-1)

Day I - Systems of Numbers and Conversion_ 7

Amicable numbers or friendly numbers


refer to two integers where each is the
sum of all the possible divisors of the
other.
The smallest known amicable numbers are
220 and 284.
The number 220 has the following
factors/divisors: 1, 2, 4, 5, 10, 11, 20, 22,
44, 55, & 110 which when added sums up
to 284, while the number 284 has the
following divisors 1, 2, 4, 71, and 142
which adds up to 220.
There a.re more than 1000 pairs of
amicable numbers have been found. Th~
following are the ten smallest pairs of
amicable numbers.

220 and 284


1184 and 1210
2620 and 2924
5020 and 5564
6232 and 6368
10744 and 10856
12285 and 14595
17296 and 18416
63020 and 76084
66928 and 66992
What is a factorial?
A factorial denoted as n!, represents the
product of all positive integers from t to n,
inclusive.

0!(1)=1!
0! =1

Significant figures or digits are digits


that define the numerical value of a
number.

Examples:
3.14159 shall be rounded up to 3.1416
3.12354 shall be rounded down to 3.1235
Truncation refers to the dropping of the
next digits in order to obtain the degree of
accuracy beyond the need of practical
calculations.This is just the same as
rounding down and truncated values will
always have values lower than the exact
values.
Example:
3.1415 is truncated to 3 decimal as 3. 141
What is

A digit is considered significant unless it is


used to place a decimal point.
The significant digit of a number begins
with the first non-zero digit and ends with
the final digit, whether zerc, or non-zero.
Examples:

16.72 .
1.672 X 10
0.0016

4 significant figures
4 significant figures
2 significant figures

Example 2 is expressed in scientific


notation and figures considered significant
are 1, 6, 7 and 2 excluding 103 Example 3
has 2 significant figures only because the
3 zeros are used only to place a deciMal.
The number of significant digit is
considered the place of accuracy. Hence,
a number with 3 significant digits is said to
have a three place accuracy and a
number with 4 significant figures is said to
have a four place accuracy.

a conversion?

Conversion is the process of getting the


equivalent value in another unit of
measure of a certain value with a different
given unit of measure.
Most conversions can be done
conveniently by using a prepared
conversion table while other conversions
can be done through mathematical
computations using formulas.
The authors suggest to the user of this
book to familiarize the values in the
conversion table which is found at the last
part of this book labeled as "Appendix A".

to

How
convert a temperature in degree
Celsius to degree Fahrenheit and vice
versa?
The unit "Celsius" was named after the
Swedish astronomer, Anders Celsius
(1701 -1744). In this unit of temperature,
the boiling point and freezing point are 100
degrees and 0 degree, respectively.

What are the forms of approximations?


There are two forms of approximations,
namely rounding and truncation.
Hounding of a number means replacing
IIi" n11rnber with diH >flier JHirnber having
I w ., ;1qnlfle<HII tl< ,, II I I'll tl1qils, m for

The unit "Fahrenheit" was named after the


German physicist, Gabriel Daniel
Fahrenheit (1686- 1736). In this unit of
temperature, the boiling point and freezing
point are 212 degrees and 32 degrees,
respectively.

8 10.01 Solved Problems in Engineering Mathematics (2nd Edition) by Tiong & Rojas
Solution:
For same reading, F

=C

Day .1 - Systems of Numbers and Conversion 9


How to convert one unit of an angle to
another unit?

Problema
There are how many grads in 1200 mils?

F=-C+32
'iling point
80

F=-F+32
5
O.BF = -32

70

F = -40

90

There are four units commonly used to


measure an angle. They are degrees,
radians, grads and mils.

5
9

60

1:30

100- 0~

_
_
tj: '' __ )::"'

oc-

20

212-32

~--li'oF- 32

-10

Centigrade

Fahrenheit

Scale

Scale

By ratio and proportion:


F-32
C-0
--=-212-32
100-0

F-32

-=-100

180
100
C=-(F-32)
180

Absolute temperature may be expressed


in Kelvin or in Rankine.
Kelvin was named after British physicist,
William Thompson (1824-1902) the First
Baron, Kelvin.

The formulas for conversion to absolute


temperature are as follows:

Solution:

K= C+273

R=F+490

Problem:
Express the temperature of 60C to
absolute temperature.

Solution:
K = C+273
K=60+273

Problem:
Convert 45C to F.

Unit
Degree
Radian
Grad
Mil

Problem&

F=-C+32

or

The following is a tabulation of the unit of


angle measurement and the
corresponding value in one revolution:

Rankine was named after Scottish


engineer and physicist, William John
Macquom Rankine (1820-1872).

C =~(F-32)

II

How to convert temperature in degree


Celsius or degree Fahrenheit to
absolute temperature?

The degree is the mostpommonly used


measure of an angle. The radian is an
angle subtended by an arc equal to the
length of the radius of the circle.

K =333

9
5

F=-(45)+32
F = 113F

Problema
At what temperature will the Centigrade
scale and the Fahrenheit scale will have
the same reading?

Express the temperature of 150F to


absolute temperature.
Solution:
0
' R=F+490
0

R = 150+490

R = 640

1200 mils
6400 mils

x=75 grads.
Study Appendix B - Prefixes which is
found in the last part of this book.
Proceed to the next page for your first test.
Detach and use the answer sheet provided
at the last part of this book. Use pencil
number 2 in shading your answer.
GOOD LUCK!

'Ol:ribia:

How many mils are there in 90 degrees?

x
90
6400 mils 360
x =1600 mils

Did you know that... the symbol 1t (pi),


which is the ratio of the circumference of a
circle to its diameter was introduced by
William Jones in 1706 after the initial
letter of the Greek word meaning
"periphery".

~uote:

Problema
How many radians is equivalent to 320
grads?

Solution:
_x__ = 320 grads
2Tt radians 400 grads
x = 5.03 radians

9
F=-C+32

x
400 grads

1 Revolution
360
21t
400
6400

Problem:

Solution:

Solution:

Problema
145 degrees is equivalent to how many
grads?
Solution:
X
145
400 grads = 360"

x = 161.11 grads

"I could hardly ever known a


mathematician who was capable of
reasoning."
-Plato

Dayl -Systems of Numbers and Conversion 11


B.

C.
D.

2.37 X 10"
0.2371 X 10"9
o.oo2371 x 1o 11

7. EE Board October 1994

Topics

~
Mon

D
Tue

D D
Theory

Wed

Problems

Thu

D D
D D

Solutions

Notes

Fri

Cardinal and Ordinal Numbers


Numerals and Digits
System of Numbers
-Natural numbers, Integers,
Rational numbers, Irrational
numbers & imaginary numbers
Complex numbers
Types of Fractions
Composite Numbers
Prime Numbers
Defective and Abundant Numbers
Amicable Numbers
Significant Figures and Digits
Forms of Approximation
Conversion

Sat

7 + Oi is
A.

B.
C.

D.

irrational number
real number
imaginary number
a variable

13. EE Board April1993


Express decimally: Seven hundred twentyfive hundred thousandths
A.

B.

c.
D.

14. EE Board April1993


Express decimally: Four and two tenth

8. ECE Board Marc:h 1996

A.

The number 0.123123123123 ........ is

B.
C.

A.
B.
C.
D.

irrational
surd
rational
transcendental

/"ECE Board April1991


Round off 6785768.342 to the nearest
one-tenth.

D.

6785768
6785768.4
6785768.3
None of these

Express 45" in mils


A.

B.
C.
~.fi:

ME Board October 1996

A.
B.

c.
D.

il

2
3

4
5

,icE Board April1991


Round off 0.003086 to three significant
figures.

D.

34.0

4: ME Board April 199&


'which number has three significant
figures?

A.
B.
C.
D.

0.0014
1.4141
0.01414
0.0141

~1!:CE

Board April1991

'Round off 149.691 to the nP.arest integer


A.
B.

C.

D.

0.003
0.00309
0.0031
0.00308

A.
B.
C.
D.

Y,ECE Board April1991


Round off 34.2814 to four significant
figures.
A.

B.
C.

34.2814
34.281
34.28

149.69
149.7
150
149

90
57.3
100
45

B.

Express decimally: Fourteen Ten


thousandths

C.

A.

Jl'/ CE Board May 1993

B.
C.
D.

0.0014
0.00014
0.014
0.14

u. ECE Board Marc:h 1996


MCM~CIV is equivalent to what number?

A:
B.

C.
D.

1964
1994
1984
1974

12. EE Board April 1993


Express decimally: Fourty-Sevenmillionth

&~CECE Board April1991


Round off 2.371 x 10"8 to two significant

A.

figures.

2.4 X 108

ME Board April1997

10. EE Board April1993

__.x.

A.

80 mils
800 mils
8000mils
80000 mils

What is the value in degrees of 1 radian?

A.
"ifow many significant digits do 10.097
have?

0.042
4.02
4.2
0.42

.A$~/ECE Board November 1995

D.
A.
B.
C.
D.

0.000725
0.00725
0.0725
0.725

B.

0.00000047
0.0000047
0 000047
0. 00000004 7

D.

3200 mils is equal to how many degrees?


A.

B.
C.
D.

45"
90"
180"
270"

18. ECE ~rd November 1995


An angular unit equivalent to 1/400 of the
circumference of a circle is called
A.
B.
C.
D.

mil
degree
radian
grad

12 lOOi Solved Problems in Engineering Mathematics (2nd Edition) by Tiong & Rojas
.19. EC:E Board April1999
4800 mils is equivalent to __ degrees.

A.
B.
C.

D.

135
270
235
142

,.a( ME Board April 199ft

A.

B.
C.
D.

micro
femto
tera
atto

,:Ji: If a foot has 12 inches, then how


A.
B.

c.
D.

3
4
6
8

A.

10"2

B.

10-e

C.
D.

10"3

D.

2.667 C
1.334C
13.34 c
37.8 C

u.

EE Board October 1990

:&7: RME Board October 1994

:J~ In a hydrographic survey, a certain

The prefix pico means

point below the surface of the water


measures 12 fathoms. It is equivalent to a
deep of how many feet?

What is the absolute temperature of the


freezing point of water in degree Rankine?

A.
B.

c.
D.

492

0
460
273

-~ "8 Board October 1994

What is the Fahrenheit equivalent of 100


degrees Celsius?

A..
B.

c.
D.

200
180
212
100

Z31EE Board Aprii199:J


The temperature 45 C is equal to

A.

45 F
113 F

B.
c. 5rF
D.

81 F

Z4;ME Board October 1994


How many degrees Celsius is 80 degrees
Fahrenheit?
A.
B.

c.
D.

13.34
1.334
26.67
2.667

A.
B.
C.
D.

10"

10"12 of a unit
10-e of a unit
10"15 of a unit
109 of a unit

A.

100 feet

B.
C.
D.

200 feet
400 feet
800 feet

A.
B.
C.

:&8. ME Bo8rd April1999

The prefix nano is opposite to

A.
B.
C.
D.

mega
giga
tera
hexa

:&91 foot is to 12 inches as 1 yard is to

__spans.

D.

4
6

c. 9
D.

24

/ :J{). EE Board .Juae 1990


A one-inch diameter conduit is equivalent
to

A.

254mm

B.
C.

25.4 mm
100mm
2.54mm

D.

57 EE Board October 1994

A.
B.
C.
D.

8x
8x
8X
8x

10"2
102
10"3
10_.

:JS(vVhich of the following is equivalent to


hectare?

'1

A.

a.

C.
D.

100 ares
2 acres
1000 square meters
50000 square feet

:J9. Ten square statute miles is equivalent

to

A.
B.
C.

w;

sections.

100
5
10
20

The legendary ship, Titanic that sunk


in 1912 was estimated to be at the sea
bottom at a deep of 18 cables. How deep it
is in feet?

40. The land area of the province of Cebu


'is 5088.39 sq. km. This is equivalent to

A.
B.

A.
B.

c.
D.

A.
B.

72
60
48
36

441 statute mile


414 nautical mile

' Carry out the following multiplication and


express your answer in cubic meter:
8 em x 5 mm x 2in.

:JZ~ How many feet difference is 1 nautical


' mile and 1 statute mile?

A.

12

C.
D.

many hands are there in one foot?

:&Cn EE Board October 1994


The micro or ~ means

c.

I)

10 to the 12 h power is the value of the


prefix

Day 1 - Systems of Numbers and Conversion 13

How many degrees Celsius is 100 deg~::ees


Fahrenheit?
B.

II

zs. ME Board
October 1996
1

D.

12,000
12,343
12,633
12,960

C.

D.

.u~"'u Board October 1994

. :~s: ME Board October 1994

How many square feet is 100 square


meters?

B.
C.

D.

5088.39 hectares
1964.64 sq. miles
2257907.2 acres
5.08839 acres

,/How many cubic feet is 100 gallons of


liquid?

328.1
929
32.81
1076

A.
B.
C.
D.

74.80
1.337
13.37
133.7
/

,.u;io ~rd Octo.,...1994

:56.

A certain luxury ship cruises Cebu to


Manila at 21 knots. If it will take 21 hours
to reach Manila from Cebu, the distance
traveled by the ship is nearly
A.
B

847.5km
507 15 statute mile

'

ME Board April1998
How many cubic meters is 100 gallons of
liquid?
A.
B.

1.638
3.785

14 .1001 Solved Problems in Engineering- Mathematics (2nd Edition) by Tiong & Rojas
C.
D.

0.164
0.378

4~.-ME Board October 1994


.How many cubic meters is 100 cubic feet
of liquid?

C.

D.

49_.ME Board April ~:99s


c5'ne horsepower is equivalent to

A.
A.
B.

C.
D.

3.785
28.31
37.85
2.831

4.4-'f~n (10) cubic meters is equivalent to


!l'>

10
104

B.
C.

D.

Topics

746 watts
7460 watts
74.6 watts
7.46 watts

~
Mon

$0(ME Board October 1994


"'1iow many horsepower is 746 kilowatts?

A.
B.

A.

B.

10
20
100

C.
D.

c.
D.

500
74.6
100
1000

0
0

Theory

Problems

45. ME Board Aprii199S


The standard acceleration due to gravity is
:;..:.

A.

B.
C.
D.

0
0
0
0
D
Tue

r'llow many stere?

32.2 ft/s 2
980 fUs 2
58.3 fUs 2
35.3 fUs 2

Solutions

Notes

VVed
Thu
Fri

Cardinal and Ordinal Numbers


Numerals and Digits
System of Numbers
- Natural numbers, Integers,
Rational numbers, Irrational
numbers & imaginary numbers
Complex numbers
Types of Fractions
Composite Numbers
Prime Numbers
Defective and Abundant Numbers
Amicable Numbers
Significant Figures and Digits
Forms of Approximation
Conversion

Sat

46. ME Board October 1996


A ?kg mass is suspended in a rope. What
is the tension in the rope in Sl?

ANSWER KEY

A.
B.
C.
D.

68:67 N
70 N
71 N
72 N

47. A 10-liter pail is full of water.


Neglecting the weight of the pail, how
heavy is its water content?

A.
B.

c.
D.

5kg
6.67 kg
10 kg
12.5 kg

2. B
3. c
4. D
5. c
6.A
7. B
8. c
9. c
. 10. A
11. 8
12. c
13. 8
,.,._

48;;'The unit of work in the mks system is


.<known as joule (J) and the unit of work in
the cgs system is erg. How many ergs are
there in one joule?

A.

B.

10
7
10

14. c
15. B
16. B
17. c
18. D
19. 8
20. D
21. A
22. c
23. 8
24. c
25. c
26. 8

1. D

l<

27. A
28. B
29. A
30. B
31. A
32. D
33. A
34. D
35. D
36. 8
37. D
38. A
39. c

RATING

40. B
41. c
42. D
43. D
44. B
45. A
46.A
47. c
48. B
49.A
50. D

'-~

'

c:J
c:J
c:J
0

43-50 Topnotcher
30-42 Passer
25-29 Conditional
0-24 Failed
IfF AILED, repeat the test.

16 .1061 Solved Problems in Engineering Mathematics (2nd Edition) by Tiong & Rojas

Ill

The number 10.097 has 5 significant


figures.

MCMXCIV

II

El

The number 0.003086 when rounded off


to three significant digitsbecomes
0.00309.

II
:t

The number 34.2814when rounded off to


four significant digitsbecomes 34.28.

II

0.0014 has two significant figures


1.4141 has five significant figures
0.01414 has four significant figures
0.0141 has three significant figures m-Ans

II

M
CM . XC
IV
= 1000 + 900 + 90 + 4
= 1994

Using the formula,

c=~(F-32)
9

c=%(100-32)

-~
1000000

Fourty-seven millionth = 0.000047

lEI
Seven hundred twenty-five hundred
725
= 0.00725
thousandths =
100000

Ill

Four and two tenth

=4.2

C=37.8C

Ell
The freezing point of water is equal to
32F oroc.

The number 2.371 x 10' when rounded


off to two significant digitsbecomes 2.4 x
10'8 .

II
7 + Oi = 7 thus, the answer is, "real
number".

II
Repeating decimal number is a "rational
number".

II

II
Fourteen ten thousandths=

14

10000
Fourteen ten thousandths= 0.0014

R= "F+460
R=32+460

R =492R

Using the formula,

F =~(c)+32

45
6400 mils= 360
x =800 mils
X

lrl
By ratio and proportion:
x
1 rad
360 .= 2Jt rad

By ratio and proportion:


x
3200 mils
360 = 6400 mils

1 span is equivalent to 9 inches


1 yard = 3 ft = 36 inches, thus
.
1 span
36 mches x - - - - :::: 4 spans
9inches

El
.

10mm

1 mch = 2.54 em x--- =25.4 mm


1 em

hand

1 foot= 12 mches x - .-h4 1nc es


1 foot = 3 hands

F=113F

Let: x = the difference between a nautical


mile and a statute mile
6080 - 5280
x = 800 feet
X ::

=~(a0-32)
9

oc = 26.67c

X =270

1 statute miie = 5280 ft.


1 nautical mile = 6080 ft

oc

By ratio and proportion:


x
4800 mils
360 = 6400 mils

to o

The prefix nano is equivalent


1 9 of a
unit while the prefix giga is equivalent to
9
10 of a unit
.

F=~(45)+32

Ill

El

c=~(F-32)

Grad

The prefix pico is equivalent to 10'12 of a


unit

1 hand is equivalent to 4 inches, thus

Using the formula,

El

F = 212F

F=*(c)+32

106 means micro

II

Using the formula,

Ill

F=~(100)+32
5

X= 57.3

X= 180

The number 6785768.342 when rounded


off to the nearest one-tenth becomes
6785768.3.

0
0

El

By ratio and proportion:

The number 149.691 when rounded off to


the nearest integer becomes 150.

II

II

Fourty-seven millionth

________________D_a...y_l_-_S_y._s_t..,..e_ms
__o_f_N_u_m
__,bers and Conversion 17

m
I hu prefix tera is equivalent to 1012 of a
unit

El
1 fathom is equivalent to 6 feet, thus
12 fathoms= 6(12) = 72 feet

Ell
1 cable is squivalent to 120 fathoms,
thus:
18 cables= 120(18) = 2160 fathoms
6feet
18 cables= 2160 fathoms x--fathom

18 100 1'Solved Problems in Engineering Mathematics (2"d Edition) by Tiong & Rojas
ft3
3

18 cables= 12 960 feet


'

100 gallons x _
= 13.37 ft
7.48 gallons

1 meter is equivalent to 3.281 ft, thus


2
1 m 2 = (3.281 )2 ft
2
1 m 2 = 10.76 ft
2
100m2 = 100(10.76) ft
2
100m2 = 1076 ft

El

3.79 li
m
100 gallons x - - x - - gallon
1000 li

Iii

x-~-=

3.281 ft

m
m

1 hectare = 100 ares


&Core: 1 are= 100 sq. meters
1
1 hectare = 100 ares x - 0.:..0.::_::_sq.:.o.-.-m.:..
1 are
1 hectare= 10,000 sq. meters

The unit of force (tension) in the Sl


system is newtons (N).
9
Tension= 7 kg
m) = 68.67 N

x(

m
1 square statute mile = 1 section
10 square statute mile = 10 sections

-~~

El
Density of water ( p ) = 1000

_k~
mo

,,~
kg
Density of water ( p ) = 1 -.liter

1 square km = 0.386102 sq. miles

1 cubic ft. = 7.48 gallons

The following are the standard


gravitational acceleration:
2
32.2 ft/s
981 cm/s 2
9.81. m/s 2

A= 5088 _39 km 2 x 0.3861 02" miles


km
A = 1964.64 sq. miles

x(-m-)

= 100 ft 3
3
V = 2.831 m

II
1

hp = 746 watts
1 hp = 0.746 kilowatts

746 kW x.

Given volume is 100 cu. ft.

1 cubic meter = .1 stere, thus,


10m 3 = 10 steres

1m
0.8 m
100 em
1
m
= 0.005 m
5 mm x
1000 mm
4
3
0.08(0.005)(2) = 8 x 1
m
8 em

V = 21 knots = 21 nautical miles


hour
D=21(21)
D =
nm x 1.15 statute miles
441
nautical mile
D 507.15 statute mile

II
1 horsepower = 746 watts

1 gallon = 3.79 liters


1000 liters = 1 cubic meters

100 gallons= 0.379 m

Solving for distance,


D = Vt

Day 1 -Systems of Numbers and Conversion 19

W= pV

W=1

~ x 10 liters = 10 kg
liter

m
7

1 joule= 10 ergs

hp

=1000 hp

22 100 1 Solved Problems in Engineering Mathematics ~2~d Editionl by Tiong &: Roiaa

Topics

Mon

Tue

I"

Wed

Theory

0
0
0

Problems

Solutions

Thu

0
Fri

LJ
Sat

Notes

What are properties of integers?


Integers have special properties.
Computations of integers will become
easier by understandir;g these special
properti~s. The commutative property, for
instance, allows you to change the order of
adding or multiplying while the associative
property allows you to change grouping.
The properties of adpitiori of integers:
\.

2. Commutative property

5. Inverse property
a +(-a)= 0

The properties of multiplication of


integers:

:\ Associative proper1y
1 <

The num 2r 0 is called the


additive identiy

a(b+c) = ab:t-ac

a+b=b+a

ll)

a+O=a

6. Distributive property

a + b = integer

4. Identity property

The number -a is called the


additive inverse

Closure property

(a

Properties of Addition of 1nteger


Properties of Multiplication of
Integers
Properties uf Equality
Properties of Zero
Properties of Exponents
Properties of Radicals
Surds
Special Products
Properties of Proportion
Least Common Denominator
Least Common Multiple
Greatest Common Factor
Remainder Theorem
Factor Theorem

"

II'

Closure properly
1 '

o~h

illl<i<'l

24

~ 00 I Solved

Problems in Engineering Mathematics (2nd Edition) by Tiong & Rojas

2. Commutative property

4. Substitution property
If a =b, then a can be replaced
by b in any expression inl'olving .

ab=ba

3. Associative property

a
5. Addition I Subtraction property

(ab)c=a(bc)

a+1=a
~r

would say that 3 is the power and that


"power" and "exponent" mean the same
thing. The misuse has probably come from
a misunderstanding of statements such
"nine is the second power of three".
The exponential notation states that if a is
a real number, variable or algebraic
expression and n is a positive number,
then:

If a = b, then a + c = b + c
If a = b, then a - c = b - c

4. Identity property

Day 2 -Fundamentals in Algebra 25

5. Inverse property

= ~ with c

c c

~0

lfa+c=b+c,thena=b
If ac = be and c 0 , then a

The number ~a is called the .


multiplicative inverse

6. Distributive property

=b

The properties of zero:


Consider a, b and c as integers or real
numbers or variables of an algebraic
expression.

a(b+c)=ab+ac

a+O=a and a-O=a

2.

a(O)=O

3.

~ == 0 ,

4.

5.

If ab = 0, then a = 0 or b

7. Multiplication property of zero


a(O) =0

The properties ofequality of integers:


Consider a, b and c as integers or real
numbers or variables of an algebraic
expression.

with

2. Symmetric property

=a

3. Transitive property

If a = b and b = c, then a = c

cya cyb = 'fab

~ =(Wf =22
~=4

5 ~675
= ~(5)(675)
= ~3375 = 15

3.

- = n - b;t:O
% b'

cya~

we=
3rso =5
V10 \/10

Property

Example

4.

'ifiFa = mzya

~~=1tfl5

1. am +a"= am+n

x2 + xa = x2+3 = xs

5.

(cya)" =a

(~f =2x

?t

!fa"= lal

~(-12) 4 =l-121

2.

am
m-n
-=a
a"

-=X
x3

a-3

s
=X

2
(y6) = y12

3. (am)" =amn

:::12
(For n =even no.)

5.

a ;t 0

(2x) =2 4 x 4 =16x 4

(abt =ambm

(~r=24=~
4
4

(~r = : :

6.

is undefined

=0. This is

known as Zero-Factor property

Exponent is a number that gives the


power to which a base is raised. For
example, in 32 , the base is 3 and the
exponent is 2.

a=a

cya)

!fa"= lal

What Is an exponent?

1. Reflexive property

If a = b, then b

=(

The properties of exponents with


corresponding examples:

4.
1. .

r,;;

vam

n factors

7. Cancellation property
a(;)=1

2.

"---y---J

If a = b, then ac = be
If a = b, then

1.

Example

a" = a a a a

6. Multiplication I Division property

The number 1 is called the


multiplicative identiy

Property

Exponent should not be misunderstood as


"power" Power is a word that is almost
never used in its correct, original sense
any more. Strictly speaking, if we write
32 = 9, then 3 is the base, 2 is the
exponent and 9 is the power. But almost
everyone, including most mathematicians,

7.

8.

5
(4x)3 = 3 (4x)

1
=am

(x

+2t

index of the radical. For example,

=1

What is a radical?
Radical refers to the symbol that indicates

F.

It was first used in 1525 by


a root,
Christoff Rudolff in his Die Coss.

cya ,

In the expression,
n is called the
Index, a (the expression inside the
symbol) is called the radicand while the
symbol

is called radical

What is a surd?
Surd is a radical expressing an irrational
number. The surd is described after the

-5 =1I(
x5

a0 = 1 (a;;, 0)

= -12

(For n =odd no.)

((44

a"=~
-m

~(-12)3

.J3 is a

quadratic surd, ~ is a cubic surd, ~ is a


quartic surd and so on.
Different types of surds:
Pure surd, sometimes called an entire
surd contairls no rational number and all
its terms are surds.
Example:

.J3 + J2 .

Mixed surd is a surd that contains at least

s.J3 is a mixed surd


because 5 is a rational number while .J3 is
one rational number.

a surd.

26. 1001 Solved Problems in Engineering Mathematics (2nd Edition) by Tiong & Rojas
Binomial surd is an expression of two
ter111s with at least one term a surd.
Example: 5 +

Trinomial surd is an expression of three


terms with at least two or. them are surds
and cannot be expressed as a single surd,
otherwise it will become a binomial surd.
Example: 5 +

Solution:

What is a proportion?
Proportion is a statement that two ratios
are equal.

F2

What is a special product?

( x + y) = x + 2xy + y
(x -

y) = x 2

2xy +

=x

3x y + 3xy2- y

4. Difference of two cubes


3

x -y =(x-y)(x

+xy+l)

5. Sum of two cubes


3

x +y

=(x- y)(x

xy +

l)

6, Square of a trinomial
2

( x + y + z ) = x + y + z + 2xy :t 2xz + 2yz

GCF = 14
What is a Remainder Theorem?

4.

LCD= 360
What is a least common multiple
(LCMI?

5.

If~=~ then a- b = c- d
b

If~ = ~ then a + b = c + d
b

If

d.
d.

~ = ~ then a + b = c + d
b

d'

~a:x=y:dl

means

23 (32 )(5)

a c
b d
3. If - =- , then - =b d
a c

extremes

(x + y) = x + 3x y + 3xy + y

(x- yf

d.

a-b

c-'d

antecedent

3. Cube of a binomial

= 32

LCD=

In number ( 1), quantities a and d are


called extremes while x and y are called
means. If x = y, then its value is known as
mean proportional. In the ratio xly, the
first term x is called the antecedent while
the second term y is called the
consequent.

2. Square of a binomial

a x
1. If - = - . then a : x = y : d

(X + y){X - y) = x2 - y2

=2 3

12=32

a c
a b
2. If - =- then - =-

1. Sum and difference of same terms or


Difference of two squares

15=35

With x, y and z as real numbers or


variables or algebraic expression, the
following are the special products.

GCF = 2(7)

Properties of proportion

F2 + J3

Special products are the expressions


where the values can be obtained without
execution of long multiplication.

Day 2 - Fundamentals in Algebra 27

r:
o

consequent

What is a least common denominator


(LCD)?
Least common denominator (LCD)
refers .to the product of several prime
numbers occurring in the denominators,
each taken with its greatest multiplicity.

A common multiple is a number that two


other numbers will divide into evenly. The
least common multiple (LCM) is the
lowest multiple of two numbers.

Remainder Theorem states that if a


polynomial in an unknown quantity x is
divided by a first degree expression in the
same variable, (x- k), where k may be
any real number or complex number, the
remainder to be expected will be equal to
the sum obtained when the numerical .
value. of k is substituted for x in the
polynomial. Thus,

remainder = f(X)
x->k

What is a Factor Theorem?

Problema
What is the least common multiple of 15
and 18?
Solution:
15=35
18

= 3 .2
2

LCM = 32 (5}(2)
LCM

= 90

What is a greatest common factor


(GCFI?
A factor is a number that divides into a
larger number evenly. The greatest
common factor (GCF) is the largest
number that divides into two or more
numbers evenly.

Problema
What Is the greatest common factor of 70
and 112?
Solution:
70

=2. 5. 7

Problem:

112=2 4 -7

What is the least common denominator of


8,9,12and15?

'

Common factors aro 2 and 7.

Factor theorem states that if a polynomial


is divided by (x - k) will result to a
remainder of zero, then the value (x- k) is
a factor of the polynomial.
Both remainder theorem and factor
theorem were suggested by a French
mathematician, Etienne Bezout (17301783).

Proceed to the next page for your second


test. Detach and use the answer sheet
provided at the last part of this book. Use
pencil number 2 in shading your answer.
GOOD LUCK I

'Orribia:
Did you know that. .. the two long parallel
lin;)S (=)as a symbol for equality was
introduced by Robert Recorde in 1557!

~note:
"Among the great things which are found
among us, the existence of Nothing is the
greatest,"
- Leonardo da Vinci

Day 2 - Fundamentals in Algebra 29

,.,,}

SS'~CME Board October :1995

D.

Solve for the value of x and y.


4x + 2y = 5
13x -3y = 2

c.o. EE Board Apri11997

A.

Topics

Properties of Addition of Integer


Properties of Multiplication of
Integers
Properties of Equality
Properties of Zero
Properties of Exponents
Properties of Radicals
Surds
Special Products
Properties of Proportion
Least Common Denominator
Least Common Multiple
Greatest Common Factor
Remainder Theorem
Factor Theorem

Mon

:t-

Theory

Problems

0
0

Solutions

Notes

~
Tue

0
0
0
0

Wed

Thu

Fri

Sat

<,sj~<ECE Board April :1999

10[~+~ ]=A

If 16 is 4 more than 4x, find 5x- 1.

A.
B.
C.

D.

2[3~-4~]=A

14
12
A.

si~ EE Board October :199:l

F .md t h e va Iue o f x .m -+-1 + -2x = 4 7 - 2 x .

A.
B.
C.
D.

16.47
12.87
18.27
20.17

n: EE Board October :199:1


'"Find the value of x in the equations:

50/9

B. 80/9
C. 70/9
D.

60/9

54. EE Board October :1997


' Find the values of x and y from the
equations:
X - 4y + 2 = 0
2x + y -4 = 0

A.
B.
C.
D.

11/7, -5/7
11/9, 8/9
4/9, 8/9
3/2, 5/3

B:
C.
D.

y = 1/2, X = 3/2
y =3/2, x =112
y = 2, X= 1
y = 3, X = 1

x=-2,y=-3,z=-1

Multiply the following: (2x + 5y)(5x- 2y)

A.

B.

C.
D.

10x - 21xy + 10/


2
-10x + 21xy +
10x2 + 21xy- 10y
-10~- 21xy -10y2

10t

Sb. ME Board October :199&

61. EE Board March 1998

Solve the simultaneous equations:

Determine the sum of the positive valued


solution to the simultaneous equations: xy
= 15, yz = 35,.zx = 21.

2x 2 -3y2 = 6
3x 2 +2y 2 = 35

A.
A.
B.

C.
D.

x = 3 or -3;
x = 3 or -3,
x = 3 or -3,
x 3 or -3,

y = 2 or -2
y = -2 or 1
y = -2 or -1
y = 2 or -3

B.

c:
D.

15
13
17
19

c.z. ECE Board April1991


57. ECE Board May :1997
Find the value of w in the following
equations:
3x- 2y +w = 11
x + 5y -2w = -9
2x+y-3w = -6

A.
B.
C.
D.

B.

Solve for the value of x.


2x-y+z=6
x-3y- 2z = 13
2x- 3y- 3z = 16

c.
D.

6~.

4
3
2
1

x2y1zs

2Y
Xy
1

~
Xy
1

~
Xy
ECE Board November 199~

5x

-=---- 2x2 + 7x + 3

Solve the simultaneous equations:


x+y=-4
x+z-1=0
y+z+1=0

y = -5, Z = 3
y =2, Z =-3
1, y =: -3, Z = 2 '

C.

X "- -1,
X ::: 1,

X ~

t (x-3 yz3 f~

Simplify the following equation

S91 ME Board October 199&

A
II
t;

( xyz-3 )-~

A.

3
2
4
-2

ss. EE Board October 199~

A.
B.
C ..
D.

2 3

Simplify: (x y z-

B.

4
X+3
2

x-3

_j_
x-3

x +3
2x 2

3x - 2

2x+1

+ -:::--2
x +x- 6

Day 2 - Fundamentals in Algebra 31

30 1001 Solved Problems in Engineering Mathematics (2"d Edition) by Tiong & Rojas
D.

D.

2
X+3

&4, ECE Board April1.99:l


s;mplify

~,l [,-!,-l (,,,-Tl

rr

&8. ME Board October :199&


X+2
.mto part1a
'Ifract1on.
.
Reso Ive

A.

A.
B.

C.

X
3
y2

D.

~
y2

&s. ECE Board April :199:1


Simplify: 7

8(7) "" 1 + 5(7)8 + 49(7)8 - 2


8

c.
~

A.
C.

-5a
-3a
-7a

D.

-4a

B.

B.

2
2
( b -4b+16) ( b -16)
= ~-----:::-'--'-----<
b3 +64

A.
B.
C.
D.

b+4
b-4

"'!1/1.

b+2

A.

B.

b -4
c. -

D.

C.
D.

b+2
b-4

&7. ECE Board April

y = 5"
y =9
Y = 52n
y = 18

:199~

x-z
x+z
a+ b

C.
D.

3x ( X - 3 )( X + 2 )
3x ( X + 3 )( X + 2 )
3x ( X + 3 )( X - 2 )
3x (X- 3 )(X- 2)

so.

If p - q = 5 and pq =

81
Evaluate: 64x . 4Y.
256"Y
4x+3y
64x+ 3y
43x+y

A.
B.
C.
D.

= 9Y

A.
B.

G.

81Y 3-x = 243

x2 .

k
25k
25 + k
k
25

81.. ME Board Apri11.995

2a -2x

Simplify bm/n

( a + X )( a -X )
( a + x )( a + x )
2x-2a

A.

Jbffi

7&. ME Board April1.99&

B.

n
bm+n

Factor the expression x 2 + 6x + 8 as


completely as possible.

c.

ifbiTi

D.

A.
B.
C.

{ X+ 4
( X- 4
( X- 4
0 .. ( X + 6

)( X + 2 )
)( X + 2 )
)( X- 2 )
)( X + 2 )

bm
n

82. ME Board Aprill998


Find the value of x which will satisfy the
following expression:

77. ECE Board November 1990


(a-b) 3 =?
A.
LJ.

3a 2 b + 3ab 2 + b 3

--

3a 2 b- 3ab 2
2

b3

<:

a:I + 3a b + 3ab 2

b3

Il

;~

b3

~ , then p2 + q2

equals

Give the factors of a 2

729
140
243

1.5

B.
C.
D.

D.

A.

12
9
11
10

Solve for x in the following equations.

A.
B.

2.25

75. ECE Board November 1.991.

-1~

27x

D.

36
2.5

Factor the expression 3x 3 - 3x 2 -18x

Find a:
A.
B.

A.
B.
C.

79 ME Board Aprill.995

~=10
am

/(2. ECE Board Aprlll99~

Solve for : _x_ = _Y_ = _z_


Y (b-e) (a-c) (a-b)

A.
B.
C.

~
1~

11o. ME Board October 199&


The value of (3 to 2.5 power) square is
equal to:

&&. Solve for x:

A.

4( 52n+1) _ 10( 52n-1)


,Evaluate: y =
2(5 2n)

Given: (an)( am)= 100,000

+ x 2 +2x+5

is a perfect square.

74. ECE Board April :1990

78. Find the value of k so that 4~ + 6x + k

. 7~d:ECE Board April :199~

A.
B.
C.
D.

&9. CE Board May :199&


Find the value of A in the equation:
A
B(2x + 2)
x 2 + 4x + 10
--=--~- = - + -::-'------"x3 + 2x 2 + 5x
x x 2 + 2x + 5

y2
3

D.

6
2
x-4 x-3
3
- - -5x-4 x-3
-6- - -5x-4 x-3
7
- - -5x-4 x-3

-----

2
2.5

X
5

c.

x2 - 7x + 12

B.
y2

C.
D.

a- b

3a b + 3ab

A.
B.
C.
D.

3/2
9/4
18/6
None of these

,Jx- 2 = JX + 2.

3Z 1001 Solved Problems in Engine~ring Mathematics (2"d Edition) by Tiong & Rojas
B.
C.

8~. Simplify V~

A.

D.

B.

89. What is the least common multiple of


15 and 18?
"'

Jab
ab
c. 'Jab
B.

D.

(n -1)!
n!
(n- 1)"

ab

A.
B.

3
5

C.

90

D.

270

I~"

,. If x to the 3/4 power.equals8, x equals

A.
B.

A.

c.

c.

B.

320
2
180

D.

16

D.

90

8S.
Solve for x:

.Jx + 2J2x + 3 -3 = 0

A.

B.
C.
D.

23
3 and 23

20

2
4
6

D.

192

.c,!S;EcE Board April 1999


"'t>iven: f(x) = (x + 3)(x- 4) + 4. When f(x) is
divided by (x - k), the remainder is k. Find

A.

B.
C.

4
6

D.

Solve for X from the given equation:

A.
B.

C.
D.

a= 5, b = 7

a= -5, b 7
a=-5,b=-7
a= 5, b = -7

What is the mean of x, y and z?

92. EE Board April 1996

A.

EE Board March 1998


The polynomial x 3 + 4x 2 - 3x + 8 is
divided by x - 5, then the remainder is,

B.

c.

4
2
3

D.

A.
B.
C.
D.

175
140
218

B.

C.
D.

200

9~ Find the quotient of 3x5 - 4x + 2>t? +


3
36x + 48 divided by x -2>t? + 6.

a+b+c
3
a+b+c

2
a+b+c
abc
abc

--

a+b+c

..,..u

Board October 1991

Hf(x) = 2>t? + 2x + 4, what is f(2)?

C.

4x + 2
16
,(l +X+ 2

D.

A.
B.

A.
B.
C.
D.

88. EE Board Aprll199'7

3>t? - 4x - 8
3>t? + 4x + 8
3>t? - 6x - 8
3>t? + 6x + 8

94~<CE Board November 1991


Find the remainder if we divide

If ri is any positive integer, when (n-1)(n2)(n-3) ... (3)(2)(1)

4y 3 + 18y 2 +By- 4 by (2y + 3).

A.

A.

e<n1l

10

A.
B.

42.31
50
38.62
5'7.12

:lOO.

97. The mean of x and y is a, the mean of


y and z is b and the mean of x and z is c.

~8 ~2.J8X =2

If two numbers namely 250 and 850 are


removed, what is the arithmetic mean of
the remaining numbers? .

D.

96. The expression x4 + ax3 + 5~ + bx + 6

.s'- CE Board November 1991

A.

13

when divided by (x - 2) leaves a


remainder of 16 and when divided by (x +
1) leaves a remainder of 10. Find a and b.

9:1. The numbers 12 and 16 has the


greatest common divisor of

A.
B.
C.

D.

99 ECE Board April :1998


The arithmetic mean of 80 numbers is 55.

c.

What is the lowest common factor of 10


and 32?

-9
6
9

15

k..

90. ECE Board April 1998

,)14. ME Board April 1996

11.

c.

98. ECE Board April 1999


Find the mean proportional of 4 and 36.
A.
B.
C.
D.

72
24
12

20

ECE Board Aprill.998

The arithmetic mean of6 numbers is 17. If


two numbers are added to the
progression, the new set of numbers will
have an arithmetic mean.of 19. What are
the two numbers if their difference is 4?
A.

21, 25

B.
C.
D.

23,27
8, 12
16,20

Day 2 - Fundamentals in Algebra 35

II

14

18
10
140
X=18
70
X=-

16=4X+4
x=3
5x-1=5(3)-1
5x -1= 14

Topics

D
Mon

.__

!"

Tue

D D
D D
~l D
D D
Wed

Theory

Problems

Solutions

Thu

Fri

froperties ~~Addition of Integer


Properties of Multiplication of
Integers
Properties of Equality
Properties of Zero
Properties of Exponents
Properties of Radicals
Surds
Special Products
Properties of Proportion
Least Common Denominator
Least Common Multiple
Greatest Common Factor
Remainder Theorem
Factor Theorem

Bl

II
[ x; 1 +

2:

= 47- 2x} 2

4x+4+6x = 564-24x
34x=560
X= 16.47

10 [A + A]=A}-1
X y
10A

' ' '

-~

"

'

-o

"'"''"''

0
0
D
D

43-50 Topnotcher

Substitute equation 1 to equation 2:

2(4y-2)+y-4=0
8y-4+y-4=0
9y=8
--+ Equation 3

Substitute equation 3 to equation 1:

--+ Equation 1

{2[3~-4~]=A}1

30-42 Passer
25-29 Conditional

--+ Equation 2

32
X=--2

RATING

40. 8
41. c
42. D
43. D
44. 8
45.A
46.A
47.C
48.8
49.A
50. D

2x+y-4,;0

X=4(%)~2

ANSWER KEY
27.A
14. c
28. 8
15. 8
16. 8
29. A
30. 8
17. c
31. A
18. D
32. D
19. 8
33.A
20. D
21. A
34. D
35. D
22.C
36. 8
23. 8
37. D
24. c
38.A
25. c
39.C
26. 8

--+ Equation 1

y =%

.!.--101 x
1

1. D
2. 8
3. c
4.0
5. c
6.A
7.8
8. c
9. c
10.A
11. 8
12. c
13. 8

x-4y+2 =0
x=4y-2

Ill
1 1
1
-+-=X
Y 10

Sat

. Notes

14
X=9

4x+2y = 5

~-.!!. =1
y

--+ Equation 2

0-24 Failed

y=

%- 2x

13x- 3y = 2

--+ Equation 1
--+ Equation 2

Substitute equation 1 in equation 2:

If FAILED, repeat the test.

Substitute equation 1 to equation 2:

~-a(_!__.!.)= 1
10

13x-3(%-2x) = 2

---+-=1
X
10 X

.!i= 1+~
X

10

18x=2+~
2

18x=~
2

X=.!_

Day 2 - Fundamentals in Algebra 37

36 100 !solved Problems in Engineering Mathematics (2nd Edition)' by Tiong & Rojas
Substitute equation 5 in equation 7:

=%-2(~)=%

~~

( 2x

9[7w1;38]-w=-12
y+(5+y)+1=0

63w- 342 -17w = -204


w=3

3y = 6 ) 3

6x 2 -9y 2

=18-7 Eq.

)fl

-7 Eq. 1

= 13

-7 Eq. 2

+ 4y = 10 -7 Eq. 2

6x + 4y

- (

6x 2 - 9y2 ) = 70-18

Z=2
-=X

X= -1

Consider above equation as Eq. 4


-

(2x + 5y){5x- 2y) = 10x2

9(2) = 18
6x 2 =54
X=3

Iii

5x
2x 2 + 7x +3

ID

x-z=3
x=3+z

3x - 2y + w = 11

-7 Eq. 1

3x+15y-6w=-27

-7Eq.4

-10y 2

Subtract equation 2 from equation 5:

Ill

5x + 5z = 5
z = 1-x

-7 Eq. 3

Multiply equation 2 by equation 3:

4XY+ 25xy

(2x + 5y)(5x- 2y) = 10x2 +21xy -10y 2

x + 5y - 2w = - 9 -7 E;q. 2
2x + y- 3w = - 6

Multiply equation 1 by equation 2:


6x - 3y + 3z = 18 -7 Eq. 5

(5x- 3y + 3z)- (x- 3y- 2z) =18-13

(xy)(yz)(zx) = 15(35}(21)

Substitute

xy = 15, in equation 4:

17y -7w = -38


7w-3
y=-17
Value of y above is considered as Eq. 5

2x =4
X=2

II

-7 Eq. 6

Subtract equation 6 by equation 3:

(2x + 10y -4w) -(2x + y- 3wl= -18- (-6)


9y-@= -12
Consider the above equation as ;q. 7

4(2x + 1)(x- 2)
(2x + 1)(X+ 3)(x- 2)

+ y =-4

+ z - 1 = 0 -7 Eq. 2

-7 Eq. 1

X=3
Substitute zx = 21, in equation 4:

Subtract equation 1 from equation

2:

-:+

x~ [ xsyJ (x y2 r~
2

21y = 105

Eq. 4

Thus, x + y + z = 3 + 5 + 7

Jr

Y=5

xtz-(x+y)=1-(-4)

z =5 + y

X+3

35x = 105

y + z + 1 = 0 -7 Eq. 3
Multiply equation 2 by 2:
2x + 10y- 4w = -18

2x+1
(x + 3)(x- 2)

- 3x -.2)
- (2x + 1)(x + 3)(x ~ 2)

Substitute yz = 35, in equation4:

z3

+~~-

4(2x 2

15z =105
Z=7

x=3+(1-x)

(3x + 15y -6w) -(3x -2y + w) = -27-11

x +3
(2X+ 1)(x- 21

8x 2 -12x-8
(2x + 1)(x + 3)(x- 2)

-:+ Eq. 4

X=3+Z
Subtract equation 4 by equatiqn 1:

5x 2 -10x-x 2 -6x-9+4x2 +4x+1


(2x + 1)(x + 3)(x- 2)

xyz = 105

Substitute equation 6 in equation 4:

2x +1
x +3
2.
+-2-2x - 3x - 2 x + x - 6

5x
(2x + 1)(x + 3)

(xyz) . = 11025

Consider above equation as Eq. 6

2-

X.Y

5x(x- 2)- (x + 3)(x + 3) + (2x + 1)(2x + 1)


(2x + 1)(x + 3)(x- 2)

Multiply the three given equations:

.\1~

y 7 z -3 =

-2

X= -4- (-3)

( 2x - 3 y - 3z) - ( x - 3 y - 2z) = 16 -13

y =2
6x

x~y-~zlf

-9-1+~
6-1-U
=x -6+1+~
22y
22Z22

Solving for x:

13y2 =52

(xyz 3 r~

2x- 3y- 3z = 16 -7 Eq. 3

Subtract equation 2 from equation 3:


2

(x...sy-9zs){x~y--lz-!)

z = 5 + (-3)

Subtract equation1 from equation 2:


2

(x2y3z2t (x-3yzT!

=6

y = -3

2x- y + z = 6
x- 3y- 2z

t>x

2y

Solving for z:

( 3x 2 + 2y = 35 ) 2
2

Substitute equation 4 in equation 3:

= 15

=x

[x-~yi

= x4 [

x4 x 3 y- 2 x- 3~ y3
1

x 5 y~ J

]3

38 100 i Solved Problems in Engineering Mathematics (2"d Edition) by Tiong & Rojas
~ Eq.1
A=1-B
Equate constants:
2 = - 4A - 3B ~ Eq. 2

x-1y~ = '_

r 2

(3) '(3t' ':' (3)


6x- x = 5

5x =5
8(7)8 +1 + 5(7)8 + 49(7)8 -

= r7 2 - (8)7.7 1 + (5)r +

49 7
< ">
72

Substitute equation 1 in equation 2:


2 ::; - 4(1 -B) - 3B
2 =- 4 + 4B- 3B
B=6
A=1-6=-5

=r(49-56+5+1 >

i"

Day_g_- f'll.rldamentals in Algebra 39

=-7"

2
( b2 - 4b + 16 ) ( b -.16 )
x-b3 + 43
2
( b - 4b + 16 ) ( b- 4 )( b +.4 )
x-~----~~~~~
( b + 4 )( b2 - 4b + 42 )
X=b-4

Thus,

X+2
6
5
=----x2 - 7x + 12 x- 4 x- 3

(b-e) by-cy
x=y--=-.-a-c
a-c
(a-b) ay-by
z=y--=--a-c
a-c
by -cy ay -by
X+Z=--+-a-c
a-c
ay-cy y(a-c)
X+Z=---=--a-C
a-c
X+Z=Y

ID

X+2
A
B
=--. +-x2-7x+12 x-3 x-4
x+2
=A(x-4)+B(x-3)
(x- 3)(x- 4)
(x- 3)(x- 4)
x+2=A(x-4)+B(x-3)
x + 2 = Ax - A4 + Bx - B3
Equate coefficients of x:
1=A+B

4( 52n+1) -1

+ 2
x +2x + 5

x 3 +2x 2 + 5x
By equating constants:

3
(a-b) =a3 -3a 2b+3ab 2 -b 3
2
4x + 6x + k = 0

(X+

52"
2

"

[\1

(a"){am) = 100,000

Eq.1

a"
= 10 ; a" = 10 am
a

Eq. 2

J=243
Substitute am

ED
p-q=5
By squaring both sides:

(P- q)2

=100, in equation 1:

1000m = 100000

m=2
~

Eq.2

Substitute equation 2 in equation 1:


(3)4Y(3t' = (3)5

Substitute

m = 2, in equation 3:
2

a = 100
a= 10

=52
2
2pq + q = 25
p 2 + q2 = 25 + 2pq

=25+2(~)

p2 +q 2 = 25 + k

~ Eq. 1

(a")( am)= (a")"'= 100000

2
2
p +q

a" = 1000
(a")(am) =am"= 100000

Squaring both sides:


(3)ox = ( 3)4y

r=0.~5k

3x 3 - 3x 2 - 18x = 3x ( x 2 -- x - 6 }
3x 3 - 3x 2 -18x = 3x ( x-3 )( x + 2 )

am= 100

= {9)Y

=0

II

Ill

(amt = 10000

(3)3x = {3)2y

\n2

k = 2.25

1oamam = 100000

{81)Y(3tx = 243
(3) 4Y(3tx = {3)5

Since it is a perfect square, then

a" (100) = 100000

{27)x

2(52n+1) _ 5( 5 2n-1}

A =2

Iii

Iii

x 2 + 1.5x + 0.25k = 0

Substitute (2) in (1):

10=5A

25
(3) .

2
x +6X+8 = (x+4)(x+2)

y=9

Ax 2 + 2Ax + 5A + 2Bx 2 + 2B)\. + Cx

11 [
II

IZI

o( 52n-1)

1
2
1
-5 )-5(5 " 5- )
y=
52"
1
1
y=2(5 )-5(5- )

A
B(2x+2)
.
= ~ + -=-'"----'x3 + 2x 2 + 5x
x x 2 + 2x + 5

2
2
a - x =(a+ x)(a- x)

2(5 2")

y=

2(5

x2 +4x+10

64x4y = (4)3x(4)Y = (4}3x+y

X+2
X+2
x 2 -7x+12- (x-3)(x-4)

Ill
y=

A(x 2 +2x+5}+Bx(2x+2)+Cx
2
x('x + 2x + 5}

X= 1

~~~

Ell

bm/n = (bm)~ =

Ill

Jx- 2 = JX + 2
By squaring both sides:
( Jx- 2

=(

JX + 2

x-2=x+4JX+4
4JX =-6

40 __! 00 i~olved Problems in Engineering Mathematics (2nd Edition) by Tiong & Rojas

x = 23

-+ Absurd

X=3

~8 ~2$x

Ell

=2

x +2x 2 +6}3x5

~2$x

= (2)4

~2../8X

=2

=16

.J8X=4

By raising both sides to exponent 2:

v~=~

X= (8)3

By squaring both sides:


x +2J2x +3 = 9
2../2X+ 3 = 9 -

El

By squaring both sides:


4(2x + 3) = (9- x)

8x + 12 =81-18x + x 2

x2 - 26x = -69

""' '
\

f(x) = 2Y! + 2x + 4
f(2) = 2(2)2 + 2(2).+ 4 = 16

II

f(r) =Sa+ 2b + 42
16 =Sa +2b+42
-26 =Sa +2b
b=-13-4a
When divisor is (x + 1), r

=-1 & f(r) = 10

f(r)=(-1t +a(-1) +5(-1t +b(-1)+6

Remainder= f [-%]

10=-a-b+12
2=a+b
Substitute equation 1 in equation 2:

liiilll

= 3

f(r) = 2 + a(2} + 5(2t + b(2) + 6

0 .
f(y) =4y 3 + 1Sy2 + Sy _ 4

=2 & f(r) = 16

f(r)=-a-b+12

n! = n(n-1)(n-2)(n-3) ... (3)(2)(1)


(n -1)! = (n-1)(n-2)(n-3) ... (3)(2)(1)

.Jx + 2~2x + 3

When divisor is (x- 2), r

(-) 8x 3 -16x 2 +48

f(x) = x + axl + 5Y! + bx + 6

Note: Remainder = f(r).

12x3 + 36x

Note: Using remainder theorem,

11!9!11

X= 16

divisot= 2y + 3 = y- [ -%]

X=2

EFI

8x 3 -16x 2 + 48

Sx =(4)2 =16

[x~ =8J

3x 2 +6x+8
4x 3 + 2x 2 + 36x +48

(-) 6x 4

2$x = (2)3 = 8

/ ab

3(5) + 8 = 218

k=4
k=-2

6x 4 -4x 3 -16x 2 +36x

By raising both sides to exponent 3:

fab'
v7a--'
b =(ab)'

(-) 3x 5 -6x 4 +18x~

By raising both sides to exponent 4:

lab r>
vif.ili
"(ab)'
lab [
Vifab
= (ab)"']1'

ID

Remainder= (5)3 + 4(5) 2

lei)

VVab- ~(ab)~

IIJ

k -2k-8 =0
(k-4)(k+2)=0

3x + 8 ; divisor = x- 5

Note: Using remainder theorem,


remainder= f (5).

x-13=10

Etl
jab-

(x -13) = 100

Note: Since x = 9/4 will not satisfy to the


given general equation when substituted,
this equation is classified as defective and
thus, the a~swer is "None of these".

k =k2 -k-8

f(x) = x3 + 4x 2

(x-13) =-69+{13t

9
4

X=-

1!1

By completing the square:

JX =-~

Day 2 - Fundamentals in Algebra 41

15 =53
18 =63
LCM = 5 6 3 = 90
10 =52
32 = 2 . 2 . 2 . 2 . 2
Lowest common factor

t[-%]=4[~%J +18[-%J +8[-~]-4

a =-5

ID

b=-13-4(-5}

f(k) =(x + 3)(x- 4) + 4


f (X) =

12=4.3=43
16=4.4 =44
Greatest common divisor = 4

=15

t[-%]=11

f(x)=x 2 -4x+3x-12+4

=2

2=a+(-13-4a)
-3a

X2 - X- 8

Remainder = f(k)

f(k) = k2 - k - 8
Substitute the given remainder = k in
equation 1:

b=7

Iii
X+Y=a

'

y+z=b

X+Z=C

. '

By adding a, b and c:
a+b+c= X+y +Y+Z+X+Z

'

2
1

a+ b + c = -[2x + 2y + 2z]

4~ -1001 Sb1ved Problems in Engineering Mathematics (2nd Edition) by Tiong & Rojas

'~

'.

~~

a+b+c=x+y+z
Mean = x + Y + z _ a + b + c

Topies

Let: x = the mean proportion of 4 and 36

= __>5_

Jx

Mon

36
= 144

X=

12

Tue

D D
D D
D D
[I] D

m
Let: x = the arithmetic sum of 80
numbers,

II

Arithmetic mean = __>5_ = 55


80
X 80(55) 4400

y = new Arithmetic mean


_ x-(250+850)
y. 80-2

Theory

Wed

Problems

Thu

Solutions

Fri

Notes

y = 42.31

111

'~~

Let:

x = the first number


x + 4 = the second number
y = sum of the original 6 numbers.

y+x+(X+4) 19
6+2
102 + 2x + 4 =
19

2x

= 46

X=

23

'---~ ~-- ---~..,.-,- ~~---r.

T
[

y = 17(6) = 102

=19(8)

--

Sat

Arithmetic mean = 'j_ = 17

106+2X

-~

Properties of Addition of Integer


Properties of Multiplication of
Integers
Properties of Equality
Properties of Zero
Properties of Exponents
Properties of Radicals
Surds
Special Products
Properties of Proportion
Least Common Denominator
Least Common Multiple
Greatest Common Factor
Remainder Theorem
Factor Theorem

;\.:;\;
~--i-i
il'l'r

li
!~
\

'J

X+ 4 = 27
l

44

r--.

Dj
I
D
Mon

I"

'II

Tue

I>JJ II

Theory

~
Wed

D D
D D
D D

Problems

Thu

Solutions

Fri

Notes

Sat

What is a Quadratic Equation?

'

Topics
-~--

Quadratic Equation
Quadratic Formula
Properties of Roots
Discriminant and Nature of Roots
Binomial Theorem
Binomial Expansion
Properties of Binomial Expansion
Pascal's Triangle
Degree of Polynomial or Equation
Logarithms
Properties of Logarithms
Modulus of Logarithms

The following is the quadratic formula:

Quadratic is an expression or an equation


that contains the variable squared, but not
raised to any higher power. Quadratic
equation in x contains x2 but not x3 .
I he general quadratic equati'm is
~~xpressed

as:
Ax 2 +Bx+C = 0

where, A, Band Care real numbers and


wrth A 0.
When B = 0, quadratic equation is known
o~s a pure quadratic equation.
1\ quadratic equation in x is also known as
.r ~u~cond-degree

polynomial equation.

1111 :;olution to a quadratic equation is


rill<~~ by factorin~J or by th(~ liSe of the
<Jlloldl.tliL IO!IIIItla

-BJB2 -4AC

X=-----

2A

The quantity Js 2 - 4AC in the above


equation is known as the discriminant.
The discriminant will determine the nature
of the roots of the quadratic equation.
The table below shows the value of the
discriminant and its corresponding nature
of roots.

JB

-4AC

0
>0
<0

Nature of roots
Only one root .
. (Real and ~qual)
Real and unequal
Imaginary and unequal

The sum and product of the roots of a


quadratic equation can be solved even
without using factoring or quadratic

Day 3 ~Quadratic Equation, Binomial Theorem, Logarithms 47

4~ 100 i Solved Problems in Engineering Mathematics (2"d Edition) by Tiong & Rojas

formula as long as the general equation is


given.

The following are just a few examples of a


binomial expansion:

The following are the properties of the


roots of a quadratic equation:

(X+Y)

Sum of the roots:


When the two roots are added, the result
is:

r1+r2=-;;.

"

Product of the roots:


When the two roots are multiplied, the
result is:

Binomial
(x + y)o
(X+ y)1

What is a Binomial Theorem?


Binomial is an expression containing two
terms joined by either+ or-.
Binomial theorem gives the result of
raising a binomial expression to a certain
power. The expansion and the series it
leads to are called the binomial
expansion and the binomial series,
respectively.

il

The binomial theorem is expressed as


follows:

(x'f:y)" =x~ +nx"-!;y +p(n-: )xn-~y~ +

. .'f'

. "

..

21

... +nxy't!~1 + yn
Binomial coefficient is a coefficient of x
in the expansion of ( x + y)", The binomial
coefficient

(x + y) = x + 3x y + 3xy + y

As observed in the binomial expansions


above, some properties were established
and are enumerated as follows:
Properties of Binomial Expansion
of (x + y)":
1. The number of terms in the resulting
expansion is equal to n + 1.

0
rir2""A

ncm gives the number of ways

of picking m outcomes {not in any


particular order) from n possible outcomes.
The binomial coefficient forms the rows of
the Pascal's triangle.

(x+y)":
Sum= n(n + 1)

( x + y ) = x 2 + 2xy + y
3

Sum of expon~nts of the expansion of

=1

(x+yf=x+y
Let r1 and r2 be the roots of a quadratic
!3quation:

In Italy, this triangular pattern is known as


Tartaglia's triangle while in many parts of
Asia, it is referred to as Yang Hui's
triangle.

2. The exponent of x decreases by 1 in


succeeding terms, while that exponent
of y increases by 1 in succeeding
terms.
3. The sum of the exponents of each term
is equal ton.
4. The first term is x" arid the last term is
y" and each of the terms has a
coefficient of 1

Pascal's Tiangle
1
1 1

1 2

{x+d
{x+d
{X+ y)4
(X+ y)s
(X+ y)6

What is a Degree of a Polynomial or


Equation?

1 3 3 1
1 4 6 4 1
15101051
1 6 15 20 15 6 1

Another way to determine the coefficient of


any term in the binomial expansion is to
use the following formula:

Example:
C =(Coeff. ofPT)(Exponent of x of PT)
E;xponent of y of PT + 1
where: C = coefficient of any term
PT = preceding term
The r1h term of the binomial expansion of
( x + y )" may be calculated using the
following formulas:

2)- .. (n- r + 2) x(n-r+1)

Each number in the triangle is equal to the


sum of the two numbers immediately
above it.

2. What is the degree of the


polynomial3x 4 y- 2x 3 z4 + 7yz 5 ? .
Answer: 7.
7 is the sum of 3 and 4 in the second
term.

(r -1)!

rtt\ = nCr-1X(n-r+1)yr-1

The coefficients of a binomial expansion


can also be conveniently obtained by
arranging them in a triangular array or
pattern. This is known as Pascal's Triangle
named after the famous French
mathematician Blaise Pascal (16231662).

1. What is the degree of the monomial 7x5 ..


Answer: 5

What is a Logarithm?
n(n -1)(n-

th

5. The coefficient increases and then


decreases in a symmetric pattern.
The Pascal's Triangle:

Degree of a polynomial or equation with


only one variable refers to the exponent of
the variable. For a polynomial or equation
that contains two or more variables, the
degree is the maximum sum of the
exponents of the variables in a single term.

A term involving a variable with a specific


exponent is obtained by using the
following formula:

y'

t-

needed to give x.

-J- - -..1

Logr6=4 maybewOttena

----n

r
~

=16

_
=-n(n-1)(n-2)(n-r+1)
x" 'y'
r!

Sum of the coefficients of 'the expansion of

(x

The logarithm of a number or variable x to


base b, Iogb x , is the exponent of b

y)":
Sum = ( Coeff. of x + coeff. of y )"

The term "logarithm" comes from Greek


words, "logus" meaning "ratio" and
"arithmus" meaning "number". John
Napier (1550 -1617) invented logarithm
in 1614 using e = 2.718 ... for its base.
Logarithm with base e {loge or In) is called
the natural logarithm or the Napierean
logarithm. In 1616, through the
suggestion of John Napier, Henry Briggs
(1561 0 1630), a professor of Geometry at

48 1001 Solved Problems in Engineering Mathematics (2nd Edition) by Tiong & Rojas
Gresham College in London, improved the
logarithm using 10 as the base. The
logarithm with base 10 is known as
common logarithm or the Brlggsian
logarithm.
The number e which is the base of the
natural logarithm is known as Euler's
number, named after the Swiss
mathematician, Leonhard Euler (17071783) and is defined as,

= um(1+!)n
-

Binary logarithm (denoted as lb) is a


logarithm with a base value of 2.
Relation between natural logarithm and
common logarithm:
The natural logarithm can be converted
into a common logarithm and vice versa.
To obtain this, a factor known as the
modulus of logarithm is necessary, such
as:
logx = 0.43431nx

Proceed to the next page for your third


test. Detach and use the answer sheet
provided at the last part of this book. Use
pencil number 2 in shading your answer.
GOOD LUCK!

Topics

ll

~ribia:

Mon

Did you know that ... Isaac Newton while


a student at age 22, invented differential
and integral calculus, discovered the law
of universal gravitation, formulated the
three laws of motion, developed the new
theory o flight in just 18 months and set a
record of the most productive periods of
achievement by an individual in the history
of science!

<!auote:

Tue

D
'

Problems

What are the Properties of Logarithms?

=logx +logy

1.

log(xy)

2.

log(~)= logx -logy


=nlogx

3.

logx"

4'.

logx
logbx=Iogb

5.
6.

logbx
1og.x= logba
log.a=1

Wed

Solutions

Notes

'~o

~-

~-

Thu

D [_]
D D

"The art of asking the right questions in


mathematics is more important than the art
of solving them."
- Georg Cantor

lnx = 2.30261ogx

The coefficients 0.4343 and 2.3026 are the


referred, to as the modulus of logarithm.

Quadratic Equation
Quadratic Formula
Properties of Roots
Discriminant and Nature of Roots
Binomial Theorem
Binomial Expansion
Properties of Binomial Expansion
Pascal's Triangle
Degree of Polynomial or Equation
Logarithms
Properties of Logarithms
Modulus of Logarithms

'o.,.,,...,._.--~

J11
~~~:.

1,,

.~

tJ

'

l
1

i
~

Fri '

Sat

1011 ECE Board March 1996


!"he equation of whose roots are the
reciprocal of the roots of 2x2 -- 3x - 5 = 0
IS,
2

5x + 3x - 2
2x2 + 3x - 5
3Jt - 3x + 2
2x-2 + 5x- 3

1\
ll.
C.
U.

=0
=0
::: 0
=0

8.

c.
D.

3
4
5

J:04: ME Board October :1.996


Solve for x that satisfies the equation 6x2

...:7x-5 = 0.
A.

5
-1
-or-

lOZI

qual to

B.

3
3
-or-

EE Board October 199~


In the equation Jt + x = 0, one root is x
f\
1!.

I:.
I>

1
5
1/4
none of these

( O:Jt ECE Board Aprilt990


. nlw for the value of "a" in the equation
" I ,,, I 1G '~ 0.
I\

t .'

C.
D.

2
8
7
-7
-or5
15
3
3
-or5
4

lOS: EE BoCJ~rd Oetober 1:997


Find the values of x in the equation 24~ +
5x- 1 "= 0

SO

A.

100 1 solved Problems in Engineering Mathematics (2"d Edition) by Tiong & Rojas
1.1.1.: Given the equation 3x + Bx + 12 =
0. What is the value of B so that the roots
of the equation are equal?

(~.1)

t~ ~)

A.

(~

~)
2'5

B.
C.

c.
D.

~8' -~)
3

J...,;;

B.

6'5

1.06: EE Board October 1.990

i"

D.

Determine k so that tl.e equation 4x + kx


+ 1 = 0 will have just one real solution.

A.
B.

3
4

C.
D.

5
6

A.
B.

C.
D.

10

8064
8046
8046
4680

x l
5
x
10
x
x5 y

Find the 5th term of the expansion of

/ ( x2 +1 )10

B.

A.

- 0.113,- 0.887

C.

B.

- 0.331;- 0.788
-0.113,-0.788
-0.311,- 0.887

D.

D.

J.08: If 1/3 and -3/2 are the roots of a

B.

C.
D.

6~ + 7x - 3 = 0
6x2 - 7x + 3 = 0
6~ - 7x- 3 = 0
6~ - 7x + 1 = 0

0?
A.

B.

0.6
-0.6

D.

0.75

c. -o.8

A.

C.

-16
16

D.

-8

B.

D.

D.

-39396
128a11

1.24: ME ~ard April 1.997


What is the value of log tci base 10 of

D.

B.
C.

1.181 What is the coefficient of the term


'free of x of the expansion of (2x- 5y) 4 ?

256
526
265
625

-548~l

103
1650 x
103
161700 X
103
167100 x
100
167100 x

-154,288 xV
- 1,548,288 xV

A.
B.
C.
D.

1
2
3

1.21.:

ECE Board April 1.995

c.
D.

What is the value of (log 5 to the base 2)


+(log 5 to the base 3)?

A.
B.
C.
D.

7.39
3.79
3.97
9.37

1.26: Find the value of log4 (log3 5).

A.
B.

C.
D.

1.460
0.275
1.273
0.165

:l%7: Given: log4 7 = n. Find log4!


7

4
/the term next to 495x y ?
660
792
990
1100

10.9
99.9
9.9
9.5

1.25: ECE Board April 1998

What is the sum of the coefficients in the


expansion of (x + y- z) 8 ?

A.
B.

A
13.
C.
D.

.1.-1.6: What is the numerical coefficient of


A.
B.

A.
B.
C.

D.

-148,288 x3

A.
B.
C.

3.76
5.84
4.48
2.98

1000 33 ?

What is the sum of the coefficients of the


expansion of (2x -1 )20 ?

2 100
'expansion of (x + x ) ?
A.
B.
C.

(A.

tzo:ECE Board November 1.995

1.1.0: What is the discriminant of the


equation 4~ = 8x- 5?

-33669
256a1t

D.
63,360
126,720
506,880
253,440

usf What is the fourth term of the

1.09: Which of the following is a root of


this quadratic equation, 30~ + 49x + 20 =

Evaluate the log6 845 = x:

d:;~Find the 6th term of (3x- 4y~~)

'In the expression of ( x + 4y ) , the


numerical coefficient of the 5th term is,

D.

1.98

C.

D.

12

B.
C.

178

-66339
128att

C.

260 x
8
5040 x
8
210 x
8
420 x

quadratic equation, then the equation is

A.

C.
D.

B.

A.

ulli'ECE Board April 1.998

A.

1.86
1.68

-66939
256att

B.
A.

)t6

B.

A.

C.

A.

1.23: CE Board November 1.997

1.07: ME Board April 1.996

2
Solve for x: 10x + 10x + 1 = 0

\_2a

10
-12

.lJ:il

".n7: CE Board November 1.996


Find the 6th term of the expansion of
1--3

Find the term involving y5 in the


.;expansion of (2x 2 + y) 10
2

Day 3 - Quadratic Equation, Binomial Theorem, Logarithm 51

0
1
2
3

t:u:r CE Board November 1.99~


ECE Board Nov. 199:J
I 1nd the valut! of loq., 'Ill

C.

1/n

n
-1/n
- n

1.28: CE Board November :199:2


CE Board May 1.994
If log a 10 = 0.25, what is the value of
log 1oa?

A.

2
11

I
.::_f
,.~

52 l 00 l.Solved Problems in Engineering Mathematics (2"d Edition) by Tiong & Rojas

C.
D.

:134: ECE Board November :199:1

6
8

Given: Iogb 1024

:l:t9: ECE Board November :1995


Given: log b y = 2x + log b x. Which of the
following is true?

A.
B.

Y = b2x
y = 2xb
y=-

D.

y = xb2x

-7x
10 to the -7x power

D.

:1401 Solve for the value of x:

Find b.

log2x 3 +log-= 6.278


X

.~

B.
C.

i:l

D.

-1

B ..

C.

-1 and

D.

1 and- 9

-7 log to the base 10

9~

:13&: ECE Board April :1993


Solve for the value of x in the following
equation: x 3109 x = 1OOx .

A.
B.

c.
D.

12

8
30
10

it
,~

'l

'" --~

:137: EE Board October :l99:t


Given: log 6 + x log 4 = log 4 + log (32 +
4). Find x.

logx

n log x
C. log (x to the 1/n power)

B.

n
D.

(n -1)1ogx

:l:J:tl ECE Board November :1990


Log (MN) is equal to:

A.
B.

C.
D.

Log M- N
Log M + N
N log M
Log M + Log N

:l:J:JI ME Board April :1997


What expression is equivalent to log ( x )
-'log ( y + z )?
A.

B.
C.

D.

log
log
log
log

x + log y + log z
[ XI ( y + z )]
x -log y - log z
y + log ( x + z )

A.
B.

2
3

D.

c. 4
ECE November :1998
If log of 2 to the base 2 plus log of x to the

:1:581

base 2 is equal to 2, then the value of x


is,

A.
B.

4
-2

D.

-1

c. 2

:1391 ME Board October :1997


Find the value of x if log12 x = 2.

.~1

A .. 144

B.

414

D.

524
425

c.

A.

'>'~ .
,'j~t

:l:J:l: ME Board April :1996


Log of the n1h root of x equals log of x to
1/n power and also equal to:

A.

c.

2560
16
4

A.

:l:JO: ME Board October :1996


Which value is equal to log to tlte base e
of e to the -7x power?

B.
C.
E.

A.
B.

=~

:l:JS: Given: log3 (~- 8x) = 2. Find x.

2x

c.

A.

Day 3 - Quadratic Equa!1e_n, Bin_o!'lial Theorem, Logarithm 53

,f-

379.65
365.97
397.56
356.79

Day 3- Quadratic Equation, Binomial Theorem, Logarithm 55

ml

2x 2

7 ~(7) -_
4(6)(-5)
__
X=--'--'-,.:__,_
2(6)

3x- 5 = 0

(2x+2)(x-25)=0

7 13
X=-12

X= 2.5

Topics

D
tv! on

D
Tue

[QJ

D
D D
D
D D
Theory

Problems

Wed

Thu

Solutions

Fri

Notes

Sat

RATING

ANSWER KEY
101. A
102. D
103. A
104.A
105. D
106. 8
107. A
108. A
109.
110. 8

111. D
112. A
113.
114. B
115. 8
116. 8
117. 8
118. D
119. D
120.A

121. 8
122.A
123.A
124.C
125. 8
126. 8
12"1. D
128.8
129. D
130. A

Quadratic Equation
Quadratic Formula
Properties of Roots
Discriminant and Nature gt;Roots
Binomial Theorem
Binomial Expansion
Properties.of Binomial Expansion
Pascal's Triangle
Degree of Polynomial or Equation
Logarithms
Properties of Logarithms
Modulus of Logarithms

131. A
132. D
133.8
134. 8
135.
136. D
137. 8
138.
139. A
140.C

c
c

c:J

34-40 Topnotcher

c:J 26-33

o
o

Passer

X= -1
Thus,
Thus. the roots of the second equation
are:

X1 = - = 0.4
2.5
1
x2 = - = -1
-1

5
x1 =3

1
and x 2 = - 2

24x 2 + 5x -1 = 0

Using the quadratic formula;

-5~(5) 2

Solving for the second eqution:


X

2(24)

(x-0.4)(x+1)=0

-5 11

X=-48

x + x- 0.4x- 0.4 = 0
{ x 2 + 0.6x- 0.4 = 0 ). 5
5x 2 + 3x -2 = 0

x(x+1)=0

X=O
where: A = 4; 8 = k & C = 1

X= -1

8 2 -4AC = 0

a8 -17a 4 +16 = 0
2
8
Let: x = a4 and x = a
x2 -17,+16=0

Using the quadratic formula;

17~(17)
-4(1)(16)
_.!....:__:__ __
X= _
2

k 2 -4(4)(1)=0
k

17 15
X=--

X=

Thus,

a= 1

= 16

k = 4

x1 = 1
a4 = 1

10x2 + 10x + 1 = 0
Using the quadratic formula:

If FAILED, repeat the test.

Thus, x1 = B and x2 = -

Note: There is only one solution to the


equation (4x2 + kx + 1 = 0), if the
discriminant (8 2 - 4AC) is equal to zero.

20-25- Conditional
0-19 Failed

-4(24)(-1)

= ----L'----'--,-::-c---

2(10)
-10 7.746
X=---20

X2 = 16

a4

= 16

a= 2
Thus, x1

ml
6x 2 - 7x- 5 = 0
1J:;lll~J the quadratic formtila;

-10~(10) 2 -4(10)(1)

lml

=- 0.887 and x2 =- 0.113


x1=

and

x 2 =-~

S6 ..100 I Solved Problems in Engineering Mathematics (2"d Edition) by Tiong & Rojas

-B

x1 +x2 = -;;;.

where: A = 4; 8 = - 8 & C = 5

Day 3 - Quadratic Equation, Binomial Theorem, Logarithm 57

(X+X2)100

Substitute:

.c
A

82

i+(-%)=~~

il'

=0
2
8 = 144

4{3){12)

B=~A

liB

C =_:!A

is the 61h term


Note: The term involving
10
of the expansion (2~ + y)

Substitute to the general quadratic


equation:
2

Ax + Bx + C

=0

lx +(-=~At,,
0
2 )

6x?. + 7x- 3 = 0

0!- - ( 32x 10) y5


= - -1-

(10-5)!5!
61h term= 8064 x 10y5

~~~

-49 J(49)2 -=4(30).(20)


X=----------2(30)

5th term =

X=---

10

60

Thus, x,
X2

11.1

-491

= - - = -0.8
60
-49-1

(__!__)n-r+ (-3)'-1
2a
where: n = 16; r = 6

1
6th term = 1s C 5 - a
(2

(~}. 6 (lx..!.)

)11 (-3)

)( 243)
16!
(
1
- (16-5)!5! 2048 a 11 -

12 (

1)
x4

4368(-243) 16
= 2048 a 11 + 16

5th term= 210 x8

4x - 8x + 5 = 0
where: A = 4; B = - 8 & C = 5
Discriminant = B2 - 4AC
= (- 8) 2 - 4(4)(5)
= -16

Noie: The roots of the equation (3x2 + Bx


+ 12 0) are equal, if the discnminant (82
- 4AC) is equal to zero.

1111

5th term= nc,_, (x)n-r+ 1(4y)'" 1

Note: The term in the expansion (2x sd which is free of x is the last term
or the 5th term.

where: n = 12; r = 5
5th term= 12 C 4 (x) 8(4y}

i2!

"" - - - - - - (x )(256y )
(12-4)!4!
5th term = 126,720

x8 l

(3x-4d

6th term= ncr-1 (3x)n-r+ \-4y)'" 1

sth term= ncr-1 (2x)"r+ 1(-5y)'" 1


where: n

= 8; r = 6

where: n
8 C5

(3x) 3(-4y) 5

81
3
5
)
(27x )(-1024y )
8-5 !5!
6th term = - 1,548,288 x 3y 5

El
Note: To solve the sum of the coefficients
20
of (2x -1 ) , substitute one ( 1) to x,
calculate, then subtract a value of (-1 )20
from the result
Sum of coefficients= [(2)(1) -1t- (-1)20
=0

IBI
Note: To solve the sum of the coefficients
8
of (x + y -z) substitute one(1) to all the
variables and calculate.
Sum of coefficients = ( 1 + 1 - 1 )8 = 1

lfD

lED

6th term = 66339


128 a11

(x + 4y)t2

~~~

(1)(625 4 )
.
y

= 625 l

r-1

60

IDI
=

10!

= - - = -0.833

)16

= (10-4)!4lx

and

1
-3
(2a

6th term = C

5th term

!4!

= (

.
495 (8)
Coefficient of next term "" - - 4 +1
= 792

( <, )r-1

(4-4)

6th term=

Coefficient of next term =

1 )10
( X2+;:

where: n = 10; r = 5

-49 1

1&1

1h
5 term = n c r-1 (x2)n-r +1

30x + 49x + 20 = 0
Using the quadratic fon'ru.l)a;

10!
97
6
x (x )
(100-3)!3!
4th term= 161,700 x103

{Exponent of y) + 1

6th term= 1oCs (2x2)s Ys

' 7
1
"J'6
2
[ Ax + 6Ax- 2.A = 0 A

= 100 c3 (x)97(x2)3

(Coefficient of PT)(Exponent of x}

61h term= ncr-1 (2x2 )n-r+ 1 (yf 1


where: n ::: 10; r = 6

A:x +(!_A
5 )

4th term

8 = 12

where: n = 100; r = 4

4AC = 0

~(-%)=~
6

41

4th term= ncr-1 (x2tr+ 1(~)'"1

x1x2 = -

5th term = 4 C 4 (2x) 0 (-5y) 4

=4; r = 5

log8 48

= 10log
9to 48
6 = 1.86
10

logs 845 = log1o 845


= 3. 76
log
10 6

IBI

log10 1000

ml

log 2 5 + log 3 5 =

33

= 3.3 log10 1000


= 9.9

~91o ~ + log10 5
log1o 2' log1o 3

= 3.79
---.._....

.JII

58. 1001 Solved Problems in Engineering Mathematics (2"d Edition) by Tiong & Rojas .

IBI

log (1og 5) = log1o(log3 5)


3
4
log1o4

Iogb 1024 =

2
log1o 1024 5
log1ob = 2
log.10 b -_ ~..:.:!_--=.
log1o 1024
.
2.5

!.,

Day 3 --Quadratic Equation, Binomial Theorem, Lo$J!rithm 59

'~

II
log6 + xlog4 = log4 + log(32 + 4")
log6 + log4" = log4 + log(32 +4")
log(6){4"} = log(4)(32+4")

log1o b = 1.204

1&1
,

log 4 ~=log

4 1-log 4 7

(6)( 4"} = ( 4}(32) ~ (4)( 4)

Note: Logarithm of 1 to any base is eq~Jal


to zero.
1
log 4 - = 0 - log 4 7 = - n

IB1

le~g 3 ( x2

8x)

=2

log10 (x -8x) =
2
'log 10 3
log10 (x 2 - 8x) = 2 log 10 3 = log 10 (3) 2
log10 (x 2 - 8x) = log10 9
x 2 - 8x = 9

log.10=0.25
log1o 10 = 0.25
log 10 a
log 10.
Iog1oa = - 10
--= 4
0.25

x2

x=9

Iogb y -Iogb x = 2x

ml

log1o Y- log1o x = 2x
log 10 b log10 b
log10 y -log 10 x

=2x log10b

x31ogx

Take logarithm on both sides:


log4" =log64
xlog4 = log64
log64.
X=--

log4

X=3

ml

=1OOx

Take logarithm on both sides:


log 3109 x = log 1OOx

log2 2 + log2 x = 2
log10 2 + log10 x =
2
log10 2 log10 2

1+ log1o x = 2
log10 2

log1o 'j_ = log1o b2x


X

1!1

Ill!.

= (-'7X)(1)

Equating factors to zero:

=-7x
log

vx

1
=log(x)' =-log(x)
1

31ogx +2 = 0
31ogx = -2

-2

log MN

X=2

(3(1ogx) + 2)(1ogx -1} = 0

log. e-7 = (-7x)log. e

nC

log1o x = 1
log10 2

log10 x = log10 2

3(1ogx)(logx) = lcg1 00 + logx


3(1ogx) 2 -logx- 2 = 0

= xb2x

= log M + log N

logx=3
x = 0.215 (absurd)
logx -1 = 0

(x) -log (y + z) =log(, :z]

logx = 1

log2x -logx = 6.278 -log6


2x 3
log-= 5.49984
X

2
2

(x +1)(x -9) = 0
X= -1

iog2x3 + log6.: logx = 6.278

2x

4" = 64

8x -9 = 0

Iogb y = 2x + Iogb x

log2x 3 +log.= 6.278

log2x 2 = 5.49984

2(-4") = 128
2

ml

(6}(4"} = (4)(~2+4x)

b = 16

El

log12 x = 2
X:::

122

X= 144

=antilog5.4998

x ""158055.6425
X= 397.56

62 100 1 Solved Problems in Engineering Mathematics (2nd Edition) by Tiong & Rojas

Topics

D
D
D
Mon

Tue

I~

Theory

Age Problems
Work Problems
Mixture Problems
Digit Problems
Motion Problems
Coin Problems

Wed

D ~
D D
D D

Problems

Thu

Solutions

Fri

Notes

Sat

Age Problem

1/5 -?This is what the person


One of the most common problems in
Algebra is the age problem. This type of
problems must be solved meticulously by
qiving more emphasis to the tenses (i.e.
past, present or future) of the statement.

Example:
fhe ages of a certain person in the past,
present and future in terms of x are as
lui lows:

II

This is the work


For a complete job,
rc;tte

Work Problem
;11ppuse that a person can do a certain
w< '' k in 5 days. This means that the said
1"'""'1 c.1n finish 1/5 of the work in one
""Y lllu~;. lli~; rah~ IS 1/~i of the work P 'I

d.iy

finished in 1 day

x time,= 1 .

When there is a specific work and specific


time and manpower, the rate of doing the
work may be computed using the number
of man-hours.

64 100 I Solved Problems in Engineering Mathematics (2"d Edition) by Tiong & Rojas
.. ,..,

Example:
If 20 bakers can bake 40 pizzas in 8 hoars,
how many bakers can bake 10 pizzas in 2
hours?
Solution:
Get the rate (in man-hour) of baking a
pizza.
Rate=

(20bakers )(8hours)

Day 4- Age, Work, Mixture, Digit, Motion Problems

Digit Problems
Let h, t and u be the hundreds, tens, and
units digit, respectively. A three-digit
number must be represented in the
following manner.
Number= (h ){100) + (t)(1 0) + (u)

Dime
10 cents

Quarter
25 cents

A two-digit number is represented by;

40pizzas
Number= (t)(1 0) + ( u)

Rate= 4baker- hour


pizza

Half Dollar
50 cerits

Motion Problems
This means that to bake a pizza, you need
either 4 bakers to work in 1 hour or 1
baker to work in 4 hours.
No. of bakers= ( 4

ba~er-hourJ(10 pizzas)
p1zza

In Algebra, the problems pertaining to


motion deals only with a uniform velocity,
i.e., no acceleration or deceleration in the
process. The following is the relationship
between the distance, time and velocity.

2 hours

= 20 bakers

Time= 0

Time= t

Proceed to .the next page for your 4th test.


Detach and use the answer sheet provided
at the 1ast part of this i:)ook. Use pencil
number 2 in shading your answer.

GOOD LUCK I

Mixture Problems
The easiest way to solve a mixture
problem is to draw a rectangle or square
which will illustrate the content of the
mixture as shown in the following
illustration.

D=Vt

'ijtribia:

V=~
t

t=~
v

Consider a 5 cubic meter mixture


containing 65% alcohol and 35% gasoline.
Coin Problems

.65%
Alcohol

!~

The entire
mixture

35%
Gasoline

Problems in Algebra about coins are more


focus on the dollar denomination than local
Philippine currency.
The following are the equivalent value for
each coin.

V= 5m 3
The quantity of alcohol is (0.65)(5) = 3.25
cubic meters while that of gasoline is
(0.35)(5) = 1.75 cubic meters.
Penny
1 cent

Nickel
5 cents

Did you know that... 161h century italian


mathematician and physician Gerolamo
Carda no, was the first to introduce the
concepts of probability and define it as the
number of favorable outcomes divided by
the number of possible outcomes.
Because of this, he is regarded as the
"Father of the Theory of Probability".

~uote:
"Where there is matter, there is geometry."
- Johannes Kepler

Day 4- Age, vvork, Mixture, Digit, Motion Problems 67

uS: A girl is one-third as old as her


,1"
/ brother and 8 years younger than her
sister. The sum of their ages is 38 years.
How old is the girl?

"ISO: Debbie is now twice as old as Jerry.


Four years ago, Debbie was three times
as old as Jerry then. How old is Debbie?
A.

D
D
D D
tvlon
Tue

Theory

,-------

Topics

...-----~

! Age Problems
j Work Problems
1 Mixture Problems
I Digit Problems
j Motion ~roblems
Coin Problems

Vl/ed

IQJ
Problems.

Notes

B.
C.
D.

4
5

c. 6
D.

lSi: ME Board April1998


A pump can pump out water from a tank
in 11 hours. Another pump can pump out
water from the same tank in 20 hours.
How long wiil it take both pumps to pump
out the water in the tank?

A.
B.
C.

5
7
8

A.

D.

10

C.

B.
D.

Thu

age now when you were born." If the


father is now 38 years old, how old was
his son 2 years ago?

Fri

Sat

A.
B.
C.

19

D.

21

Mary is 24 years old. Mary is twice as old


as Ann was when Mary was as old as
Ann is now. How old is Ann now?
A.
B.
C.
D.

16
18
12
15

.14i: EE Board April1997


The sum of Kim's and Kevin's ages is 18.
In 3 years, Kim will be twice as old as
Kevin. What are their ages now?

A
B.
C.
D.

4,14
5, 13
7, 11
6, 12

14-(:"GE Board February 1994


"Robert is j 5 years older than his brother
Stan. However "y" years ago, Robert was
twice as old as Stan. If Stan is now "b"
years old and b>y, find the value of (by).

A.
B.

C.
A.
B.
C.

15
16

D.

18

17

.144:

JJ is three times as old as Jan-Jan.


Three years ago, JJ was four times as old
as Jan-Jan. The sum of their ages is
A.
B.
C.
D.

20
24
28
36

D.

"A. 400-mm 0

hours. With all the three pipes open, how


long will it take to fill the tank?

B.
C.
D.

hours
hours
hours
hours

A tank is filled with an intake pipe in


2 hours and emptied by an outlet pipe in 4
hours. If both pipes are opened, how long
will it take to fill the empty tank?
A.

3 hours

B.
C.

4 hours
5 hours

D.

6 hours
~":,.,.-4:"" .

Js.:i: A tank can be filled in 9 hours by


one pipe, 12 hours by a second pipe and
can be drained when full by _aJ!l_jrd pipe in
15 hours. How long will it take to fill an
empty tank with all pipes in operation?

B.

A.

B.

(:)

C.

()

2.00
2.50
2.25
2.75

,63:

s49i

pipe can fill the tank alone in

5 hours and another 600-mm 0 pipe can


fill the tank alone in 4 hours. A drain pipe
300-mm 0 can empty the tank in 20

17
16
15
14

At present, the sum of the parents'


ages is twice the sum of the children's
ages. Five years ago, the sum of the
parents' ages was 4 times the sum of the
children's ages. Fifteen years hence, the
sum of the parents' ages will be equal to
the sum of the children's ages. How many
children are there?

hours
hours
1/2 hours
1/2 hours

,,~

15
17

1.!18i""'Six years ago, Nilda was five times


as old as Riza. In five years, Nilda will be
three times as old as Riza. What is the
present age of Riza?

7
6
7
6

J.Si: CE Board November 1993

A.

.w:'ECE Board April1995


ECE Board April1999

14
16"
18
24

, ...,
J46: Paula is now 18 years old and his
/colleague Monica is 14 years old. How
many years ago was Paula twice as old
as Monica?

14'7: A father tells his son, "I was your

D D
D D

Solutions

A.
B.

7 hours and 12 minutes


7 hours and 32 minutes
7 hours and 42 minutes

68 100 LSolved Problems in Engineering Mathematics (2"d Edition) by Tiong & Rojas
7 hours and 50 minutes

cr
,iSS: ME Board April :1995
If A can do the work in "x" days and Bin
"y" days, how long will they finish the job
working together?
x+y
xy

A.

~y

B.

2
__!'!_

C.

I'

FY

.s6; ECE Board November :1995


Pedro can paint a fence 50% faster than
Juan and 20% faster than Pilar, and
together they can paint a given fence in 4
hours. How long will it take Pedro to paint
the same fence if he had to work alone?

A.
B.
C.

"while Stewart can paint the same house


in 16 hours. They work together for 4
hours. After 4 hours, Stewart left and
Glenn finished the job alone. How many
more days did it take Glenn to finish the
job?

A.

2.75
2.50
2.25
3.00

B.
C.
D.

hours
hours
hours
hours

CE Board November :1993


It takes Butch twice as long as it takes
Dar'l to do a certain piece of work.
Working together they can do tile work in
6 days. How long would it take Dan to do
it alone?

.J:S81

.~
;

A.
B.
C.
D.

D.

8, 13

A.
B.
C.
D.

10, 15
6, 11
7, 12

12
13

'I'

,Ja,

B.
C.
D.

9 days

A.
B.
C.
D.

9 hours
18 hours
12 hours
14 hours

.:u;i; ECE Board April :1.999


Mike, Loui and Joy can mow the lawn in
4, 6 and 7 hours respectively. What
fraction of the yard can they mow in 1
hour if they work together?
A.
B.
C.
D.

47/84
45/84
84/47
39/60

~6:1:

A farmer cari plow the field in 8


days. After working for 3 days, his son
joins him and together they plow the field
in 3 more days. How many days will it
require for the son to plow the field alone?

10 days

11 days
12 days

A.
B

10
11

C.
D.

(.'''

~~ A goldsmith has two alloys of gold,

c:
D.

20 days. In how many days can all of


them do the work together?
19
17
21
15

20 m of solution with 35% alcohol,


3
40 m of solution with 50% alcohol
3
50 m of solution with 35% alcohol,
3
20 m of solution with 50% alcohol
3
20 m of solution with 35% alcohol,
3
50 m of solution with 50% alcohol
3
40 m of solution with 35% alcohol,
3
20 m of solution with 50% alcohol

B.

:1.64: ECE Board November :1.99:1.


Crew No. 1 can finish installation of an
antenna tower in 200 man~hour while
Crew No. 2 can finish the same job in 300
man-hour. How long will it take both
crews to finish the same job, working
together?
A.
B.

A:6o: EE Board April :1.99&


'A and B can do' a piece of work in 42
days, Band C in 31 days and C and A in

A.

A.

"""'"'#.[

100 man-hour
120 man-hour
140 man-hour
160 man-hour

the first being 70% pure and the second


being 60% pure. How many ounces of the
60% pure gold must be used to make 100
ounces of an alloy which will be 66%
gold?

..,_.<"'""

. u)s: ME Board October :1.994

,'jl;h,
'i 'f~

w'

It takes Myline twice as long as


Jean a to do a certain piece of work.
Working together, they can finish the work
in 6 hours. How long would it take Jeana
to do it alone?

,_s{:"'Glenn can paint a house in 9 hours

C.

J.&"i;

6
8
10
12

D.

Day 4- Age, Work, Mixture, Digit, Motion Problems 69

.<1.59: ME Board April :1.995


A and B working together can finish
painting a house in 6 days. A working
alone can finish it in 5 days less than B.
How long will it take each of them to finish
the work alone?

,.,..,,<"~

X+y

D.

,_

On one job, two power shovels excavate


20,000 cubic meters of earth, the larger
shovel working 40 hours and the smaller
for 35 hours. On another job, they
removed 40,000 cubic meters with the
larger shovel working 70 hours and the
smaller working 90 hours, How much
earth can each remove in 1 hour working
alone?

A.

il

B.
C.

D:

169.2,
178.3,
173.9,
2P0.1,

287.3
294.1
347.8
312.4

:lflft: EE Board October :1.997


Ten liters of 25 % salt solution and 15
liters of 35 % salt solution are poured into
a drum originally containing 30 liters of
10% salt solution. What is the per cent
concentration of salt in the mixture?

A.
B.
C.
D.

19.55%
22.15%
27.05%
25.72%

:1.67: ME Board October :1.99:Z


A Chemist of a distillery experimented on
two alcohol solutions of different strength,
35% alcohol and 50% alcohol,
respectively. How many cubic meters of
each strength must he use in order to
produce a mixture of 60 cubic meters that
contain 40'Vo alcohol?

A.
B.
C.

40
35
45
38

D.
/

,d9: ME Board October :1.994


Two thousand (2000) kg of steel
containing 8% nickel is to be made by
mixing a steel containing 14% nickel with
another containing 6% nickel. How much
of each is needed?
A.

B.
C.
D.

1500 kg of steel with 14% nickel, 500


kg of steel with 6% nickel
750 kg of steel with 14% nickel, 1250
k g of steel with 6% nickel
500 kg of steel with 14% nickel, 1500
k g of steel with 6% nickel
1250 kg of steel with 14% nickel, 750
k g of steel with 6% nickel

,t!fo: How much water must be


/evaporated from 10 kg solution which has
4% salt to make a solution of 10% salt?
A.
B.
C.
D.

4kg
5 kg
6kg
7 kg

. :1.'f1: EE Bo!li'd October :1.994


If a two digit number has x for its unit's
digit and y for its ten's digit, represent the
number.
A.
B

1ox+y
10y +X

Day 4- Age_,_Wor}S_Mixture, Digit, Motion Problems 71

7 0 100 1 Solved Problems in EnQ'ineering Mathematics (2nd Edition) by Tiong & Rojas

c.
D.

yx
xy

1?.1-:z: EE Board October 1994

.,;771 GE Board February 199Z


The product of

and

iof

a number is

500. What is the number?

P:.Une number is 5 less than the other. If


their sum is 135, what are the numbers?

A.
A.
B.
C.
D.

85, 50
80, 55
70, 65
75,60

B.

c.
D.

50
75
100
125

.J78: If 3 is subtracted from the


,1?3: ECE Board March 1996
Ten less than four times a certain number
is 14. Determine the number.
A.

B.
C.
D.

7
8
9

ECE Board March 1996


the sum of two numbers is 21 and one
number is twice the other. Find the
numbers.

B.
C.
D.

6, 15
7,1'4
8,13
9,12

J'15: EE Board April1993


lf eight is added to the product of nine
and the numerical number, the sum is
seventy-one. Find the unknown number.

A.
B.
C.
D.

5
6
7
8

1'76: Find the fraction such that if 2 is


(ubtracted from its terms its becomes 1/4,
but if 4 is added to its terms it becomes
1/2.

A.

3/5

B.

5/12
5/14
6/13

c.
D.

A.

35/55

B.

36/55

c.

~4:

A.

numerator of a certain fraction, the value


of the fraction becomes 3/5. If 1 is
subtracted from the denominator of the
same fraction, it becomes 2/3. Find the
original fraction.

D.

3/7
32/41

,,f:t79: ECE Board November 1997


' The denominator of a certain fraction is
three more than twice the numerator. If 7
is added to both terms of the fraction, the
resulting fraction is 3/5. Find the original .
fraction.

A.

8/5

B.

13/5

c. 5/13
D.

3/5

180: Find the product of two numbers


' such that twice the first added to the
second equals 19 and three times the first
is 21 rnore than the second.

A.
B.

c.

24
32

18

D. 20
j
'1:81: The tens' digit of a number is 3 less
than the units' digit. If the number is
divided by the sum of the digits, the
quotient is 4 and the remainder is 3. What
is the original number?

A.
B.

36
47

C.

58

/1,S&: CE Board November 1.994

D.

69

An airplane flying with the wind, took 2


hours to travel 1000 km and 2.5 hours in
flying back. What was the wind velocity in
kph?

l8%: The second of the four numbers is


three less than the first. the third is four
more than the first and the fourth is two
more than the third. Find the fourth
number if their sum is 35.
A.
B.
C.
D.

10
11
12

..,..,.,,.,z,

l83: EE Board April1997


A jogger starts a course at a steady rate
of 8 kph. Five minutes later, a second
jogger starts the same course at 10 kph.
How long will it take the second jogger to
catch the first?
20 min
21 min
22 min
18 min
,c''

.1114: EE Board April1997


A boat man rows to a place 4.8 miles with
the stream and back in 14 hours, but finds
that he can row 14 miles with the stream
in the same time as 3 miles against the
stream. Find the rate of the stream.
A.
B.
C.
D.

1.5 miles per hour


1 mile per hour
0.8 mile per hour
0.6 mile per hour
/"

_J.SS: ECE Board November 1998


A man rows downstream at the rate of 5
mph and upstream at the rate of 2 mph.
How far downstream should he go if he is
to return in 7/4 hours after leaving?
A.
B.
C.
D.

2.5
3.3
3.1
2.7

50
60
70
40

,;::187: CE Board May 1.998

13

A.
B.
C.
D.

A.
B.
C.
D.

miles
miles
miles
miles

A boat travels downstream in 2/3 of the


time as it goes going upstream. If the
velocity of the river's current is 8 kph,
determine the velocity of the boat in still
water.

A.
B.

c.
D.

40 kph
50 kph
30 kph
60 kph

/188: Two planes leave Manila for a


southern city, a distance of 900 km. Plane
A travels at a ground speed of 90 kph
faster than the plane B. Plane A arrives in
their destination 2 hours and 15 minutes
ahead of Plane B. What is the ground
speed of plane A?

A.
B.
C.
D.

205 kph
315 kph
240 kph
287 kph

1.89: EE Board April 1.997

#'

A train, an hour after starting, meets with


an accident which detains it an hour, after
which it proceeds at 3/5 ot its former rate
and arrives three hbur after time; but had
the accident happened 50 miles farther
on the line, it would have arrived one and
one-half hour sooner. Find the length of
the journey.
A.
B.
C.
D.

910/9
800/9
920/9
850/9

miles
miles
miles
miles

72 l 00 I Solved Problems in Engineering Mathematics (2"d Edition) by Tiong & Rojas


190: On a certain trip, Edgar drive 231
km in exactly the same time as Erwin
drive 308 km. If Erwin's rate exceeded
that of Edgar by 13 kph, determine the
rate of Erwin.
A.

39 kph

B~

44 kph

D.

48 kph
52 kph

c.

')~,

'L
I'!

'\''
:'I:'

,!

Topics

.,,,

-------"-'""'""-"".._..._u,_____,,__,

(
(

Age Problems
Work Problems
Mixture Problems
Digit Problems
Motion Problems
Coin Problems

Mon

,:

1,\\

Tue

0
0

0
0

Theory

1v

nr.
:{f
i~

Problems

Solutions

Notes

Wed

[QJ
Thu

Fri

Sat

ANSWER KEY

ill' I

141. B
142.8
143. A
144.0
145.C
146. D
147. B
148.A
149. c
150. B
I 151. A
152. B
153. B

154. 0
155. c
156.C
157.A
158.A
159. B
160.A
161. B
162.A
163. c
164. B
165.C
166.A

167. 0
168. A
169. c
170. c
171. B
172. c
173. A
174. B
175.C
176.C
177. c
178. B
179. c

180. A
181. B
182. 0
183. A
184. D
185. A
186.A
187.A
188.C
189. B
190. D

RATING

43-5{] Topnotcher

c::J 33-42 Passer


c:J 25-32

1:11

Conditional

0-25 Failed
lfFAILEO, repeat the test.

74

100'1 Solved Problems in Engineering Mathematics (2nd Edition) by Tiong & Rojas

Ill

x-12=3
X=9

Present
24
X

Nilda
Riza

3x = 3(9)

Sum of ages = 9 + 27 =36

X= 18

Ill

Ill
Future
X +3
v+3

,,.
X+ y =18
y=18-x
(y + 3) = 2(x + 3)

~Eq.1
~

Eq. 2

Y= 3x

Eq.1

z.= x + 8
X + y + z = 38

Eq. 2
Eq. 3

X=6

lEI

X=5

x = 1On- 5

Ill
X

Present
38

Past
(b+15-y)=2(b-y)
b + 15- y = 2b- 2y

Father
Son
38- X=

b-y=15

X-

X =19

Past
3x -3
x-3

3x- 3 =4(x- 3)
3x- 3 = 4x -12

Present
3x
X

Two years ago, the son was


(19- 2) = 17 yea'rs old

n=5

1
2

1
4

-+---=5 4 20 X
x = 2.5 hours

---=x = 4 hours

1 1
1 1
-+---=9 12 15 X
x = 7.826 hours
x = 7 hours & 50 minutes

Present
2x
X

Let: x = time needed to complete the work


1
1 1
-+-=20 11 X
x = 7,.096 hours

Let: n = number of days needed to


complete the work
1 1 1
-+-=-

~'

'!:

2b - b- 2y + y = 15

-+ Eq. 1

2x + 30 = x + 15n
X = 15n- 30 -+ Eq. 2

=10

liD

15n-30=10n-5

18-x = 28-2x
X

Equate equations 1 and 2:

(18-x)=2(14-x)
Present
b + 15
b

Future
2x+30
x+15n

Present
2x

2x = 20n-10

y =13

Past
b+ 15- v
b-y

Past
2x-10
x-5n

2x -10 = 4x- 20n

Present
18
14

ml

X =17

2x-10=4(x-5n)

y=18-5

-2x+45=111

Parents
Children

x+3x+(x+8)=38

21-x=2x+6

Let: x = age of the girl


y age of her brother
z = age of her sister

(18- x) + 3 = 2x + 6

Ill

Future
3(x+5)
x+5

3x + 15- 5x + 30 = x + 5- x + 6

Substitute equations 1 & 2 in equation 3:

Substitute y in equation 2:

Present

3(x + 5) -5(x -6) = x +5-(x -6)

2x =36

I
I

Past
5(x-6)
x-6

3x= 27

24- x =x-12

Robert
Stan

Day 4- Age, Work, Mixt~e, Digit, Motion Problems 75

2x- 3(x- 4) = x -(x -4)


2x - 3x + 12 = x - x + 4
-x+12=4
X=8
2x=16
Thus, Debbie is now 16 years old.

y n

_!=x+y
n xy

n=.2L
X+Y

Let: A = number of hours, Pedro can


paint the house
B = number of hours, Juan can paint
the house
C = number of hours, Pilar can paint
the house
1 1 1 1
-+-+-=~Eq.1
A B C 4

_!_
= 1.5(1) _!_ = 0.666(2-) ~ E . 2
B'B
A
q
A

Day 4- Age, Work, Mixture, Digit, Motion Problems 77

76 100 i Solved Problems in Engineering Mathematics (2nd Edition) by Tiong & Rojas

~=2(~)

*=1.2(~} ~=0.833(l) ~ Eq. 3


28-5
8 2 -58
128 - 30 = 8 2 - 58

=e-

Substitute equations 2 & 3 in equation 1:

A.

82

1
(1) +0.833 (1)
A+0.666
A = 1

Note: (rate)(time) = 1(complete job)


1 1l
1
=1
( -+-14+-(x)
9 16;
9
0.6944 + 0.111x = 1
x = 2. 75 hours

ml

x = time for l!utch to finish a certain


job working alone
y = time for Dan to finish a certain
job working alone
1 1 1
-+-=-

y 6

X= 2y

1 1
1
-+-=A B 42
1 1
1
-+-=8
31
1 1
1
-+-=A C 20

Add the three equations:

1+ 2

+
[ _!_
A B

y = 9 days

-+-=A B 6
A=B-5

Eq. 1

Eq.2

Substitute equation 2 in equation 1:

1
1 1
-+-=B-5 B 6
B+(B-5} 1
B{B-5) =5

,.
'

-+-+- = 0.106
A B C
1
1 1 1
-+-+- =0.053 =A B C
x
x = 18.87days

Let: x = time for Myline to finish the jo


y = time for Jeana to finish the job

1
-+-=X y 6

~Eq.1

(1)

1 [1 1]

-(3)+ -+- (3)=1


X
X
y

.~..

x~19days

.!)

4 6 7
42+28+24
X=_:_-~-=-..
168
94
X=168
47
X=84

'

Substitute x

--+ Eq. 2

70( 500- 0.875y) + 90y =40000


35000-61.25y+90y = 40000
y = 173.9 m3 /hr
X= 500-0.875{173.9)

c:]+I
10

x = 347.8 m3 /hr

35%

I+ c:J

15

c:J

30

55

Eq.1

=8 in equation 1:

r~(3)+[i+~ J(3)= 1H

70x + 90y ,;, 40000

Substitute equation 1 in equation 2:

Let: x = time for the farmer to ~_!}he


field
y = time for the son to flow the field,

Eq. 3

=0.106

..!] +[..!B + _!_]


+[_!_ + ..!] = ~ + __!_ + ~
C
A C
42 31 20

2Y=6
1

Eq.t

~~q.

Eq.2

Substitute equation 2 in equation 1:

=(..! + _! +

40x + 35y = 20000


X= 500- 0.875y --+ Eq. 1 .

Let: x = fraction of the lawn that can


mowed after one hour
X

A B

y =18 hours

=15 - 5 =10 days

Let: x = number of days needed by A, B


and C to finished the work
working together.
1 1 1 1
-+-+-=-

Eq. 1

-+-=2y y 6

-+-=200 300 X
x = 120 man-hours

Let: x = capacity of the larger shovel in


m3/hr
y = capacity of the smaller shovel in
m3/hr

Substitute B = 15 in equation 2

-=-

B = 15 days
B = 2 (absurd)

-+-=y y 6

{B-15}(8-2)=0

El

Let:

Eq.2

Substitute equation 2 in equation 1:

178 + 30 = 0

A= 10 hours

1 1 1 1
-+-+-=8 8 y 3
y = 12 days

Let: x =number of man-hours needed by

crew number 1 and number 2


to finish the job.

0.25{10}+0.35{15) + 0.10(30) = x(55)


X=19.55%

c=]+lso%1=14o%J
x

60-x

60

For the 35% solution:


0.35(x) + 0.50(60- x) =0.40{60)
0.35x + 30- 0.5x = 24
X=40 m3

18 1001 Solved Problems in Engineering Mathematics (2"d Edition) by Tiong & Rojas
For the 50% solution:
.Let:

60-x=20~ m 3

c=:J c=J c::J


+

70- 0.7x + 0.6x = 66


x =40 ounces

X=5

2x = 140

I~i1i

c:J= c:J
2000-x

2000

y = 14

x-5 =65

Ill

L:et:

1111

x = the number

X=6

DD

5x + 35

Thus, the numbers are 7 and 14.

2000 - X = 1500 kg

c::J- c:J= 6
X

10-x

0.04{10)- O(x) = 0.10(10- x)


0.4 = 1-0.1x

~(x) J= 500

X=6 kg

Let: y = tens' digit of the number


x = units' digit of the number

The two digit number is represented by:


1Qy +X.

~~~~

Let x = the number

~~l

Let:

=100

~ = the fraction
y

X -3

3y =

5x -15
~Eq.1

3x=y+21
~Eq.2

y~2

Substitute equation 1 in equation 2:

4x-i3 = y -2

,-;,_"

y=4x-6

Eq.1

=2[~x -5 ]-2

10
3x=-x-10-2
3
0.333x = 12

X+4

--=y+4 2
2x + 8 = y + 4

3x

Eq. 2

=36

2x+8=y+4Jar

Eq. 1
Eq. 2

Substitute equation 1 in equation 2:


3x=(19-2x)+21
5x=40
x=8
y=19-2{8)
y=3
:. prqduct of the numbers is 8(3)

(36) 5 -55

13

2x+ y = 19

-=-

~.

x = the first num.ber


y = the second number

Let:

y = 19- 2x

3x=2y-2

x-2

Therefore, the original fraction is

IB!I

--=-

X
y-1

y=2(5)+3

y = 13
Let:

~ = the fraction

X=5

x 2 = 10000

X=7

Ill

Eq. 2

5x + 35 = 6x + 9 + 21

20

5
y=-x-5

9x+8=71

5x + 35 = 3(2x + 3) + 21

~=500

2x =.14

=3y + 21

Substitute equation 1 in equation 2:

.~

0.14x + 120- 0.06x = 160


X= 500 kg

y+ 7

x=7

Eq. 1

Let: x = the number

[ l(x) ][

Let: x =-the first number


2x = the sec;ond number

3
5

X+7

Therefore, the fraction is - .


14

Thus, the numbers are 70 and 65.

0.14(x) + 0.06(2000- x) = 0.08(2000)

10

Y = 2x + 3
<q,

x + 2x = 21

For 6% substance:

x = numerator of the faction


y = denominator of the fraction

y=4(5)-6

For 14% substance:

Let:

Ax-10=14

~+

ml

10 =2x

X= 70

. . 36
Thus, the fractlon 1s -

55

2x +8 = (4x -6)+4

x+(x-5)=135

. 100

0.70(1 00..., x) + 0.60(x) = 0.66(100)

Substitute equation 1 in equation 2:

x = the first number


x - 5 = the second number

100-x

Day 4- Age, Work, Mixture, Digit, Motion Problems 79

= 24:

''II

'li

I
80 IOO"l Solved Problems in Engineering Mathematics (2"d Edition) by Tiong & Rojas

Ill
Let:

w + (w- 3)+(w +4) +(w + 6) = 35

=u-~

W=7

'~~

r.

,''.if

z =13
t+U
10t+u-3

,:\r
.I

Ill

t+U

4
t+U
10t+ u -3 = 4t + 4u
6t-3=3u ~ Eq. 2

Let: V 1 = velocity of the first jogger


V2 = velocity of the second jogger

t1

. Substitute equation 1 in equation 2:

Point where
jogger 2
starts running

14
3
----------

t, =25 min.
'.!'..

Z=7+6

~ Eq.1

at,= 101,-50
V1 +V2

,I

~~
'.':.~

Substitute w = 7 in equation 3:

10t+U= 4 +~

.,

4w=28

t = ten's.digit of the number


u = unit's digit of the number
Number= 10t + u

--------'-----.:D=ay'-4 -Age, Work, Mixture, Digit, Motion Problems ~J.

14V1 -14V2

t2 = 25-5

Let:

'""

V1 -V2

3V1 + 3V2

11V1 = 1'7V2

t 2 = 20 min.

IIEII

'::':.

v, = velocity of boatman
v2 =velocity of stream

V1 = 1.545V2

-~ Eq. 2

Substitute equation 2 in equation 1:


2

49.6(1.545V2 ) = 14(1.545V2 ) -14V/

Total time= 14 hrs

-.( .............................................................

i4.832V2

=19.418V/

v2 "'0.76 mph

Direction of the
stream current
~~~.%,'{W"''V'

f.t----------------1

!IP.Jl!ll
ll&iill

Note: time=

dis tan ce
-v,.ei"OcitY

4.8miles

6(u-3)-3=3u
fhe two figures below that the same time.

6u-18-3=3u
3u=21
u==7

t =4

/ ................................. l.................................. c...

Number= 47

~~,
P

y =w+4

-7 Eq.2

Point where
jogger 2 catches
up jogger 1

z =w+6

V,

->Eq.4

Substitute eq1f3tions 1, 2 and 3 in


equation 4: /

I
1\

5 2 4
0.7S =d.75.

--c_~

S=;2.5 miles

3miles

~~+~=14

Skmx~=_!_km/min
60min

v2 = ~~ km/min

7 Eq. 3
..----------

w+xr35

. v,_ .. v2

hr

z = y + 2 = (w + 4) + 2

.... -

s s 7
-+-=-

~'\WI'

s2

x = second number
y = third number
z =fourth number

-7 Eq. 1

t, + t2 = ttotol

twtttfl!.t!lklfW*iP!X!lf.. ?'MP.tEt.,. !IC,.i!IM.

w = first number

x=w-;3

14miles

--=-- v2

Number= 10{4)+ 7

-v,+V2

t2=t1-5

t=7-3

Let

2__,__ ~----.r

s1

S1 =S 2
= V2t 2

v,t,
8

10

60

60

-t, =-(t,-5)

V1 +V2

60

v,- V2

4 8(V1 +V2 }+4.8(V1 + V2 )

(V1 + V2 }(V1 - V2 )

,1 IIV1 ~: 4.8V2 + 4.8V1 + 4.8V2

V/ . . v,v2 + v,V2 - v/
96V1

=14

=14

14V/-14V/~Eq.1

Let:

V, velocity of airplane
V2 =velocity of wind

11=2
.....................................................................

~
I
r

, !
!

+I

s,=1ooo

1000
v, + V2 =----= soo

'

V!+V2-+

7 Eq. 1

...

82 100 I Solved Problems in Engineering Mathematics (2nd Edition) by Tibng & Rojas
Direction of the wind -

I 2= 2.5

L.,

-< ...................................... .

"-

v.-v,
82=1000

v, - v2 = 1ooo =400
2.5

Eq.2

Subtract equation 2 from equation 1:

Let:

v, =ground speed of plane A

V2

Let: t = time needed to travel and reach


destination without any delay
V = velocity of the train

'= ground speed of plane B

~
~-v

PlaneA.................................. . .............._.............

,,
,,'

:w

s = 900

_.:1-~.
.,

']'

(v, + V2 ) - (v, - V2 ) "' 500 -

400

2V2 = 100

v2 =50 kph

Plane B .

t-2.25

..,cz~ -V+90

Day 4- Age, Work, Mixture: Digit, Motion Problems 83

RR

\:

Ill
=

V+8

Direction of
stream current

F- :,==
DireCtion of
-stream current

=0

(V-8)1=(

~V+ 163
;A3=~

I\

8
800
miles
9

Let:
V = rated of Erwin
V - 13 = rate of Edgar

~v

-V13

-,~,.,,

S- 231 km.

b .
Su st1tute t = V :

12

s
-(S- V)=-+1
5

!~ph

11

S-V =t+1

3V

v =2-:r:, )~ph
E:.

I9R

=0

5
250 5 .
v
50+V+-S----\i =S+-

lllii6ill

-V

~(~)=~

(absurd)

V=4

S=308km

t, = h
~

Eq . 1

V=40 kph
< :ondition

3V

s =

.
S-V
1+1+-3- = t+3

(V -240}(V +150) = 0
V

+~(S-50- Vh~+_!] V

S = Vt
+ 202.5 "" 0

V/

5
50+ V
[ V

Divide all by 2.25:

S 1 =S 2

50+V + 1+ S-(50+V) =I+~


v
~v
2

=--S

2.25V2 - 81000 + 202.5V = 0


2.25V2 + 202.5V - 81000 = 0

V 2 +.90V- 36000

"

S =VI

250 -~(.)=s+.!(~)- 100


50 +.+~S4 3
3- 3 4
2 4
3

. (900)
V

~v

Substitute equation 1 in above equation:

Vt = Vt- 2.25V + 90t- 202.5

2.25V- 90

S-V-50

t2

2.25V- 90t + 202.5


900
But t =

S2=S-V-SO

v +50
v

t, =

Point where
the accident
happened

Multiply both sides by V:

V-8-

.l

Condition 1: If the accident happened 1


hour after, substitute values to the

general equation:

S1 =S 2
Vt = (V + 90){1- 2.25)

J/
s, = v +50

Let: V = velocity of the boat in still water


s, = distance traveled upstream
s2 distance traveled downstream

Mili
,....
III:J.

General equation:
Time consumed by the train traveling,
before the accident + Time during
which the train was detained + Time
needed to continue the course and
Time needed
reach the destination
to travel and reach the destination
without any delay + Time of delay

\1

Point where
the accident
happened

2: If the accident happened 50

111lln'~ fnrthcr. suhstitutP. ve~luP.s to tiH'


'i<'IH!IOIIIqliOIIIOil

308

231
V-13

231 v

308 v -4004
=52 kph

:I

86 1001 S~lved Problems in Engineering Mathematics (2nd Edition) by Tiong & Rojas
~

----

-Topics

D
D
D
D 0
D ~
D D
tv1on

Tue

Theory

Wed

Problems

Thu

splutions

Fri

Notes

Sat

~I

What is a clock problem?


A clock problem is a mathematical
problem which focus on the relationship of
tile movements of the hands (hour hand,
minute hand, second hand) of the clock.
rhis type of problem is for mechanical
docks only and never for a digital clocks.

Clock Problems
Variation Problems
Diophantine Equation
Sequence
Series
Arithmetic Progression
Geometric Progression
Infinite Geometric Progression
Harmonic Progression
Figurate Numbers

Most of the problems in clock problem


involve only the minute hand and the hour
hand. The diagram shows the relation
between movement of the minute and hour
hands.

rhe longest hand is the second hand while


llle shortest hand is the hour hand. By
principle, the second hand ($H) always
rnoves faster than the minute hand (MH)
;md the minute hand always moves faster
lllan the (HH). The relation between the
thn~e hands of the clock <:~re as follows:
HH"'MH

12

HH=~
720

MH..,.!t!
12

where : SH is in number of seconds


MH is in numb('r of minutes

~
12

The earliest known clock problem was


posed in 1694 by Jacques Ozanam in his

~~ 1. Solved P:ro~~ns in Engineering Mathematics {?nd Eqition) by Tiong & Rojas

"Recreations Mathematiques
Physiques".

et

What is Variation Problem?


Variation problems are problems in
Algebra which show the relationship
between the variables in terms of
expressions such as "directly proportional
or inversely proportion or simply
proportional".
The expression x varies directly as y is
expressed as follows:
xocy
The symbol varies ( oc ) is replaced by an
equality symbol and a constant of
proportionality, k, hence:
X

=ky

The expression x varies inversely as the


square of y is expressed as follows:

Day 5- Cl_ock, Variation, Miscellaneous Problems & Progrlssion 89

Diophantine equation is named after a


Greek mathematician, Diophantus of
Alexandria (AD c200- c284) who

What is a progression?
A progression is simply another term for a
sequence.

" developed his own algebraic notation and


is sometimes called the "Father of
Algebra".

What is are the types of progression?

k_!_
y2

A Diophantine equation is an equation


that has integer coefficients and for which
integer solutions are required.

1" Arithmetic Progression (AP)


2. Geometric Progression (GP)
3, Harmonic Progression (HP)

A sequence is a set or collection of


numbers arran9ed in an orderly manner
s"uch that the preceding and the following
numbers are completely specified.

A sequence is said to be in arithmetic


progression if its succeeding terms have
a common difference"

An infinite sequence is a function whose


domain is the set of positive integer. If the
domain of the function consists of the first
n positive integers only, then it is said to
be a finite sequence.

The corresponding sum of all the terms in


arithmetic progression is called as
arithmetic series.

1+3+5+7+9 -

There are only two formulas (i.e. last term


and sum) to remember and used in solving
a problem in arithmetic sequence.

finite sequence

1,1

a0

Elements are the term used to describe


the numbers in a given sequence" An
element issometimes called a term.

S=%(a 1 +a.) or
where:

What is an alternating series?

What is the difference between a


converging series and a diver..aent
series?

=a1 +(n-1)d

,I

If an infinite series has a finite sum, it is


referred to as convergent sorlos and
divergent series if it has no sum nl all

S=%[2a 1 +(n-1)d]

a 1 = first term
an= last term (n1h term)
n = number of terms
d = common difference
d a2- a, a3- a2 = ...

What is a ge.ometric progression?


!\ sequence is said to be a geometric

progression if its succeeding terms have


common ratio.

.1

IIH~ corresponding sum of all the terms in


.,, onwtric progression is called as
quometric series"

or

= a1(1-rn)
1-r

r-1

where: a, = first term


an = last term (d" term)
n = number of terms
r= common
ratio=

a2
a1

a 3. = ...
a2

What is an Infinite Geometric


Progression?
This type of progression is a geometric
progression only that the number of terms
(n) is extremely large or infinity.
If r > 1, sum of all terms is infinite
If r < 1, the sum of all terms is

Sum of all terms:

What .is a series?

An alternating series has positive and


negative terms arranged alternately.

it,

Last term (n1h term):

What is an element?

Despite its simple appearance,


Dir;>phantine equations can be fantastically
difficult to solve. A notorious example
comes from Fermat's Last T~which
is a" = b" + c" , where

Sum of all terms:

S = a.1(rn .:_1)
What is an arithmetic progression?

What is the difference between an "


infinite and a finite sequence?

Series is the sum of the terms in a


sequence.

o)fe

" an == a1rn-1

What i~ a sequence?

The best examples of Diophantine


equations are those from Pythagorean
Theorem, a 2 + b 2 = c 2 , where a, band c
all required to be whole numbers.

Diophantine eqw;rtions may refer to a


system of equations where the number of
equations is
less ihan the number of
unknowns. Ttjese equations yield whole
a11swers"
number for

Last term (n 11' term):

The most common types of progression


are:

1 + 3 + 5 + 7 + 9 + .-,..infinite sequence
X=

Also, there are only two formulas (i.e. last


term and sum) to remember and used in
solving a problem in geometric sequence.

S=-3_
1-r
where: a, = first term
r = common ratio
What is Harmonic Progression?
A sequence of numbers whose reciprocals
form an arithmetic progression is known as
harmonic progression. In solving a
problem, it would be wise to convert all
given terms into arithmetic sequence by
getting its reciprocals. Use the formulas in
arithmetic sequence and take the
reciprocal of resulting value to obtain the
equivalent harmonic term for an answer.

Dav 5- Cloc:k, Variation, Miscellaneous Problems & Progression 91

90 1001 Solved Problems in Engineering Mathematics (2"ct Edition) by Tiong & Rojas
D.

What are the Fibonacci and Related


Sequences?
The following are the some of the famous
and related sequences:

1.

Fibonacci Numbers - Named after


the Italian merchant and
mathematician, Leonardo di Pisa or
Fibonacci (Figlio dei Bonacci, "Son of
the Bonnaccis").

1, 1, 2, 3, 5, 8, 13, 21, 34, 55, 89,


144 ...
Each number is equal to the sum of
the two preceding numbers.

ObloJOtg numbers: Numbers


which can be drawn as dots and
arranged in a rectangle shape.



2
6
12
20
30

......

E.

. (.:
5

Lucas Sequence - Named after


Edouard Lucas (1841- 1891). Like
the Fibonacci numbers, every term of
the Lucas. sequence is the sum of the
two preceding numbers.

' fit {jjJ


ril .

\~l -:-:11
12

Cubic numbers:

riD

35

22

.- If

....,__. -
~re

t=-~
~:-

....

....

27

64

1' 3, 4, 7, 11' 18, 29, 47, 76, 123 ...


Tetrahedral numbers:

Figurate Numbers:
Triangular numbers: Numbers
which can be drawn as dots and
arranged in triangular shape .

A.

.
..'







1
3
6
10
15
21
B.


1 4
16
9
C.

25

36

Gnomons:y.fmbers which can


be drawr;Vas dots on equally long
leg7:sa right angle.

'

11

4;.
4

H.

Square numbers: Numbers


which can be drawn as dots and
arranged in square shape.

Did you know that... the eminent German


mathematician, Carl Freidrich Gauss'
father was an accountant and young
Gauss corrected his father's spreadsheet
at the age of 3 !

@uote:
"Mathematics is the queen of sciences and
arithmetic is the queen of mathematics.
She often condescends to render service
to astronomy and other natural sciences,
but under all circumstances the first place
is her due."
- Carl Freidrich Gauss

G.

3.

GOOD LUCK I

\!l:ribia:

Pentagonal numbers:

F.
2.

Proceed to the. next page for your 5th test.


Detach and use the answer sheet provided
at the last part of this book. Use pencil
number 2 in shading your answer.

1
I.

4. ~
. 10

20

Square pyramidal numbers:

14

30

Supertetrahedral numbers:

By piling up tetrahedral numbers,


1, 4, 10, 20, 35, 56, etc, we make
four-dimensional numbers:
1, 5, 15, 35, 70, ...

1
1+4
1+4+10
1 + 4 + 10 + 20
1 + 4 + 10 + 20 + 3!1

=1
=5
=15
:15
{()

Day 5 -Clock, Variation, Miscellaneous Problem!J & Progression 93


/

.1.11&: EE Board October 1990

T~pics

Dr
Mon

j
j

Tue

LJ D
D
lJ ~
D D

Problems

Solutions

Thu

Fri

"'

1gt; CE Board May 1995

.fu how many minutes after 2 o'clock will


the hands of the clock extend in opposite
directions for the first time?

C.
D.

A. . 3:02.30
B. 3:17.37
C. 3:14.32
D
3:16.36

.:144: GE Board February 1997

42.8 minutes
43.2 minutes
43.6 minutes

"'At what time after 12:00 noon will the


hour hand and minute hand of the clock
first form an angle of 120?

19ii CE Board November 1995

A.
B.

the hands l;le directly opposite each other


for the first time?

D.

minutes
minutes
minutes
minut

c.

12:18.818
12:21.818
12:22.818
12:24.818

./1.95: At what time between 8 and 9


o'clock will the minute hand coincide with
the hour hand?
~

193:,/'~
CE oard.
May 1997
What ti e after 3 o'clock will the hands of
the clo k be together for the first time?

C.

D.
/

197: GE Board February 1994


From the time 6:15PM to the time 7:45
PM of the same day, the minute hand of a
standard clock describe an arc of
A.

60

B.

go

D.

540

c. fso

A.

B.

3
4

D.

A.
R

c.
D.

8:42.5
8:43.2
8:43.6
843.9

~oo: CE Board May 1993


Given that "w" varies directly as the
product of "x" and "y" and inversely as the
square of "z" and that w = 4 when x = 2, y
= 6 and z = 3. Find the value of "w" when
x = 1, y = 4 and z = 2.

A.
B.

199: ECE Board April :1990


lhe resistance of a wire varies directly
with its length and inversely with its area.
If a certain piece of wire 10 m long and
C.10 em in diameter has a resistance of
tOO ohms, what will its resistance be if it
1s uniformly stretched so that its length
becomes 12 m?
/\.
ll.

80
90

144
120

I)

3
4

C.

D.

""

~1: ECE Board November 1993


If x varies directly as y and inversely as z,
and x = 14 when y = 7 and z = 2, find the
value of x when y 16 and z 4.

A.
B.
C.
D.

14
4
16
8

~: EE Board Marc:h 1998


The electric power which a transmission
line can transmit is proportional to the
product of its design voltage and current
capacity, and inversely to the
transmission distance. A 115-kilovolt line
rated at 100 amperes can transmit 150
megawatts over 150 km. How much
power, in megawatts can a 230 kilovolt
line rated at 150 amperes transmit over
100 km?

A.

B.

42A minutes

5.22
5.33
5.46
5.54

33147
321.45
346.10
3 36.50

which is proportional to the charge. If the


charge is reduced by 50% of its original
value at the end of 2 days, how long will it
take to reduce the charge to 25% of its
original charge?

'1n how many minutes after 7 o'clock will


A.
B.
G.
D.

A.
B.

198; EE Board Aprill990


A storage battery discharges at a rate

Sat

Notes

A.
B.

VVed

Theory

il

Clock Problems
Variation Problems
Diophantine Fquation
Sequence
Series
Arithmetic Progression
Geometric Progression
Infinite Geometric Progression
Harmonic Progression
Figurate Numbers

A man left his home at past 3:00 o'clock


PM as indicated in his wall clock, between
2 to 3 hours after, he returns home and
noticed the hands of the clock
interchanged. At what time did the man
leave his home?

C.
D.

785
485
675

595

~--3~ ME Board October 1992

'ihe time required for an elevator to lift


weight varies directly with the weight and
the distance through which it is to be lifted
and inversely as the power of the motor. If
it takes 30 seconds for a 10 hp motor to
lift 100 lbs through 50 feet, what size of
motor is required to lift 800 lbs in 40
seconds through 40 feet?
A.

B.
C.
D.

42
44
46
48

Day 5 - Clock, Variation,

94 . 1001 Solved Problems in Engineering Mathematics (2"d Edition) by Tiong & Rojas

Miscellane~us

Problems & Progression 95

204: The selling price of a 1V set is


double that of its cost If the 1V set was
sold to a customer at a profit of 25% of
the net cost, how much discount was
given to the customer?

A.

C.
D.

~.7%
~.7%
~.5%

~.j%

205: A group of EE examinees decided


to hire a mathematics tutor from Excel
Review Center and planned to contribute
equal amount for the tutor's fee. If there
were 10 more examinees, each would
have paid P 2 less. However, if there
were 5 less examinees, each would have
paid P 2 more. How many examinees are
there in the group?

D.

14
16
18
20

A.
B.
C.
D.

B.
C.

D.

16
20
18
24

206: EE Board Marc:h J:998

The arithmetic mean of a and b is

A bookstore purchased a best selling


price book at P 200.00 per copy. At what
price should this book be sold so that,
giving a 20% discount, the profit is 30%?

A.
B.

JO
P 500
P 357
P 400

207: ECE Board November 1993


Jojo bought a second hand Betamax VCR
and then sold it to Rudy at a profit of 40%.
Rudy then sold the VCR to Noel at a profit
of 20%. If Noel paid P 2,856 more than it
cost to Jojo, how much did Jojo paid for
the unit?

A.
B.
C.
D.

p 4,000
P 4,100
P 4,200
P 4,300

C.
D.

a+b
2

Jab
ab
2
a-b
2

.zii: The sum of three arithmetic means

B.
C.
D.

17
18
19
20

:t:t~ECE Board November J:998


,l:i
'~

Find the 30th term of the arithmetic


progression 4, 7, 10, ..

A.
B.
C.
D.

75
88
90
91

CE Board May 1994,


CE Board November J:994
How many terms of the progression 3, 5,
7, ... must be taken in order that their
sum will be 2600?

B.
C.
D.

A.
B.
C.
D.

114
124
134
144

48
49
50
51

U.J:1!:E Board Marc:h 1998


:t:;ravity causes a body to fall 16.1 ft in the
first second, 48.3 in the 2"d second, 80.5
in the 3'd second. How far did the body
fall during the 101h second?
248.7 ft

n.

6~

C.

6~
~

. CE Board May 1998


)

~2J:'9:

Determine the sum of the progression if


there are 7 arithmetic mean between 3
and 35.

C.
D.

11
12
13
14

./'

A besiege fortress is held by 5700 men


who have provisions for 66 days. If the
garisson losses 20 men each day, for
how many days can the provision hold
out?

C.
D.

In the recent "Gulf War" in the Middle


East, the allied forces captured 6400 of
Saddam's soldiers and with provisions on
hand it will last for 216 meals while
feeding 3 meals a day. The provision
lasted 9 more days because of daily
deaths. At an average, how many died
per day?

U'7: CE Board May 1995

B.

14; 19 ... up to the 20th term?

c.
D.

A
<;

1030
1035
II 140

II

,:145

II

72
74
76
78

.~z.rf'CE Board May 199J:

\:'Jh:1! 1s

tt"' sum of the progression 4, 9,

171
182
232
216

2ZO: ECE Board April J:995

B.

In a pile of logs, each layer contains one


more log than the layer above and the top
contains just one log. If there are 105 logs
in the pile, how many layers are there?.

(~.

6~

B.

A.

U6~ME Board Apri11995

A
G.

A.

A.
B.

:.t~s: CE Board May 1993

A.

US: EE Board April1997


,.., A stack of bricks has 61 bricks in the
bottom layer, 58 bricks in the second
layer, 55 bricks in the third layer, and so
on until there are 10 bricks in the last
layer. How many bricks are there all
together?

D.

between 34 and 42 is

.{

A.

209: ECE Board Marc:h 1996


A merchant has three items on sale,
namely a radio for P 50, a clock for P 30
and a flashlight for P 1. At the end of the
day, he sold a total of 100 of the three
items and has taken exactly P 1,000 on
the total sales. How many radios did he
sale?

308.1 ft
241.5ft
305.9 ft

2~3r-if the first term of an arithmetic


'progression is 25 and the fourth term is
13, what is the third term?

850
500
550
600

,v:z.t-0:~ ME Board October J:996

B.
C.
D.

C.
D.

In a certair.~ community of 1,200 people,


60% are literate. Of the males, 50% are
literate and of the females 70% are
.literate~ What is the female po'pulation?

A.
A.
B.
C.

B.

208: EE Board Marc:h 1998

15
16
17
18

Day 5 - Clock, Variation, Miscellaneous Prob~J & ProJ.l!ession 92'


96 1001 Solved Problems in Engineering Mathematics (2nd Edition) by T'ong & Rojas

;atZ: GE Board July X99:5


"A Geodetic Engineering student got a
score of 30% on Test 1 of the five number
test in Surveying. On the last number he
got 90% in which a constant difference
more on each number that he had on the
immediately preceding one. What was his
average score in Surveying?

A.
B.

50
55

D.

60
65

c.

/,<'
~~b: ME

Board April 1999


:If the sum is 220 and the first term is 10,
find the common difference if the last term
is 30.

"pf;blem 226:

C.
D.

When all odd numbers from 1 to 101 are


added, the' result is

2~i;' Find the 9 1h term of the harmonic


progressio11 3, 2, 3/2 .....

. A.
B.
C.
D.

2500
2601
2501

2
5

c.

D.

2/3

2:1'.ci;'EE Board April 1997


Once a month, a man puts some money
into the cookie jar. Each month he puts
50 centavos more into the jar than the
month before. After 12 years, he counted
his money, he had P 5,436. How much
money did he put in the jar in the last
month?

A.
B.

c.
D.

P 73.50
P 75.50
p 74.50
P 72.50

~z71'"How many limes will a grandfather's


/clock strikes in one day if it strikes only at
the hours and strike once at 1 o'clock,
twice at 2 o'clock , thrice at 3 o'clock and
so on?
A.

210

B.

24
156
300

c.

/If

A girl on a bicycle coasts downhill


covering 4 feet the first second, 12 feet
the second second, and in general, 8 feet
more each second than the previous
second. If she reaches the bottom at the
end of 14 seconds, how far did she
coasts?
A.

782 feet

B.
C.

780 feet
784 feet
786 feet

4/5
4/9
..... -~

A.
B.
C.
D.

130
140
150
160

233: EE Board October :199:1


The fourth term of a G. P. is 216 and the
61h term is 1944. Find the 81h tem1:

<>"~

"-'. Uth CE Board May :1992


To conserve energy due to the present
energy crisis, the Meralco tried to readjust their charges to electrical energy
users who consume more than 2000 kwhrs. For the first 100 kw-hr, they charged
40 centavos and increasing at a constant
rate more than the preceding one until the
fifth 100 kw-hr, the charge is 76 centavos.
How much is the average charge for the
electrical energy per 100 kw~hr?
A.
B.
C.
D.

3/5
3/8

:a3'i: Find the sum of 4 geometric means


between 160 and 5.

B.

17649
17496
16749

I>

17964

A.

7,
7,
7,
7,

-7/12
-5/6 .
-14/5
- 716

The 3'd term of a harmonic progression is


15 and the 91h term is 6. Find the 11th
teim.

Z}Sr ECE Board April 1999


II one third of the air in a tank is removed
hy each stroke of an air pump, what
''action a I part of the total air is removed in
1; strokes?

A.

(\

B.
C.
D.

5
6
7

II

A..

B.

1/10
1/11

A.

14336

B.

13463
16433
16344

c.
D.

,.A:J's: ECE Board April :1998


The sum of the first 10 terms of a
geometric progression 2, 4, 8, ... is

:t:J9f'if the first term of a G.P. is 9 and


1he common ratio is -213, find the fifth
term.

The numbers 28, x + 2, 112 form a G. P.


What is the 101n term?

he a geometric progression.

('

~: CE Board May 1995

1596

60 centavos
62 centavos
64 centavos

1/2' 0.2, 0.125, ~ ..

P 213.23
p 202.'15
P 302.75
P 156.00

D.

II.

.. ~;;: ECE Board November 1995


Find the .fourth term of the progression

A.
B.
C.
D.

ZJ4: ECE Board April 1999


Determine x so that: x, 2x + 7, 1Ox - 7 will

1\.

A product has a current selling of P


325.00. If its selling price is expected to
decline at the rate of 10% per annum
because of obsolescence, what w\11 be its
selling price four years hence?

2046
225

58 centavos

/"

,:zj"(;~ ME Board October 1996

A
B.
C.

... -2291 CE Board November :199:5

.,&25: EE Board AJiril1997

b.

A.
B.
C.
D.

3500

D.

A.
B.

0.102
0.099

,..
ll

0.7122
0.9122
0.6122
0.8122

1023

A.
B.
C.

8/5
16/9
1517

D.

1314

.,2401 EE Board April 1997


The sev~nth term is 56 and the twelfth
term is --1792 of a geometric progression.
Find the comro~,..., ral,..; i;jnd the first term.
Assume the ratios .are equal.

A.
B.
C.
D.

-2,
-1,
-1,
-2,

5/8
5/8
7/8
7/8

98 .1 00 1. Solved Problems in Engineering Mathematicl:' (2nd Edition) by Tiong & Rojas

,~.u; A person has 2 parents, 4


grandparents, 8 great grandparents and
so on. How many ancestors during the 15
generations preceding his own, assuming
no duplication?

A.
B.
C.

131070
65534
32766
16383

D.

.....

-~Zlf$:

EE Board March J:998


Determine the sum of the infinite series:

s=
A.

20
19
18
21

B.
D.
A'

24'3: CE Board November 1:994


/In a benefit show, a number of wealthy
men agreed that the first one to arrive
would pay 10 centavos to enter and each
later arrive would pay twice as much as
the preceding man. The total amount
collected from all of them wasP
104,857.50 How many wealthy men
paid?

A.

18
19
20
21

B.

c.
D.

c.
D.

z4~: A man mailed 10 chain letters to ten


of his friends with a request to continue
by sending a similar letter tci each of their
ten friends. If this continue for 6 sets of
letters and if all responded, how much will
the Phil. Postal office earn if minimum
postage costs P 4 per letter?

B.

c.
D.

~ + 2~ + ... + ( i

B.

c.

p 6,000,000
P 60,000
p 2,222,220
P 4,444,440

5/2

D.

11/2

A.
B.
C.
D.

i, 2~

....

5/6
2/3
0.84
0.72

Find the ratio of an infinite geometric


progression if the sum is 2 and the first
term is 1/2.

A. 1/3
B. 1/2

C. 3/4

z.47: EE Board October 1:994

D. 1/4

A rubber ball is made to fall from a height


of 50 feet and is observed to rebound 2/3
of the distance it falls. How far will the ball
travel before coming to rest if the ball
continues to fall in this manner?

A.
B.
C.
D.

200
225
250
275

zs:t(EE Board April 1:997


If equal spheres are piled in the form of a
complete pyramid with an equilateral
triangle as base, find the total number of
spheres in the pile if each side of the
base contains 4 spheres.

feet
feet
feet
feet

A.
B.
C.
D.

. -ziis: EE Board April 1:990


What is the fraction in lowest term
equivalent to 0.133133133?

A.
B.
C.

D.

zs3: Find the 6th term of the sequence


55, 40, 28, 19, 13, ...

133
1\.

10

133

IJ.

9
8

()

11

777
133

l:S4: EE Board October 1:997

888

In the series 1, 1, 1/2, 1/6, 1/24, ... ,


determine the 6th term.

133
999

~~ ECE Board April J:998


.i1Find the sum of the infinite geometric
progression 6, -2, 2/3, ...

A.

15
20
18
21

666

9/2

l
'p,_:

1\

It
(:
ll

1/80
1/74
1/100
1/120

Progression 99

~:

211: ECE Board November :1998

1,073,741
1,730,74
1,073,741,823
1,037,417

Misc~U~n~ous Pro~lems &

712

Find the sum of 1. -

Under favorable condition, a single


cell bacteria divided into two about every
20 minutes. If the same rate of division is
maintained for 10 hours, how many
organisms is produced from a single cell?

A.

B.
C.

zsol CE Board May 1998

.:!&:

A.

4/5
3/4
2/3
1/2

B.

D.

C.

,..:viz: In the PBA three-point shootout


- contest, the committee decided to give a
prize in the following manner: A prize of
P1 for the first basket made, P 2 for the
second, P 4 for the third, P8 for the fourth
and so on. If the contestant wants to win
a prize of no less than a million pesos,
what is the minimum number of baskets
to be converted?
A.

Day 5 - Cl<>.c:1c, Y:ariation,

ECE Board Aprill998


Find the 198ih digit in the decimal

. Ient to 1785
. from t h e
eqUJva
- startmg
9999
decimal point.

A.
B.
C.
D.

8
1
7
5

'

' l
Day 5- Clock, Variation, Miscellaneous Problems & Progression 101

ml

ml

--

12 ---

Topics

D
D
D D
D D

Clock Problems
Variation Problems
Diophantine Equation
Sequence
Series
Arithmetic Progression
Geometric Progression
Infinite Geometric Progression
Harmonic Progression
Figurate Numbers

Mon
Tue

Theory

Wed

Problems

Thu

Solutions

. t es
Note: 1200 ( 30 minutes) = 20 mmu
180

x=40+~

12
x = 43.6 minutes

X =~+20
12
x=21.818 min

Ill

Therefore, the time is 12:21.818

~
Fri

D D
Notes

Sat

12
RATING

ANSWER KEY
191. D 208. D
192.C 209. A
193. D 210.A
194. B 211. A
195. c 212. D
196. A 213. A
197. D 214. D
198. A 215. c
199. c 216. D
200.A 217. A
201. c 218. c
202.C 219. A
203. D 220.C
204. c 221. D
205. D 222.C
206.C 223.A
L'07.C 224.A

225. c
226. B
227.C
228.A
229. B
230. B
231. A
232.C
233. B
234. D
235. B
236.A
237.A
238. B
239. B
240. D
241. B

242.A
243.C
244.0
245. D
246.C
247.C
248. D
249.A
250.A
251. c
252. B
253.A
254. D
255.A

c:J
c:J

c:J

12

x=5+~

55-65 Topnotcher

12
x = 5.454 minutes

42-54 Passer
32-42 Conditional

12

mJ

0-31 Failed

x = 43.6 min
Therefore, the time is 8:43.6

If FAILED, repeat the test.


fiWliilk

X=40+~

m&Jiill~1:t!!!JlJiiill!i

:~

X= 30+L
12

y=15+~
12

7 Eq.
~

Eq.2

'
Substitute (2) in (1 ):

15 + -~

xc15t-~-

v~,

12
1G.:lG rnin

llll't"lut<', lilt' lllll<' r; :1 11,

X=

:If>

30 + -- ! 2._
~2

'

102 100i Solved Problems in Engineering Mathematics (2nd Edition) by Tiong & Rojas

Day 5- Clock, Varia!i<m, Miscellaneous Probl~ms & Progression 103

6: 15 = 6(60) + 15 = 375 minutes

Note: When the wire was stretched, the


diameter was changed but the volume
remains constant assuming there was no
losses in the process.

7:45 = 7(60) + 45 =465 minutes

X=465-375

ED

When P = 150, V = 115, I = 100 & d = 150

When R = 100 and L = 10

x = 90 minutes

150 = k (115)(100)
150
k = 1.956

100=k'(10)
180
x=90min ( -)
30 min

k' = 1

When V = 230, I= 150 and d = 100


When L = 12:

X= 540

ml

Time when he left his home

R=1(12)
D=kC

p = 1.956[(230)(150)]
100

R =144 Q

When D

= 2 and C = 0.5 C

ml

y
2 ~ k(0.5C)

D=(

12 - - : :----Time when he returned home

When w = 4, x = 2, y = 6 and z =

15+~] 12

x=30+~
[

12x = 360 + 15 + ~
12
x = 31.4it min

So the time when he left home was


3:31.47.

Let: x = the number of minute difference


between 6:15 and 7:45.

= 0.75 C:

R=k.!:_

Let: V

30 = k[

(10~~50)]

k = 0.06

When x = 1, y = 4 and z = 2:
~

When t = 40, W = 800 and S = 40


40 = (0.06>[

Eq.1

w=3[(1)(4)]

(2)2

W=3

V=AL

ml

~ Eq.2

ml

x=kr

Substitute equation 2 in equation 1:

Wilen x = 14, y = 7 and z = 2

R"k( ~]

ii .

=48 hp

Let:

x = net col.'t
2x = selling price
d =discount
2x (1 -d) =new selling price

2x(1- d)= x + 0.25x


2x - 2xd = 1.25x

Wilen y

(800~( 40)]

New selling price = Net cost + Gain

14=k~

R=k'L2

k'=~
v

ws

When t = 30, W = 100, S =50 and P =10

k=3

= volume of the wire

A=Lv

t=k

(3)2

D= 3 days

ml

4=k(2)(6)

~ )(o.75C)

Multiple both sides by 12:

mJ

z2

When C

P = 675 megawatts

w =k xy

4
k=-

P=k VI
d

k=4

d=0.375

= 16 and z = 4,

d =-S:-7<5%

X
X

(4}(~6)
16

x
y

r;;

Let: x = number of examinees


y = tutor's fee
original fee shared per examinee

If !here were 10 more examines who wm


join,

y =(X+

10{~--2)

m
Let: x = number of men in the popuiC!tion
y = number of female in the
population

10y =2x + 20x


2

y "'0.2x + 2x

X= 1200- y
-- Eq. 1

If there were 5 examinees who will backout,

y=(x-5{~+2)

-~ Eq. 2

I5IIJI

0.2x 2 + 2x = 0.4x

m
Let: x = number of radios sold out
y = number of clocks sold out
z number of flashlight sold out

2x

4x =0.2x 2
x = 20 examinees

m
x = selling price without t!iscount
0.8x = new selling price (with discount}
Profit= Income- Expense!s
0.24x = 0.8x - 200
X= 357.14

It!

B!J

1.2(1.4x) =

x + 2856

n2 + 2n - 2600 = 0

~~~~ a,

-a 1

af = 25;

rl~

a4= 13

(,I

105 = ~(1 + n)

IIi'

ti;'

ljl

210=n+n 2

'I'
Iilli

n2 +n-210=0

i\)l

~: 'i

(n-14}(n+15)=0

d=-4

Equating factors to zero:

a3 =a,+ 2d
a 3 = 17

mJ

a2

a,= 4;

=7;

a3 = 10

By inspection, d = 3
cL 10

'a 1 1

a,0

,I

~)

111

1!:1

a 3 = 25 + 2 ( -4)

1000 = 1000
It checks!

ti~'

105=-(2+n--1)

a 4 =a 1 +Gd

in equation 1:

=~[2a 1 +(n-1)d]

105 =%[2{1)+(n -1){1)]

13=25+3d

50(16) + 30( 4)+ 80 = 1000

x= 16 radios

= 1; d = 1; S = 105
S

a 10 = 305.9

z=80

Therefore,

n=50

d = 32.2

y=4

1 .68x = x + 285
X= 4200

Equating factors to zero:

d=48.3-16.1

fJEI

=20, y =4 and z = 80 in

(n + 51){n- 50)= 0

a,= 16.1; a2 = 48.3; a3= 80.5

49(16) + 29y = 900

Substitute x
equation 2:

(4 + 2n)

n ==-51 (absurd)

16-t-4-t-z = 100

x = price Jojo paid for the VCR


1.4x = price Rudy paid for the VCR
1 .2(14x) =price Noel paid for the VCR

Thus, a2 = 36, a3 = 38 and _a4 == 40


Sum = 36 + 38 + 40
Sur:n = 114

49x + 29y = 900

Lc

. 2600 = 2n ;t n

( 50x + 30y + z)- ( x + y + z) = 1000 -1 00

=4

d=2

a10 =16.1+9(32.2)

Substitute x = 20 and y

2600=-(6+2n-2)
2600=

a 10 = a 1 +9d

0.3(0.8x) = 0.8x -200

=~[ 2(3) + (n -1){2) J

2600

a5 = a1 + 4d

Subtract equation 1 from equation 2:

Assume x = 16:

d = a2
Z

Let

nr

42 =34+4d

7 Eq. 1
7 Eq. 2

= 100
50x + 30y + z = 1000

y+

X+

a3 =7

a2 = 5;

S='2L2a 1 +(n-1)d

34, a2, a3, a4,42

Substitute equation 1 in equation 2:

Equate equations 1 and 2:

~Eq.2

y = 600 females

5y =2x 2 -10x

=3;

By inspection, d = 2

Eq. 1

600- 0.5y + 0.7y = 720

y = 0.4x 2 - 2x

0.5X+0.7y=0.6(1200)

0.5(1200- y)- 0.7y = 720

5y
y = y + 2x - - -1 0

a,

Thus, the arithmetic mean of a and b is,


a+b

X+ y = 1200

~~~~

Note: Arithmetic mean i? the same term


as average.

10y

y "' y - 2x + - - - 20

Day 5- Clocls__'{_aria!i()n, Miscellaneous Problems & Progression lOS

l 04 1Ot) 1 S_olved Problems in Engineering Mathematics (2nd Edition) by Tiong & Rojas

29d

29(3)

Ell

n = -15 (absurd)

i'l

n = 14 layers of logs

1:::

a,= 4; a2 = 9; a3 = 14;_a4
n = 20
By

ir:~spection,

d=5

S=%[2a 1 +(n-1)d]

1;,1

:=

19;

DayS- Clock, Variation, Miscellaneous Problems & Progression 107

106 1001 Solved Problems in Engineering Mathematics (2nd Edition) by Tiong & Rojas
20
[2(4)+(20-1)5]
2
=1030

S=

37620 = 571n- n2
n2 - 571n + 37620 = 0

(n- 76)(n- 495) = 0

S=

n = 495 (absurd)

Ell

By inspection, d = - 3

Ell

an =a1+(n-1)d

Let:

10=61+(n-1)(-3)
10=61-3n+3
n =18

the meal can


last
S = total number of provisions
n = number of days the total
provisions can last
d = number of soldiers died per day

ml

d = 0.50; n = 12(12) =144

144

[2a 1 +(144 -1)(0,5)]

a 1 =2
a144 =a,+ 143d

n =72+9
n =81 days

fm

8 = %[2a 1 +(n-1)d]

Total provision= 5700 (66) = 376,200

S =%[2a1+(n-1)d]

a1

1
8= ;[2(4)+13(8)]

= 30; as = 90

8= 784
as= a1+4d
90 =30+4d

a,=1;an=101;d=2

d =15
376200 = %[ 2(5700) + (n -1)( -20)]
376200 = ~[11400- 20n + 20)

376200 = 571 On -1 On

S=%[2a 1 +(n-1)d]
S=%[2(30)+4(15)]
8=300
300
Average score = - ". 60
5

a1 = 40; as= 76
a5 =a 1+4d

76 =40+4d
d=9
Therefore:
a2 =40+9=49
a3 = 49+9 =58
a4 = 58+9 = 67

a1 = 4; d = 8; n = 14

d= -18

&D

a,44 = 73.50

Therefore, 18 soldiers died everyday

Note: a1 = 5700; d = - 20

Therefore, total = 2(78) = 156 times

a,4 4 =2+143(0.5)

S = %[2a 1+(n -1)d]

ml

8=78

5436 = 144a1+5148

S = 460800 meals

~1 [2(6400)+80d]

12 .
S=-(1+12)

S=%[2a 1 +(n-1)d]

s = 6400(72)

460800 =

Note: One day is equivalent to 24 hours.

216
3
x = 72 days

By inspection: a1 = 3: an = 35;
n='7+2=9

a1 = 1; a2 = 2; a3 = 3; ... a12 = 12
n
8=-(a,+an)

d=2

5436 =

n
S=-(a 1+aJ
2
9
s =-(3+35)
2
s = 171

ml

30=10+10d

X=-

3, a2, a3, a4, as, a6, a7, aa, 35

8=2601

a11 = a1 +10d

x = number of days,

1
S= :[2(6.1)+(18-1)(-3)]

51
8=-(1+101)

(a 1 +a 2 )

n = 11

S = %[2a 1+ (n -1)d]

S=639

220 = ~(10 +30)

n = 76 days

a1 = 61; a2= 58; a3 =55; an= 10

n
8=2(a1 +an)

8 = 220; a1 = 10; an= 30

an= a1 +(n-1)d

40 + 49 + 58 + 67 + 76
A verage = - - - -- - - -

Average = 58 centavos

1
h3 = 15 a3 = '

15

hs = 6; as=

6'1

a 3 =a, +2d

= a 1 + 2d

101=1+(n-1)(2)

15

101=1+2n-2

1
a1=--2d
15

n =51

~Eq.1

a 9 =a 1 + 8d
1
6

-- =a, + 8d -> Eq.2

Day 6- Clock, Variation, Miscellaneous Problems& Progression 109


108 100 l Solved Problems in Engineering Mathematics (2nd Edition) by Tiong & Rojas
Substitute equation 1 in equation 2:

6=15-2d+8d

h 1 = 3; a, =

=~x-i(~x )=~x

r2 = 9

Volume left after 3'd stroke

=1~-2(6~)

216=a,(3)

Note: By inspection, the volume left after


each stroke forms a GP whose common
ratio is:

as= al7

ag=i+8(i)

a8 =8(3)

5
ag =3

a8

m::l

a11 =a, +10d

=~+10(~)
30
60

a,= 8

a9 =a, +8d

30

=17496

r=Jl_2

2--

a, = x; a2

=2x + 7; a3 =10x- 7

hg = __!_ = ~
a9 5

r = az = a3

a1

1
a,,=5

hg

=~x-i(~x)= : 7 x

Substitute r in equation 1:

d=2

1
a,.,.,-

=s3

a2

2x + 7

1Qx -7

2x+7

Solving for the volume left after the 61h


stroke:
as= als

BD

1
h,,=-a
11

h,,=T

5 = 160r

r=0.5

h,, =5
h, =

X=

Thus,

By inspection, d

=3

fJ!I

a 4 =2+3(3)

a 4 = 11
1

'

1994 =

al

a6 =a,r

~ Eq. -2

Thus h4 = - = ,

a4

11

mt

=7
12
35-49
7
x2 =-1-2- =-6

Divide equation 2 by equation 1:

=0.9122x
r =0.9
a, = 325(0.9) = 292.5

a4 = a,r3
a4 =292.5(0.9t
a 4 = 213.33

mt
~ Eq.1

216=al

y = x- 0.08779x

x, = ~~-+49

a4 = 216; aa = 1994

a 4 = a 1r3

a 4 =a, +3d

~ 4(6)( -49)

3549
12

Sum = 80 + 40 + 20 + 10
Sum= 150

1
h3 = 0.125 =-; a3 = 8

X=---

1
h2 = 0.2 =- ; az = 5

35 ~5)

2{6)

a2 = 160(0.5) = 80
a3 = 80(0.5) = 40
a4 40(0.5) = 20
as= 20(0.5) = 10

; a1 = 2

Thus, the total volume removed after the


61h stroke:

Using the quadratic formula:

(~x )(~J

a6 = 0.08779x

6x 2 - 35x- 49 = 0
aa = als

as=

4x 2 +28x+49 = 10x2 -7x

a, = 160; as = 5

(2x + 7) = x(10x -7)

Therefore:

ml

Volume left after 2"d stroke

r=3

2 3

d in equation 1:

a,,

al 1994
al = 216

21

1 1
d=---

60

a,

h2 = 2; a2 =

d = a 2 -a 1

d=~
Substitute

Let: x = total volume of air in thetank


y = total volume removed from the
tank after the 61h stroke

Bil

a,= 28; a2 = x + 2; a3

r = a2

a1
Volur. ;e left after 1"1 stroke = x

_13 x = -~3 x

X+2

112

28

X+2

a3
a2

= 112

Day 5- Clock, Variation, Miscellaneous Problems & Progression 111

110 100 l Solved Problems in Engineeruig Mathematics (2nd Edition) by Tiong & Rojas
2
(x+2) =112{28)

0.1(2"-1)
104857.5 = _ _,__-----!._
2 -1
2" -1 = 1048575

Divide equation 2 by equation 1:


a r 11 -1792
_1_=-56

(x+2) =56

al

X=54

1( 2030 -1)
s = --'------'-

2" = 1048576

2-1

rs = -32
Solving for r:

r = a2 = 54+2
a,
28

56= a,(-2)

1111

a, 0 = 14336

Ell

15

2(2 -1)
s = --'-2---1--'-

a,= 9; r = _ 3_

-iJ

=9G~)

a7 = a1r

a1 (r" -1)

,r:-q.2

r-1

By inspection, r =

S=~

S=-3.
1- _!

log1000001
n=--=---log2
n=19.93.
Say n = 20 baskets

a, = 0.1; r =2
S = a1(r"-1)
r -1

100/3 ft

so[3.J3 = 100.
,n=oo
3

S=~

I!IIIIW
1
1alaiiil a,= 3 ; a2 = 9 , 3 -

Take logarithm on both sides:

EJI

,>\ .- .

vv\..--.,_.... -

a,=

-1)

a,2 =a/,
-1792=<>r 11

j ,f .Iv ........
v ,, ,, .. h.~
r-1
100

S=-32
1-3
S= 100

1
27

=--'---'-

log2" = log1000001
nlog2 = log1000001

----+Eq.1

Total cost= 4{1,111,110) = P 4,444,440

2" = 1000001

56=al

~0(10

1-r
1
-

a7 =56; a12 =-1792

n =6

10 -1
S=1111110

2" - 1 = 1000000

as= a,r4

16
as=9

~~

1(2"-1)
1000000=-.
2-1

= 10;

:-~.

So~---''---

a,= 1; r=2.

a,= 10; r

5o' ft

a,(r"-1)
S=--'----'r-1

r-1

2(2 -1)
s =--'-----.-:..
2-1.
s =2046

as

-1)

s = 65534 ancestors

10

as =9(

Bill

S=-'--~

By inspection, r = 2

S = a1 (r" --1)
r-1

a, =2; r = 2; n = 15
a,(r"

a,= 2; a2 = 4; a3 = 8; n = 10

a10 = 28(2)

v\

log2" = log1048576
nlog2 = log1 048576
log1048576
n=--"--log2
n = 20 wealthy men

7
a1=-

= a,rg

S = 1,073741,823 organisms

Take log on both sides:

Substitute r in equation 1:

r=2

a,o

=-2

a,(r" -1)
r-1

S=2

B:l

a1 =1;r=2

60
20

n=-=30

31
Let: D = total distance traveled by ball
D =50+ 2S
D =50+ 2(100) = 250 feet

tm
0.133133133133 = 0.133 + 0.000133 +
0.000000133 + ......
Note: The numbers being added are in a
GP and 0.133133133133 ... is the sum of
an infinite GP.

Solving for the common ratio:


1
0.000133
r=
=-0.133
1000

S=~
1-r

112. IOOl.Solved Problems in Engineering Mathematics (2"d Edition) by Tiong & Rojas_
5=0.133
1- ___
1-

1-r
1

S=_9~33

s9?r

2=2(1-r)

1000

4 =---

999

-2

a1

The four repeated digits are 1, 7,8 & 5


1987 = 496.7G;

2
r=--=-

Layer I

Ell=

1-

Let: x

9
S=2

55

a1 = 1, a2 = -

i,

a3 =

2~

the

6th

40

term of the number series

28

19

13

in

'--y-~~
-12

-15

By inspection, r =- 5 ;

:1 1 1 1!1!~

=10

,r,l/,, 11
I

Let: y = the

6th

term of the number series

,,,_1

y,

S=~

\.'.- v-

1-r
1

~,

X=13+(-3)

5;

S=-~

X1

.~".. ..

} . . y. k

x1/2
y=

1/24

1/6

x113

xY.

!I

I~,
'Iii

""-- y--)

X 1/5

riJ'ii!

2~(i)

. ~'I

!
.>!'
Ill ~;l

l,ilr,l

y = 120

~~

s =_1__

1-r

Ell

S=~-(--1~

II
I
~ _,.~ I
1'.~:'I'

-3

-6

-9

a1
S=----1-r
1

S=2;a1=

5
7
1
8.

Layer IV

EDI

=1984

Total spheres= 10 + 6 + 3 + 1
= 20 spheres

S=-(1)
-3
B

Layer Ill

Layer II

496(4)

Therefore, 19841h digit=


19861h digit=
19851h digit =
19871h digit=

1
r=-3
a1
S=---.
1-r

=0.178517851...
'"'"'

The above number is a-repeating decimal


number.

ED

a1 =6; a2=-2

1785
9999

1
1-- r
3
r=4

s =: 133
~~~

2 = _1__

1000

Day 5- Clock, Variation, Miscellaneous Problems & Progression 11~

.JI':rl

',!
1

116. 1001 solved Problems ~n Engineering Mathematics (2nd Edition) by Tiong & Rojas

Topics

[J
Mon

IJ
IJ
Tue

~-

....

Theory

Vl/ed

!_j
[]

Solutions

D
D

Problems

Notes

Thu

Fri

"'" ,-,,.-~.~""'"-"'-'"-""'"'''"

~um"'""""'"""'"""'""

'""""" ""''''""'"'"m""

Venn Diagram
Combinatorics
Fundamental Principle of Counting
Permutation
Inversion
Cyclic Permutation
Assortment
Combination
Probability Theory
Mutually Exclusive Events
Independent Events
Binomial Distribution
Odds For and Odds Against
Mathematical Expectation

Sat

What is a Venn Diagram?


Venn diagram is a rectangle (the
universal set) that includes circles
depicting the subsets. This diagram,
named after the English logician John
Venn (1834- 1923) in 1880, is a way of
displaying the events or an experiment.
lhe Venn diagram below shows two
events A and B and their intersection.

A Venn diagram can be employed for any


number of subsets, but more tnan three
defeat the purpose of gaining increased
clarity.
Below is an example of a typical problem
that is given in the engineering licensure
examtnations.

Problem:
lilt following Venn diagram shows two
lltttltt.tily exclusive events A and B

A survey was conducted in a graduating


ECE students in a certain university on
which board subject they like best. The
result is tabulated as follows:

118 _I 00 I Solved Problems in Engineering Mathematics (2nd Edition) by Tiong & Rojas
Mathematics
Math & Electronics
Electronics
Math & Communications
Communications
Electronics & Comm
All three subjects

55
32
50
28
51
25
10

How many were there in the graduating


class?
Solution: Use Venn diagram:

.,

. /f

Mathematically,

N=m-n

II

What is Permutation?

''l'/1

Permutation is defined as an ordered


arrangement of a finite number of
elements, either all of the available n
elements or of a part of them. The

'.;.);

't

npn =(n-1)!

If taken all at once,

What is the Permutation with Identical


Elements?

npn = n!
N = 5 + 22 + 10 + 18 + 3 + 15 + 8

What is Combinatorics?
Combinatorics is the branch of
mathematics that concerns with the
selection of objects called elements.
Combinatorics traces its history back to
the ancient times when it was closely
associated with number mysticism. This
branch of mathematics led to the creation
and development of probability theory.

What is the "Fundamental Principle of


Counting?
The Fundamental Principle of Counting
states that:
"If a thing can be done in m different ways
and another thing can be done in n
different ways, then the two things can be
done in m times n different ways."

The total inversion is 2 + 3 + 2 + 1 =8. So


permutation (C, E, D, B, A) is an even
permutation

Cyclic permutation is the shifting of an


entire order of elements one or more steps
forward or backward -the first element
taking the position of \he last, or vice
versa, without changing the order of the
elements in the sequence.

n!
nPr=(n-r)!

N = 81 students

The number of permutations is reduced


when a collection contains identical
elements. The number of permutation n
objects, p of one type, q of another, s of
another, etc., is given by:

What is an Inversion?
If two elemen.ts in a permutation of distinct
elements are in reverse order relative to
their normal or natural order, they
constitute an inversion.

Example:
In the permutation (C, E, D, B, A)
Element C precedes B and A, therefore
has 2 inversions.

= 4989600

Assortment refers to a group of objects


selected from a larger group in such a way
that an object can be used more than
once.

Assortments = (No. of choices for 1st


position) x (no. of choices for the 2nd
position) x (no. of choices for the 3'd
position) x x (no. of choices for last
position)
What is a Combination?
Combination is an arrangement of the
selection of objects regardless of the
order.
The number of combinations of n different
things taken r at a time is
n!
ncr= (n-r)!r!

If taken all at once,

ll

n!
p n r-(n-r)!_p!q!s!

A permutation is said to be even if it


contains an even number of inversions; it
is odd if the number of inversions is odd.
The number of transpositions that are
required to return a sequence of elements
to their natural order is even or odd
according to the number of inversions in
the arrangement.

11p11

What is an Assortment?
Element B precedes A, therefore has 1
inversion.

What is a Cyclic Permutation?

The number of permutations of n different


things taken r at a time is

Communications

Element D precedes B and A, therefore


has 2 inversions

The number of assortments is expressed


mathematically as:

permutations that contains exactly the


same elements but not in the same order
are regarded as different.

Electronics

Day 6 -V!!l_llJ:)iagram, Permutation, Combination & Probability 119


Element E precedes D, Band A, therefore
111
11 11
has 3 inversions.
P = (11-11)12121211i111i 111i

I'~

Problem:
Find the largest number of permutations of
the letters in MATHEMATICS?
Solution:
The word MATHEMATICS has 11 letters.
The letter M occurs 2 times, letter A occurs
2 times, letter T occurs 2 times while the
rest occurs once.

ncr =1
What is the relation between
Permutation and Combination?
The relation between permutation of n
thing taken r at a time to the combination
of n things taken r at a time is expressed
mathematically as follows:

nCr=~
r!

120 100 l Solved Problems in Engineering Mathematics (2nd Edition) by Tiong & Rojas
What

is Probability?

Probability is simple defined as the


numerical assessment of likelihood. It is
. expressed as a number between 0 and 1,
where 0 means an event is impossible
while 1 means it is surely or certainly to
occur or happen.
The total probability of favorable and
unfavorable outcomes is 1.

I
f

One of the earliest mathematical studies


on probability was "On Casting the Die"
written by the 16th century Italian
mathematician and physician Gerolamo
Cardano (1501 -1576). Cardano defined
probability as "the number of favorable
outcomes divided by the number of
possible outcomes". Cardano was
regarded as "the father of the theory of
probability".
The following are some of the important
terms used in probability:

A. Experiment - is a controlled study


whose outcome is uncertain but not
entirely unknown.
B. Trial -a recorded result of an
experiment.
C. Outcome - is one of the possible
results from an experiment trial. It also
refers to the basic unit of possible
occurrences. For example, in. tossing a
coin, the outcomes are heads and tails.
D. Event- is some combination of
possible outcomes in one experiment
trial. In picking a card from an ordinary
deck of cards, the outcomes are the 52
cards themselves. Examples of events
are:
"Pick a 5" (one of the four cards
numbered 5)
"Pick a heart" (one of the 13 cards in
the suit of heart)
"Pick a face card (one of the 12 cards
showing a face i.e. king, queen or jack)

E. Frequency of the outcome - refers to


the number of times a certain outcome
will occur. For example, if you roll a die
50 times, you are conducting 50 trials
of the same experiment. The number
of times a "6" comes up is known as
the frequency of the outcome.

Day 6- Venn Diaqram, Permutation, Combination & Probability 121


The probability for a mutually exclusive
event (E or F) is:

Pe or F = Pe + PF

What are independent events?


The relative frequency of the
outcome is the ratio of the frequency
of the outcome to the number of trials.
Mathematically,

RF

= no. of occurrences
no. oftrials

What is the Principle of Probabiitv?

If an experiment has a set of distinct


outcomes, each of which is equally likely
to happen, then the probability of an event
E, is the ratio of the number of outcomes
to the total number of possible outcomes.

An event is independent if the outcome of


one trial has no effect on the outcome of
any other trial.
A good example is when a coin is tossed,
the outcome has nothing to do with what
happened on the previous toss and will not
affect the next one.
If two events are independent, the
probability that the two events occur is the
product of their individual prob.abilities.
The probability of independent events (E
and F) is:

may mean or is synonym<;>us to the word


"chance".
If a coin is tossed, the probability of
coming up head is 50% and the probability
of coming up tail is also 50%. This is an
example of 1-to-1 odds or even odds. For
a game with even odds, a one peso bet is
paid exactly one peso upon winning. This
is known as the "true odds". However,
there are instances that 90 centavos is
paid instead to a one peso bet, which
would give it an advantage on each game.
This refers to the "payoff odds". The
payoff odds are not always the same as
the true odds.
The odds of an event occurs is the ratio of
the probability that it will occur to the
probability that it will not occur.
odds for E =
prob. that E occurs
prob. that E does not occur

P.
. 1-Pe

odds for event E =-e-

Pe & F = Pe x PF

P. __ no. of outcomes
e - total outcomes

What is a Binomial Distribution?


. The probability that it is not E is:

Pnot E = '1.- Pe

What are mutually exclusive events?

If there are two possible outcomes of an


event and the-possibilities of the outcome
are independent and constant, the
distribution of probabilities is called
binomial distribution.
Binomial or repeated trial probability:

Two or more events are said to be


mutually exclusive if no two of them can
possibly happen in the same trial.
For example, in picking a card from a deck
of cards, it is not possible to pick a card
that is a diamond and a heart at the same
time.
If two or more events are mutually
exclusive, the probability that either one or
the other will occur is the sum of their
probabilities.

p =ncr pr qn-r

where:

p = probability of success
q = probability of failure = 1 - p
n = number of trials
r = number of successful trials

Problema
What is the odds for throwing a total of 5
and 10 in rolling two dice?
Solution:
First, solve the probabilitY of throwing a
total.of 5: For a total of 5, it must be (1 and
4), (4 and 1). (2 and 3) and (3 and 2).

4
1
p5 =-=36 g.
Then, solve the probab' ity ofthrowing a
total of 10: For a total of 10, it must be (4
and 6), (6 and 4), and ( ' and 5).

What is an Odd?

P1o = 36 = 12

The term "odd" is an ambiguous word


that may refer to the probability that an
event occurs, or it can be used to indicate
the payoff on a winning bet. It sometimes

Psopo =

g + 12--36

Day 6 - Venn Diagram, Permutation, Combination & Probability 123

122. 1001 Solved Problems in Engineering Mathematics (2"d Edition) by Tiong & Rojas
The odds for. throwing a total of 5 or 10 is:

odd=~
1- .2_

36

odd=

7
29

If the expectation is positive, the player will


win in the long run and lose if the
expectation is negative.

Problem:
In tossing a single coin, P 100 bet is
placed on heads i.e. if heads comes up,
the player wins P 100 and if tails comes
up, the player loses P 100. What is the
expectation?

This is expressed as 7 -to-29 odds.


Tails

y,

What is another formula for "odd for"?

-100
If the"odds for" an event are given as "a"
and "b", then the probability of that event
would be:
odds for an event =-aa+b

Expectation=

(~}1 oo) + (~ )c-1 oo)

deck of cards contains 52 cards divided


into 4 suits; spades, hearts, diamonds and
clubs. Each suit contains 13 cards labeled
Ace, 2, 3, 4, 5, 6, 7, 8, 9, 10, jack, queen
and king. Each of the four kings in a deck
represents a great leader from history;
Charlemagne (hearts), Alexander the
Great (clubs), Julius Ceasar (diamonds),
and King David (spades).
The playing cards were used also to
describe a calendar year. The 52 cards
represent the 52 weeks in a year. The 4
suits represent the four seasons of the
year and the 12 face cards (4 kings, 4
queens and 4 jacks) represent the 12
months ot' the year.

Expectation = 0

Problem:
A player in a certain card game lays odds
of 3 to 5 for him to win. What would be the
payoff for a bet of 3 pesos?

In Casino Filipino, a roulette offers winning


a bet on a single number pays 35 to 1. The
actual probability is 1 in 38. What is the
expectation?

Solution:

Solution:

Problem:

3 -~
Odds for = 3 + 5 - 8
Therefore, it is a 3-to-8 odd. A bet of 3
pesos will have a payoff of 8 pesos.

What is "odd against"?


The "odds against" an event are the
reciprocal of the "odds for" the event.
The odds of 5 to 7 for an event is
translated into odds of 7to 5 against it.

What is a Mathematical Expectation?


A mathematical expectation is the
average amount a player can expect to
win or lose on one play in any game of
chance.

The 35 to 1 is a house (payoff) odds of


Casino Filipino.
Expectation can be obtained by taking the
difference betWeen the product of the first
number in the house odds and probability
of success, and the product of the second
number of the house odds and the
probability of failure.
37
Expectation= 35(_!__) -1(
)
38
313 .

Expectat1on
. =- 2

38
Expectation = -0.0526
Thus, a player expects to lose to Casino
Filipino an average of 5.26 centavos for
every peso bet.
What are "Card Games"?

It can be found by multiplying the


probability of each possible outcomes by
its payoff, and then adding these results.

Card games are games which are played


using a standard deck of cards A standard

Poker Harid with corresponding number of


ways:
Hand
Royal flush
Stra_ight flush
~our of a kind
Full house
Flush
Straight
. Three of a kind
Two pair
Pair
None of the above
Total

No. of ways
4
36
624
3744
.5108
10200
54912
123552
1098240
1302540
2598960

Poker Hand with corresponding


probability:
Hand
Royal flush
Straight flush
Four of a kind
Full house
Flush
StraiQh!
Three of a kind
Two pair
Pair
None of the above
Total

Probability
0.00000154
0.00001385
0.0002401
0.0014406
0.0019654
0.0039246
0.0211285
0,0475390
0.4225690
0.5011774
1.00000000

Poker Hand with corresponding expected


frequency:
Hand
Royal flush
Straight flush
Four of a kind
Full house
Flush
Straight
Three of a kind
Two pair
Pair
None of the above

Expected frequency
1 in 649740 hands
1 in 72192 hands
1 in 4165 hands
1 in 694 hands
1 in 509 hands
1 in 255 hands
1 in 47.33 hands
1 in 21 hands
1 in 2.37 hands
1 in 2 hands

What are Probabilities with Dices?


Dice were invented for sole purpose of
gambling. As a matter of fact, dice gaines
have so little intrinsic interest that in the
absence of wagering they would hardly be
worth nl::>llinn

''' '',.

Dice were first used by th~ Chinese. The


sum of the opposite faces of~ die is
always equal to 7. And the sum of all the
vertical faces of a die, no matter how it
rolls is always equal to 14. When two dice

124 IO<h Solved Problems in Engineering Mathematics (2"d Edition) by Tiong & Rojas
are rolled and both will face up 1, this is
known as snake's eyes.

G8

~.8 [ZJ8

Gr:J~r:J
G rsJ ~ rsJ
l:l [i"7l
L:.J l!_!J
l:l r-;;1
L:.J l!:.!J

Proceed to the next page for your 6th test


Detach and use the answer sheet provided
at the last part of this book. Use penCil
number 2 in shading your answer.

[ZJ[J
1.1 LSJ

rel

[i"7l
l!._j l!_!J

r:;][i"7]
l!.:J l!_!J

rel ~
l!._j l!:.!J

r:eJ fe.eJ
l!.:J l!:.!J

.
G []

~8

1::1 G 1::1 G

[i"7lr.l
l!_!J L!J

r.=--11-l
LUI L!J

[]
1.1 []

GOOD LUCK I

Topics

D
D
D D

-m:ribia:

tvlon

Did you know that .. the number 1


followed by 100 zeros is called "google"
and the term "google" was coined in the
1930s by the aine-year old nephew of the
American mathematician Edward Kasner
when he was asked to come up with a
name for a very large number !

Tue

Theory

~uote:

1: :11.1 1::11.1
fe"il [i"7l

l!_!J l!_!J

~- reel

LUl

l!_!j

I!!]

LUJ

lei

The following are the two-dice


probabilities:

9
10
11
12

Probability
1/36
2/36
3/36
4/36
5/36
6/36
5/36
4/36
3/36
2/36
1/36

D
Thu

D D

- Morris Kline

Solutions

Notes

LUJ L!_..!J

~ []
[]
[]
[]

~-

Problems

l!!lr-::1
LUJ ~
I!!] reel

1: :11::1 1::11::1 1: il

Two-dice sum
2
3
4
5
6
7

"Statistics - the mathematical theory of


ignorance.

VVed

Fri

(QJ
Sat

:15&: EE Board October 199~


In a class of 40 students, 27 like Calculus
and 25 like Chemistry. How many like
both Calculus and Chemistry?

A.
B.
C.
D.

Venn Diagram
Combinatorics
Fundamental Principle of Counting
Permutation
Inversion
Cyclic Permutation
Assortment
Combination
Probability Theory
Mutually Exclusive Events
Independent Events
Binomial Distribution
Odds For and Odds Against
Mathematical Expectation

10
11
12
13

.2$8: GE Board February 1994


' A survey of 100 persons revealed that 72
of them had eaten at restaurant P and
that 52 of them had eaten at restaurant Q.
Which of the following.could not be the
number of persons in the surveyed group
who had eaten at both P and Q?

A.

B.
%57: ECE Board November 1998
A c!ub of 40 executives, 33 like to smoke

Marlboro and 20 like to smoke Philip


Morris. How many like both?
A.

c
n

10
11
12
13

C.
D.

20
22
24
26

2591 ECE Board November 1992


'The probability for the ECE board
examinees from a certain school to pass
the subject Mathematics is 3n and for the
subject Communications is 5/7. If none of
the examinees fails both subject and
there are 4 examinees who pass both
subject~. find the number of examinees
from that school who took the
examinations.

126 H>O I Solved Problems in Engineering Mathematics (2nd Edition) by Tiong & Rojas
A.
B.

c.
D.

20
25
30
28

/
,260: EE Board March 1998
In a commercial sur.iey involving 1000
persons ori brand preference, 120 were
found to prefer brand x only, 200 prefer
brandy only, 150 preferbrand z only, 370
prefer either brand x or y but not z, 450
prefer brand y or z but not xand 370
prefer either brand z or x but' not y. How
many persons have no brand preference,
satisfied with any of the three brands?
A.

B.

c.
D.

280
230
180
130

Day 6- Venn Diagram, Permutation, Combination & Probability 127

2&8: EE Board April199&

26~:

Five different mathematics books, 4


different electronics books and 2 different
communications books are to be placed
in a shelf with the books of the same
subject together. Find the number of ways
in which the books can be placed.

A.
B.
C.
D.

A.

B.
C.
D.

292 .
5760
34560
12870

D.

265: ECE Board November 1998


If 15 peop!e won prizes in the state lottery
(assuming that there are no ties), how
many ways can these 15 people win first,
second, third, fourth and fifth prizes?

A.

B.
C.
D.

B.
C.
D.

beginning and ending with a vowel


without any letter repeated can be formed
from the word "personnel"?

B.

c.
D.

40
480
20
312

What is the number of permutations of the


letters in the word BANANA?

274: EE Board October 1997

B.
C.
D.

B.

c.
D.

c.

There are four balls of four different


colors. Two balls are taken at a time and
arranged in a definite order. For example,
if a white and a red balls are taken, one
definite arrangement is white first, red
second, and another arrangement is red
first, white second. How many such
arrangements are possible?

A.

B.
C.
D.

6
12
~

275: How many different ways can 5


boys and 5 girls form a circle with boys ,
and girls alternate?

A
B.
C.
D.

28,800
2,880
5,600
14,400

276: EE Board October 1997


A
B.
C.

120
130
140
150

1440
480
720
360

2,024
12,144
480
360

In how many ways can a PSME Chapter


with 15 directors choose a President, a
Vice President, a Secretary, a Treasurer
and an Auditor, if no member can hold
more than one position?

D.

How many permutations are there ifthe


letters PNRCSE are taken six at a time?

A.
B.
C.
D.

36
60
52
42

A.

271: ME Board October 1992

267: EE Board June 1990

A.

2204

How many 4 digit numbers can be formed


without repeating any digit from the
following digits: 1, 2, 3, 4 and 6?

A.

262: How many four-letter words

D.

270: ME Board April1994

266: CE Board November 1996


Neither yes nor no
Yes
No
Either yes or no

2&9: EE Board April1997

A PSME unit has 10 ME's, 8 PME's and 6


CPM's. If a committee of 3 members, one
from each group is to be formed,. how
many such committees can be formed?

4,845
116,260
360,360
"3,003

In how many ways can 4 boys and 4 girls


be seated alternately in a row of 8 seats?
1152
2304

B.
C.
D.

144
258
720
450

A toothpaste firm claims that fn a survey


of 54 people, they were using either
Colgate, Hapee or Close-up brand. The
following statistics were found: 6 people
used all three brands, 5 used only Hapee
and Close-up, 18 used Hapee or Closeup, 2 used Hapee, 2 used only Hapee
and Colgate, 1 used Close-up and
Colgate, and 20 used only Colgate. Is the
survey worth paying for?

EE Board June 1990


EE Board April199~
CHE Board May 1994

B.

A.

and 4 engineers be seated on a bench


with the nurses seated together is

261: EE Board April i:997

A.

720
120
360
180

264: The number of ways can 3 nurses

A.
B.
C.

27~:

In how many ways can 6 distinct books


be arranged in a bookshelf?

t
~

360,360
32,760
3,003
3,603,600

There are four balls of different colors.


Two balls at a time are taken and
arranged any way. How many such
combinations are possible?

272: EE Board October 1997

36

Four different colored flags can be hung


in a row to make coded signal. How many
signals can be made if a signal consists
of the display of one or more flags?

B.
C.

3
6

D.

12

277: EE Board March 1998


A.
B.

c.
D.

64
66
68
62

How many 6-number combinations can


be generated from the numbers from 1 to
42 inclusive, without repetition and with
no regards to the order of the numbers?

850,668

1Z8 1001 Solved Problems in Engineering Mathematics (2nd Edition) by Tiong & Rojas
B.

c.
D.

5,245, 786
188,848,296
31,474,716

Z78: Find the total number of


combinations of three letters, J, R, T
taken 1, 2, 3 at a time.

c.
D.

Z8~: ECE Board April1998


A semiconductor company will hire 7 men
and 4 women. In how many ways can the
company choose from 9 men and 6
women who qualified for the position?

A.

B.
C.

8
9

A.
B.

D.

10

C.

D.
Z79: ME Board October 1997
In how many ways can you invite one or
more of your five friends in a party?

A.
B.

c.
D.

15
31
36
25

zsoa CHE November 1996


In how many ways can a committee of
three consisting of two chemical
engineers and one mechanical engineer
can be formed from four chemical
j;!n'gineers and three mechanical
engineers?

A.
B.
C.
D.

18

64
32

120
530

c.

720

D.

320

Z84a ECE Board April1994


There are 13 teams in a tournament.
Each team is to play with each other only
once. What is the minimum number of
days can they all play without any team
playing more than one game in any day?

B.

1390
1240

A.
B.
C.
D.

11

B.

12
13
14

C.

D.

Z85a EE Board October 1996


There are five main roads between the
cities A and B, and four between B and C.
In how many ways can a person di'ive
fromA to C and return, going through B
on both trips without driving on the same
road twice?

A.
B.

c.
D.

260
240
120
160

Z86: EE Board Aprll1991


There are 50 tickets in a lottery in which
there is a first and second prize. What is
the probability of a man drawing a prize if
he owns 5 ti't:kets?

B.

c.
D.

50%
25%
20%
40%

1/36
1/9
1/18
1120

Z88: Roll two dice once. What is the


probability that the sum is 7?

%93: EE Board Apll'ill996


The probability of getting at !east 2 heads
when a coin is tossed four times is,
A.
B.

c.
D.

A.
B.
C.
D.

1/6
1/8
1/4
1/7

A.

is

c.

B.

D.
A.

B.

c.

1/6
1/9
1/12
1/18

290: Determine. the probability of drawing


eithel" a king or a diamond in a single
draw from a pack of 52 playing cards.

A.

c.

B.

2/13
3/13
4/13

D.

1/13

:1911 A card is drawn from a deck of 52

playing cards. Find the probability of


drawing a king or a red card.

A
B.
C.
0.

0.5835
0.5385
0.3585
0.8535

1:91:1 CE Board November 1998

A coin is tossed 3 times. What is the


probability of getting 3 tails up?
A
B.
C.

o.

1ffl
1/16
1M

11/16
13/16
114
3/8

::t94: A fair coin is tossed three ti.mes.


What is the probability of gel!ing either 3
heads or 3 tail?

289: In a throw of two dice, the


probability of obtaining a total of 10 or 12

D.

A.

A.

zsza EE Board April 1997


How many committees can be formed by
choosing 4 men from an organization of a
membership of 15 men?

A.

680
540
480
840

None of these

Z81a EE Board Aprii199S


In Mathematics examination, a student
may select 7 problems from a set of 10
problems. In how many ways can he
make his choice?

A.
B.

1435
1365

Dart- VexmDiagram, Permutation, Combination & Probability 1Z9

118
3/8
1/4
112

295: ECE Board Mare 19'96


The probability of getting a credit in an
examination is 113. If three students are
selected at random, what is the
probability that at least one of them got a
credit?
A.
B.
C.
D.

19/27
8/27

2/3
1/3

:Z9fl: There are 3 questions in a test. For


each question 1 point is awarded for a
correct answer and none for a wrong
answer. If the probability that Janine
correctly answers a question in the test is
2/3, determine the probability that she
_gets zero in the test.
A.

B.
C.
D.

8/27
4/9
1/30
1/27

:Z97Z EE Board April :199:1


In the ECE Board Examinations, the
probability t.hat an examinee will pass
each subject is 0.8. What is the
probability that an examinee will pass at
least two subjects out of the three board
subje~ts?

Z87a Roll a pair of dice. What is the


probability that the sum of two numbers is

A.

11?

B.

70.9%
80.9%

130 . rOO! Solved Problems in Engineering Mathematics (2nct Edition) by Tiong & Rojas
C.
D.

85.9%
89.6%

298: In a multiple choice test, each


question is to be answered by selecting 1
out of 5 choices, of which only 1 is right If
there are 1 0 questions in a test, what is
the probability of getting 6 right of pure
guesswork?

302: ME Board April 1.99&


An urn contains 4 black balls and 6 white
balls. What is the probability of getting 1
black and 1 white ball in two consecutive
draws from the urn?

A.
B.

D.

A.
B.
C.

D.

10%
6%
0.44%
0.55 %.

:&99: ME Board April1994


From a box containing 6 red balls, 8 white
balls and 10 blue balls, one ball is drawn
at random. Determine the probability that
it is red or white.

A.
B.
C.
D.

1/3
7/12
5/12
1/4

A.
B.
C.
D.

A.

A.

C.
D.

25/81
16/81
5/18
40/81

301: CE Board May 1996


A bag contains 3 white and 5 black balls.
If two balls are drawn in succession
without replacement, what is the
probability that both balls are black?
A. 5/16
B. 5/28
c. 5/32
D. 5/14

B.
C.
D.

1/6
1/4
1/2
1/8

305: From 20 tickets marked with the


first 20 numerals, one is drawn at
random. What is the chance that it will be
a multiple of 3 or of 7?
A.
B.

c.
D.

Theory

16/81
25/81
20/18
40/81

300: EE BoardOctober 1990


From a bag containing 4 black balls and 5
white balls, two balls are drawn one at a
time. Find the probability that both balls
are white. Assume that the first ball is
returned before the second ball is drawn.

1/2
8/15
3/10
2/5

Venn Diagram
Combinatorics
Fundamental Principle of Counting
Permutation
, Inversion
Cyclic Permutation
j Assortment
1 Combination
1 Probability Theory
Mutually Exclusive Events
Independent Events
Binomial Distribution
Odds For and Odds Against
Mathematical Expectation

tv1on

303: EE Board October 1.990


From a bag containing 4 black balls and 5
white balls, two balls are drawn one at a
time. Find the probability that one ball is
white and one ball is black. Assume that
the first ball is returned before the second
ball is drawn.

304: EE Board October 1.997


A group of 3 people enter a theater after
the lights had dimmed. They are shown to
the correct group of 3 seats by the usher.
Each person holds a number stub. What
is the probability that each is in the
correCt seat accordi1g to the numbers on
seat and stub?

B.

Topics

0.24
0.27
0.53
0.04

Problems

Solutions

0 I
I
0
0 I
0
Tue

\JVed

Thu

Fri

[QJ

Notes

Sat

ANSWER KEY
256.C
257. D
258.
259. D
260. A
261.
262.A
263.
264.C
265.C
266.A
267.C
268. A

c
c

269. B
270.
271. A
272.A
273.
274.
275. B
276.C
277. B
278.A
279. B
280.A
281. A

c
c
c

282. D
283. B
284.
285. B
286.C
287.
288.A
289. B
290.
291. B
292.A
293. D
294.

c
c
c

RATING

295.A
296. D
297. D
298. D
299. B
300.A
301. D
302.
303. D
304.A
305 D

c:J 43-50

.,

c:J
c:J
0

Topnotcher

33-42 Passer
25-32 Conditional
0-25 Failed

If FAILED, repeat the test.

,jil

132 lQOl Solved Problems in Engineering Mathematics (2"d Edition) by Tiong & Rojas

Day 6- Venn Diagram, Permutation, Combination &Probability 133

ml

Let: x = number of students who like


both subjects
Calculus

Let:

ED

x = number of examinees who took


the examination

Given word:

Math Communication

Chemistry

N = (5!}(4!)(2!}(number of patterns)

I s

Let:

x =Hx-4)*4+(%x-4)

x = number of executives who


smoke both brand of cigarettes
Marlboro

Philip Morris

8
7

Any of the five consonants can be filled in


the second letter.

Any of the remaining four consonants can


be filled in the third letter.

- Let: x = number of persons who have no


brand preference
Brand X

Brand Y

Brand Z

+ 150+100
X=280

(72-x)+x+(52-x)=100
72 + 52 - X = 100

n
E

n J

Number of ways the 3 nurses can be


arranged = 3!
Number of ways the 4 engineers can be
arranged = 4!

FD

Number of ways the books in ELEC can

N = 5(4)(3)(2)
N= 120 ways

I 'I

total nwnber of wctys

Five patterns

l1c arranged= 4!

l11 arranned = 2!

N = 360,360 ways

Number of ways the books in COMM can

X =24

Number of ways the books in MATH can


be arranged = 5!

The survey is not worth paying for. One


error is that according to the said survey,
there are 6 people who used 9.!JJ.h_ree
brands but only ~~ used !he brands
Hapee and Close-up.

E
E

N;, 15(14)(13)(12)(11)

Six patterns

Note:

;E

COMM
ELEC
COMM
MATH
ELEC
MATH

-~

\_

fll

N =720 ways

ELEC
COMM
MATH
COMM
MATH
ELEC

E.

E.
n

N = (3!)(4!)(5)

MATH
MATH
ELEC
ELEC
COMM
COMM

1000 =X+ 120+ 50+ 200 + 100

n
n

N = (3!)(4!)(number of patterns)

Ell

= 40

X= 13

x = number of persons who have


eaten in both restaurants

E
E
E

n
n
E
E

Let: N = number of words

(33- X)+ X +(20- X)= 40

Let:

Let: N = total number of ways

N= 40 ways

Ell

The remaining bowel can be filled in the


fourth letter.

N = 2(5)(4)(1)

33+20-x

Ill

x=-x-4
X=28

Bil

Four-letter word
Note:
Any of the two vowels can be filled in the

first letter.

(27 ~ x)+ x +(25- x) = 40


X= 12

N = 34,560 ways

Number of vowels= 2 (E & 0)


Number of constants= 5 (P, R, S, N & L)

27 +25-x = 40

N = (5!)(4!)(2!)(6)

PERSONNEL

:I

Day 6- Venn Diagram, Permutation, Combination & Probability 135

l34 100 l Solved Problems in Engineering Mathematics (2"ct Edition) by Tiong & Rojas

!,

N = 2" -1

N = npn = n! = 6!

ml

N=720

N=nPn

Given word: BANANA


Number of A's = 3
Number of N's = 2

p!q!

GoBO?
0
08oG

3!2!

08
GO

OG

Number of ways of selecting a


mechanical engineer:
31
N = C =
.
=3
2
3 1 (3-1)!1!

8 '~

\'~~

This seat is permanently


occupied by one of the children!

"'--..__

I. ~;c I Tr7;s I ~f]

N = 6(3) = 18 ways

.~

Ell

10!
N= 10c1 = ( 10 _7 )!7!

Number of ways the girls can be


arranged = 5!

N=120 ways

N = 15(14)(13)(12)(11)

N=(4!)(5!)

N= 360,360

N = 2880 ways

ml

ml
N = 4p1 + 4P2 + 4P3 + 4P4

Number of ways of selecting a chemical


engineer:
41
N - C =
=6
1- 4 2 (4- 2)!21

Number of ways the boys can be


arranged = (5-1)! =4!

Cit! ~~

ml

N= 60 ways

N = 10(8)(6)
N=480 ways

= 2" - 1 =2

N = 31 ways

B0

N=_I1J__=~

ml

N = 12 ways

N= 720 ways

=23 -

N = 7 ways

4!
N= 4p2 = ( 4 -2)!

n!=6!

ml

ml

N=(4!)(4!)
N = 576 ways

N= 15 C 4 =~
(15-4)!4!
N = 1,365 committees

4!
N= 4c2 = (4 -2)12!

Number of games that can be played per


day:
13
N = - = 6.5

6 games/day

Number of days needed to complete the


tournament:
78
N=-=13 days
6

ED
Number of ways to travel from A to B = 5
Number of ways to travel from 8 to C = 4
Number of ways to travel from C to 8
without using the same road to travel from
8 to C =3
Number of ways to travel from B to A
without using the same road to travel from
A to B =4
N = 5(4)(3)(4) = 240 ways

P = probability of winning a prize in the


lo_ttery

N=--4_!_+ __4_!_+ __4_!_+ __4_!_


(4-1)! (4-2)! (4~3)! (4-4)!

N =6 ways

ml

N = 64 signals

Ell

Number of ways of hiring men:


91
N1 = C7 =
. =36
g
(9- 7)!7!

- c-~

N-42 6-(42-6)!6!
Num_ber of ways the 4 boys can be
arranged= 4!

Total number of games


131
N= 13 C 2 = .
=78
(13- 2)!2!

Number of ways of hiring women:


6!
N.,"' .C 4 =------=15

(>
(6-4)14!

N = 5,245,786 ways

Number of ways the 4 girls can be


arranged = 4!

2
1
P=-=50 25
P = probability for the man to win
= number of tickets he bought x.
probability of winning a prize

P= 5(_!_)
.25
p =0.20

:16( 15) = 540 ways

136 1001 Solved Problem~ngineering Mathematics (2nd Edition) by Tiong & Rojas

Total number of trials

r----

f-

=36

2. I
3

r=a
4

.3

Let:
p = probability of getting a head in a
single tl)row of a fair coin

--+-----1

L- L-+--J---1! I

Day 6- Venn Diagram, Permutation, Combination & Probability 137

q = probability of getting a tail in a single


throw of a fair coin

P = number of successful trials

total number trials

~'---

Number of trials with a sum of 11

=2

P = number of successful trials_


total number trials

36

18

II!.'P.fJII

laiiill

34!]

Total number of trials = :36

1
9

P=--=36

--1

Number of trials with a sum of 7 = 6

P = number of successful _trials


total number trials

6
1
P=-=36

PKorD =PK +Po-PK&D

F.ll
Total number of trials = 36
Number of trials with a sum of 10 or 12

=4

3 0 3
1)
1) (1)2
p3H=3C3 (2 '(2
1

3H =8

3c3(~J(~J =(1{iJ

P3T =

=(1)

1 1
p = p3H +P3T =B+S

p:;;:~
4

P=n CrPr qn-r

where: p =

; q = 1 ; n = 4;
2

P2H =

4c2(iJ(iJ

p2H:;;:

4!
( 1)
(4-2)!2! 2

r=2

:;;:.S3

13

ED
Note:
Probability of getting a predit = 1/3
Probability of not getting any credit

P3H =

PK = probability of drawing a king


PR = probability of drawing a red card
PK & R = probability of drawing a king at

the same time

a red card

4c3(~J (~J
1

=4

26

52

52 52

PKorD

= 0.5385

4)

3!
(
12
(3-1)!1! 21 = 21

P2 = probability that exactly two students

(21)2(1)2

got a credit

1) (2)
= c (3 3 =
2

4
2

1
:;;:16

2) = 21.6

3!
(
(3-2)!2! 21

P3 = probability that all three students

=--+----

P,

PKorD =PK +PR -PK&R

1) (2)
= c (3 3 =
1

4!
( 1)
p3H:;;: (4-3)!3! 2

1)
p4H:;: ( 2

= probability that only one student


got a credit

P4H=4c 4

= 2/3

Let:

P1

Note: In a pack of 52 playing cards, there


are 4 king cards, 26 red cards and 2 king
&.red cards at the same time

Koro

p3T :;;:_

4
13 1
16
R =-+-----=Koro 52 52 52
52

.
1
1 :n= 3:r= 3
where:p=2";q=

PK = probability of drawing a king


Po = probability of drawing a diamond
PK & o = probability of drawing a king at
the same time a diamond

(1) (1) (1)2


2 2

r=3

RKoro =-

1
:
q= : n=3;
2 2

P=-.

P=3C3

Note: In a pack of 52 playing cards, there


are 4 king cards, 13 diamond c.ards and 1
king & diamond card at the same time

P=-=--

where: p=

P=n CrPr qn-r

=(1)

P=j, CrPr qn-r


I

Ill

p"" Pm + P3H
I'~

+ p4H-

3 1 1
8 +4- +16-

got a credit

11
16

r3

=3

J( J

c3 (i j

= (1 {

2~) = 2~

Day 6- Venn Diagram, Permutation, Combination & Probability 139

138 IOOlSolved\Problems in Engineering Mathematics (2nd Edition.) by Tiong & Rojas


P = probability that at least one student
got a credit

P = probability of passing at least two


Subjects

_ __g+~+~
p = p1 + p2 + p3 - 27 27 27

P=0.384+0512

19
p = 27

p = 0.896

p2

10

P =-x24
p =1
90

=~
9

P = probability that both balls drawn are


all wh1te

Assume the first draw is white and the


second draw is black:

Let:

Note:
The only way she c'an get zero is, if all her
3 answers were wrong.

Let:
p = probability of getting a correct answer
p 2/3

p = probability of getting a correct answer


p = 1/5

1
P=3C3 ( 1 ) ( 2 ) =(1) ( 1 ) =27

ED

= 10 c61 5

Probability of failing in any of the three


subjects is 0.2

Let:
P1 = probability of passing exactly two
subjects

= 3C 2 (0.8)
31
(3 -2)121

(o.2f

(0. 128) = 0.384

P2 = prol;>ability of passing all the three


subjects
3

P2 = 3C 3 (Q8) (02) =(1)(0512)

1\

4r

5) 5;

P1- -5P2 = probability of drawing a black ball in


the second draw
Note:
The 1st ball was not returned in the bag
before the 2"d ball was drawn

P = probability of getting a red or a white


ball from tile box

p2

_ number of red or white balls


Ptotal number of balls
-------~~----

7
12

P = probability that both balls drawn are


all black

1m

5 4
P=PxP=-X-

Let

P= 20

=~

56

14

1\ssume the first draw is black and the


:;econd draw is white:

P2 = probability of drawing a white ball 111


the second draw

p1

= pblack X pwhite
9 9

p = 20
1
81
Assume the first draw is white and the
second draw is black:

mJ

_5
P1 - 9

Assume the first draw is black and the


second draw is white:

P=-=-

P1 = probability of drawing a white ball in


the first draw

4 5
p =-x-

=~

24 24
p = p1 + p2 = 90 X90
p =0.533

p = 0.0055

14
24

Let:

P1 = probability of drawing a black ball in


the first draw

Let:

Probability of getting a passing score in


each subject is 0.8

P = probability that one ball is black and


the other is white

Let:

l5 !

mJ

Notes:

10

p = 24
1
90

)v ( 4 ~4

p = nC,p' qn-r
3

p2 = pwhite X pblack
1

81

P = probability of getting 6 correct


answers out from 10 questions

101
P= (10-6)!61

p = 25

q = probability of getting a wrong answer


q = 4/5

P=P1 X p2 =g-Xg

P =-x-

q = probability_.of getting a wrong answer


q = 1/3

pt-

P1 = Pblack X pwhite
1

ml

ED

P1

Note
The 1st ball was returned in the bag
before the 2"d ball was drawn

P2 = pwhite X pblack

..

5 4

'") =-x
9 9
p2 = 20
81

Let:
P

=probability that one ball is black and


the other is white

p2 = 0.512

140 1001 Solved Problems in Engineering Mathematics (2nd Edition) by Tiong & Rojas

P=P1 +P2-20
20
--x
81 81
P= 40
81

D
D
D D
D D
D D

ID

Mon

Probability that A is correct= 1/3,


assuming he is to sit down first

Tue

Probability that B is correct= 1/2,


assuming he is to sit down after A
Probability that C is correct = 1, assuming
he is the last to sit down
Let:
P =probability that A; Band C

~re

correct

1 1
P=-x- x 1

I~

Topics

1
P=6

Theory

Wed

Problems

Thu

Solutions

Fri

[I]

~
Sat

Notes

Numbers from 1 to 20, which is divisible


by 3 = 6 numbers (3,6,9, 12, 15, 18)
Numbers from 1 to 20, which is divisible
by 7 = 2 numbers (7,14)

Total numbers from 1 to 20, which is


divisible by 3 or 7 = 8 numbers
Let:
P = probability that the ticket nu!llber is
divisible by 3 or 7
P = successful outcomes
total outcomes

P=20

2
P=-

T
T

Venn Diagram
Combinatorics
Fundamental Principle of"Counting
Permutation
Inversion
Cyclic Permutation
Assortment
Combination
Probability Theory
Mutually Exclusive Events
Independent Events
Binomial Distribution
Odds For and Odds Against
Mathematical Expectation
'!

142 100 l Solved Problems in Engineering Mathematics (2nd Edition) by Tiong & Rojas

Topics
' "

< '

<

[QJ

~ '< ~ 'o '

Mon
'tfi',<~'''"">~<iA'<":<'

,"<>

( <-,

-'<-

~ ,

"'

),

;_

...

~ < .,.

4- ~ .~ ~ ~ ~

.i , '

.{ .. ~

'It~

'<

0
L 0

i>..~.~:~

Tue

' ,?

< '

'

~;.,

"

'

...

'"

...

Theory

Wed

0
0
0

0
0
0

Problems

Solutions

Notes

Thu

Fri

Sat

What is Geometry?
,,.

"

'

"

...

' > ; ., ,.

)0 i>

< )' ~

:0 0

"<

'

<

),

-t

'

+ ,.

<; ,. ~

".

~ '

< ' <;

</ ... ' "' ,_

>.

'0

<

.,

.< ,, .< ' .,

~ , . .,. ~ -~ .(.

'")<

Branches of Geometry
Basic Postulates of Euclid
Geometric Elements and Figures
Types of Angles
Units of Angles
Polygons
Classifications of Triangles
Other Types of Triangles
Types of Quadrilaterals
Bramaguptha's Theorem
Ptolemy's Theorem
Terms Related to Circles
Theorems Involving Circles
Ellipses

Geometry is the branch of mathematics


which deals with the properties of shapes
and spaces. The term "geqmetry" was
derived from the Greek words, "ge"
meaning earth and "metria" meaning
measurement.
Euclid (c. 330- c 275 B.C.) in his best
known book in geometry "Elements" give
more emphasis on Plane geometry while
Archimedes (287- 212 B.C.) contributed
so much to Solid geometry.
What are the different branches of

2. Solid Geometry - deals with the


properties of geometrical shapes of
three dimensions, such as cones,
pyramids, cylinders, prisms, spheres,
etc.
3. Euclidean Geometry- a geometry
that is based on the assumptions of
Euclid, who, about 300 B. C., collected
the mathematical knowledge of that
time in the 13-volume "Elements". The
problems of this type of geometry can
be solvd by logical reasoning from an
initial core of postulates (axioms), a
method commonly referred to as the
classical' axiomatic method of Euclid.

Q~s>metry?

1. Plane Geometry- deals with the


pmperties of plane figures or
qt~ometncal shapes of two dimensions,
:;11d1 <ls angles, tnangles, squ;1rr!,
f"llyqow;, Ulllic s(dron~;, (~!c,

4. Non-Euclidan Geometry- a
geometry that is not based on the
assumption of Euclid ..
5. Projective Geometry- deals with the
study of those properties of plane
frqures that arf' rmch;mqed when ;1

- Da_y_I -- Plane Qeq_m~.!Dt:..~

144 1001 oSo1ved Problems in Engineering Mathematics (2nd Edition) by Tiong & Rojas

What are the Basic Geometric Elements


and Figures?

given set of points is projected onto a


second plane.

1. Point- a.dimensionless geometric

6. Trigonometry - a geometry which

figure having no properties other than


location or place.

specializes on the study of triangles.


This subject is divided into two
branches, namely Plane. Trigonometry
and Spherical Trigonometry.

5. Straight angle- equal to 180 or


n radians.

2. line -.the shortest distance between

--~

any two points. A line is implicitly a


straight line. A line segment is a piece
of a line with definite endpoints.

7. Analytic Geometry- a geometry


which deals with geometric problems
by using the coordinates systems, and
transforming them into algebraic
problems. This type of geometry was
invented by Rene Descartes. Plane
analytic geometry deals primarily with
the analysis of equations in two
variables while Solid analytic geometry
deals with equations hi three variables.

lines or two planes that meet. The point


where the two line me~t is known as
the vertex. Angle comes from Latin
"angulus'' for "sharp bend".

r cr

What are the different types of angles


according to their measure?

8. Differential Geometry - a geometry

1. A unique straight line can be drawn


between any two points.
2. Such a line can be extended
indefinitely in either direction.

3. A circle can be drawn in a plane using


a given point (a center) and a given
distance (a radius).

4. All right angles are equal.


5. Given a line and a point not on the line,
there .exists exactly one line parallel to
the original line passing through the
given point
Postulate number 5 above is commonly
known as the "parallel postulate"

2. Acute angle- greater than 0 but less


than goo or

2:
2

..-

( ~-o2:__).

\___,..J - - -

5. Vertical angles - angles formed by two


intersecting lines. Vertical angles are

7. Full angle or Perigon- equal to 360"


or 2rr radians.

radians

.&__

G
What are the other types of angles?

3. Right angle- equal to goo or ~

1. Adjacent angles -two angles with a


common leg.

radians

k_

l:_
2.

4. Obtuse angle- greater than 90 or ~


2
radians but less than 180 or n radians

Complementary angles- two angles


whose surn is a right angle (90).

L, /

LL~--

s_/

equal',

-........,:r- ( ~\A
A\ / ;r 1-

0~
'

Euclid's Elements is based upon five basic


postulates or assumptions which are as
follows:

1. Null or Zero angle -equal to 0.

What are the Basic Postulates of


Euclid?

----'~

4. Explementary angles -two angles


whose sum is <l perigon (360').

6. Reflex angle- angle great than 180


or n radians but less than 360 or
2rr radians.
270"

3. Angle - The opening between two

that applies differential and integral


calculus to cuiVes, surfaces and other
geometric entities.

3. Supplementary angles -two angles


whose sum is a straight angle (180).

7"~

Angles A and B are vertical angles.


These angles are called vertical angles
because each side of one is an
extension through the vertex of a side
of the ot~1er
Wh~..!!..tl!.~

A bisector is a straight line which divides


a geometric figure into two equal figures.
An angle bisector is a light which divides
an angle into two equal angles.

;
Angle bisector

146 . I 00 Solved Problems in Engineering Mat!tematics (2nd Edition) by Tiong & Rojas
What are the different units of angles?

A regular polygon is a polygon having all


sides equal and a!: angles equal.

1. Degree- The degree is a non-S! unit


approved in 1969 by the General

Table showing th; regular polygons:

Conference on Weights and Measures.


This unit was established by the
Babylonians more than 4000 years
ago. A degree is divided into 60
minutes; a minute is divided into 60
seconds.
2. Radian -The radian is the standard
angular measure in the International
System.of Units (SI). A radian is the
angle between two radii of a circle
which cut off on the circumference an
arc equal in length to the radius.

3. Gon - The gon or centesimal degree is


1/4001h of the full circle. A right angle is
100 gons A gon is divided into 100
centesimal minutes; one centesimal
minutes is divided into 100 centesimal
seconds. This unit is also known as
grad or grade and is used in surveying
works and aircraft navigation.
4. Mil- The mil is ameasure of an angle
which is 1/6400 of the full circle. A right
angle is 1600 mils.
The following is a table showing
different measure of an angle with
corresponding value in one revolution.
Unit
Degree
Radian
Gon
Mil

One Revolution

360
2n
400
6400

Name
Equilateral triangle
Square
Regular pentagon
Reqular hexagon
Regular heptagon
ReQular octaQon
Regular nonagon
ReQular decagon
Regular hectagon
ReQular meqagon.
ReQular googolgon

Sides

Angle

3
4
5
6
7
8
9
10
100
106
100100

6Qo
goo
108
120
"'128.57
135
140
144
176.4
179.99964
"'180

D~Plane

Reentrant angle is the inward-pointmg


angle (anglt= A 1n the f1gure) of the
concave polygon wh1le the other angles
are called salient angles.

.'

The diagonal of a polygon is the line


connecting two opposite vertices. The
number of d1agonals in a polygon can be
calculated using the following formula:

3
4
5
6
7
8
9
10
11

12
100
1000000
10100
n

Name
Triangle
Quadrilateral or TetraQon
Pentagon
Hexaqon
Heptagon
OctaQon
Nonagon
Decagon
UndecaQon
Dodecagon
Hectaqon
MeqaQon
Googoigon
n-gon

A convex polygon is a polygon having


each interior angle less than 180 while a
concave polygon is a polygon having an
interior angle greater than 180

What is a Polygon?

Concave polygon

A triangle is a polygon with three sides.


All the three sides are .contamed in a
plane.
What are the classifications of triangles
according to their sides?

equal and all angles are equal. An


equilateral triangle is also equiangular.
Each interior angle is 60
2. Isosceles triangle- two sides equal
and corresponding two angles are also
equal

Diagonals= -'2(n2

3. Scalene triangle - no two sides are


equal.

3)

What are the classifications of triangles


according to their angles?

where n = number of sides


The interior angles of a polygon are the
angles inside the polygon. The sum of the
mterior angles of the polygon can be
calculated using the following formula:

1. Acute triangle.- each interior angle is


less than a right angle.
2.. Right triangle -one angle is a right
angle.
3. Obtuse triangle - one angle is greater
than right angle.

Sum of interior angles= ( n- 2)180


where n

I he deflection angle of a polygon is the


,mgle subtended by the prolongation of
'n1e s1de to the next s1de.

~;lllll

of dofll)C[IOII cHID IUS

Note: A triangle which is not a right


triangle is known as oblique triangle.
-Both acute triangle and obtuse triangle
are oblique triangles.
What are other types of triangles?

=number of s1des

A polygon is a plane figure with three or


more angles. It has as many sides as
angles. The sides of the polygon are
straight lines. The term "polygon" comes
from Greek words "poly" which means
"many" and "gonia" which means "angles".
Convex polygon

What is a triangle?

1. Equilateral triangle- all sides are

Polygons are named according to the


number of sides or vertices.
Number of sides

Geometry 147

''

360"

1. Egyptian tri;mgle- a triangle with


sides 3, 4, 5 units.
2. Pedal triangle- a triangle inscribed in
a given triangle whose vertices are the
feet of the three perpendiculars,to the
sides from some point inside a g1ven
triangle.
3. Golden triangle- an isosceles
triangle with sides is to its base in the
golden ratio; its angles are 72, 72",
36"

Day 7 - Plane Geometry 149

148 100 l Solved Problems in Engineering_ Mathematics (znct Edition) by Tiong & Rojas
Note: The formulas to use in solving
problems involving triangles are found in
Chapter 9 (Plane Trigonometry) of this
book
What is a quadrilateral?
A quadrilateral is a polygon with four
sides. A quadrilateral is also known as
quadrangle or tetragon.
What are the types of quadrilateral?

1. Square- a regular quadrilateral. All its


sides are equal and all angles are
equal to right angle. A square is a
rectangle with all sides equal.

aD

A= ab
'h

-------

4. Parallelogram - both pairs ot opposite


sides are parallel. Another term for
parallelogram is rhomboid.

A= bh

A Given base and altitude

B. Given two diagonals and their

where: d1 and d 2 are lengths of


diagonals
B. Given four sides and opposite
angles: (Using Bramaguptha's
formula)

A=

...... d1 .....

{-"><fo

9. Cyclic quadrilateral- a quadrilateral


whose vertices lie on a circle

What are the formulas used in solving


-

!!!e areq_of a 9!J.?drilateral?

" 7

where: d1 and ch are lengths of


diagonals

A= absine

C. Given side and included angle:

I ---:

tJ

GLih

A=a sin0

A= a

A= ~(s-a)(s-b)(s-c)(s-d) -abcdcos 0
2

where:

t)

=A+C
2

A.=

--(B + b)h
7

or

t)

a+b+c+d
S=------2

7. Cyclic quadrilateral
5. Trapezoid

C Given two sides and included angle:

1
A= 2(d 1 )(d 2 )

adjacent sides are equal in pair.

-~(d 1 )(d 2 )sine

where: d 1 and d2 are lengths of


diagonals

dz \ ..

For rhombus, 0 is a right angle.

8. Deltoid -concave quadrilateral whose

~(d 1 }(d 2 )sin0

included angle:

7. Kite- convex quadrilateral whose


adjacent sides are equal in pair.

A:=

B. Given two diagonals

5. Trapezoid-- only two sides are


parallel.
6. Trapezium- if no two sides are
parallel.

d~

3. Rhombus

A =bh

3. Rhombus - all sides are equal but no


angle equal to right angle.

aD

A. Given two diagonals and included


angle:

A. Given base and altitude:

2. Rectangle - a right-angled
parallelogram.

1. Square

6. Trapezium (General quadrilateral)

4. Parallelogram or Rhomboid

2. Rectangle

~_:+:_Q
2

Day 7- Plane GeolllEillY 151

150 1001 Solved Problems in Engineering Mathematics (2nd Edition) by Tiong & Rojas
where: n = number of sides

What is Ptolemy's Theorem?


A= .J(s-=a)(s- b)(s- c)(s- d)
where: s =a+ b + c + d
2
The radius of the circle circumscribing
the quadrilateral may be cal<;ulated
using the following formula:

This was named after the mathematician


astronomer, and geographer, Ptolemy
Alexandria.

J( ab + cd)( ac + bd)( ad+ be)

What are the formulas used in solving


the areas and perimeters of other
regular polygons?

r=~--~~~~--~

4R

where: A = area of quadrilateral

2. Regular polygon inscribed in a


circle

"The sum of the products of two pairs of


opposite sides of a convex cyclic
quadrilateral is equal to the product of the
lengths of the diagonals."

perpendicular to the radius of the


circle.
3. Secant of a circle - a !ine cutting the

circle in two places.


4. Diameter of a circle -the longest

chord of a circle that passes through


the center.

cl

5. Radius -the distance from the center

to the circle. It is one-half of the


diameter.
A=

1. Regular polygon

1 . (360)
n
2 nr
2

sm

P =2nrsin(

8. Quadrilateral circumscrib;ng a circle

6. Chord- the segment of a secant


bounded by the circle.

1~0)
secant

where: n = number of sides

Wl:tiit is a perimeter?

A perimeter is the distance around a two-

a
A=
A = rs "'

v'abed

where: s = a + b + c + d

dimensional shape. The perimeter of the


square is the sum of the length of its sides.
The formula for perimeter will vary
depending on the geometric figure or
poly~ on.

1 0

~na 2 cot( ~ )
P=na

What is a circle?

where: n = number of sides

2
What is Bramaguptha's Theorem?

A circle is a plane curve that is the locus


of all points in the plane equidistant from a
given point, called center. The term "circle"
comes from the Latin 'circus", which
refers to a large round or rounded oblong
enclosure in which the famous Roman
chariot races were held. The circle is a
conic section whose eccentricity is zero.

2. Regular polygon circumscribing a


circle

"In a cyclic quadrilateral having


perpendicular diagonals, the perpendicular
to a side from the point of intersection of
the diagonals always bisects the opposite
side"
Bramaguptha (A.D. 598- 665) was a
Hindu astronomer and mathematician who
became the head of the observatory at
Ujjain- the foremost mathematical center
in ancient India.

What are common terms related to a


circle?
A= nr

tanC~O)

1. Arc of a circle- length of circle


between to points on the circle or
between two radii.

P = 2nrtan(. ~0 I
'- n J

2. Tangent of a circle- a line touching


the ctrcle in one place A tangent is

I
i

6. Circumference- the perimeter of a


circle. This is also known as periphery ..
The circumference of a circle is
2n(radius) or n( diameter) .
7. Radian- the measure of an angle
whose arc length is equal to the radius
of the circle.
8. Sector of a circle- area bounded by
two radii and the included arc.
9. Segment of a circle- area bounded
by a chord and the arc subtending the
chord.

~9b 1 Solved Problemsi!l Engineering Mat!:ematics (2~~ Editio~by Tiong & Rojas

3. Inscribe angles subtended by the

10. Central angle --an angle whose


vertex is at the center of a circle and
whose sides are the radii. The central
angle is an angle subtended by an arc.

360

e is in degrees

4. Area of segment of a circle:

'--

=-ts

e =a(a+b)
e = 13 =goo

A.= Area of sector - Area of triangle AOB


The angle 0 is the central angle while
is the angle subtended by chord "x".

What are the for111ulas used in sqjyin.Q_<!

circle1
1. Area of a circle:

A= ltr~

or

A=

6. Secant- Tangent Theorem:

diameter of a circle are right angles.

A= _:<r2e

where:

11. Angle subtended by a chord - an


angle whose vertex is along the
periphery or circumference and its
sides are chords.

Day 7 -Plane Geometry 153

What are useful theorem involving a


circle?

What is an ellipse?

Therefore, the triangle formed in the


figure is a right triangle.
4. Chord Theore;n:

1. If a central angle and a peripheral


angle are subtended by the same arc,
then the central angie is twice as large
as the peripheral angle

ab = cd

Ellipse is a locus of a point which moves


so that the surn of its distances to the fixed
points (foci, denoted as F in the figure) is
constant and is equal to the length of the
major axis.

----r--.. . .

rt02

where: r = radius
D =diameter

2. Circumference of a circle:
C =2nr

or

5. Secant Theorem:

C=:rcD

2, Inscribed angles subtend the same arc


are equal.

-~

Ellipse has two axis of symmetry, a major


axis (2a), betvveen the vertices (V) of the
ellipse, and a minor axis (2b), which
intersect at the center of the ellipse.
Area of an ellipse, A
A= n:ab

3. Area of sector of a circle:

0=13

where:
a = semi-major axis
b = semi-minor axis

1
A= --rc

a(a+b)=c(c+d)

i
A ""-r2e

where:

e is in radians

t,l,

II
,.

154 1001 Solved Problems in Engineering Mathematics (2"d Edition) by Tiong & Rojas
Proceed to the next page for your 7th test.
Detach and use the answer sheet provided
at the last part of this book. Use pencil
number 2 in shading your answer.

,I..
'

,t',

GOOD LUCK I

Topics

~
Mon

~ribia::
Did you know that. .. the symbols + and for plus and minus, respectively were
introduced by German mathematician and
astronomer, Johannes Regiomontanus in
1456!

D
D D
D
D DI
D D

'/-:

Tue

,:'',

<!auote:

Branches of Geometry
Basic Postulates of Euclid
Geometric Elements and Figures
Types of Angles
Units of Angles
Polygons
Classifications of Triangles
Other Types of Triangles.
Types of Quadrilaterals
Bramaguptha's Theorem
Ptolemy's Theorem
Terms Related to Circles
Theorems Involving Circles
1 Ellipses

"Man is like a fraction whose numerator is


what he is and whose denominator is what
he thinks of himself. The larger the
denominator the smaller the fraction."
-Tolstoy

Theory

Wed

Problems

Thu

Solutions

Fri

Notes

Sat

30&: ECE Board November 1998


Find the angle in mils subtended by a line
10 yards long at a distance of 5000 yards.
A.
B.
C.
D.

1
2
2.5
4

307: ECE Board April1999


Assuming that the earth is a sphere whose
radius is 6400 km. Find the distance along
a 3 degree arc at the equator of the earth's
surface.
A.
B.
C.
D

335.10 km
533.10 km
353.10km
353.01 km

:J08: EE Board April199:1


1111' <Hl\Jie subtended by an arc is ?-1". If
llu r<~dllrs ot thr~ circlr~ is 4~, em, turd llw
II'IHJth "' <IIC

A.
B.
C.
D.

16.85 em
17.85 em
18.85 em
19.85 em

309: ME Board April1990


A rat fell on a bucket of a water wheel with
diameter of 600 em which traveled an
angle of 190 before it dropped from the
bucket. Calculate for the linear em that the
rat was carried by the bucket before it fell.

A
B.
C.
D.

950
965
985
995

310: ECE Board November 199:1


Given a circle whose diameter AB equals
2 m. If two points C and D lie on the circle
and angles ABC and BAD are 18 and 36,
respectively, find the length of the major
arc CD.

156 100 l Solved Problems in Engineering Mathematics (2"d Edition) by Tiong & Rojas

A.
B.
C.
D.

1.26 m
1.36 m
1.63m
1.45 m

3u:: A certain angle has a supplement 5


times its complement What is the angle?

A.
B.
C.
D.

67.5
58.5
30
27"

31::&: ECE Board November 1998


Each angle of a egular dodecagon is
equal to

A.
B.
C.

D.

135
150
125
105

313: CE Board May 1997


How many sides have a polygon if the sum
of the interior angles is 1080?
A.
B.

5
6

C.

D.

314: ECE Board March 199&


The sum of the interior angles of a polygon
is 540. Find the number of sides.
A.
B.
C.
D.

12
24
20
48

3:t:Z: EE Board April 1991


From a point outside of an equilateral
triangle, the distances to the vertices are
10m, 18m and 10m, respectively. What
is the length of one side of a triangle?

317: How many diagonals are there in a


polygon of 20 sides?

A.
B.
C.

D.

200
170
100
158

A.

A.
B.
C.
D.

17.75
18.50
19.95
20.50

m
m
m
m

318: ME Board April1999


Find each interior angle of a hexagon.

3::&3: EE Board April1991


The sides of a triangle are 8 em, 10 em
and 14 em. Determine the radius of the
inscribed circle.

A.
B.
C.
D.

A.
B.
C.
D.

90
120
150
180

319: EE Board April1994


Given a triangle, C = 100, a= 15m, b =
20m. Find c.
A.
B.

C.
D.

26m
27m
28m
29m

2.25 em
2.35 em
2.45cm
2.55 em

3::&4: CE Board May 199&


What is the radius of the circle
circumscribing an isosceles right triangle
having an area of 162 sq. em.?

A
B.
C.

D.
3::&0: CE Board November 1994
In triangle ABC, angle A = 45 and C =
70: The side opposite angle Cis 40 m
long. What is the length of the side
opposite angle A?

3::&7: ECE Board March J99&


A circle with radius 6 em has half its area
removed by cutting off a border of uniform
width. Find the width of the border.

12.73 m
13.52 m
14.18m
15.55 m

B.
C.
D.

1.76cm
1.35 em
1.98cm
2.03 em

3%8: ME Board April :199&


The area of a circle is 89.42 sq. inches.
What is its circumference?

A.
B.
C.
D.

32.25
33.52
35.33
35.55

in.
in.
in.
in.

3::&9: ECE Board April 1991


A square section ABCD has one of its
sides equal to x. Point E is inside the
square forming an equilateral triangle BEC
having one side equal in length to the side
of the square. Find the angle AED.
A.
B.
C.
D.

130
"140
150
160

3ZS: EE Board April 1991


The sides of a triangle are 8 em, 10 em
and 14 em. Determine the radius of the
circumscribing circle.

330: CE Board November :1.995


The area of a circle circumscribing about
an equilateral triangle is 254.47 sq. m.
What is the area of the triangle in sq. m?

A.
B.

A.

ii r

3
4

5
6

315: ECE Board April :1991


Fioo the sum of the interior angles of the
vertices of a five pointed star inscribed in a
circle.
A.
B.
C.
D.

A.
B.
C.
D.

_ _ _ _ _ _ _ _ _ _ _ _ _ _ _ _ _ _ _ _ _ _ _.:::D~a::..;yc....:..7_-Plane Geometry 157

150
160
170
180

A.
B.
C.
D.

m
m
m
m

3::&1: CE Board May i995


In triangle ABC, angle C = 70, A= 45,
AB = 40 m. What is the length of the
median drawn from vertex A to side BC?

C.
D.

7.14
7.34
7.54
7.74

em
em
em
em

3261: CE Board May 1996

Two sides of a triangle are 50 m and 60 m


long. The angle included between these
sides is 30 What is the interior angle
opposite the longest side?

A.

~.3m

B.
C.

~.6m

A.

~.9m
~.2m

n
c

D.

31&: ME Board April1999


How many sides are in a polygon if each
interior angle is 165 degrees?

26.1
27.1
29.1
30.1

(l

93."14"
9) l4
90.? 11"
llCI J()'

B.
C.
D.

100.25
102.25
104.25
105.25

3311 CE Board May 1.995


What is the area in sq. em of the circle
circumscribed about an equilateral triangle
with a side 10 em long?
A.
B.
C.

104.7
105.7
106.7

D.

107.7

'if

1~8 1001 Solved Problems in Engineering Mathematics (2"d Edition) by Tiong & Rojas
33:11 CE Board November 199:1
The area of a triangle inscribed in a circle
is 39.19 cm 2 and the radius of the
circumscribed circle is 7.14 em. If the two
sides of the inscribed triangle are 8 em
and 10 em, respectively, find the third side.

A.
B.
C.
D.

11 em
12 em
13 em
14 em

C.
D.

64 1t
16n

3371 ECE Board }lllovember 1993


The arc of a sector is 9 units and its radius
is 3 units. What is the area of the sector in
square units?

A.
B.
C.
D.

12.5
13.5
14.5
15.5

3331 CE Board November 1994


The area of a triangle is 8346 sq. m and
two of its interior angles are 37"25' and
56 17'. What is the length of the longest
side?
A.
B.
C.
D.

171.5 m
181.5 m
191.5m
200.5 in

A.
C.

59.8
89.5
58.9

D.

81:;CJ

B.

Two perpendicular chords both 5 em from


the center of a circle divide the circle into
four parts. If the radius of the circle is 13
em, find the area of the smallest part.
30
31
32
33

cm
cm 2
cm 2
2
cm

A circle having an area of 452 sq. m is cut


into two segments by a chord which is 6 m
from the center of the circle. Compute the
area of the bigger segment.

A.
B.
C.
D.

354.89
363.68
378.42
383.64

sq.
sq.
sq.
sq.

m
m
m
m

339: ECE Board April 199:1


A swimming pool is constructed in the
shape of two partially overlapping identical
circles. Each of the circles has a radius of
9 m and each circle passes through the
center of the other. Find the area of the
swimming pool.
A.
B.
C.
D.

The distance between the centers of the


three circles which are mutually tangent to
each other externally are 10, 12 and 14
units. The area of the largest circle is

727t
23

1t

Determine the area of the quadrilateral


shown. OB = 80 em, AO = 120 em, OD
150 em c;nd q, = 25.

A.

W.5~

B.

21.5cm

c.
D.

n.5~
n.5~

A trapezoid has an area of 36 m 2 and an


altitude of 2 m. Its two bases have ratio of
. 4:5. What are the lengths of the bases?
A.

12, 15

B.
C.

7, 11
8, 10

D.

16, 20

Tf::>,o

;V

'\1

<

A rhombus has diagonals of 32 and 20


inches. Determine its area.
A.
B.
C.
D.

380m 2
2
390m
2
400m
2
410m

360 in
280 in 2
2
320in
400 in 2

If the sides of a parallelogram and an


included angle are 6, 10 and 100,
respectively, find the length of the shorter
diagonal.

A.

Find the difference of the area of the


square inscribed in a semi-circle having a
radius of 15 m. The base of the square lies
on the diameter of the semi-circle.

B.
C.
D.

A.
B.
C.
D ...

2721.66
2271.66
2172.66
2217.66

cm 2
cm 2
cm 2
2
cm

347: CE Board Oc:tober 1997

343: EE Board Marc:h 1998

340: ME Board April 1991

10.63
10.37
10.73
10.23

Find the area of a quadrilateral have sides


12m, 20m, 8 m and 16.97 m. if the sum
of the opposite angles is equal to 225,
find the area of the quadrilateral.
A.
B.
C.
D.

!:!i,

'I

'r'j

I
1!,,,

1
1i

'I'

'j:

100m
124m2
168m2
158m2

i
1,

348: ME Board Oc:tober 1996


ME Board April :1997
The area of a regular hexagon inscribed in
a circle of radius 1 is
A.
B.

c.
D.

,,

1.316
2.945
2.598
3.816

'I

349: EE Board April 1990


2

Find the area (in em ) of a regular octagon


inscribed in a circle of radius 10 em?

3451 CE Board Nove.mber 1996 .

A.
33fn ECE Board April 1998

A.
B.

Problem 346: EE Board October 199:1

A rectangle ABCD which measures 18 em.


by 24 em. is folded once, perpendicular to
diagonal AC, so that the opposite vertices
A and C coincide. Find the length of the
fold.

344: ECE Board April 1998

3351 EE Board Ai"ll'il 199:1

A.
B.
C.
D.

341: ECE Board November 1995

342: ECE Board April 1998

338: CE Board May 1998

334: ECE Board April 1998


The angle of a sector is 30 and the radius
is 15 em. What is the area of the sector in
cm 2?

Day 7- Plane Geometry 159

B.
C.
D.

171.5 cm 2
172.5 cm 2
173.5 cm 2
174.5 cm 2

Find the area of a quadrilateral having


sides AB = 10 em, BC = 5 em, CD= 14.14
em and DA = 15 em, if the sum of the
opposite angles is equal to 225.
A.
B.
('

A.
B.

C.
D.

283
289
298
238

96 sq. em

350: GE Board February 199:1

100 sq. em
94 sq. em
qg sq. em

A regular hexagon is inscribed in a circle


whose 'diameter is 20m. Find the area of
the 6 segments of the circle formed by the
sides of the hfiXilCJOil

llnl"l''l:;;'r'

iiJj!

I 'I!
I

.:

:_
'.l.__ .
~-h

Ill'~

160 1001 ~olved Proble!ns in Engineering Mathematics (2nd Edition) by Tiong & Rojas
A.

B.
C.

D.

36.45
63.54
45.63
54.36

sq.
sq.
sq.
sq.

m
m
m
m

351: EE Board Aprii199:J


Find the area of a regular pentagon whose
side is 25m and apothem is 17.2 m.

A.
B.
C.
D.

li'

355: EE Board March 1999


Determine the area of a regular 6-star
polygon if the innerregular hexagon has
10 em sides.
A.
B.
C.
D.

441.66
467.64
519.60
493.62

II

cm 2
cm 2
cm 2
cm 2

Topics

1075 rn
2
1085 m
2
1080 m
2
1095 m

Mon

TtJe

3S:l: ME Board October 1!.99&

The area of a circle is 89.42 sq. inches.


What is the length of the side of a regular
hexagon inscribed in a circle?
A.
B.

c.
D.

5.533
5.335
6.335
7.335

Theory

[1

in.
in.
iio.
in.

Problems

353: EE Board April1990

Solutions

In a circle of diameter of 10m, a regular


five-pointed star touching its circumference
is inscribed. What is the area of that part
not covered by the star?
A.
B.
C.
D.

40.5
45.5
50.5
55.5

sq.
sq.
sq.
sq.

0
306. B
307.A
308.C
309. D
310.A
311. A
312. B
313. D
314.C
315. D
316. B
317. B
318. B

354: EE Board March 1998


A regular pentagon has sides of 20 em. An
inner pentagon with sides of 10 em is
inside and concentric to the larger
pentagon. Determine the area inside and
concentric to the larger pentagon but
outside of the smaller pentagon.
A.
B.
C.
D.

430.70
573.26
473.77
516.14

cm

cm
3
cm
3
cm

Thu

0
Fri

[]

ANSWER KEY
319. B 332. D 345.8
320. D 333.8 346.A
321. A
334.
347.
322.C 335. B 348.
323.C 336.C 349. A
324.A 337. B 350.0
325.A 338. B 351. A
326. D 339. D 352. B
327.A 340.C 353.
328. B 341.
354. D
329.
342. D 355.
330. D 343.
331. A 344.

I~

Sat

Notes

m
m
m
m

[WedJ
[l

;I

Branches of Geometry
Basic Postulates of Euclid
Geometric Elements and-Figures
Types of Angles
Units of Angles '
Polygons
Classifications of Triangles
Other Types of Triangles
Types of Quadrilaterals
Bramaguptha's Theorem
Ptolemy's Theorem
Terms Related to Circles
Theorems Involving Circles
Ellipses

c
c
c

c
c

c
c

RATING

'

0 ~3-50
c::J
c::J
0

Topnotcher

33-42 Passer

25-32 Conditional
0-25 failed

If FAILED, repeat the test.

:l

il. iIl

II
i

il :'

;,j[,,

162 100.1 Solved Problef!lS in Engineering Mathematics (2nd Edition) by Tiong & Rojas

Day 7- Plane Geometry 163

Ell

Ill
5

Note: A dodecagon has 12 sides, n

9 = (n- 2)(180')
n

8 = (12-2)(180")
12

tan8=-5000
8 = tan

5
= 0.0573
5000

28 = 2(0.0573) = 0.1146"
= .
x 6400 mils
0 1146
360'
20 = 2.037 mils

n rad)
360

1080 = (n- 2)(180)


n=8sides

Ill
S=(n-2)(180")
540=(n-2)(180)

8=36'

n-2 =3

28=72"

n = 5 sides

Let:

c = length of. arc CD

C = 18.85cm

C=r8

360

C = 3oo(190o x 2n: rad)


360

N=-(n-3)

2
20
N =-(20-3)
N = 170 diagonals

a
Note: A hexagon has 6 sides, n = 6.
8= (n-2)(180")

Ell
28 = 360

e =the angle
go - e = the complement of angle e
180- e =the supplement of angle e

Elll

e =120

Let:

C=995cm

n =24 sides

8 = (6-2)(180')

C=1.26m

c =re

15n = 360

C=1(nx~)
.
360

Ell

18+ e + 36 =go

C=45(24~x 2nrad)

= (n-2)(180")

165n = 180n- 360

n2=6

C =335.10km

C=re

165

S=(n-2)(180")

Refer to the first figure:

Ill

S= (n-2)(180')

'looo<..-,.:

Ell

For the same intercepted arc (arc CD),


the value of the central angle is twice that
of the inscribed angle.
If one side of the triangle inscribed m a
circle is equal to the diameter of the
circle, then the said triangle is a right
triangle.

C =r9
C = 6400.(3 x

Theory:

a
n

a= 1so

( --)

=12.

5
0=36

Az=s:B
c =?

By cosine law:

Let:
S = sum of the interior angles of the five

180 - e = 5(90 -e)

vertice!O

180-8 = 450-58
48= 270

e = 67.5'
c

c=Ja +b -2abcosC

s;:; 50-:;; 5(36')

c=~(15) 2

:; - 11l0

c =27

+(20) -2(15)(20)cos100'

!'',1

,j:l,
I'

'. '.I'

""

Day 7- Plane Geometry 1&5

164 1001 Solved Problems in Engineering Mathematics (2nd Edition) by Tiong & Rojas

x. = fc 2 + (at2/ - 2( c)( a/2)GOS B

A= Js(s-a)(s-b)(s-c)

x=~(40) 2 +(15) 2 -2(40)(15)cos65'

A= J16(16- 8)(16 -1 0)(16 -14)


A=39.19cm

x=36.3m

S=

Mf _......_---:c--~40
\
_ ___: 'o B

sinA

4r

By sine law:
sin 30

sin (60 + ~)

10

18

a+ 4.158 + 90 = 180

By sine law:

sin 10

sin45"

40

a=30 m
a/2=15 m

sin 30

10

By cosine law:

S=

2r = .J2x 2.

x =19.95m

11,1
'i :1

II'!;
1

1!:1'
Iii.'
2
-

2(50)(60)co:;30'

lL
II''

1:,.

r
i

ml

b = 10

2
ac:8

It

=----

S=:16cm

sin30"

60
30.06
B=86.38

8+10+14

c = 30.06m

sillB

2r=J2(1W
r=12.73cm

a+b+c

I',,II:

By sine law:

d=Jx 2 +x 2

A+B+C=180
45 + B + 70 = 180

B=65

c = ~ (50) + (60)

Using Pythagorean theorem:

sin 85.842

b =60

c=Ja 2 +b 2 -2abcosC

2
x =18 em

30"

By cosine law:

1
162 = -(x)(x)

a =85.842'

A= -(r<~se)(height)

\l.,ll' .
:[ .

a + ~ +60 + 30 = 180

II
II

39.19 = 8(10)(14)

Note:
Since the isosceles right triangle, is
inscribed in the circle, then its hypotenuse
must be equal to the diameter of the circle

~=4.158'

sinA

II

A= abc

sinC

II

4r
r=7.14cm

sin45 J
a=40 - (
sin70
a-= 30.1m

By sine law:

~,I

A= J16(16 -8)(16 -1 0)(16 -14)

sinAJ
a=c (
sinC

ml

A=39.19cm 2

sinG

A= Js(s-a)(s-b)(s-c)

A=rs
39.19 = r(16)
r=2.45 em

By sine law:

8+10+14

=----

S = 16 em

ml

a+b+c

Day 7 - Plane Geometry 167

166. 100 I Solved Problems in Engineering Mathematics (2"d Edition) by Tiong & Rojas
Note:
Since half of the area was removed, then
the area (A} left is also one-half of the
total area.

30' +a+a=180o

111

a=75

Ill

60' +2a+ 8=360

A=.:!_[ n(R/ J=:_[n(6) J


2

60 + 2(75) + El = 360.

A= 18n
But "A" is also equal to the area of the
small circle. (Refer to figur~
A= nr 2
18n = nr

Bd"

Note: Since an equilateral triangle, then


angles A, B and C are equal to 60.

:f

.A +B+C = 180

c =86.301'

A =-(x) sin A
2
1
2

By sine law:

A= -(10) sin 60

x=6-4.24
A= nr

r=1.76cm

El
= circumference of the circle

C = 2nr
C = 2n(5.335)

By cosine law:

Solving for area of circle:

x=) r 2 + r 2 - 2(r)(r)(cos 9)

A= nr 2

J:JA

::;:-acsinB

2
1

8346 =-(0.609c)(c)sin 56.283)


16,692 =0.5065 c 2
c=181.5

'I

b = 10

2
1

A= abc

.,.- tJs

Atriangle

II'

A= ~(15.59) 2 (sin 60)

--

Jc

sin37.416
a =0.609c

I!

A= ~(x)(x)(sln A)
X

sin86:301
(

Ill

Ill

Note: Since the triangle is an equilateral


triangle, then angles A, B 'and C are each
equal to 60 degrees.

a=

A =104.7cm 2

x=15.59m

A= 7t(5.774) 2

x.=A9) +(9) -2(9)(9)(cos120)

C=33.52in

n.--

4r
r =5.774cm

e= 120

r =5.335in

4r

89.42 = nr 2

43.3 = (10)(10)(10)

360
9=-

sin37.416

'!l'r

A= abc

r=9m

A= nr 2

sin86.301

A=43.3cm 2

254.7 = nr 2

c~

'DA

37.416 +56.283 +C = 180

.1

x=R-r

'

c =?

A= 3r25' = 37.416
B = 5617' = 56.283

r =4.24cm

Let: C

'

9 =150'

A =105.24 m

4r
39.19 = -~O)c
4(7.14)

Note:
Triangle CDE is an isosceles triangle.

l
j

cco14crn

1 .
A =-r 2 e
2
1
A =--.(15)
2

, I'I::if
I r:

2(30. x--J
7t

180'

A =58.9cm 2

lflilll'
,l
,11'1
,

I
j

LLJ
ttl

II:

168 ._LOOl Solved Problems_in Eng_!~eering Mathematics (2nd Edition) by Tiong & Rojas

12 = y+5

Day 7 -Plane Geometry 169

y=7

A =As -2A 1

A 2 = Ac - A1 = 452-84.44
A 2 = 363.56 m 2

5
A= 66-2[;(base)(height)]

A= 66-(7)(5) = 31 cm 2

1
A=-rC
2

A=-(3)(9)=13.5sq. umts
2

Ell

stna =--

4.5
cose=-=60
9

13
o;

=22.62"

Let:
4.5
A = area of the pool
A, = area of triangle ABC
Ac = area of the sector
As= area of the segment (shaded portion.)

2a+0=90
2(22.62) + e = 90

.9=44.76

Let:

1 9=-(13)
1
As =-r
2

r2 +r3 =10-tEq.2
r1 +r3 =14-tEq.3

2[ 44.76x-1t ]

A1 = area of the smaller segment


A2 = area of the bigger segment
Ac = area of the circle
As = area of the sector
A, = area of the triangle ABC

r1 +r2 =12-tEq.1

Let:
A area of the shaded part
As == area of sector ABC
A, = area of triangle COB

180"

=_!.(9) 2 (120"

Subtract Eq.3 by Eq.2:

(r1 +r3 )T(r2 +r3 )=14-10

A 5 =66cm 2

A. =Ac -A 1
1
1
=-r 2 (29)--r 2 sin29

r1 -r2 =4-+Eq.4

A. = 1tr

452 = 7tr

x-n:-}
180"

--<W sin120"
2

r=12

A, =49.75 m 2

Add Eq.4 and Eq.1:


(r1

~r2

cos8=~=~
r

)+(r1 -r2 )=10+4

'

=21t<W -2(49.75)

r1 =8
r2 =12-8=4
r3 =10-4=6

~c

By Pythagorean theorem:

Let: A = area of the largest circle

{13) 2 =(5) 2 +(y+5) 2

A=

1tr/ =1t(8)

=2(1tr 2 )-2A.

9=600

2r1 = 16

'

A'=2A- -2A.

12

1 2
1 2
A 1 =A. -A, =-r (29)--r stn29

=_!.(12) 2 (120/' x-1t-}

2
1
--(12) 2 sin 120"
2
.A 1 ::::84.44

= 647t

144=(y+5) 2

.'~

180'

A =409.44 m 2

Ill

x/2

x/2

:11!

170 1001 Solved Problems in Engineering Mathematics (2"d Edition) by Tiong & Rojas

r2 = x2

+(;r

Day_J-:()l_ane Geometry 171

II

i',l

A=

15 =1.25x
= 13.416

-abcdcos 2 9

Let:
A, = area of the semi-circle
Az = area of the square
A = difference of A1 and Az
1
A=A 1 -A 2 =-1tr 2 -x 2

1
A =-1t(15) 2 -(13.416) 2

(22.07- 5)(22.07 -14.14)

Ill
24

o~c
X

24-x

5
4

29 + 100 + 100 = 360

9=8o

Substitute Eq.2 in Eq.1:

By cosine law:

36=;(~b+b}2)

d2 =6 2 +10 2 -2(6)(10)cos80.
d = 10.73

x =324+576-48x+x

4
5
36 =-b+-b
5
5
9
36=-b
5
b=20m
4
a=-(20)==16m

Ell

Length of the fold= 2y = 2(11.25)


Length.of the fold= 22.5cm

'!1:'1

:-=oo

i ~I
,:1
r'

By secant law:

a =5

.AO BO =CO DO
120 80 =C0150
C0=64 em

(s- a)(s- b)(s- c)(s- d)

Let:
A = area of the quadr lateral
A, = area of triangle 1100
Az = area of triangle FOG

abcdcos 2 9

A =A 1 -A 2

a+b+c+d
S=----

(18.75) 2 =(15) 2 +y 2
y = 11.25

V(

li

llillllil!ili

L 'Ill,
1',

l1

~~~

A=

+(y)2

t'J

Ill

X= 18.75

=(;r

c =15

x2 =18 2 +(24-x) 2

A=100cm 2

Total interior angle= 360'

d=-J18 2 +24 2 =30

-(5)(14.14)(15}(10)cos 2 112.5"

Total interior angle= (n- 2)180


=(4-2)180

A~B

18

A= 1(22.07-15)(22.07 -10)

10

A=-(a+b)h~Eq.l

a=-a~Eq.2

A =173.44

x2

ii'I'

(s- a)(s- b)(s -c )(s- d)

Ill

Substitute:

A=-d 1 d2
2

s=

5+14.14+15+10

A= -(32)(20)
2
A=320in 2

A +C 225"
9=--=--

2
0=112.5"

=22.07

1
1
A = -(AO)(OO)(sin E - -(BO)(CO)(sin 9)
2
2
1
A= -(120)(150)(sin '5")
2
1
--(80)(64 )(sin 25")
2
A= 2721.66 cm 2

,,'i
,,

ir
I'

':

;I,!

,,

172 1001 Solved Problems in Engineering Mathematics (2nd Edition) by Tiong & Rojas .

II

A1

(21)

Let:
.
r 2 Sln9

Ah = area of the hexagon


At = area of triangle
Ac = area of the circle
A = area of the 6 segments of the circle

A 1 =.2,(1) sin60

Day 7 -Plane Geometry 173

A0~'~--7b-=~2~0---

A 1 =0.433

360
9=-=60

oo

Atotal =6At = 6(0.433)


(s- a)(s- b}(s- c)(s- d)
A=
abcdcos 2 9
a+b+c+d
S=----

2
S=

A total = 2.598

A 1 =(;} sin9

360
9=--=60

1
2
At =-(10) sin6o

= 7tr2
89.42:;; 1tr 2
Ac

2
At =43.3m 2

r = 5.335in
Ah = 6A 1 = 6(43.3)

12 +20+8 + 16. 97 = 28.485

By cosine law:

Ah = 259.8 m 2

2
5
9- A+C
- . -_ 22 =112.5
- 2
2
Substitute:
(28.485 -12)(28.485- 20)
A=

-(12)(20)(6)(16.97)cos 2 112'.5"
A=168m

Let:
Atota~

Let:

=area of the hexagon

A:: 1tr 2 - Ah

Atotat :;; area of the octagon


At = area of triangle

A= 54.36

360
9=--=45
8

I (28.485- 8){26.485 -16.97)


A1

x=Jr 2 +r 2 -2(r)(r)cos9

A =Ac -Ah

= 1t(10)2 - 259.8

2(5.335) cos60

x =5.335in

m2

Ill

.
=(1)r 2 stn9

A 1 =-(10) sin45

. A 1 = 35.355 cm 2
b=25
A total =SA, = 8(35.355)

Let:

A 10181 = 262.84 cm 2

Ah = area of the hexagon


At = area of triangle

A 1 =-(base}(height)=-(25)(17.2)
A 1 =215m 2

At = area of triangle
360
9=-=60

x = b(5.335)

A= 5A 1 = 5(215)

A=1075 m 2

.Let:
A = area not covered by the star
As = area of the star
Ac = area of the circle

360"
29=-

8=36c

8/2 18
9/2

"'-~ ________ [}liy_7,_ !'lal\f?

174 . 100 1 Solved Problems in Engineering Mathematics (2nd Edition) by Tiong & Rojas

Qeometry 175

9+-+a =180

A= n(;-(base)(height)J = n(;xh J

2
36 +18 +a= 180
a= 126~

A =;xCt:neJ
nx 2
A=-4tan9

By sine law:
sin18"

sin126"

X=

180

180

9=-=~=30"

n
6
6(10) 2
A1=--4tan30"
A 1 = 259.8 cm 2

x = 1.91 m
1

A 1 = -rx sine= -(5)(1.91 )sin 36


A 1 =2.806m 2
X2=

x= 10

20

\ll

A=Ac -10A 1
180" 180
9=--=--=36"
n
5

=7tr 2 -10A 1
= 1t(5)2 -1 0(2.806)
A=50.5m 2

Note:
By inspection, the triangle must be
equilateraL

Let:
A1 = area of the inner pentagon
A2 = area of the outer pentagon
A = area of the shaded portion

Ell

A 2 = -x 2 sin 60

A =A 2 -A 1

2
1

2
nx ___
nx
=--2
. _1_

=
x/2
x/2

2h

10

x/2

tan9=-=~

il'

= -(10) 2 sin 60
2

4tane 4tane
5(20) 2 5(10) 2

llil!l

II,
I'
I

A 2 =43.3

---

4 tan 36 4tan36
A= 516.14 cm 2

A=A 1 +6A 2
= 259.8 + 6(43.3)

A =519.6 cm 2

Ell

h=-2tane
360" 180'
9=-=2n
n
Let: A = area of an n sided polygon

Let:
A1 = area of the hexagon
A2 = area of the triangle
A = total area

IIIII:
, ... 1

I
d

178 l 00 l Solved Problems in Engineering Mathematics (2nd Edition) by 'riong & Rojas

' ' ~ ;'"T(.,

:,;. '

<' '> "<- ~

.0<

'

< f',

"

'

Topics

.,.. .

!v!on

[QJ
Tue

,, ... .', '' ~ :. ""N


.;. ...&

"v .,.

"'

Theory

V.

>"" . .;. .:. '-0 ~ ., ~ ~ ~ .... ' ,..:,:,. ~i'"

>

"!' ~

:<-.;. '

'

,,.~

,' "'

'<- ~

i'< .., " :

" " :!<"'

<

* ;t ~

-:;..

:>.,

)<""

* ~ :t 't ~

"<-...

: " ..

....

"~ ;:

0
0
0

~**~'1<-i<'..f$-<

.,:.

."

~ '

Problems
" ..,_

"

'

"'!" "' "

< ..... "" ,,: .;. ",.

'" ' '

~~

o>;,ifj!>.-$Mio;

~,

"

"

~-." 4: ~

'''

'f ., "

" ,. . . . . '

' ~ .... ~ "-'<- ~ ., '

-~

"

~ 'i

>X~

., ~ .;, ::> f .'<' '< J' ::>

I .. ,

~ l ""' "? ~

,,

> <" '>. ~

;<

v.

~ ~

.:

Solutions

'o

' ,_ ,< ~ ~ .. ~

$- .., ' <- :<- ' "'

Notes

" ~ .. <:".,, ' ... ;: "->:..

'

"0 ..

'!'" ' / , ' ; ,

'

~"'. " .

.,

0
0

Wed

Thu

0
Fri

Sat

~"' ' ' <., ".

\'' ""v,

What is a polyhedron?

~'<">0~{~~-.j.-l:>

,, ""' ' ~- ..,


;, <

<'- <

"<-

o.

+ ,.

''

0'

<

<

-~1<-<v<

;. ' '

<

::>'I<"' .., < ' ...

'< ~-$ <f: -i. 0 ~A,~ '$ ~

~ ~

?: ~ "" ~

!>

<

z<

' ' -~ ,,:

p..,."'

<

'
<>

. "'""

"<-.(<< ...

.,. * .,.

,'

~ ~ ~

<

.<

> ->

->.

< ": '

;<

~ P > ~- ~ -., P 4

.;. ~

<o

~-" :f '

'"'"''

~-~"'

:.c-~~{'<'->i"'~~''"=<'-""<'

.,.,.,,..

.....

.. ~,,.~,~~~*0"~1'< ~.,~~-$~'~"'
4 {

<

< '

>'

-.!"<-~"~"-"'~~...

~ ~.,., '~'."'

Polyhedron is a solid whose faces are


plane polygons. The term "polyhedron"
comes from the Greel "poly" for "many"
and "hedron" for "base or face". Polyhedra
are named according to the number of
faces. A polyhedron with six faces is
called hexahedron while the one with
three faces is called tetrahedron.

r<.j

~ -< : 0. ,_,.. 6:.-:f ,. ~ t '<

~. .,0:<<:<~0{"'>'0"'J<{o.{"'~"'~A""'

'

Polyhedrons
Platonic Solids
Properties of Platonic Solids
Prisms
Cubes & Parallelepiped
Cylinders
Pyramids
Cones
Frustum of Pyramid or Cone
Prismatoid
Sphere
Zone
Spherical Segment & Sector
Spherical Pyramid & Wedge
Torus
Ellipsoids

"j'

-l<

...

'

. . . . ,. ""'

-~.?

Convex polyhedron is one that lies


r~ntirely on one side of a plane that contain
;111y of its faces while a concave
polyhedron has at leart one face so
located that there are parts of the
l'olyhedron on both sides of a plane
Lorrt.Hninn that face. If a line that connects
.'"Y lw<J JH >rrrts on the surface of a
1" >lylir dr '>ll 1s cornpletely rnside or on the
i'' 1lylu rlr orr, lilr, polylwdron IS convc:x,
"ll111wr:;r rl r:; <.<llC.lVI'

-~.~,~~~""~"'

,,,l"?'

~;.

,. ~

,;

~ .,

.,.

....,

'

'

'

"

~ ~

'

'

...,

Every polyhedron in three-dimensional


space consists of (two-dimensional)
faces, (one-dimensional) edges and
(zero-dimensional) vertices. The faces of
the polyhedron intersect each other along
the edges, which meet at the corners
known as vertic.es
What are the 5 regular polyhedrons?
A regular polyhedron is a solid with all its
faces identical regular polygons. There are
only five regular convex polyhedra,
namely the tetrahedron, the hexahedron
(cube), the octahedron, the dodecahedron
and the icosahedron. These solids are
also known as Platonic solids in honor of
Plato (427- 348 B.C)
The following are the Platonic solids with
its properties:

Day 8- Solid Geometry 181

180 1001 S~lved Problems in Engineering Mathematics (2"d Edition) by Tiong & Rojas
1.

3.

Tetrahedron

5.

Octahedron

Icosahedron

Surface area, A:
A=2(ab+bc+ca)
Right prism is one which has its lateral
faces perpendicular to the base.

Faces
No. of faces
No. of edges
No. of vertices
Polygon angle
Sum of angles
Radius of
circumscribed sphere

Triangle
4
6
4
60
180

Faces
No. of faces
No.ofedqes
No. of vertices
Polyqon angle
Sum of anqles
Radius of
circumscribed sphere

eJ6
4

--

12

Total area

2e

Volume

4.

Trianqle
20
~----1
12
60
300

~~2(5+J5)

~t+3.J5

./3

5e 2.f3

Volume

Dodecahedron

Total area

!e3.J2
3

~e3.J2
12

2.

e.J6
-

Total area

e2.f3

Volume

e../2
___?__

Radius of inscribed
sphere

e.J6

Radius of ir.scribed
sphere

Faces
No. of faces
No.ofedqes
No. of vertices
Polygon angle
Sum of anqles
Radius of
circumscribed
sphere
Radius of inscribed
sphere

Trianqle
8
12
6
60
240

~~3 (3 + .J5)

What is a prism?

Hexahedron

A prism is a polyhedron with two faces


(bases) parallel and congruent and whose
remaining faces (lateral faces) are
parallelograms.

e
Faces
No. of faces
No. of edges
~_of vertices
,.l'_l)lygon angle

Square
6
12
8

~ofanqies

270

e~

.!-Radius of inscribed
sphere

6e 2
eJ

Total area
------

~..Qt:l_angle

goo

Radius of
circumscribed sphere

~olume

Faces
No. of faces
No. of edges
-~o. of vertices

~-

~-~---

Sum of angles
-Radius of
circumscribed

Cube is a prism with all six faces a


square. It is a regular hexahedron.and one
of the five Platonic solids.

Pentagon
12
30
20
108
324

Rectangular parallelepiped is a prism


with all six faces a rectangle.

~--

Total area

--

Volume

71 c

~J3(1 + JS)
a

A= h Pb
where:
B = area of the base
h =altitude of prism
Pb =perimeter of base
Oblique prism is one in which the lateral
faces are not perpendicular to the base.

~,Ih
Volume of prism, V

V =Bh = Ke

A= e Pk

Volume, V:

Gt/
'I ( 1 1)

Lateral area of prism, A

Lateral area of prism, A

-------------
e:~

V=Bh

'---------"b

~ J.~9.~~~-~I5

!--.-------------

Volume of prism, V

--

~_her~---~ ----------------Radius of inscribed


sphere

Ih

V =abc

7 J~,)
I

where:
B = area of the base
h = altitude of prism

182 100 l Solved Problems in Engineering Mathematics (2"d Edition) by Tiong & Rojas

=area of right section


=lateral edge
PK =perimeter of right section

K
e

183

Oblique cylinder is one which has its


cylindrical surface not perpendicular to the
base.

1
3

V=-Bh

Volume of cylinder, V

=altitude of cylinder

K = area of right section


e = lateral edge
What is a pyramid?

What is a c_ylinder?
Cyiir;ader is a solid bounded by a closed
cylindrical surface and two parallel planes.

Right cylinder is one which has its


cylindrical surface perpendicular to the
base.

+rz + Rr)

where:

What is a frustum of a pyramid? r~

8 = area of the base


h

V = 1t3h (Rz

R = radius of the lower base


r = radius of the upper base
h = altitude of the frustum of a cone
Frustum uf a pyramid is a portion oft~- .
what is a prismatoid?
pyramid included between the base and a
section parallel to the base.
Prismatoid is a polyhedron having for
bases two polygons in parallel planes and
for lateral faces triangles or trapezoids with
one side lying in one base, and the
opposite vertex or side lying in the other
base of the polyhedron.

where:

s(h1 + hz: h3 + h-)

or

where:
B = area of the base
h = altitude of cone

V = 8h = Ke

v "' ~( 81 + 82 + ~8182)
where:
81 and 82 = area of the bases
h = altitude of the frustum of cone

Volume of cone, V

v"'

_,}

,_

Truncated prism is a portion of a prism


contained between the base and a plane
that is not parallel to the base.

Volume of the frustum of cone, V

--lh

Pyramid is a polyhedron of which one


face, called the base, is a polygon of any
number of sides and the other faces are
triangles which have a common vertex.

~l.L

Volume of the frustum of pyramid, V

Ih

)"

v =~(81 +82 + ~8182)

where:
81 and 82 = area of the bases
h = altitude of the frustum of pyramid

Volume of pyramid, V

Volume of cylinder, V
V=_!Bh

V=8h
Lateral area of cylinder, A

What is a frustum of a cone?

where:
B = area of the base
h = altitude of pyramid

Frustum of a cone is a portion of the


cone included between the base and a
s.ection parallel to the base.

A = (circumference of base )(h)


What is a cone?
where:
8 = area of the base
h = altitude of cylinder

Cone is a solid bounded by a conical


surface (lateral surface) whose directrix is
a closed curve, and a plane (base) which
cuts all the elements.

Volume of the prismatoid, V

V=s{A 1 +4Am+A2 )
where:
At and A2 = end areas
Am = area at the midsection (at half oft)
L = distance between end areas
This formula is known as the Prismoidlll
Formula.

184 lOQ 1 Solved Problems in Engineering Mathematics (2nd Edition) by Tiong & Rojas

Dav 8- _SoU!iGeometrv 185

What is a sphere?

lh

Sphere is a solid bounded by a closed


surface every point of which is equidistant
from a fixed point called center.

Cross-section of torus:
:+-- generating axis

J,~~=t)

Volume of spherical segment, V

Volume of torus, V
Volume of spherical pyramid:

rch2

V =-{3R-h)
3

Great circle

V=

540
Volume of sphere, V

What is a spherical sector?

4
3

Spherical sector is a solid generated by


rotating a sector of a circle about an axis
which passes through the center of the
circle but which contains no point inside
the sector.

V=-nR 3
Surface area of sphere, A

Jh

A=4'ltR 2
What is a zone?

What is a spherical wedge?


Spherical wedge is a portion of a sphere
bounded by two half great circles and an
included arc.

where : A

What is a spherical segment?


Spherical segment is a solid bounded by
a zone and the planes of the zone's base.

where:
R = distance from axis to center of
generating circle
R = radius of generating circle

What is an ellipsoid?

Volume of spherical sector:

1
3

A= 2nRh

A =4tt2Rr

Ellipsoid (Spheroid) is a solid formed by


revolving an ellipse about its axis.

V=-AR

Area of zone, A

Lateral area of torus:

where:
E = spherical excess of polygon ABCD
in degrees

Zone is that portion of the surface of a


sphere included between two parallel
planes.

.!h

V 2'lt 2Rr2

itR E

= area of zone

Volume of spherical wedge:


V= 1tR39

maJOr axiS

270
Volume of general ellipsoid, V

What is a spherical pyramid?

What is a torus?

Spherical pyramid is a pyramid formed


by a portion of a surface of a sphere as
base and whose elements are the edges
from the vertices of the base to the center
of the sphere.

Torus is a solid formed by revolving a


circle about a line not intersecting it.

4
3

V =-ltabc
Prolate spheroid is a solid formed by
revolving an ellipse about its major axis.

4
2
V ::,...nab

"

..

186 1001- Solved Problems in Engineering Mathematics (2nd Edition) by Tiong & Rojas
Oblate spheroid is a solid formed by
revolving an ellipse about its minor axis.

4 2
V=-na b
3

Topics

Proceed to the next page for your 8th test.


Detach and use the answer sheet provided
at the last part of this book. Use pencil
number 2 in shading your answer.

Polyhedrons
Platonic Solids
Properties of Platonic Solids
Prisms
Cubes and Parallelepiped
Cylinders
Pyramids
Cohes
Frustum of Pyramid or Cone
Prismatoid
Sphere
Zone
Spherical Segment & Sector
Spherical Pyramid & Wedge
Torus
Ellipsoids

tvlon

[QJ

GOOD LUCK I

Tue

~ribia:

Theory

Did you know that ... the Pascal's triangle


which is used to determine the coefficient
of a binomial expansion was named after
the French mathematician, philosopher
and physicist E31aise Pascal but die:! not
claim recognition for the discovery
because such triangle was first introduced
by a Chinese mathematician, Chu Shihchieh in 1303 !

Problems

0
0

Solutions

Notes

~uote:
"Mathematics consists of proving th~ most
obvious things in the least obvious way."

0
0
0
0

Wed
Thu
Fri

Sat

31!0&: ME Board October 1.99:1

A circular piece of cardboard with a


diameter of 1 m will be made into a conical
hat 40 em high by cutting a sector off and
JOining the edges to form a cone.
Determine the angle subtended by the
sector removed.

- George Polya

A.
B.
C.
D.

358: CE Board May 1995


A sphere having a diameter of 30 em is cut
into 2 segments. The altitude of the first
segment is 6 ern. What is the ratio of the
area of the second segment to that of the
first?

. A.
B.
C.
D.

144
148
152
154

4:1
3:1
2:1
3:2

359: CE Board November :199&


:li>S7: CE Board November :1994

What is the area in sq. rn of the zone of a


spherical segment having a volume of
1470.265 cu. rn if the diameter of the
sphere is 30 m?

,.ll

465.5 m
5G5.5 m 2
f!G~>-~i m 2

IJ

!l~lfifJ

rn

If the edge of a cube is increased by 30%,


wj how much is the surface area
increased?

A.
B.
C.
D.

W%
~%
~%

W%

LL

188 1001 Solved-Problems in Engineering Mathematics (2nd Edition) by Tiong & Rojas
3601 ECE' Board April1995

3&51 CE Board November 1997

Each side of a cube is increased by 1%.


By what percent is the volume of the cube
increased?

Find the volume of a cone to be


constructed from a sector having a
diameter of 72 em and a central angle of
210".

A.
B.

c.
D.

1.21%
2.8%
3.03%
3.5%

3&11 ECE Board November 199Z


Given a sphere of diameter, d. What is the
percentage increase in its diameter when
the surface area increases by 21 %?

A.
B.

c.
D.

5%
10%
21%
33%

3&ZI ECE Board November 199Z


Given a sphere of diameter, d. What is the
percentage increase in. its volume when
the surface area increases by 21 %?

5%
10%
21%

~%

A.

3&31 EE Board October 1991


How many times do the volume of a
sphere increases if the radius is doubled?

A.
B.
C.
D.

D.

cm
3
cm
3
cm
3
cm

A conical vessel has a height of 24 em and


a base diameter of 12 em. It holds water to
a depth of 18 em above its vertex. Find the
3
volume (in cm ) of its content.
188.40
298.40
381.70
412.60

368: CE Board May 1995


A.
B.
C.
D.

4 times
2 times
6times
8 times

3&41 CE Board May 1997


A circular cone having an altitude of 9 m is
divided into 2 segments having the same
vertex. If the smaller altitude is 6 m, find
the ratio of the volume of the small cone to
the big cone.
A.
B.
C.
D.

0.186
0.296
0.386
0.486

What is the height of a right circular cone


having a slant height of
diameter of 2x?

A.
B.

2x
3x

C.

3.317x

D.

3.162x

J10x

and a base

3&9: CE Board November 1995


The ratio of the volume to the lateral area
of a right circular cone is 2:1. If the altitude
is 15 em, what is the ratio of the slant
height to the radius?

A.
B.

5:6

C.
D.

5:3
5:2

5:4

I}:

14
15

375: CE BOard November 1995

B.

C.
D.

4
5

cm
3
cm
cm 3
3
cm

3731 EE Board April1993 :.


In a portion of an electrical railway .cutting,
the areas of cross section taken every 50
mare 2556, 2619, 2700, 2610 and 2484
sq. m. Find its volume.

D.

A.
B.
C.

3.50
3.75
4.00
4.25

3761 CE Board May 199&

What is the volume of a frustum of a cone


whose upper base is 15 em in diameter
and lower base 10 em. in diameter with an
altitude of 25 em?
3018.87
3180.87
3108.87
3081.87

A circular cylinder with a volume of 6.54


cu. m is circumscribed about a right prism
whose base is an equilateral triangle of
side 1.25 m. What is the altitude of the
cylinder in meters?

D.

37Z.: EE Board April 199Z

c.

te

413
3
311 ft
313 ft3
391 ft3

c.

A.

A.
B.

A.
B.

1~

The volume of the frustum of a regular


triangular pyramid is 135 cu. m. The lower
base is an equilateral triangle with an edge
of 9 m. The upper base is 8 m above the
lower base. What is the upper base edge
in meters?

A.
B.
C.
D.

the triangular base and have the height of


8.6 ft., 7.1 ft. and 5.5 ft. respectively.

13

371: CE Board November 1995

3&71 CE Board November 199&

A.
B.
C.
D.

A regular triangular pyramid has an


altitude of 9 m and a volume of 187.06 cu.
m. What is the base edge in meters?

C.

Find the volume of a cone to be


constructed from a sector having a
diameter of 72 em and a central angle of
150"
5533.32
6622.44
7710.82
8866.44

3701 CE Board November 1994

A.
B.

12367.2 cm
3
13232.6 cm
3
13503.4 cm
3
14682.5 cm

3&61 CE Board May 1998

A.
B.
C.
D.

Day 8- Solid Geometry 189

522,600 m 3
520,500 m 3
3
540,600 r'n
534,200 m 3

374r ME Board April199&


Determine the volume of a right truncated
triangular prism with the following
definitions: Let the corners of the triangular
base be defined by A, B and C. The length
of AB = 10ft., BC =9ft. and CA =12ft.
The sides A, 8 and C are perpendicular to

A circular cylinder is circumscribed about a


right prism having a square base one
meter on an edge. 'T:he volume of the
cylinder is 6.283 cu.m. Find its altitude in
meters.

A.
B.
C.
D.

4.00
3.75
3.50
3.25

377: CE Board November 1997


The bases of a right prism are hexagons
with one of each side equal to 6 em. The
bases are 12 em apart. What is the volume
of the right prism?
A.
B.

c.
D.

1211.6cm3
2211.7 cm 3
1212.5 cm 3
11.22.4 cm 3

378: EE Board April199&


Two vertical conical tanks are joined at the
vertices by a pipe. Initially the bigg~r tank
is full of water. The pipe valve is open to
allow the water to flow to the smaller tank
until it is full. At this moment, how deep is
the water in the bigger tank? The bigger
tank has, a diameter of 6 ft and a height of
10 ft, the smaller tank has a diameter of 6

190 1001 Solved Problems in Engineering Mathematics (2"d Edition) by Tiong & Rojas
ft and a height of 8 feet. Neglect the
volume of water in the pipel.ine.
A.

'1200

B.

C.~
D.~
~79:

The central angle of a spherical


wedge is 1 radian. Find its volume if its
radius is 1 unit.

A.

2/3
1/2
3/4.
2/5

B.
C.
D.
~80:

A regular octahedron has an edge


2m. Find its volume (in m 3 ).

A.

3.77
1.88
3.22
2.44

B.

c.
D.
~81:

CE Board May 1996

38~: An ice cream cone is filled with ice


cream and a surmounted ice cream in the
form of a hemisphere on top of the cone. If
the hemispherical surface is equal to the
lateral area of the cone, find the total
volume (in cubic inches) of ice cream if the
radius of the hemisphere is 1 inch and
assuming the diameter of hemisphere is
equal to the diameter of the cone.

A.
B.
C.
D.

Topics

D
tvlon

[QJ

3.45
3.91
4.12
4.25

D
D D
~ D
D Di

384: ME Board April1997


A cubical container that measures 2
inches on a side is tightly packed with 8
marbles and is filled with water. All 8
marbles are in contact with the walls of the
container and the adjacent marbles. All of
the marbles are of the same size. What is
the volume of water in the container?
0.38 in 3
2.5 in 3
3.8 in 3
4.2 in 3

A.
B.
C.

D.

Tue

Polyhedrons
Platonic Solids
Properties of Platonic Solids
Prisms
Cubes and Parallelepiped
Cylinders
Pyramids
Cones
Frustum of Pyramid or Cone
Prismatoid
Sphere
I Zone
I Spherical Segment & Sector
j Spherical Pyramid & Wedge
Torus
. Ellipsoids

Theory

VVed

Problems

Thu

Solutions

Fri

Notes

Sat

A mixture compound of equal parts of two


liquids, one white and the other black, was
placed in a hemispherical bowl. The total
depth of the two liquids is 6 inches. After
standing for a short time, the mixture
separated, the white liquid settiing below
the black. If the thickness of the segment
of the black liquid is 2 inches, find the
radius of the bowl in inches.

~8S:

A.
B.

A.

D.

7.33
7.53
7.73
7.93

~82:

CE Board November 1996

B.

C.

CE Board May 1997

The corners of a cubical block touched the


closed spherical shell that encloses it. The
volurrte of the box is 2744 cubic em. What
volume in cubic centimeter inside the shell
is not occupied by the block?

C.
D.

ANSWER KEY
356. A
357. B
358.A
359. D
360.
361. B
362. D
363. D
364. B
365.

2714.56
3714.65
4713.56
4613.74

c
,,

The volume of water in a spherical tank


having a diameter of 4 m is 5.236 rna
Determine the depth of the water in the
tank.

A.
B.
C.
D.

1.0
1.2
1.4
1.8

366.
367.
368.
369.
370.
371.
372.
373.
374.
375.
~

c
c
B
D
A

c
A

'

376.A
377. D
378.A
379.A
380.A
381. A
382. A
383. B
384.C
385.

,,

RATING

c:J
c:J
c:J
0

26-30 Topnotcher
20-25 Passer
15-19 Conditional
D-14 Failed

If FAILED, repeat the test.

11111;:'11,11:.,11111

192 1001 Solved Problems in Engineering Mathematics (2nd Edition) by Tiong & Rojas

A= 21trh
= 21t(15)(6)
A= 565.49 m 2

:II

.193

@
AI

I I

2\ =8
=[~)
=(
v1 r,
\)
3

v2

V 2 =SV1

A2= 1.21A1

Ill

A2 -r~;l2
-

x=~ =f<Sol-(40)

A1 . d1

x=30
c1 = circumference of the circle
c2 = circumference of the base of the
cone
C = length of arc

C=C 1 -C 2
= 21t(50)- 21t(30)
c =401t

:: =

A1

d2 =1.1d1 .

2--m:.h 1

~
.

401t =(50)0
401t 180
0=--x50
1t

X2

X1

[::)' t:J'

'

~
..

X1

X2

1 01
V2 =[2) =( ' ] =1.0303
1

X2

V1

=(1.21~1

A1

312

_
-1.331

'

Thus, V2 is increased by 3.03%

= circumference of the base of the

cone
C = length of arc of the sector
C =C 1 -C 2

re =2n:r-2:n:x

Thus, V2 is increased by 33.1%

36(210 x-1t-) =21t(36)-27t(X)


180"
x=15cm

ml!f\

C9
r,

..

=circumference of the circle

v2 =1.331V1

II

h=~

x
vV2 = 1.0303
v,

= 0.296

Let:
c1
c2

X2=1.01 X1

1404 = 45h 2 -h 3
h=6 m

-[A

2 )
v2 - -

X1

1th2
1470.265 = -[3(15)-h]

312

\:E;}

~=[2)2 =(1.3']2 =1.69

II

~
= [-~) = (~)
v2
112
9

Thus, A 2 is increased by 69% .

1th2
V=-(3r-h)

X2=1.3x1

A 2 =1.69A 1

0=144'

=11

Thus, d2 is increased by 10%

X1

X1

C=re

A 2 :A 1 =4:1

= 27tr- 21tX

J:: =r~~

A2 2--m:.h 2 24
-=--=-=4

II'

h = .[362 -15.

r2"' 2r,

h=32.726cm

I
illl

~!

194 I 00 i Solved Problems in Engineering Mathematics (2"d Edition) by Tiong & Rojas
1

V = -nx 2 h = -1t(15) 2 (32.726)

V =7710.88cm 3

24 18
r =4.5

Let:
c1 = circumference of the circle
= circumference of the base of the
cone
C = length of arc of the sector

50.625 = 35.074 + 0.433x

C=C 1 -C 2
re=2nr-2nx
36(150" x-n-)=2n(36)-2n(x)
180
x=21cm

=-/r 2 -x 2

h = -/36 2 -21 2
h=29.24cm

1
V =-nx 2 h

V =-n(21) 2 (29.24)

3
V = 13,503.44 cm 3

0 =(x-3)(x +12)
x=3 em

mJ

Ox

=;-1[1;x SinSJh

187.06 =

2 .

~(; }

Let:

---

h=3x

AL = nrl
1

A1 =area of the lower base


A2 = area of the upper base
A 1 = nr,Z = n(5) 2 = 25 n

'

A2

=nr/ =n(7.5) 2

= 56.25n

'

h
V=-(A 1 +A 2 + -/A 1 A 2

V =-nr 2 h

Note: 8 = 60, since equilateral triangle.


Substitute:

-=2~asgiven

AL
V=2AL

Let:
A1 = area of the lower base
A2 = area of the upper base

..!_ 'n/\h=2'7tt:_L
3
rh =6L
L

15

r
L

2
1
2
2
A 2 = -(x) sin60o = 0.433x

2
V-=

h
:l

(A 1 +A 2 +-}A 1 A 2

25
V = - ( 25 1t + 56.25 n +-} (25n )(56.25n:))

3
V

A 1 =-(9) sin60=35.074

----2

T25

sin 60(9)

ED

h 2 = 9x 2

x=12m

+x2

10x 2 =h 2 +x 2

e "'60, since equilateral triangle.

By Pythagorean theorem:

&

0 = x 2 + 9x -36

=h2

+
2

V=-Bh

..

(...fi0x)2

+ 3.897x

Note:

ml

c2

h1= 24

8[35074+0.433x
135 =-

3 )(35.074)(0.433x

V =381.7cm 3

'

Substitute:

1 2
1
2
V = -nr h 2 = -n(4.5) (18)
3
3

195

By ratio & proportion:

=3108.87 cm 3

Note:
Since the areas being cut is at the same
distance, then the given solid is a
prismatoid. And since there are five
different areas being cut then, this solid is
equivalent to two rrisrnatoids

Day 8 - Solid Geometry 197

196 1001 Solved Problems in Engineering Mathematics (2nd Edition) by Tiong & Rojas
h
V =-(A 1 +4Am +Az)

6
where:
A1 = area of the first base (base 1)
A2 = area of the second base (base 2)
Am = area of the middle section
h = distance between base 1 and base 2

v =(~d

1t
nd,Z
2
v1 =-1 ( - ) h1=-(6)
(10)

)h

6.283=.::.(1.4142) h
4
h=4m

Let
V1 = total volume of the two prismatoid

v,

x2 =r 2 +r 2 -2r 2 cos9

Ill

(1.25)2 = 2r 2 - 2r 2 cos 120


1.5625 = 3r 2
r=0.72m

h21

A
a+b+c

9+12+10

S=---

1h3

s =15.5

nx 2
A=-4tan9

6
2

6 6
A= ( )( ) = 93.53 cm 2
4 tan 30
V = Ah = 93.53(12)

V:-:1122.4cm 3

A =)s(s-a)(s-b)(s-c)
A= j15.5(15.5- 9)(15.5 -12)(15.5 -10)

A = 44.039 ft

= (10)

18.849

6.54 = 1t(0.72)h
h=4m

~ =(h~
94.247

18o 180
9=-=--=30

V=1tr 2 h

t::

= 75.398 ft 3

V = V1 - V2 = 94.247-75.398

x=6
Let:
A = area of one base
x = length of each side of the base

By cosine law:

...-...~

1(1td/)
1t
2 (8)
V2 =- - h 2 =-(6)
3
4
12

By similar solids:

360
9=-=120
3

V1 =522,600m 3

..."

v =18.849 ft 3

100
+-[2700 +4(2610)+ 2484]
6

.li... a
. c ....... B
. . .

12

V1 = 94.247 tt

V2

100
=-[2556+4(2619)+2700]

h1

h =~200ft

1tR 3 9

V=-2700

V=

180
1t(1 )3 1 rad x -)
(
1t
~ 180
270'

270

V =- cubic units
3

Top view

V =A(h 1 +h; +h3

d=~(1)

+(1)

Ell

d=1.4142

v = 44.039( 8 6 + 7~1+ 5.5 J


V=311ft 3

Let:

v1 =total volume of the bigger tank

V2 = total volume of the smaller tank


V := volume left in the bigger tnnk

. '.!.~

J i

198 1001 Solved Problems in Engineering Mathematics

J2

(2nd

Edition) by Tiong & Rojas

Day 8- Solid Geometry 199

Jth2

V=-x 3

Ell

V=-(3r-h)

x=2

7th2

J2(2)3
V=-__:___:_-

5.236 = --(3(2)- h)

5 = 6h

V=3.77m 3

.J x 2 + x 2 + x 2

d=

.[3(14)2

d = 24.24 em

'

d=

r=12.12cm

h=1m
x=2

ED

ml

V=Vs -Vc

Let:
r = radius of each marble
Vw = volume of water inside the cube
Vc = volume of the cube
Vm = volufl1e of each marble

,y

v=(;nr .)-x3
3

v=(;n(12.12) )-14

V =4,713.55em 3

x =4r
2 =4r

Let:

r = 1t2

vb = volume of the black mixture


Vw = volume of the white mixture

!Vw =Vc -8Vm


Let:
Ac = surface area of the cone
Ah = surface area of the hemisphere
V1 = total volume
Vc =volume of the cone
Vh = volume of the hemisphere

Vb =Vw
V 1 =Vb+Vw
V 1 =2Vw

Yw

V.,

Ac =Ah
V 1 =2Vw
2

[ 1r(h )
)
-1r(h1-(3r-h
-(3r-h 2 )
1) = 2 3
3
2

1t(6)2

[ 1t( 4}2

-(3r-6)=2 -

-(3r-4)

3
36(3r-6) = 32(3r-4)

108r-216=96r-128
12r = 88
r=7.33in

Vw =(2)

Substitute:

=x

~L =2(4\rl)

-s(;nr
-s(;n(0.5)
3

=3.8in 3

Ill

2
L = 2r = 2(1)
r=1

L =2in

h=~
= )(2)2 -(1)2
h = 1.732in

Let:
V = volume inside the sphere but outside
the box
V. = volume of the sphere
Vb = volume of the box

V1 =Vc+Vh

1(4 nr
V 1 = 1 nr h+;

3J

v, =(;}1) {1.732)+ :1t(1)

vw = x3

2744 = x 3

V1 = 3.91in 3

14 em

,,

I,

;,.,,lr

J
-~"Et.

202 1001 Solved Problems in Engineering Mathematics (2nd Edition) by Tiong & Rojas

Topics

0
0

Mon

Tue

Theory

0
0
0

Problems

Solutions

Notes

~
VVed

Thu

0
0
Fri

Sat

What is Trigonometry?
Trigonometry is the study of triangles by
applying the relations between the sides
and the angles. The term "trigonometry"
comes from the Greek words "trigonon"
which means "triangle" and "metria"
meaning "measurements.

What are the two branches of


trigon(lmetry?
Trigonometry is divided into two branches,
namely:

'i

!i
Plane Trigonometry deals with triangles
in the two diiT!::nsions of the plane.
Spherical Trigonometry concerns with
triangles extracted from the surface of a
sphere

I
j

Plane Trigonometry
Solutions to Right Triangles
The Pythagorean Theorem
Special Triangles
Solutions to Oblique Triangles
Law of Sines & Cosines
Law of Tangents
Trigonometric Identities
Exponential Form of Identities
Other Parts of Plane Triangle
Radius of Inscribed Circle
Radius of Circumscribing Circle
Properties of Triangles
Points in a Triangle
Conditions for Congruency
Conditions for Similarity
What are the two general classifications
of plane triangles?
There are two general types of triangles,
namely:
Right triangle - a triangle that has a right
angle.
Oblique triangle- a triangle that does not
have a right angle. Acute triangle and
obtuse triangle :;~re oblique triangles.
Isosceles triangle (triangle with two sides
er;ual and two angles equal) could be a
right triangle or an oblique triangle.

'
204 1061 Sohred Problems in Engineering Mathematics (2nd Edition) by Tiong & Ko]as '
What are solutions to a plane right
triangle?

Day 9- Plane Trigonometry 205

3. Special Triangles .

a
A. Egyptian triangle:

sinA

LJ.

1. Fundamental Trigonometric
Functions
side opposite

= sinB = sinC

1603).

te
co

a2

= b2 + c 2 - 2bccosA

b =a2 +c2 -2accosB

e2 = a2 + b2 - 2abcosc

side adjacent
side opposite

_ hypotenuse
sec e - .
.
s1de adJacent
hypotenuSe
esc e = --:-''-'------,...
side opposite

Pythagorean theorem states that "In a


right triangle, the sum of the squares of
. the sides is equal to the square of the

/"1

hypotenuse
0

L_ja

'2~
~.1

This solution to an oblique triangle is


used when the given are two sides and
the included angle.
This was demonstrated by the French
mathematician Francois Viete (1540-

.J3

1603).
What are solutions to a plane oblique
triangle?

This was first described by a Danish


mathematician and Physician Thomas
Fincke (1561-1656) in 1583.

1. Laws of Sines
This solution to an oblique triangle is
used when the given are:

_a_-b_
a+ b

A. two angles and one opposite side

or

= tan 2(A-B)

secA= cosA

2.

cscA= sinA

Even-odd identities
sin(-e) "".~sine.
cos'(-9) =cose
tan(-e) =~tan a
cot(-e) =-cote
sec(-e)

=sece

~
c

sine= cos(90~e)
cose = sin(90- 9)
tane

=cot{90- 9)

cote= tan(90 -e)


seQe ::;;cs9(90- e)

esc a... s~~(9o -.e)


4. Pythagorean relations
sin 2 A+ cos2 A =1

tan

1+ cot2 A cse2 A

(A+B)

Trigonometric identities are equations


that express relations among ttigonometric
functions which are true for all values of
the variables involved.

a +'b*c

3. Cofunction identities

il

II
f

The following are the trigonometric


identities:

1+tan2 A "'sec2 A

5. Sum of angles formulas


sin(A+B)= sinAcosB+c:oSAsinB
cos(A +B)= cosAcosB-sinAsinB
tan( A+ B)= tan A+ tanB

1-tanAtanB

I
t,l

What are trigonometric identities?

This was demonstrated by Ptolemy of


Alexandria in 150AD.

-~1.____

B. two sides and an opposite angle

.b
2

1
.cosA=secA

esc( -9) =-esc

2. The Pythagorean Theorem


Pythagorean theorem is the most
renowned of all mathematical
theorems. It is considered as the most
proved theorem in mathematics. This
was formulated by Pythagoras .(c.580
- c 500 B.C.) about 500 B.C.

cotA

3. Laws of Tangents

C. 30 - 60 Right Triangle

1
=tanA

sin'A= cscA

tan A"' cotA

A. two sides and the included angle or


B. three sides
This was demonstrated by the French
mathematician Francois Viete (1540-

tane = side opposite


side adjacent

This solution to an oblique triangle is


used when the given are:

B. 45- 45 Right Triangle

.
side opposite .
s1n 9 = -:---'-'--hypotenuse
side adjace. nt
cos e = ---=--,-hypotenuse

1. Reciprocal relations

2. Laws of Cosines

side adjacent

- 11

Day 9 - Plane Trigonometry 207

20& 1001 Solved Problems in Engineering Mathematics (2nd Edition) by Tiong & Rojas
triangles. This phenomenon is known as
the ambiguous case.

11. Difference of two functions

6. Difference of angles formula

2cos~(A + B)sin~(A- B)

sin(A- B)= sinAcosB -cosAsinB


cos( :A B)"' cos A, cos 8 + sin A sin 8

sin A- sinS=

tan(A .. B)= tl;inA.~tan B



1+tanA tans

cos A- cosB = 2sin1(A+ B)sin.!(A- B)

tan A- tanS= _s_in_,(_A_-_B..:.)


cosAcosB
Note: sinp = sin(180- f3)

7. Double angle formulas


12. Product of two functions

sin2A = 2sinA.cosA
cos2A = cos2 A - sin2 A
tan2A = 2tanA
1tan2 A

What are the other parts of general


triangle?

2sinAsinB =cos( A B)- cos( A +B)


2sinAcosB =sin( A+ B)+ sin( A -B)
2cosAcosB =cos( A+ B)+ cos( A -B)

1. Height of a general triangle

8. Powers of functions
What are the exponential forms of the
fundamental trigonometric functions?

sin 2 A"" .2(1-cos2A)

cos 2 A
2

tan,

The following are the trigonometric


functions expressed in terms of
exponential functions and the imaginary
unit.

,t- C;QS2A
.,, .
1+cos2A

9. Functions of half angles

sin A
1+cos A

10. Sum of two angles

co~AcosB

ArRIANGLE

a+b+c

cotx = i( eix + e-ix}


ex -eix

R=

2. Equilateral triangle

c
2

where: e = base of the natural logarithm


i = imaginary unit

' .1
1
cosA+cosB"" 2cos-(A +B)cos-(A -B)
2
. 2
sin(A+B)
. .
.
t 8=
tan A +an

1. General triangle

2. Median of a general triangle

sin A+ sinB = 2sin1(A +B)cos.!(A -B)

let R be the radius of the circumscribing


circle and r be the radius of the inscribed
circle.

a+b+c
2
Note that he is perpendicular to side c.

tanx = -i(eix -e-ix}


ex + e-ix

. 2

What are values of the radius of


inscribed circle and the radius of
circumscribing circle in a triangle?

where: s =

A l~cos.A
cos-=

tan~= 1- cosA
2
sin A

=./~b[1-(a:bJ]

h -. 2Jf>(s- a)(s'-b}(s- c:)


. cc

sinx = eix- eix


2i
COSX = eix + e-x

sin~ =l~cosA
2

tc

=~(1+cos2A)
2
,

What is an ambiguous case?

me

If you are given a triangle with two sides


and an opposite angle known, you can
solve one of the two unknown angles
using the law of sines. Since sin e = sin
(180- 8), a unique solution is obtained
only in right triangles. When rlmlinCJ with
oblique triangles, you will f111d lw"

c
2
2
2
=1.J2a
+ 2b~2
. c.

3. Angel bisector of a general triangle

abc
4ATRIANGLE

208 100 I SOlved Problems in Engineering Mathematics (284 Edition) py Tiong & Rojas

,,.aJi

.... a,/3
R=3

6
3. Right triangle

Day 9 Plan.!!!9.!?nOI!!_et!)!'~~J.

--------------

.~

a+b -+ c an d a 1s
th e s1de
where: s = -

fi.--------

2
physically tangent to the circle

A= y\s- a)(s- b)(s- c)(s -d)


Note: A+ C = 1so
B + 0 = 180"

A= .Js<s- a)(s- b)(s c)

1. Given two diagonals and an


included angle

where: s= a-t:b+c
2

Ptolemy's Theorem states that The sum


of the two pairs of opposite sides of a
convex quadrilateral inscribed in a circle is
equal to the product of th1:1 lengths of the
diagonals"

4. Triangle inscribed In a circle

ac + bd "' did2

A=

Whi!t~l.f!m.Q..rtant ~arQJ.!2~

5. Triangle circumscribing a circle

AL
A=

___.JD

1.

The sum of two sides of a triangle is


greater than the third side and their
difference is less than the third side.

J(s :a)(s -b)(s- c)(s- d) .. abcdcos2 9

.2.

The perpendicular bisectors of the


sides and the blsectors of the angles
of a triangle, meet in points which are
the center of the circumscribing circle
the inscribed circle, respectively.

3.

The altitudes of a. triangle meet in a


point.

4.

The median of a triangle are


concurrent at a point which is 2/3 of
the distance from any vertex to the
midpoint of the opposite sides.

where:

a+b+c+d

A=rs

S=-----

2. Given 2 sides and Included angle

O='~~= B+D
2
2

where: s= a+b+c
2

3. Cyclic quadrilateral (All vertices lie


on a circle)

6. Triangle with escribed circle

~
r

Using Heron's Formula: Named after


Heron of Alexandria (1'1 Century AD)

_
d

t ...._

3. Given 3 sides

This theorem w<)s named after the


geographer, mathematician and
astronomer. Ptolemy or Claudh.1s
Pto:~maeus (<:.100- c.16U A.D.) of
Alexandria.

triangle?

. ......_WIT
A ..
....
l

A=tabstne

sine

1d2

abC
A=. 4r

1. Given bllse and altitude

Id

2. Given 4 sides and 2 opposite angles

What are the formulas for the area of a


triangle?

LJ

where : d1 and d2 are diagonals of a


quadrilateral

....

A= r(s-a)
/./

Circumcenter- the point of concurrency


of the perpendicular
bisectors of the sides of a
triangle.

!I

!['
210 .1 00 1 Solved Problems in Engineering Mathematics ~2nd Editioll) by Tiong & Rojas
lncenter- the point of concurrency of the
angle bisector of a triangle.
Orthocenter - the point of concurrency of
the altitudes of a triangle.

Proceed to the next page for your 9th test.


Detach and use the answer sheet provided
.at the last part of this book. Use pencil
number in shading your ansWer.

GOOD LUCK!

Topics

What are the conditions for two


triangles to be congruent(:::~)?
Two triangles are congruent if the
elements of one triangle are equal to the
corresponding parts of the other triangle.
The following are the conditions for
congruency:

1.

Two sides and the included


angle are equal.

2.

Three sides are equal.

3.

On~

side and adjoining angles


are equal.

D
D

~ribia:
Did you know that... Pythagoras was the
first to connect mathematicS and music. In
fact, he is credited with discovering octave
and the fifth interval of a note !

Tue

<muote:
"Do not worry about your difficulties in
mathematics, I assure you that mine are
still greater."

Theory

The following are the conditions fbr similar


triangles:
Two corresponding sides are
proportional; included angles are
equal.

2.

Three sides are proportional.

3.

Two angles are equal.

[g
Wed

D
D D
D D

- Albert Einstein

What are the conditions for two


triangles to be similar?

1.

Plane Trigonometry
Solutions to Right Triangles
The Pythagorean Theorem
Special Triangles

Solutions to Oblique Triangles


La"'\S of Sines & Cosines
Law of Tangents
Trigonometric Identities
Exponential Form of Identities
Other Parts of Plane Triangle
Radius of Inscribed Circle
Radius of Circumscribing Circle
Properties of Triangles
Points in a Triangle
Conditions for Congruency
Conditions for Similarity

Mon

Problems

Thu

Solutions

Fri

Notes

Sat

386: ECE Board April :1999


Sin (B -A) is equal to _ _ , when B =
270 degrees and A is an acute angle.
A.
B.
C.

D.

-cosA
cos A
-sinA
sinA

387: ECE Board "pril :1999


2
If sec A is 5/2, the quantity 1 -sin 2 A is
equivalent to

A.
B.
C.
D.

2.5
1.5
0.4
0.6

:188: ECE Board April :1999


(cos A) 4 - (sin A) 4 is equal to _ _ .

1\

r:o~;

ll

C()~; :)/\

~~Ill ~)/\

D.

sin 4A

389: ECE Board April :1999


Of what quadrant is A, if sec A is positive
and esc A is negative?

A.

IV

B.

II
Ill
I

C.
D.

390: ME Board October :1996


Angles are measured from the positive
horizontal axis, and the positive direction is
counterclockwise. What are the values of
sin B and cos B in the 4th quadrant?

A.
B.
C.
D.

sin B > 0 and


sin B <0 and
sin B > 0 and
sin' B < 0 and

cos
cos
cos
cos

B<0
B<0
B>0
B>0

'1/\

. . t!

212. 100 t Solved Problems in Engineering Mathematics (2"d Edition) by Tiong & Rojas

Da~ Plan_eTrigonometry

:!91: ECE Board November 1998


Csc 520 is equal to

397: EE Board April1992


Find the value of A between 270 and 360
if 2 sin 2 A- sin A =1.

B.
C.
D.

A.
B.
C.

A
B.

403: CE Board November 199:!


Find the value of y in the given: y = (1 +
cos 28) tan 8.

D.

cos 20
esc 20
tan 45
sin 20

:!921 ECE Board Aprii199:J


Solve for 8 in the following equation: Sin
28 =cos 8
A.
B.
C.
D.

30
45
60
15

:J93: CE Board November 1993


If sin 3A = cos 68, then
A.

B.
C.
D.

A+ B = 90
A+ 2B =30
A+ B =180
None of these

394: EE Board October 1996


So lye for x, if tan 3x = 5 tan x.

A
B.
C.
D.

20.705
30.705
35.705
15.705

39S: EE Board October 1997


If sin x cos x + sin 2x = 1, what are the
values ofx?
A.
B.
C.
D.

32.2, 69.3
-20.67", 69.3
20.90, 69.1 o
-32.2, 69.3

396: EE Board April1991


Solve for G is esc (11G -16 degrees)=
sec (5G + 26 degrees).
A.
B.
C.
D.

7
5
6
4

degrees
degrees
degrees
degrees

C.
D.

300
320
310
330

A.
B.
C.
D.

:J98: CE Board November 199:!


If cos 65 + cos 55 = cos 8, find 8 in
radians.
A.
B.
C.
D.

0.765
0.087
1.213
1.421

A.

fj ).

B.
C.
D.

8/11
8/19
8/15
8/17

c.
D.

6
3

410: EE Board March 1998


Solve for x in the equation: arc tan (x + 1)
+ arc tan (x- 1) = arc tan ( 12).

B.

sin 2 8

C.

sin

D.

sec e

e sec 28

406: ME Board October 199S


Simplify the expression

A.
B.

cos 8
cos e

C.
D.

sin 8
sin 8

411: ECE.Board November 1998


Solve for A for the given equation cos 2 A =
1 - cos2 A.
A.
B.
C.
D.

45,
45,
45,
45,

125,
125,
135,
150,

225,
225,
225,
220,

335
315
315
315

degrees
degrees
degrees
degrees

If tan x = _.!_ tan y = _.!_ what is the value

A.

3'

R.

of tan (x + y)?

IJ

'1/7

412: ECE Board April1991


Evaluate the following:
sin oo +sin 1 o +sin 2 + ... +sin 89 + sin90
cosoo +cos 1 +cos 2 + ... + co.s89 + cos90

402: CE Board November 199:l't

yo

2'

1.5
1.34
1.20
1.25

sec 8- (sec e) sin 28

407: ME Board April1998


112
Arc tan [2 cos (arc sin [(3 ) I 2]) is equal
to

A.
B.
C.

D.

If sec 2A = _ __.:!___ , determine the angle


sin 13A

A in degrees.

so

D.

sin 8
cos 8
tan 8
cot 8

0.149
0.281
0.421
0.316

A.

40_1: EE Board March 1998

B.
C.
D.

A.
B.
C.

Simplify the equation sin 8 ( 1 + cot 8)

4/3
5/4
4/5
3/4

A.

7t

400: EE Board October 1991

B.

30"
45
60
90

(2x) + arc tan (x) = -

40S: ME Board April1996

The sine of a certain angle is 0.6, calculate


the cotangent of the angle.

A.

2
2
2
2

A.
B.
C.
D.

409: ECE Board March 199&


Solve for x in the given equation: Arc tan

sin 8
cos 8
sin 28
cos 28

Find the value of sine+ cos e tan e


cose

Find the value of sin (arc cos

B.
C.
D.

408: EE Board October 1992


Evaluate arc cot [2cos (arc sin 0.5

404: CE Board May 1992

399: CE Board November 1992

A.

1/6
2
1

213

n/3
n/4
)[/16
11/?

A.
B.
C.

D.

413: ECE Board April1991


Simplify the following:
cos A +cosB
sin A+ sinB
- - - - , - - - + ---,----=sin A -sinB
cosA-cosB
A.

1
0
45.5
10

Day 9 - Plane Trigonomet!Y 215

ZI 4 1001 Solved Problems in Engineering Mathematics (2"ct Edition) by Tiong & Rojas
B.

sinA

C.

D.

cosA

A.
B.
C.
D.

A.

39.49
35.50
30.74
42.55

B.
C.

D.

19.7",
20.1,
21.1.
22 3",

307.4
309.4
321.8
319.2

mph
mph
mph
mph

4141 ECE Board April1991


2sin8cos8-cos8
Eva Iuate: -------;:,....-----,;o--2
2
1- sine+ sin 8 - cos 8
A.

sin 8

B.

cos 8

C.

tan 8

D.

cot 8

D.

If sin A= 2.5J 1x, cos A= 3.06x and sin 2A


= 3.939x, find the value of x?
0.265
0.256
0.562
0.625

B.
C.

D.

A.

A.
B.

6 and 12
3 and 9
5 and 11
4and10

C.
D.

C.
D.

A.

B.

so

3.68
4.03
5.12
4.83

422: ME Board April

54.23
48.23
42.44
46.21

D.

427: CE Board November 1997


Points A and B are 100 m apart and are of
the same elevation as the foot of a
building. The angles of elevation of the top
of the building from points A and Bare 21"
and 32 respectively. How far is A from the
building in meters?
A.

B.
C.
D.

An aerolift airplane can fly at an airspeed


of 300 mph. If there is a wind blowing
towards the cast at at 50 mph, what should
be the plane's compass heading in order
for its course to be 30? What will be the
plane's ground speed if it flies in this
course?

The captain of a ship views the top of a


lighthouse at an angle of so with the
horizontal at an elevation of 6 meters
above sea level. Five minutes later, the
same captain of the ship views the top of
the same lighthouse at an angle of 30
with the horizontal. Determine the speed of
the ship if the lighthouse is known to be 50
meters above sea level.

m
m

24 ft, 53.13
24 ft, 36.87
25 ft, 53.13
25ft, 36.87"

A.
B.
C.

D.

The angle of elevation of the top of tower


B from the top of tower A is 28 and the
angle of elevation of the top of tower A
from the base of tower B is 45. The two
towers lie in the same horizontal plane. If
the height of tower B is 120 m, fine! the
height of tower A.

C.

m/sec
m/sec
m/sec
m/sec

429: ME Board April1997

B.

66.3 m
79.3 m
872 m

0.265
0.155
0.169
0.210

An observer wishes to determine the


height of a tower. He takes sights at the
top of the tower from A and B, which are
50 feet apart, at the same elevation on a
direct line with the tower. The vertical
angle at point A is 30 and at point B is
40. What is the height of the tower?

4261 CE Board November 1997

A.
B.

259.28
265.42
271.64
277.29

4281 ECE Board November 1991

A.

:i99~

90.7 m

A wire supporting a pole is fastened to it


20 feet from the ground and to the ground
15 feet from the pole. Determine the length
of the wire.and the angle it makes with the
pole.

c.

A man standing on a 48.5 meter building


high, has an eyesight height of 1.5 m from
the top of the building, took a depression
reading from the top of another nearby
building and nearest wall, which are
and
respectively. Find the height of the
nearby building in meters. The man is
standing at the edge of the building and
both buildings lie on the same horizontal
plane.

m
m

4251 ME Board November 1994

A ship started sailing S 42.35' W at the


rate of 5 kph. After 2 hours, ship B started
at the same port going N 46.20' W at the
rate of 7 kph. After how many hours will
the second ship be exactly north bf ship
A?
A.
B.

418: ME Board April1991

m
m

angle of elevation of the sun is 61 . If the


pole is leaned 15" from the vertical directly
towards the sun, determine the length of
the pole.

421: GE Board August 1994

c.
o. go

76.31
73.31
73.16
73.61

ECE Board April 1994


A pole cast a shadow 15 m long when the

Two triangles have equal bases. The


altitude of one triangle is 3 units more than
its base and the altitude of the other
triangle is.3 units less than its base. Find
the altitudes, if the areas of the triangles
differ by 21 square units.

D.

30"
45
60

ECE Board April 1998

~;

C.

If conversed sin 8 = 0.134, find the value


of8.

so

D.

B.

417: CE Board May 1994

A.
B.

C.

364m
374m
384m
394m

4201 ECE Board November 1998

32.47
33.68
34.12
35.21

416: ECE Board November 1996

A.
B.
C.
D.

A man finds the angle of elevation of the


top of a tower to be 30. He walks 85m
nearer the tower and finds its angle of
elevation to be so. What is the height of
the tower?
A.

Solve for the value of "A" when sin A = 3.5


x and cos A= 5.5x.

C.

42~:

Points A and B 1000 m apart are plotted


on a straight highway running East and
West. From A, the bearing of a tower C is
32 W of N and from B the bearing of C is
26 N of E. Approximate the shortest
distance of tower C to the highway.
A.
B.

4151 ECE Board April1994

A.
B.

419: ECE Board April 1998

D.

D.

85.60 feet
92.54 feet
110.29 feet
143.97 feet

4~0:

ME Board April199~

A PLDT tower and a monument stand on a


level plane. The angles of depression of
the top and bottom of the monument
viewed from the top of the PLOT tower at
13 and 35" respectively. The height of the
tower is 50 m. Find the height of the
monument

Zll 1001 oSolved Pr~blems in Engineering Mathematics (2"d Edition) by Tiong & Rojas

29.13 m

B.

30.11 m

A.
B.
C.
D.

c. 32:t2m'
D. ~ 33.51 m

240
420
320
200

431: ECE .Boart November 1:998


If an equilateral triangle is circumscribed
about a circle of radius 10 em, determine
the side of the triangle.
A.

Topics

D
D
Mon

34.64 em
64.12 em
36.44 em
32 ..10 em

B.
C.
D.

Tue

oz: EE .Board Oetober 1997

The two legs of a triangle are 300 and 150.


m each, respectively. The angle opposite
the 150m side is 26". What is the third
side?

A.
B.

197.49
218.61
34{78
282.15

C.
D.
43~

Theory

~
Wed

EE Board October 1997

Problems

[=]
ra,,

Thu

Solutions

Fri

r-Jotes

Sat

C.
D.

120m
130m
125m
128m

386. A
387.
388. B
389. A
390. D
391. B
392.A
393. B
394. A
395.
396. B
397. D
398. B

434: EE Board October 1:997


The sides of a triangle are 195, 157 and
210, respectively. What is the area of the
tria~~gle?

A.
B.
C.

D.

73,250
10,250
14,586
11 ,260

sq.
sq.
sq.
sq.

-----:----~

ANSWER KEY
B.

Plane Trigonometry
Solutions to Right Triangles
The Pythagorean Theorem
Special Triangles
Solutions to Oblique Triangles
Laws of Sines & Cosines
Law of Tangents
Trigonometric Identities
' Exponential Form of Identities
Other Parts of Plane Triangle
Radius of Inscribed Circle
Radius of Circumscribing Circle
Properties ofTriang les
Points in a Triangle
1 Conditions for Congruency
Conditions for Similarity

D
DI
D DI

m
m
m
m

The sides of a triangular lot are 130 rn.,


180m and 190m. The lot is to bl'l divided
by a line bisecting the longest side and
drawn from the opposite vertex. Find the
length of the line.

~-~---------

units
units
units
units

435: ECE Board April1998


The sides of a triangle are 8, 15 and 17

unies. If each side is doubled, how many


square units will the area of the new
trioogle be?

399. D
400.A
40'1. B
402. D
103.
404.C
405.A
406. B
407. B
408.A
409. 8
410. B
411.

412. A

413. A
414. D
415. A
416. B
417.
418.A
419. B
420. D
421. B
422.
423. D
424.A

425. D
426. B
427.A
428.C
429. B
430. D
431. A
432.C
433.
434.C
435.A

c '

~------------.
RATING

0
0

13-50 Topnotchel'

33-42 Passer

I [ ] 25-32 Conditional

0-25 Failed

IfFAILED, repeat the test.

rull

!l,i(
218 lOOi Solved Problems in Engineering Mathematics (2nd Edition) by Tiong & Rojas

II

Ill

sin (270. -A)= sin 270' cos A


In the 4th quadrant:

3A =90' -68

+cos A= 1
1-sin 2 A=cos 2 A

- 12 --------

A+2B=30'

ml

5/2

ml

Complementary angles have the same


values of their sine functions. Thus, the
other angle is equal to: 90" - 20.9" = JU.:

esc(11G-16" )=sec(5G+26')

tan 3x = 5 tan x
tan (2x + x) = 5 tan x
tan2x+tan x
- - - - - = 5 tan x
1-tan 2x tan x
tan 2x +tan x = 5 tan x- 5 tan 2x tan 2 x

Thus, sin B < 0 and cos 8 >0

sin(11G-16')

--

cos(5G+26')

cos(5G+26' )=sin(11G-16'

)~Eq.1

Note: sine = cos (90 - a )


Let: 9 = 11G-16"

tan 2x = 4 tan x -5 tan 2x tan 2 x

lliiiiill
cos 4 A-sin 4 A=cos 2 Acos 2 A
- sin 2 A sin 2 A
= cos 2 A(1- sin 2 A)
2

esc 520' =esc (520' - 360' )

- sin A(1- cos A)

=cos'2A-~

-sin 2 A+~
= cos 2 A -sin 2 A

=esc160'

sin(11G-16' )=cos(106' -11G)~Eq.2

4~=( 2 ~ J(1+5tan 2 x)

Substitute Eq.2 in Eq.1:

5G+26' =106' -11G


16G=80'
G=5'

Substitute:
2 sin 8cos8=cosa
2sin8 ;,1

!llllllillll

IDI

sin x cos x + sin2x = 1 ~ Eq.1

sin 2 A- 0.5 sin A= 0.5

Note:
2 sin x cos x =sin 2x
sin x cos x = 0.5 sin 2x ~ Eq.2

By completing square:

,,

2 sin 2 A- sin A= 1

In the 4th quadrant:


hypotenuse
c
, =adjacent side b
hypotenuse
c
c
escA=
=-=-opposite side -a
a

Cbs..,(5G+26' )=~(106' -11G)

. X= 20.7048

Note: sin 29 = 2 sin 9 cos 9

-a

4 tan x =tan 2x (1 + 5 tan 2 x)

2=14tan 2 x
tan x = 0.377964473

esc 160' = C$C 20'

sin 29 =cos 9
b

sin (11G -16' ) =cos (90 -(11G -16)

4-4tan 2 x =2+10tan 2 x

esc160' =esc(180' -160')

Ell

Note: cos 2A = cos2 A- sin2 A

4 tan x =tan 2x + 5 tan 2x tan 2 x

1-tan 2 x

Thus esc 520 = esc 20

cos 4 A- sin 4 A= cos 2A

secA=

=20.9'

A =30' -28

-b
b
.
Opposite side
s1nB=
=-=-hypotenuse
c
c
adjacent side
a
cos B .
hypotenuse
c

sin 3A =sin (90'' - 68)

-b

sin(270' -A)=-cosA

sec A
1-sin 2 A=0.4

2x=41.8'

sin3A =cos6B

-$in A cos 270'


= (-1 )cos A- sin A (0)

sin 2 A

Day 9- Plane 'I'l"igonometry 219

Substitute Eq.2 in Eq.1:

sin9=0.5

0.5 sin 2x +sin 2x = 1

8=30'

1.5 sin 2x = 1
sin 2x = 0.6667

(sin A- 0.25) 2 = 0.5 + (0.25) 2


(sin" A- 0.25) 2 = 0.5625
sin A -0.25 = 0.75
sin A =-0.75 + 0.25

1' 1111''1'
1'1 ,,:

,I

A =sin1 (-0.5)
A =-30' or

A= 360-30 = 330'

ill
j

'

Day 9- Plane Trigonometry 221

220. l 001 Solved Problems in Engineering Mathematics (2nd Edition) by Tiong & Rojas

Note:

a=~ =J5 2 -3 2
a=4

=(1-sin 2 8)-sin 2 8

cos 65 + cos 55 = cos e

e = 5"
1t

e=5" x - 1800
e = 0.087 rad

b~?~
a =15

e=cos-

15)

~ j~Asgiven

15 adjacent side
cose=-=

17
hypotenuse

y= (1 +cos 28) tan 8

Note: sin

e = cos (90 - 9}

Substitute:

b=~ =J17

-15

90-13A =2A

b=8

90=15A

.
opposite side b
stne =
=hypotenuse
c
8
sine=-

1-tan xtan Y

-+-

=~

1-(;)(;J

hypotenuse

b~3bh

tan (x + y) = 1

y = ( 1 +cos 2e} tan e

a=?

8=tal')-1 [2cos(60' )]

=2cos8sin8

8=tan- 1 (1)
1t

a= cot-1 [ 2 cos (sin-1 0.5)]

cos8
sine Cos-e_tan8
=--+
Cos-a_
cos
= tan8+tan 8

8 = cot- 1 [ 2 cos (30

8=cot- (1.732)
cot 8 = 1.732
1
--=1.732
tan8
1
tane=(-1.732

= sin 2 8 + sin 2 8 cot 2 8

~[coseJ
2

=sin 2 8+

=sin 2 8+cm~ 2 8
X= 1

8=30"

tan-1 2x+tan-1 x =2:or45'


4
Let:
A= tan-1 2x
tanA=2x

SeC

tl sin 2 8
7

-csec0(1-sin 0)

l]

x = sin 2 8(1 + cot 2 8)

X= sec; 8-

1t

sin 8+cos8 tan 8

J]

8=45' x - - = - rad
180'
4

D
1

sine= 0.6 or~= opposite side

sinS J
= 2(cos ) 8) ( Cbse_

x =2tan8

A=6"

tan (x + y) = tan x +tan y

17

e = tan-1 [ 2 cos (60" l]

x=

sin 13A = cos 2A

=2(1-sin 8)tan8

y =sin28

cos 2A
sin 13A
sin 13A = cos2A

Cbs-a_

e = tan-1 [ 2 cos( sin- 1 ;

= (2- 2sin 2 8) tan 8

I
.: i

1
= - -- ( cos 'a e)

=(1+1-2sin 2 8)tan8

1
sec2A=--sin 13A
1

e)

X= COS8

Substitute:

cote=~

Cbs._ (90 -13A} =do&_ 2A


2

cos 2e = 1- 2sin 2 8

.
adjacent side a
cote=
=opposite side b

0.99619=cose

x=sece(cos 2

cos 29 = cos 2 8- sin 2 e

B=>=tan-1 x
tan B = x

Day 9- Plane Trigonometry 223

Z22. 100-1 Solved Problems in Engineering Mathematics (2"a Edition) by 'l'iong & Rojas
Substitute:
A+B=45
tan (A+ B)= tan 45
tan A +tanB
-----1
1-tanA tanB
2x+x
--=1
1- 2x(x)

= --'------'---'---'2(12)

-234
x=---

x=~----------~

24

(sin A- sin B)(cos A- cos B)

-2+34
x=--=1.33
24

(cos 2 A- cos 2 B) +(sin 2 A- sin 2 8)


(sin A- sin B)(cos A cos B)
2

-3J(3) -4(2)(-1) = -34.123

-3+4.123
X

4
=0.28

tan1 (x + 1) + tan1 (x -1) = tan-1 12


Let:

cos A= 1-cos A
2cos 2 A =1
cos 2 A = 0.5
cosA=0.707
cosA =0.707

A =ta'ril-1 (x+fj'

tan A =(li! + 1'

eos O+cos 1+ ... cos 2 89+cos 2 90

tan(A+B)=tan(tan- 1 12)

Thus, the above equation can be written


in the following format:

--~-=12

tan A +tanB

(sin 2 0 + sin 2 90) + (sin 2 1+ sin 2 89)

1-tanA tanB
(x + 1) + (x -1 )

+... +(sin 2 44+sin 2 46)(sin 2 45)

12

1- (x + 1)(x -1)
2x = 12-12(x 2 -1)
2x=12-12x 2 +12
12x 2 +2x-24=0

x=

2 sine cos e -cos

1-sin e + sin 29 -cos 29


cos 9(2 sine -1)
(1- cos 2 9) + sin 2 9-

sinS

X=---~----~-

sin29 + sin 2 e- sine


cos 9(2 sin e -1)

II
!
I

provided, A+ B = 90

A+B=tan-1 12

sin 2A = 2 sin A cos A


3.939x = 2(2.511 x)(3.06x)

X=

Note:
sin 2 A +cos 2 B =1andcos 2 A +cos2 B = 1

Substitute:

Substitute:

coversed sin

e = 0.134

Note: coversed sin 9 = 1 - sin 6


Substitute:

cos 9(2 sin e -1 )

tanB = x-1

Note: sin 2A = 2 sin A cos A

3.939\ = 15.367x 1.
3.939
x=-15.367
X =0.256

~=
2

B = tan - (x -1)

sin A= 2.511x
cos A =3:06x
sin 2A = 3.939x

(sin A- sin B)(cos A- cos B)


1-1
x=---------------(sin A -sin B)(cosA -cos B)
x=O

A =135" or 225

sin 2 0 + sin 2 1+ ... sin 2 89 + sin 2 90

X=

(cos A+ sin A)- (sin B + cos B)

A =45" or335
cos A =-0.707

x=----------~-------~

x=--------------~

Ill

Using the quadratic formula:

sin A +sin B
x=
+~------sin A - sin B
cos A - cos B
(cos A+ cos B)(cos A cos B)J
( +(sin A+ sin B)(sin A- sin B)

-2 J(2) -4(12)(-24)

2x 2 +3x-1::r0

2(2)

cos A +cosB

3x = 1-2x 2

X:

1111

Using the quadratic formula:

x=--~------~--~-

(cos2 0 + cos 2 90) + (cos 2 1+ cos 2 89)


+... + (cos 2 44 + cos 4 46)(cos 2 45)
X= 1

"

2 sin 2 e- sin e
cos9(~)

coversed sine= 0.314


1-sin9=0.314
sin9=1-0.314
sin9=0.866

X=--------sinS(~)
x =cote

9=60'

ml

'I

I'

II

I
'

f;!:'I
,I

,1,.!

sin A= 3.5x; cos A= 5.5x


sin A

3.5x

cosA 5.5x
tan A= 0.636363
A= 32.4T

!'iillil
1:

1~1!

224

J_Q_OJ oSolv~d Pro~em~ in Engineering Mathef!lati~~nd Edition) by Tiong & R~~


X

~Jso-h

\l\j50

1\
I :h, \ L ih\

X.

x=8.816m
50-h

h1 = b + 3

h2 =b-3

!an 50 - -----

50-h
tan 50 -------

c
i

d.

32

'

l>A

85+x
sin 13

Substitute x in Eq.2:

B
?t

-~-

50

sin111.7

:>in8.3

BC
Note:

7t = total distance traveled by ship B


10 + 5t = total distance traveled by ship A
d

----------sin 111.7
V=--(50)
sin.3
V = 321.8 mph

(j o: :\7~ I'll

sin 42.35'
71

sin 46"70'
I()

15

e + 61 + 90 + 15 = 180
6=14

By sine law:
h

By sine law:

852.719

d =: 852.719(sin26

..~..

ml

By sine law:

BC = 852.'119m

sin26

h = 42.5 (tan 60)


h = "73.61m

a=111.7

sin 58

sc

x(~~ 60 )

x =42.5m

a+8.3+60=180

4620'

a +p +60 = 180

10

sin 26

sin 60

13 =8.3

h 2 =7-3=4units

By sine law:

,d

,,I

\ tan30
85+x =3x

300

50

= 42

0=96

1000

to Eq.2:

(85 + x) tan 30 = x tan 60

b=7

9 + 26 + 58 = 180

sin 96

By sine law:

h 1 =7+3:::1Qunits

,.-<_t\:
ei

1000

"'

tan60=-

_;;;.

I--

{~

Equate Eq.1

b(b + 3)::: b(b- 3) + 42

6b

B cf

B5+x
h = (85 + x) tan 30-+ Eq.1

tyz_ + 3b = ~. - 3b + 42

I"

h
tan30=--

1
1
--bh1 =--bh2 +21

10.506 '0 50- h


h = 39.49m

./

A 1 =A 2 +21

tan50 (8.816):c::50-h

85 +X

= 4.03 hrs

h = x tan60 -+Eq.2

8.816

2.3231 = 9.354
_t

~ ill

~h~h

sin42.35')
{1 0 + 5t) -_-.----- :: 7t
(
stn46.20'
9.354 + 4.6771 = 7t

L ____~ -

50

tan 80 = --

Iii,

- - - - - - - - - - - - - - - - - - - - = D a y 9-~ane Trigonome~!!_

r~r

sin14"

sin61

15

II

x = 54.23m

II

'ol

II
j:

lb

ZZ6 . 100 1 Solved Problems tn Engineering Mathematics (2nd Edition) by Tiong & Rojas

I_

Equate Eq.1 to Eq.2:


120-h
tan 28'

_ _ _ _ _ _ _ _ _ _ _ _ _ _ _ _ _ _ _D_a..._y_9_-_Pl_a_ne Trigonometry 227

/
l
~h~h
11

---tan 46

5(@+

tan28'
120-h= - h
(
tan46'
120-h =0.513h
h=79.31m

tan 40'

x=J(15) 2 +(20) 2
X= 25ft

~44

~h

11h

100 +X

120
h

tan21 = - 100+x
h=(100+x)tan21' ~Eq.1
X

......---1,20-h

./"1

&_j

120-h
tan28 = - X

120-h

x=--~Eq.1

tan28"
h
tan46' =-

x
h

x=--~Eq.2

tan46"

tan32'
h

1144

=x

=x tan 32'

+X

41

=a-h
=m--tan40
h

Eq.1

tan30' = - 50+x
h

nil

X =----50~

Eq.2

tan 30

Equate Eq.1 to Eq.2:

44
tan60=x
x =25.4m

~&J

~~

15
tane=20
9=36.87

6X

By Pythagorean theorem:

- - =----50
tan400 tan30"
119175h=1.73205h-50
h = 92.54 ft.

44
tan30=-S+25.4
s + 25.4 = 76.21
S=50.81m

V=t
50.81
V=-5(60)
V =0.169m/s

35

~ Eq.2

ml

Equate Eq.1 to Eq.2:

(100 + x) tan 21' = x tan 32'

tan 32'
100+x=x - (
tan 21'
100 + x = 1.6278x
x=159.286m
Thus:

: + 100 = 259.286 m

~~
50

50
tan35' = x
X'"71.407m

50

228 I 00 ["Solved Problems in Engineering Mathematics (2nd Edition) by Tiong & Rojas
By sine law:

50-h
tan13. = - -

150

-~

--------

50-h
tan13'' = - -
71.407
tan13. (71.407) =50- h

b=30

sin 26 sin 92.75


c =341.78m

c= 34

m1

h =33.514m

1111

Day 9 -Plane Trigonometry 229

S=

a +b +c

16 + 30 +34
-----

8=40

Be{ \

~-

vu..

"oA

c=190
"'" <

Bd

_'_?QC

By cosine law:
2

180 2 =130 2 +190 2 -2(130)(190)cosB


B =65.35'"

r
tan30" = - 0.5x
10
tan30' = - 0.5x
x =34.64cm

r~
0.5x.

By cosine law:
x 2 = a 2 + (c/2) 2 - 2a(c/2)cos B

x2 = 130 2 + 95 2 - 2(130)(95) cos 65.35'


x =125m

Bcf ----=c==-:-?-~r::.J_,:' 'o A

c=210

By sine law:
S=
150

a+b+c

300

sin26'

sinB
B = 61.2!5
A+B+C=f80
26 +61.25 +C = 180

c =92.75

A= .J 40(40 -16)(40- 30)(40 -34)


A= 240 square units

b =a +c -2ac cosB
Note:
Since equilateral triangle, A = B ::: C = 60

A= .J s(s- a)(s- b)(s- c)

195+157+210

=----2

s =281
A= .Js(s-a)(s-b)(s-c)
A= .J281 (281-195)(281-157)(281- 210)
A= 14,586.2 square units

"'l

!IIi

il

232 100 !'Solved Problems in Engineering Mathematics (2nd Edition) by Tionsz & Rojas

[]
Mon
,, <

' , . .,. ..~

~-

< -'

:v "~

~ \

'

>

,. .
r

Tue

~.

. ' ~

101 0

.-

---'

Theory

Wed

[j

[QJ

Problem<;

Thu

Solutions

Notes

0
0
Fri

Sat

Topies
Spherical Trigonometry
Great Circle & Small Circle
Poles and Polar Distance
Spherical Wedge & Lunes
Propositions of Spherical Triangle
Solutions to Right Spherical
Triangle
Napier's Rules
Quadrantal Spherical Triangle
Solutions to Oblique Spherical
Triangle
Law of Sines & Cosines
Area of Spherical Triangle
Terrestrial Sphere
GMT & UTC
Latitudes and Longitudes

I
[ii

'r:

:!11
',

I
111,1

:11
'I

What is SQherical Trigonometry?

,,._,..,,,.,

:I

I
'

Spherical Trigonometry is the part of


trigonometry which deals with triangles on
the sphere. The object of this subject is to
study the relations connecting the sides
and angles of a spherical triangle'. This is
of great importance for calculations in
astronomy and navigation.

Circle A

What is a Great Circle'?

1{
c'

II

Great circle is a circle whose center


coincides with the center of the sphere On
the surface of the sphere, the closest
analogue to straight lines is great circles.

A great circle is a circle on the surface of a


sphere that has the same diameter as the
sphere, dividing tl1e sphere into two equal
hemispheres. A great circle is the
intersection of a sphere with a plane going
tluouqh its center. J\ great circle is the
l<ifl)"~,t circlr' tlrdt c::rrr ht~ dr:1wn on CJI)ivrm
;plrrrr

Circles A and circle B are example of


great circles because their centers
coincide with the center of the sphere. The
m~idians and the equator of the earth
are also examples of the great circles.

.I.

,
I

~ !I

.,

~~.~~

I!

Great circle paths are used by ships and


aircmfts where currents and winds are not
a siqnificant factor. For aircraft traveling
w<>;l\;rly between r:on11nents in the

:lillii

I
I,

234 1001 Solved Problems in Engineering Mathematics (2nd Edition) by Tiong & Rojas
northern hemisphere these paths will
extend northward near or into the arctic
region, while easterly flights will often
fly a more southerly tract to take
advar>+age of the jet stream.
What is a Small Circle?

Day I 0 -Spherical Trigonometry 235

What is a Polar Distance?

What is right spherical triangle?

The polar distance of a circle is the least


distance on a sphere from a point on the
circle to its pole. The polar distance of a
great circle is goo.

A right spherical triangle is one with a


right angle.

ill I
I

A spherical triangle with two right angles is


called birectangular spherical triangle
while the one with three right angles is
known as trirectangular spherical
triangle.

;!,

r:

A small circle of the sphere is the circle


constructed by a plane crossing the
sphere not in its center. Circle C belowis
an example of a small circle. '

'111111111'11

:I

Center of Sphere
What are the important propositions of
a spherical triangle?

The following are the important proposition


of a spherical triangle:

p1 is the polar distance of the small circle


and P2 is the polar distance of the great
circle. P2 is an arc that measures goo.

1.

If two angles of a spherical triangle


are equal, the sides opposite are
equal; and conversely.

2.

If two angles of a spherical triangle


are unequal, the sides opposite are
unequal, and the greater side lies
opposite the greater angle; and
conversely.

What is a Spherical Wedge?

What is a Pole?

A line through the center 0 of a sphere


perpendicular to the plane of a circle on
the sphere cuts the sphere into two points
called poles.

Two diametral planes that form an angle


divided the sphere into four spherical
wedges, opposite wedges being
congruent. The surfaces of the we_dges are
called sphericallunes or digons.

,II' I

3.

The sum of two sides of a spherical


triangle is greater than the third .side.

What are the solutions to right


spherical triangle?

A right triangle can be solved by using the


Napier's Rules.
Napier circle (sometimes called Neper's
circle or Neper's pentagon) is a mnemonic
aid to easily find all relations between the
angles and sides in a right spherical
triangle.

a+b >C

tune
4.

Pole

The sum of the sides of a spherical


triangle is less than 360 degrees.

0 < a + b + c < 360


5.

The sum of the angles of a spherical


triangle is greater than 180 degrees
and less than 540 degrees.
180 < A + B + C < 540"

6.

What is a Spherical Triangle?


Pole

Spherical triangle consists of three arcs


of great circles that form the boundaries of
a portion of a SP.herical surface.

The sum of any two angles of a


spherical triangle is less than 180
degrees plus the third angle.
A +B< 180 +C

Napier's Rules:
Hule no. 1 (Tan-ad Rule):
The sine of any middle part is equal to the
product of the cosines of the opposite
parts.
Rule no. 2 (Co-op Rule):
The sine of any middle part is equal to the
product the tangent of the adjacent
parts.

of

il
I

Day 10 -Spherical Trigonometry 237

Z36 1001 Solved Problems in Engineering Mathematics (2nd Edition) by Tiong & Rojas
How to apply the Napier's Rules?
Consider the right spherical triangle below:

Important Rules:
1.

2.

Convert it to its equivalent Napier's circle:

A. Law of Sines:

sinb "'sinBsinc

3.

In a right spherical triangle an oblique


angle and the side opposite are of the
same quadrant.
When the hypotenuse of a right
spherical triangle is less than 90, the
two legs are of the same quadrant
and conversely.
When the hypotenuse of a right
spherical triangle is greater than 90,
one leg is of the first quadrant and the
other of the second and conversely.

What is a quadrantal spherical triangle?


A quadrantal spherical triangle is a
spherical triangle having a side equal to

goo.

B. Law of Cosines for sides:

=cosbcosc + sinbsinccosA

cosa

cosb == cosacosc + sinasinccosB

=cosacosb + sinasinbcosC

cosc

C. Law of Cosines for angles:

A meridian is half of a great circle on the


earth terminated by the north pole and the
south pole.
In October 1884, it was agreed upon that
the reference meridian is the one that
passes through the Royal Greenwich
Observatory at Greenwich, England. This
meridian is called .the Prime Meridian or
sometimes known as the Greenwich
Meridian.

cos A= -cos8cosC + sirt8sinG~cosa.


cosB = -cosAcosC + sinAsinCcosb
cosC = -cosAcos8 + sinAsin8cosc

What are the formulas for area of


spherical triangle?

If "b" is considered as the middle part, then


the opposite parts are

sina
sinb
sine
sinA = sinB =sinG

The equator is the great circle whose


plane is perpendicular to the axis. The
equator has a latitude of zero degree.

c and B while the

adjacent parts are A and a.


Applying rule no. 1:
sinb =tan A tan a

Eq. 1

But TanA=tan(90-A)

~8

Substitute Tan A in equation 1


sinb = tan(90- A)tana

An oblique spherical triangle is one with


no angle equal to a right angle.

.
1
But tan(90-A)=cotA=-tanA
.
1
smb =--tana
tan A
sinbtanA = tana

What are the solutions to an oblique


spherical triangle?

sinb = cosBcosc

But cos(90- B)= sinS


cos{90- c)= sine

A=-.--

taoo

where: R = ~adius of the sphere


E spherical excess in degrees
E =A+ 8 + C -180

What Is a Terrestrial Sphere?


Terrestrial sphere refers to the earth
(though slightly ellipsoid) as sphere with a
radiusof 3959 statute miles.

Applying rule no. 2:

sinb = cos(90- B)cm:(90- c)

1tR2E

What Is an oblique spherical triangle?

The terrestrial sphere rotates about a


cjiameter called its axis which pierces the
.sphere in the north pole and the south
pole.

The meridian on the side of the earth the


lie opposite to the prime meridian is known
as the international date line. This
imaginary line on the surface of the earth
offsets the hours that are added or
subtracted as one travels east or west
through successive time zones.
If one crosses the international dateline
precisely midnight, going westward one
skips and entire day while going eastward,
the day repeats.

l
~

238 1001 Solved Problems in Engineering Mathematics (2"ct Edition) by Tiong & Rojas
What is a Greenwich Mean Time (GMT)?
Greenwich Mean Time I GMT) is the
mean solar time at the Royal Greenwich
Observatory, in Greenwich, England.
Theoretically, noon Greenwich Mean Time
is the moment when the sun crosses the
Greenwich meridian (and reaches its
highest point in the sky in Greenwich).
Because of the earth's uneven speed in its
elliptic orbit, this even may be up to 16
minutes off apparent solar time. GMT
became a world time and date standard
because it was used by the Britain's Royal
Navy and merchant fleet during the
nineteenth century.
In January 1, 1972, GMT was replaced as
the international time reference by
Coordinated Universal Time (UTC}. The
UTC is maintained by an ensemble of
atomic clocks (a much more stable
timebase) around the world.

UTC uses a 24-hour system of time


notation, "1:00am" in UTC is expressed as
0100, pronounced as "zero one hundred".
Fifteen minutes after 0100 is expressed as
0115; thirty-eight minutes after 0100 is
0138 (usually pronounced as "zero one
thirty-eight"). The time one minute after
0159 is 0200. The time one minute after
1259 is 1300 (pronounced "thirteen
hundred"). This continues until 2359. One
minute later is 0000 ("zero hundred"}, and
the start of a new UTC day.

A parallel of lati~de or briefly parallel, is


the small circle cut from the earth by a
plane parallel to the equatorial plane.
latitude

Colatitude is the complement of latitude.

DayJO- SpliE!rical Trigonometry 239


What are Terrestrial Sphere constants?

.!

-:~

: 1minute:
; angle

N,,rth

'\

(+)

31)

~~-L

::r>~~:,_.;.,:.~.~ :.-:..~ \ . . . .
\

Equator If 1:.

':1 .

J ..

T'~':'''l

Radius of earth = 3959 statute miles


1 minute on the great circle arc = 1 NM
1 NM (nautical mile)= 6080 feet
= 1852 meters
1 statute mile = 5280 feet = 1760 yards
1 statute mile = 8 furlongs = 80 chains

S>Ut.L

Tl<le longitude of a point is the angular


distance between the prime meridian and
the meridian through the point. The
angular measurement ranges from 0 at
the prime meridian to +180 eastward and
-180 westward. A degree of longitude
corresponds to a distance from 0 to 111
km. The degree of longitude computed as
111 km times the cosine of the latitude,
when the distance ir.> laid out on a circle of
constant latitude.
l.Dngitude
180

The!. statute mile (sometimes known as the


international mile) is typically meant
when the w9rd "mile" is used without
qualification.
The nautical mile is used universally in
aviation, naval and maritime purposes.

Proceed to the next page for your 1Oth


test. Detach and use the answer sheet
provided at the last part of this book. Use
pencil number 2 in shading your answer.
GOOD LUCK I

90

; . !

'. /~~- .. '

East
(+)

0
Prime
meridian

Did you know that..: in the ancient times,


the number 40 was used to indicate "many
or too many" and the number 1001
signifies a kind of "finite infinite" as in
Alibaba and the 40 thieves, Moses leave
his people for 40'days and 40 nights and
the famous Arabian "A thousand and one
nights"

uotr:
"To be a scholar of mathematics, you must
be born with talent, insight, concentration,
taste, luck, drive and the ability to visualize
and guess"

Lf',t o

')(!

'Ql:ribia:

''(

\1 Nautical mile

The UTC is also kncwn as "Zulu Time" or


"World Time".
The latitude of a point is the angular
distance of the point from the equator, and
will be considered positive if the point is
north of the equator and negative if the
point is south of the equator. This angular
measurement ranges from 0 at the
equator to 90 at the poles. One minute of
the of the arc of latitllde is approximately
one nautical mile or 1852 meters. A
degree of latitude always corresponds to
about 111 km or 69 miles.

1l

- Paul Halmos

-----'=D-"'aY.JQ.- SEherical Trig_9~!.Y 24_1


441: Solve for side b of a right spherical
triangle ABC whose parts are a ::: 46, c "'
75 and C =90

tv! on

0
lJ 0

Tue

Theory

Problems

Solutions

Notes

V'./ed

[Q]
Thu

0
Fri

Sat

436: If Greenwich Mean Time (GMT) is 6


A.M., what is the time at a place located
30 East longitude?
A.

7A.M.

B.

BA.M.

C.

9A.M.

Spherical Trigonometry
Great Circle & Small Circle
Poles and Polar Distance
Spherical Wedge & Lunes
Propositions of Spherical Triangle
Solutions to Right Spherical
Triangle
Napier's Rules
Quadrantal Spherical Triangle
Solutions to Oblique Spherical
Triangle
Laws of Sines & Cosmes
Area of Spherical Triangle
Terrestrial Sphere
GMT & UTC
Latitudes and Longitudes

C.

30 minutes

D.

1 hour

B.

68

48
74

A.

45"

B.
C.
D

90
60
30

437: If the longitude of Tokyo is 139E and


that of Manila is 121E, what is the time
difference between Tokvo and Manila?

73.22

D.

1 hour and
1 hour and
1 hour and
1 hour and

12 minutes
5 minutes
8 minutes
10 minutes

spherical triangle whose given parts are A

= B = 80 and a

A.

B.
438: One degree on the equator of the

C.

earth is equivalent to _ _ in time.

D.

A.
B.

1 minute
4 minutes

15812'
16221'
16831'
17212'

What is the value of the side opposite the


right angle?

A.
B.

8330'
8445'

C.

8615'

D.

8515'

448: ECE Boar4 April 11.997


The area of sphencal tnangle ABC whose
parts are A= 9::140'. B = 6412', C =
i 1651' and the radius of the spl1ere i~ 100
rn is:

443: Determine the value of the angle B of


an isosceles spherical triangle ABC whose
given parts are b = c 5428' and a =

A.
B.
C.

D.

15613 sq. m.
16531 sq. m.
!8645 sq. m.
25612 sq. rn

9230'.
A.
B.

C.
D.

449: A spherical triangle has an area of


327.25 sq. km. What is the radius of the
sphere if its spherical excess is 30?

8945'
5545'
8425'
4145'

A.
B.
C.
D.

4554'
8042'
97"09'
7243'

445: Solve for angle C of the oblique


spherical triangle ABC given, a = 80, c
115 and A = 72

74.33
75.44
76.55

440: Solve the remaining side of the

A.
B.
C.

44:.t: Given a right spherical triangle whose


parts are a = 82, b = 62 pnd C = 90.

A.

20 !\m

B.
C.
D.

22 krn
25 krn
28 krn

4$0: EE Board. April :1:991

A.
B.
C.
D.

74

444: Solve the angle A in the spherical


triangle ABC, given a = 10625', c = 4216'
and B = 11453'.

439: A spherical triangle ABC has an


angle C 90 and sides a = 50 and c =
80 Find the value of "b" in degrees.

D. 4A.M.

'

9057'
9845'

C.
D.

.~.

D.

447: What is the spherical excess of <>


spherical triangle whose angles are all
right angles?

A.

Topics

c.

= b =89

A.

B.
C.
D.

A
B.
C.

61
85
95
119

D.

441&1 Determine the spherical excess of


the spherical triangle ABC given a = 56, b
65" and c 78

:n"33'

ll

(ill":l7'

A ship on a certain day is at latitude 20N


and longitude i 40E. After sai!ing for 150
hours at a uniform speed along a great
circle route, it reaches a point at latitude
10S and longitude 170E. If the radius of
the earth is 3959 miies. find the speed in
miles per hour.

17.4
15.4
16.4
19.4

Day 10- Spherical Trigonometry 243

sin co-c = cos a cos b


cos c = cos a cos b

diff. in til11e

Topics

D
D
D D
D ~
D
D D
Mon

Tue

Wed

Theory

Problems

Thu

Solutions

Fri

Notes

Sat

436. B
437. A
438. B
439. B
440.C

441. B

442.

443. D
444.C
445. D

c:J 13-15

c:J
c:J
0

Topnotcher

9-12 Passer
b-8

Conditional

0-b Failed

If FAILED, repeat the test.

30-

oo

-------

24
360"
diff. in time = 2 hours
Note: The time in the place is 4 hours
ahead of GMT because the place is at the
East. Thus, the time is 8 AM.

diff. in time , diff. in longitude


360
139-121 o

----=~---

24
360
diff. in time= 1.2 hours
= 1 hour and 0.2(60) min
= 1hour and 12min

c
il:

sin co-B =tan co-a tan c/2

f3

=cot a tan c/2

1
cos B =--(tan c/2)
tan a
1
cos 80 = --(tan c/2)
tan 89
tan c/2 = cos 80 tan 89

c/2 =84.26
c = 168.52" or 168 31'

Note: 360 degrees = 24 hours

'

b = 74.33''

cos

24
diff. in time

RATING

446.A
447.8
448. B
449.
450.C

360

24
diff. in time

Spherical Trigonometry
GreatCircle & Small Circle
Poles and Polar Distance
Spherical Wedge & Lunes
Propositions of Spherical Triangle
Solutions to Right Spherical
. Triangle
Napier's Rules
Quadrantal Spherical Triangle
Solutions "to Oblique Spherical
Triangle
Laws of Sines & Cosines
Area of Spherical Triangle
Terrestrial Sphere
GMT & UTC
Latitudes and Longitudes

ANSWER KEY

=diff. in longitude

cos 80 = cos 50 cos b

time=1 \

Ill

241m;_)
x (- x [60
- minJ
-.
360\

. 1 ~

time =4 minutes

a=46

A
sin co-c = cos a cos b
cos c = cos a cos b
cos 75 =cos 46 cos b
b=68.12' or68'07'
a=so

.,,1'1'11111
l

II

It

'

244. 1001 Solved Problems in Engineering Mathematics (2"d Edition) by Tiong & Rojas;

'
cos b = cos a cos c + sin a sin c cos B

cosb = (cos 106 25')( cos 42 16')


+

(Sir 106

25' )(sin 42 16' )

Day 10 - Spherical Trigonometry 245

B=72

E=(A+B+C)-180
E = (93'40' +54 12' + 116 51') -180

(cos114 53')

E =9443'

b =11843'

a=82
Using law of sines:

sin co-c = cos a cos b


sinA

cos c = cos 82 cos 62

sin 10625'
sin A

sin b
sin 11453'

sin 10625'

sin 11843'

----

A=82'51'or
A= 97"09' (its supplement)

a=9230'

8=72

cos a= cosb cosc +sin b sin ccosfl,.


cos 56 =(cos65 cos 78)
+ (sin 65 sin 78 cos A)
A =5T53'
Using law of sines:
sine
sine
sin C

tan 54'28'

sinC

sine
sin C

B =41"75'or41"45'

sin 115

II

8=11453'

Using law of sines:

[ t a92'30')
n--

sinA

Using law of cosines for sides:

sinS

sin 80

sinA

sinb
sin B

sina
sin 57" 53'

sin65"

sin 56"

sina
sin 72

8=67"48'

1tR 2 E ~t(100) 2 (94.43')


A=-------180"
180
A.= 16,531.17m 2

,I,,

E =(5T53'+67 48' +8T52')-180

C = 118 56' (its supplement)

E=33"33'

~tR 2 E
A= 180

327.25 =

~tR

1.,

!',,I

(30)

180
R=25km

North

:: \h;~~~~:~}"N)
u
\... _ C

I,

,II
"
i:lillll.i'

B(170J'E, 108)

1'.1

'i

~>b-l

E = (A + B +G) -180

C=61"04'or

Note:
Since side c is greater than side a then
angle C is greater than angle A. Thus use
= 11856'

sin a
sin 5753'

sin 56
C =>8T52'

cos B = cot c tan a/2

cosB=

sinA

sin 78

sin co-B = tan co-c tan a/2


1
cos B =--(tan a/2)
tan c

Using law of cosines for sides:

sinS

-------

c=86.25 or86'15'

b=65

South

E =(A+ B + C)-180

E=(90 +90' +90 )-180


E=90

B
C a=3o

cos b = cos a cos c + sin a sin c cos B


;11:

,,

246 100 1 Solved Problems in Engineering Mathematics (2nd Edition) by Tiong & Rojas
sin co-c = cos a cos b
cos c = cos 30 cos 30

c =41.4096

1\

C=41.4096\ X ( 60NMJ

Topics

c = 2,484.58 NM

distance

Mon

time

2484.58

D
D D
D ~
D D
D

= 16.56NM/hror
150
V = 16.56 knots

Spherical Trigonometry
Great Circle & Small Circle
Poles and Polar Distance
Spherical Wedge & Lunes
Propositions of Spherical Triang~e
Solutions to Right Spherical
Triangle
Napier's Rules
Quadrantal Spherical Triangle
Solutions to Oblique Spherical
Triangle
Laws of Sines & Cosines
Area of Spherical Triangle
Terrestrial Sphere
GMT & UTC
Latitudes and Longitudes

Tue

Theory

Wed

Problems

Thu

Solutions

Fri

Notes

Sat

"'

,,.,,-o,..,

,,

' '

, , ,,. '

,,

;;

'

,.

'00 '*'"'""<

. .

~ <

< '~

,A,,,;,

,.,.

<

248 100 l Solv:ed Problems in Eng!!'!~~g Mathematics (2nd Edition) by Tiong; & Rojas

E'ii'IW I
Topics

0
0
0
~J
l] 0
0 ~
0 0
Mon

'I

..: ;,,

< '

~ 0'

Tue

~il

'

Theory

V\fed

Problems

Thu

Solutions

Fri

Notes

Sat

Analytic Geometry
Rectangular Coordinates System
Distance Between Two Points
Slope of Line
Angie Between Two Lines
Distance BetWeen a Point & a Line
Distance Bel.\-IJeen 2 Parallel Lines
Division of Line Segment
Area of Polygons by Coordinates
Equations of Lines
Conic Sections or Conics
General Equation of Conics
Geometric Properties of Conics
Equations of Circles

Wha!Js Analytic Geometry?


Analytic geometry deals with geometric
problems using coordinates system
thereby converting it into algebraic
problems.
Rene Descartes (1596- 1650, Cartesius
in Latin language) is regarded as the
founder of analytic geometry by
introducing coordinates system in 1637.

First Quadran!

Second Quadrant

P(5,3)

abscissa

ordinate

I t--4>~---t

-4 -3 -2 -1
Rectangular Coordinates System (Also
known as Cartesian Coordinates System)

Th;ro ""''""'

-1

Fomth Qoed"ot

lx
5

Day 11 -Analytic Geometry (Points, Line11 & Circles) 251

250 1001 Solved Problems in Engineering Mathemati<;:s (2"d Edition) by Tiong & Rojas

A line parallel to the x-axis has a slope of

Point 0 is the origin and has coordinates


(0,0). The x-coordinate or ~bscissa is
always measured from the y-axis while the
y-coordinate or ordinate is always
mei;lsured from the x-ax1s. The point P has
5 and 3 as abscissa and ordinate,
respectively.

(Xt, Yt)

zero while a line parallel to the y-axis has


a slope of infinity {co).

...

"-~-~

For parallel lines with slopes of m1 and m2,


respectively, the slopes are the same.

(X2, Y2, Z2)

Ax+By+C=O

~--~--~--~--~x

m~=m2

What is distance between two points in

i elane?
Consider two points whose coordinates ,
are (Xt, y,) and (x2, y2), respectively. A
right triangle is formed with the distance
between two points being the hypotenuse
of the right triangle.

For perpendicular lines with slopes of m,


and m2, respectively, the slope of one is
the negative reciprocal of the other.

y
d = J(x2- x1)

+ (Y2- Y1) + (z2- z1)

(x2, Y2)

.~//./'!'
(XI, Y1

) I...

X2- XI

Using Pythagorean theorem, the distance


between two points can be calculated
using:

Line 2

d ..

...'
(X1, Y1)

X2- X1

Line 1
~~

The angle, e between these lines (line 1


and line 2) may be calculated using the
. following formula:

What is distance between two I?Oints in


space?

1:

,j

This formula is known as the distance


formula.

rise t:..y
slope=m = - =-run
t:..x
where: !!.. denotes an increment

x2

-x~

tane = m

m = Y2 -y2
x2 -x~
I
I

-m~

Ax;t-By+C1 =0

tane "' 1 + m m
2 1

t~

The (perpendicular) distance, d, between


the two lines is:

c,":'c2

,I
'.',

But

m2

tane = Y2 - Y2
Consider three axes namely x,. y and z and
two points with coor<;linates (x,, Yt. Zt) and
(x2. y2, z2), respectively.

What is distance between two.Qarallel


lines?
Consider two parallel lines with equations
as shown in the figure.

----~-----------------------X

+(Ya -Y,)

if B is positive and the point is above.


the line or to the right of the line
if B is negative and the point is below
the line or to the left of the line
If otherwise

Consider two lines with slopes of m, and


m2.

,l

d;:J{; -xt)

+
What is angle between two lines?

~---+-----------------------X

Consider two points whose coordinates


are (x,, y1) and {x2. y2}, respectively. A line
is formed by connecting the two points.
The slope of the line is defined as the rise
{vertical) per run (horizontal)

use

m2 =--.
m,
What is slope of a line (m)?

d=Ax +By1+C
A~ +82

d=
What is distance between a point and a

.JA2 +B~

!!.!!!1
Consider a point with coordinates (x1, y,)
and a line with equation Ax + By + C = 0.

Use the sign (either+ or-) that would


make the distance positive.

Day 11 -Analytic Geometry (Points, Lines & Circles) 253

252 1001 Solved Problems in Engineering Mathematics (2nd Edition) by Tiong & Rojas
What are the coordinates of a point that
divides a line segment?
Consider two points with coordinates (x,,
y,) and (x2. y2). The line segment formed
by these two points is divided by a point P
whose coordinates are (x, y).
Let r, and rz be the corresponding ratio of
its length to the total distance between two
points.

Y./\ . .
,\

(h Y2)

E. Intercept Form:

(XJ, Yt)

~+1'.=1
a .. b

~~y~

The arrow shown in tire figure moving in a


counterclockwise direction ir1dicates that
the vertices must be written in the equation
below in counterclockwise direction.
.~

//

A=

(x,, y,)

..~

.~

.!((XWz +i<2Y3 +x3y,) 1


2 -(y,x2 +y2x3 +yax,)j

Conic section (or simply Conic) is the


locus of a point which moves so that its
distance from a fixed point (focus) is in
constant ratio, e (eccentricity) to its
distance from a fixed straight line
(directrix) .
The term "conic" was first introduced by a
renowned mathematician and astronomer
of antiquity, Apollonius (c.255- 170 B.C.)
Also, the term ~conic section" was due to
the fact that the section is formed by a
plane made to intersect a cone.
Circle

The abs.cissa of the point, P is:

What is line?

K= (Xl2)+(XifQ

r1 + r2

A line is defined as the shortest distance


between two points. The followi'ng are the
equa~ons of the lines:

The ordinate of the point, P is:

A. General Equation:

Y"' (y,r2)+(Y:l1)
r1 +r2

Ellipse is produced when the cutting


plane is not parallel (o~ inclined) to the
base of the cone.

What is a Conic Section?


~z.Y0

A-~lx, . ~2./~3 )'1r


- 2 Y1/ Y2"1_-a ~Y1 . . ~

~P(x,y}

Circle is produced when the cuttiryg plane


is parallel to the base of the cone.

What is the General Equation of a


Conic Section:

Ax2 + Bxy + Cy 2 +Ox+ Ey + F =0


When B is not equal to zero, then the
principal axes of the conic are inclined (not
parallel to the coordinates axes). The
curve can be identified from the equation
given by determining the value of the
determinant; 8 2 - 4AC.

8 2 -4AC
<0
=0
>0

Ellipse

y:::::Y2+Y1

C. Slope-Intercept Form:

y=mx+b
What is the formula for the area of a
polygon using the coordinates of its
vertices?
Consider a polygon whose vertices have
coordinates of (x,, y,), (Xz. Yz) and (x3, y3).

D. Two-Point Form:

Parabola

1E
Hyperbola

_ Yc y, (x ... x,)

y-y,-xz-x,

Conic
Section
Ell~

Parabola
Hyperbola

Eccentricity

< 1.0

= 1.0
> 1.0

When B is equal to zero, then the principal


axes of the conic are parallel to the
coordinates axes (x and y axes). To '
identify the curve, compare the
coefficients of A and C.

B. Point-Slope Form:
y-y,=m{x-x 1)

and

Hyperbola is produced when the cutting


plane is parallel to the axis of the cone.

Ax+By+C=O

If the point, P is at the mid-point of the line


segment, then the abscissa and ordinate
of the point are the following:

x, +X2
X=
2

Parabola is produced when the cutting


plane is parallel to the element (or
generatrix) of the cone.

~~

If A = C, the conic is a circle.


If A C but the same signs, the conic is an
ellipse.
If A and C have different signs, the conic
is a hyperbola.
'If either A or C is zero, the conic is a
parabola.

The conic sections have geometric


properties that can be used for some
engineering application such as beams of
sound and reflection of rays of light.

Day 11 -Analytic Geometry (Points, Lines & Circles) 255

254. 100 1 Solved Problems in EngineeljC,Q Mathematics (2nd Edition) by Tiong & Rojas
Circle reflects rays issued from the focu;s:
back to the center of the circle.

If D & E

= 0,

Center (h,k)

center is at the origin (0,0)

If either D or E, or both D & E "" 0, the


center is at (h,k).

2.

Standard Equations:

Radius ( r)

~I/

; I

Proceed to the next page for your 11th


test. Detach and use the answer sheet
provided at the last part of this book. Use
pencil number 2 in shading your answer.

x2 *Y2 =rt
C(h,k)
Ellipse reflects rays issued frorn the
focus Into the pther focus.

GOOD LUCK I

y
'(ll;ribia:

'.,

What Is a Circle?
A circle is a locus of a point that which
moves so that it is equidistant from a fixed
point called center.
1.

General Equation:

J!- +Yt +DX+Ey+F = 0

2A

D2 +E 2 -4AF
r = .1---.-4A 2

Parabola reflects rays issued from the


focus as a parallel (with respect to I~
axis) outgoing beam.

Hyperbola reflects rays issued from the


focus as if coming from the o~er
focus.

k= -E

2A

C(O,O)

8/~

h"' -D

h
2

{-J(-.h) +(y-kf

Did you know that. .. nature seems to


know its math and never wastes its
resources! The reason why bubbles are
spherical because with this shape, bubbles
can enclose the most volume with the
least material. This was first shown by
Archimedes.

<!euote:

r2

When the equation given is general


equation rather than standard equation,
the center (h,k) of the circle and its radius
(r) can be determine by converting the.
general equation to standard using the
process known as completing the
square. Or using the following formulas:
General equation:

Afi!. +Qy2 +'Ox +Ey + F =0

"Can we actually know the universe? My


God, it's hard enough finding your way
around Chinatown!"
-Woody Allen

Day 11 -Analytic Geometry (Points, Lines & Circles) 257


451 If (-2,-4) is the midpoint of (6.-7) and
(x,y), then the values of x and yare

Topics

D
D
D D

Analytic Geometry
Rectangular Coordinates System
Distance Between Two Points
Slope of Line
Angle Betw~en Two Lines
Distance Between a Point & a Line
Distance Between 2 Parallel Lines
Division of Line Segment
Area of Polygons by Coordinates.
Equations of Lines
Conic Sections or Conics
General Equation of Conics
Geometric Properties of Conics
Equations of Circles

Mon

Tue

Theory

Problems

Solutions

Wed

D
Thu

~
Fri

[] D
Notes

x= 2, y

=1

B.
C.

X= -10,

0.

X:::;

10, y = -1
8, y -1

45'71 ECE Board November 1998


Determine the coordinates of the point
which is three-fifths of the way from the
point (2,-5) to the point (-3,5).

A.

(-1,1)

B.

(-2,-1)
(-1,-2)
(1,-1)

c.
D.

4S8a ECE Board April 1998


The segment from (-1 ,4) to (2,-2) is
extended three times its own length. The
terminal point is

A.
B.

c.
D.

Sat

X::

=-1

(11,-24)
(-11,-20)
(11,-18)
(11,-20)

4S9t The points (a, 1), (b,2) and (c;3) are


collinear. Which of the following is true?
451: ECE Board A.pril :t999
The linear distance between -4 and 17 on
the number line is

C.
D.

19
-5or19

B.

c.
D.

13
21
--17
-13

A.
B.

4601 If the slope of the line connecting the


origin and point P is 3/4, find the abscissa

c.

(2,
(3,
(3,
(2,

-2)
-2)
-3)
-3)

D.

(-2,5).

455: lEE Board April 1995


The line segment connecting (x,6) and
(9,y) is bisected by the point (7,3). Find the
values of x and y.

11
9
10
8

A.
B.

5 or-5
5 or 19

of P if its ordinate is 6.
A.

B.

6
7
8

c.
D.

4a ECE Board April 1999

453: If the distance between points (3,y)


and (8,7) is 13, then

equidistant from (1, -6), (5, -6) and (6, -1 ).

452: EE Board Apri!l 1.994


Find the distance between A (4,-3) and B
A.
B.
C.
D.

c-b=c-a
c-b b-a
c-a =a-b
c-a = b- a

B.
C.
D.

454: Find the coordinates of a point

A.

A.

y is equal to

A.

14,6

B.
C.

33, 12
5, 0

D.

14,6

Find the inclination of the line passing


through (-5,3) and (10,7).

B.

14.73
14.93

C.

14.83

D.

14.63

. A.

42: Find the angie formed by the lines 2x


+ y -8 = 0 and x + 3y + 4 0.

A.
B.

c.
D.

30"

35"
45

so

4:J Find the angle between the lines 3x +


6 and x + y = 6.

2y

A.

B.

1220'
1119'

D.

1425'
13.06'

c.

44a What is the acute angle between the


lines y= 3x + 2 andy= 4x + 9?
A.
B.

C.

4.4"
28.3"
5.2c

D.

18.6"

465: EE Board October 1997


Find the distance of the line 3x.+ 4y
from the origin.

B,

4
3

D.

2
1

A.

c.

=5

4 ... CE Board November :1'!'92

The two points on the lines 2x = 3y + 4 0


which are at a distance 2 from the line 3x +
4y-6 = 0 are

A.
B.
C.

D.

(-5, 1) and (-5,2)


(64,-44) and (4,-4)
(8,8) and (12, 12)
(44,-64) and (-4,4)

471 CE Board November :1992


The distance from the point (2, 1) to the
line 4x- 3y + 5 0 is

A.
B.

1
2

D.

c. 3

258 . 100 1 Solved Problems in Engineering Mathematics. (2nd Edition) by Tiong & Rojas
4681 CE Board November 1996
Determine the distance from (5, 10) to the
linex- y = 0.

A.
B.

c.

3.33
3.54
4.23

D.

5.45

4
5
6
7

ME Board April 1998


Find the slope of the line defined by y - x

units
units
units
units

1
1/4
-1/2
5+x

0?

What is the x-intercept of the line passing


through (1,4) and (4,1)?

3/2
2/3
-3/2
-2/3

A. 3
B.

c.
D.

In a cartesian coordinates, the vertices of


a triangle are defined by the following
points: (-2,0), (4,0) and (3,3). What is the
area?

4711 EE Board April 1995


Find the distance between the lines, 3x + y
- 12 = 0 and 3x + y - 4 = 0.

A.
B.
C.
D.

8 sq. units
9 sq. units
10 sq. units
11 sq. units

4761 EE Board April 1994

A.
B.

c.
D.

16

J10
4

J10
8

J10

47:&1 ME Board October 1996


What is the length of the line with a slope
of 4/3 from a point (6,4) to they-axis?
A.
B.
C.
D.

Given three vertices of a triangle whose


coordinates are A (1, 1), B(3,-3) and
(5,-3). Find the area of the triangle.

12

J10

10
25
50
75

A.
B.
C.
D.

D.

A.
B.
C.
D.

3x + y3x- y +
X+ 3y +
X- 3y-

1 =0
1=0
1 =0
1 =0

481: ECE Board April1999


If the points (-2,3), (x,y) and (-3,5) lie on a
straight line, then the equation of the line is

A.
B.
C.
D.

2y -1 = 0
2x + y -1 = 0
X+ 2y -1 = 0
2x + y + 1 =0
X-

The equation of a line that intercepts the xaxis at x = 4 and the y - axis at
y =- 6 is,

4771 ECE Board November x99o


In a cartesian coordinates, the vertices of
a square are: ( 1,1 ), (0,8), (4,5) and
(-3,4)~ What is the area?
blnits
units
units
units

478: EE Board April1997


A line passes thru (1 ,-3) and (-4,2). Write
the equation of the line in slope-intercept
form.

A.
B.
C.
D.

y- 6x =0
y = -6 l
X+ y = -6
6x + y =0

intercept and y-intercept are -2 and 4,


respectively.

48ZI ME Board April :1998

3
4
5
6

A. 20 sq.
B. 30 sq.
C. 25 sq.
D. 35 sq.

B.

Find the equation of a straight line with a


slope of 3 and a y-intercept of 1.

5
6.

4.5

480: ME Board April1997

4751 ECE Board November 1990

484: Find the equation of the line passing


through the origin and with a slope of 6?

485: Find the equation of the line if the xA.

c. 4

4701 CE Board May 199:1

A . ., -4 =X
B. y =-X - 2
C. y =X- 4
-D. y- 2 =X

479: EE Board October 1997

4741 CE Board November 1995


What is the slope of the line 3x + 2y + 1 =
A.
B.
C.
D.

Find the distance between the given lines


4x- 3y = 12 and 4x- 3y -8.

5.
A.
B.
C.
D.

4691 The distance from a point (1 ,3) to the


line 4x + 3"y + 12 = 0 is
A.
B.
C.
D.

47~1

Day 11 -Analytic Geometry (Points, Lines & Circles) 259

A.
B.
C.
D.

3x + 2y = 12
2x- 3y = 12
3x- 2y = 12
2x- 3y = 12

A.
B.
C.
D.

y- 2x-4
y + 2x-4
y- 2x + 4
y + 2x + 4

=0
=0
=0
=0

486: ECE Board April 1998


Determine B such that 3x+ 2y - 7 = 0 is
perpendicular to 2x- By + 2 = 0.
A.
B.
C.

5
4
3

D.

487: The line 2x - 3y + 2 = 0 is


perpendicular to another line L 1 of
unknown equation. Find the slope of l.,.
A.
B.
C.

312
-3/2
2/3

D.

-2/3

488: A line through (-5,2) and (1 ,-4) is


perpendicular to the line through (x,-7) and
(8,7). Find X.

A.

B.
C:
D.

-4
-5
-6
-19/3

483: A line with an inclination of 45


passes through (-5/2,-9/2). What is the xcoordinate of a point on the line if its
corresponding y-coordinate is 6?
A.
B.
C.

6
7
8

f)

<)

489: CE Board May 1996


What is the equation of the line that
passes thru (4,0) and is parallel to the line
X- y- 2 = 0?
A.
B.
C.
D

X- y + 4 = 0
x+'y+4=0
X .... y --4 =.- 0
X y
0

260 100 l Solved Problems in Engineering Mathematics (2"d Edition) by Tiong & Rojas
49e: Find the equation of the line through
point (3, 1) and is perpendicular to the line
X+ 5y + 5 = 0.

49&: CE Board May 1.997


Find the slope of a line having a
parametric equation of x =2 + t and y = 5

-3t.
A.
B.
C.
D.

5x- 2y = 14
5x- y = 14
2x- 5y = 14
2x + 5y = 14

491.: Find the equation of the


perpendicular bisector of the line joining
(5,0) and (-7,3)
A.

B.
C.
D.

8x + 2y + 11 = 0
8x - 2y + 11 = 0
8x - y + 11 = 0
8x + y + 11 = 0

492: Which of the following lines is parallel


to the line 3x- 2y + 6 = 0?
A.
B.
C.
D.

3x + 2y - 12 = 0
4x- 9y = 6
12x + 18y = 15
15x-10y-9=0

49:J: The equation of the line through (-3,5) parallel to 7x + 2y - 4 = 0 is

C.

-2

D.

-3

C.
D.

7x
7x
7x
2x

+ 2y + 31 = 0
- 2y + 30 = 0
+ 2y - 4 = 0
+ 7y + 30 = 0

A.
B.

C.

D.

A.
B.

C.
D.

2x + 3y = 0
4x- 5y = 22
4x + 5y = 2
5x + 4Y = 7

C.

A.
B.

3
4

C.
D.

5
2

A.
B.

A.
B.

C.
D.

Theory

Problems

C.
D.

2x+y-2=0
2x - Y,- 2 = o
2x- y + 2 = 0
2x + y + 2 = 0

Wed
Thu

[IJ
Solutions

Notes

499: ECE Board April 1.998


Find the area of the triangle which the line
2x - 3y + 6 = 0 forms with the coordinate
axis.

passing through (-2,6) with the x-intercept


half the y-intercept?
x-y=6
2x + 2y + 2 = 0
3x- y + 2 = 0
2x + y- 2 = 0

0
0

4x- y = 14
4x + 4y = 14
X+ 4y = 12
x-4y=-14

soo: ECE Board November 1998


A line passes through point (2,2). Find the
equation of the line if the length of the !ine
segment intercepted by the coordinates
axes is the square root of 5.

49S: What is the equation of the line

Tue

498: ECE Board Aprill:999


Two vertices of a triangle are (2,4) and(2,3) and the area is 2 square units, the
locus of the third vertex is
A.
B.

0
0
0
0

Mon

1
2

494: What is the equation of the line


joining the points (3,-2) and (-7,6)?

Topics

497: CE Board May 1.998


Find the slope of the line having a
parametric equation y = 4t + 6 and x = t +
1.

D.

A.
B.

A.
B.

Fri

Analytic Geometry
Rectangular Coordinates System
Distance Between Two Points
Slope of Line
Angle Between Two Lines
Distance Between a Point & a Line
Distance Between 2 Parallel Lines
Division of Line Segment
Area of Polygons by Coordinates
Equations of Lines
Conic Sections or Conics
General Equation of Conics
Geometric Properties of Conics
Equations of Circles

Sat

ANSWER KEY
451.B
452.C
453.D
454.C
455.C
456. B
457.A
458. D
459. B
460. D
461. 8
462.C
463.8

464.A
465. D
466. B
467. 8
468. 8
469. 8
470. 8
471. D
472. A
473.A
474.C
475. 8
476. 8

477.C 490. B
478. B . 491. B
479. 8
492. D
480. 8 493.A
481. D 494.C
482.C 495. D
483. C . 496. D
484. A 497. D
485. 8
498. D
486.C 499.A
487. B 500. 8
488.C
489.C

RATING

c:J
c:J
c:J
0

43-50

T~pnotcher

33-42 Passer
25-32 Conditional
0-25 Failed

IfFAILED, repeat the test.

262 l 00 l Solved Problems in Engineering Mathematics (2nd Edition) by Tiong & Rojas

ml

ml

Day 11 -Analytic Geometry (Points, Lines & Circles) 263

Let
Xm and Ym. the coordinates of the midpoint

17
Xm-

'----y---__)

x,

Iii

P2(-3,5)

+X.2
__

2.1

7= x+9

The linear distance from -4 to 17 is equal


to 21.

2
x=5

P2(-2,5)

(1 ,~6)

(5,-6)

Ym = Y1 + Y2

2
d, =d2
2 +-(-y-+6) 2 = Jr-(x---5)7"
2 -+(-y-+-6)7'
2
J'"(x---1)::-(x-1) + ~ =(x-5)
2

+~

'JR.- 2x + 1 ='JR. -1 Ox+ 25


8x=24
x=3
P,(4,-3)

3= 6+y
P,(2,-5)

2
y=O

6 8

y= Y1r2 +y 2 r1

+(5+3) 2

(3-5) +.(y+6)

Let:
Xm and Ym. the coordinates of the midpoint
Xm

(3/5) + (2/5)

P,(-1,4)

= x, +x2
2

10y::: -30

P2 (x 2 ,y 2 )=(8,7)

::: 6+ X
2

y =-3

Using distance formula:

Thus the pointis at (3, -3)

-x, )2 +(y2 -y, )2

mJ

13 2 =(8-3) 2 +(7y) 2

X =-10

Ym = y, + Y2

P,(x,6)
6

169:::25 + 49 -14y + y 2
2

2
2
=(x-6) +(y+1)
2
2
=(3-6) +(y+1)

4+ ) \ +12y+36=9+ ) \ +2y+1

P1 (x 1 , y 1 ) = (3, y)

O=y

P,(6,-7)

(x-5) +(y+6)

d=~(x2

r1 + r2

Substitute x = 3:

d=10

= -5(2/5)+5(3/5)

y =1

2
2
2
2
(x-5) +(y+6) =(x-6) +(y+1)

d=~(x2 -x, )2 +(y2 -Y, )2

(3/5)+(2/5)

-1

2 = J,-(x---6--=)2:-+-(-y-+-1)"""
2
J,-(x---5--=)2:-+_(_y-+-6)-=-

Using distance formula:

2(2/5) + ( -3)(3/5)

-~~~~~

r1 +r2
X=

d2 ::: d3

d=J(-2~4} 2

x 1 r2 + x2r1

-4=

-14y95

O=(y-19)(y+5)

y=19or

y =-5

2
-7 + y
-

2
I

10.-1

1---..

P2(9,y)

y::: -1

x-

r1 +r2
2

::: -'-(--'-1)("'--3-'-)_+
d _:X2:..:(d-'-)
d+3d

-3'li.+'Cl_x 2
2=--4'Cl_
X1

x1 r2 + x2 r1

=11

'

264 . 100 I Solved Problems in Engirieering Math:emaacs (a" Edition) by Ttong~ Rojas

Substitute:

y= Y1r2. + Y2r1
r1 +r2

3 6-0
----

4(3d)+y2(d)
-2=
.
d+3d

3x+2y=6

x-0

x=8

4)(

m1 =-3/2
x+y=6
y=-x+6
m 2 =-1

Y2 =-20

Let 9 = angle of inclination


_ ry<c.3)
(b,2)

'Y

Let: 9 = angle ~tween the two lines

Note: P1 (-5,3); P2 (10,7)

9 =tan1

Since collinear,

m1 = m2.

m 1 =m 2

2-1

3-2

b-a

c-b

--=---=-b-a c-b
c-b=b-a

X2

-1- (-3/2) )
1 + (-3/2)(-1)

9=11.3099" or 11" 18' 35"

Ill

2x+y-8=0
y = -2x +8
m1 =-2

Y=~x+2

m1 =3

1 4
y=--x-.3
3
m 2 = 113

d (-)1

Note:
The negative sign only denotes that the
point is below',the line.

Let: 9 = angle between the two lines

-m, )

9=tan1[ m2
1+m1m 2

Let: 9 = angle between the two line!

-m~

e =tan

whenx~O.y = -J.JJ
= 2.0

4 3
( -----)
1+(4)(3)

'9=4.398'"
9=tan1 [ -113-(-2) )

-x 1

t-13 2 +4 2

y=4x+9
m2 =4

x+3y+4 =0

9=tan1[ m2
)
1+m,m2

m=Y2Y1

[ 1+m m
1 2

9 = 14.93'

x2 -x1

m2 -m 1 )

9=tan -1 (

9 = tan 1 ( 4/15)

m=Y2Y1

d= A(xt)+B(y 1 )+C
__:......:..J.:.,:A=:2::::+::8=;:2:-d= 3(0)+4(0)-5 = -5

Y2 -y,
7-3
m=--=--=4/15
x 2 -x 1 10+5
tan9=m

Given two points, the slope of the line is,

Y=~-x+3

3x=24

-2= 12M_+ M,y2

Day ll -Analytic Geometry (Points, Lines & Circles) 265

when y=O, x

+d,

=A(x 1)+B(yd+C.
.JA2 +82

Ill

1+ (-~)(-1/3)
9=45

P1 (0,0)-+x 1 =Oandy 1 =0
P2 (x,6)-+X 2 =xandy 2 =6
m=3/4

Givenline:3x +4y-5 =0
A =3;B=4;C::: -5
Given point: (0,0)
XI "'0;y1 tt:Q

2=

3(x 1)+4(y 1 )+(-6)

.
./32 +42
10=3x1 +4y 1 -6
16=3x1 +4y 1
y1

:t

4-0. 75x 1 -+ Eq.1

266 . .I 001 Solved Problems in Engineering Mathematics (2"d Edition) by Tiong & Rojas
2x 1 +3y 1 +4 =0 --)Eq.2

!Q

[4x- 3y + s =

Substitute Eq.1 in Eq.2:

2x 1 +3y 1 +4=0
2x 1 +3(4-0.75x 1 )+4;::0

Given line: 4x + 3y + 12 = 0

whenx~O.y ~1.67

when y-o. x "" -/.25

Day II -:-Analytic Geometry (Points, Lines & Circles) 267

P(2,1)

A= 4; 8 =3; C = 12
Given point: (1,3)

Given line 1: 3x + y -12 = 0

A=3;8=1;C 1 =-12
Given line 2 : 3x + y - 4 =0

x 1 =1;y 1 =3

-0.'25x 1 =-16

A= 3; B = 1; C 2 = -4

x 1 =64
y 1 =4-0.75(64)
Y1 = -44
Thus, the first point is at (64,- 44)
A(x 1)+8(y 1)+C
-d1 = -.J-,.A=::=2=+=8::.=2=-_2 = 3(xd+4(yd+(-6)
-)32 +42
-10=3x 1 +4y 1 -6
-4=3x 1 +4y 1
Y1 =-1-0.75x 1 -+Eq.3
Substitute Eq.3 in Eq.2:
2x 1 +3y 1 +4=0

_j A2

../16

-~42 +(-3)2

10
d=-

-5

A(x 1)+8(y 1)+C


d =--'--';::===='==--JA2 +82

Given line : x - y = 0

d=

Given point: (5,1 0)

+J42 +32
25
d=5
d = 5 units

x;=5;y 1 =10

10
8

y 1 =-1-0.75(4)

Y1 = -4

when x "(), y 0
whenx~J.y of

Thus, the second point is (4, -4).

Given line : 4x- 3y + 5 = 0


A =4; 8 =3; C = 5
Given point: (2,1)
x1

= 2; Y1 = 1

Using point slope form:

y- y 1 = m(x- x 1 )
4

y- 4 = -(x- 6)--+ Equation of line


3

Thus, at x = 0; y = - 4
Using the distance formula:

Given line 1 : 4x- 3y -12 = 0


A=4;8=-3;C 1 =-12

A(x 1)+8(y 1)+C

d = ---;::==:::=:~
-JA2 +82
d = 1(5)+(-1)(10)+0

-~f

+(-1)

-5

d=-

-.fi

(0,-y)

4(1) + 3(3) + 12

__:_'--========-

A=1;8=-1;C=O

2x 1 +3(-1-0.75x 1 )+4=0

ml

d = (-) 2 units

x1 =4

J32;"12

d=--units

--'--7::=====-

-0.25x 1 = -1

=--4_-(_-1_2_)

+82

4(2)+(-3)(1)+5

d-

c 2 -C 1

d=

A(x 1)+8(y 1)+C


d = _..:__~===--
-JA2 +82

d=~(x2

2
-x1 ) +(y2 -y1 )2

Given line 2 : 4x - 3y + 8 = 0

d=~(6-0) 2

A =4; 8 =-3; C 2 = 8

d = 10 units

d=

C 2 -C 1
-JA2+B2

20
d=-.
5
d = 4 units

8-(-12)

= --;=:====

~42+(-3)2

+(4+4) 2

y-x = 5
y=x+5

From the point-slope form:. y = mx + b

d = 3.54 units
lly inspection, the slope (m) is equal to 1

268 100 1 Solved Problems in Engineering Mathematics (2nd Edition) by Tiong & Rojas

1 [(1(-3)+3(-3)+5(1))

A=;- -(1(3)+(-3)(5)+(-3)(1))

3x:t-2y+1=0
2y = -3x-1

From the point-slope form: y

=mx + b

A=~l

x,

2 Yt

x2
Y2

P2(4,0)
x3
Y3

x,
Yt

P2(4,5)

2 0 0 3 0

A =~[(-2(0)+4(3J+3(0))

2 -(0(4)+0(3)+3(-2))
A

=9 square units

11 x 1 x 2
A=2 Yt. Y2

x3
Y3

Y2- Yt
=
-( x-x 1 )

x2 -x 1

P2(3,-3) P3(5,-3)
x3 x1
Y3 Yt

A-111 3 5 .1,
21-3-31

,.

2+3
y.+3=-(x-1)
-4-1
y+3=-1(x-1)
y+3 =-x+1
y =-x-2

x2 - x1

5-3
y-3=--(x+2)
-3+2
2
y-3 =-(X +2)
-1

-y+3=2x+4
2x+y+1=0

....

(4, 1)

" .

A=; -(1(4)+5(0)+8(-3)+4(1))

Using two point form:

x2
Y2

x1
Yt

A = 25 square units

Y2- Yt
y-y 1 =--(x-x 1 )

x2 - x1

_1,140-311
A-215841

y-y 1

_ 11 x 1
A-2 Yt

x4
Y4

P1 (1,-3)-+x 1 =1andy 1 =-3


P2 (-4,2)-+x 2 =-4andy 2 =2

Pt(1, 1)

Using two point form:

1-4
y-4 =-(x-1)
4-1
y- 4 = -1 (X -1)
y-4 = -x+1
x+y=5

1[(1{5)+4(8)+0(4)+(-3)(1) )]

. A-_1,-243-21

Geometry (Points, Lines & Circles} 269

Y2 - Yt
y-y 1 =--(x-xd

Pa(3,3)

P1(-2,0)

~ytic

P1 (-2,3)-+x 1 =-2andy 1 =3
P2 (-3,5)-+x 2 =-3andy 2 =5

Using two-point form:

Pa(0,8)

By inspection, the slope is equal to -3/2.

P1 (1,4)-+x 1 =1andy 1 =4
P2 (4,1)-+x 2 =4andy 2 =1

A = 4 square units

y=--x-2
2

Day 11 -

Substitute y
intercept:

x-interc~pt ~~

=Oto solve for the x-

x+y=5
x+0=5
x=5

[Y-~

By inspection, a

=4 and b =6

Using the intercept form:

As given: m

=3; b =1

Using point slope form:

X y
-+-=1
a b
X

-+-=1
y=mx+b
y=3x+1
~x-y+1=0

4 -6
6x-4y=24
3x-2y=12

Day 11 -Analytic Geometry (Points, Lines & Circles) 271

270 1001 Solved Problems in Engineering Mathematics (2nd Edition) by Tiong & Rojas

7
3
y=--x+-

P1 (-5/2,-9/2) - H 1 = -5/2 and y 1

P2 (x,6)~x 2

= -9/2

=xandy 2 =6

9=45

1
Since perpendicular, m 2 = - -

m,

By inspection, m, = -3/2

1
1
m 2 =--=--=1
m1
(-1)

P3 (x,-7) ~ x 3 = x and y 3 = -7

2x-By+2=0
tan9= Y2 -y 1
X2

-(-5/2)

1 = 21/2
x+5/2
x=8

P(O,Q)~x 1

y=-x+-

-x 1

tan45 = 6-(-9/2)

=Oandy 1 =0

m=6

Using point-slope fonn:


y-y 1 =m(x-x 1 )
y -0 = 6(x- 0)
y=6x

P4 (8,7)~x 4 =8andy 4 =7

1
Since perpendicular, m 2 = - -

m2

Using point-slope form:

1= 7+1

B
-3/2
B=3

8-x
x=-6

Using slope-intercept form: y = mx + b


2x-3y+2 = 0
3y =2x+2

y =-x+3
3

y-1=5(x-3)
y -1 = 5x -15

x4 -x3

m,

By inspection, m,

5x-y = 14

P1 (5,0)~x 1 =5andy 1 =0

Using slope-intercept form: y = mx + b

P2 (-7,3) ~ x 2 = -7 and y 2 = 3

x-y-2=0
y =x-2

m,

=Y2 -y 1 = 3-0
x2 - x1

By inspection, m = 1

mz = m,

=-~

-7-5

m,

= 2/3

X y
-+-=1
-2 4
-2x + y = 4

y -2x-4 = 0

Using slope-intercept form: y = mx + b

1
Since perpendicular, m 2 = - -

Using point-slope fonn:

m,

- -1- - -1m2 -

m2

m1
=-3/2

2/3

y-0=1(x-4)

y =x-4

Using slope-intercept form: y = nix + b

P1 (-5,2)~x,-=-5andy 1 =2
P2 (1,-4)~x 2

y-y 1 =m(x-x 1

=1andy 2 =-4

X +5y

+5 = 0

1 5
y =----X--
5
5

1
Since perpendicular: m 2 = - -

P(4,0), thus x1 = 4 and y, = 0

X y
-+-=1

m,

1
1
m2 =--=---=5
m1
(-1/5)

y-y 1 =m(x-x 1

= Y4 -y3

Since parallel:

Using the intercept fonn:

1
Since perpendicular: m 2 = - -

P(3, 1), thus x; = 3 <md y, = 1

By inspectiQn, niz = 2/B

y -6x =0

By inspection, m 1 = -1/5

Y2 -y,
-4-2
m1 = - - - = - - = - 1
x2 - x1 1-(-5)

3x +2y -7 = 0

1
1
m 2 =--=---=4
m1
(-1/4)

Dayll-.AnlllYlic GeometrY <PointS. Lines & Circles) Zf3

272 100 1--Solved Problema in Engineering Mathematics (2"ll Edition) by 'l'iong &: Rojas

x, =- 3 andy,= -5

Solving for the midpoint of P, and P2:

P(-3,-5), thus

x 1 -x 2 5-7
x=---.=--=-1

Using point-slope form:

y-y 1 =m(x-x 1 )
7
y.+5=--(x+3)
2
2y +10 = -7x-21
7x+2y+31=0

_y,-y2 0+3 3
y---=-=-

Using point-slope form:

y-y 1 =m(x-x 1 )
y-312 4(x +1)
2y-3=8x+8
8x-2y+11=0

P1 (3,-2)-.x 1 =3andy 1 =-2

P2 (-7,6)--+x 2 =-7andy 2 =6

Using two poi.ntform:

Using slope-intercept form: y

=mx + b

Y2 -y,
y-y 1 =--(x-x1 )
x 2 -x 1
6+2
y+i=--(x-3)
-7-3

3x-2y+6 =0

y=-x+3
2

=312
Since parallel: m2 =m,
By inspection. m,

From the choices, the answer is choice D.

15x-10y-9=0

10

y=-x--

a =_;b

By inspection, m2 =.312

y+2=-(x-3)
. -10
-10y-20=8x-24
8x+10y-4=0
4x+5y-2=0

2
b=2a

Using slope-intercept form: y


7x+2y-4 =0

y=--x+-

By inspection, m, -712
Since parallel:

mz m,

=mx + b

Using the intercept form:


X

-2

-+-=1
a 2a
-4+6
--=1
2a
a=1
b=2

Substitute:

- 11 x1
A--

~.!=1
a b
X y

-+-=1

1 2
4

2x+y=2
2x+r-2=o

Substitute: x =- 2, y = 6 and b = 2a:

-2 6
-+-=1
a 2a

-2

2 4 3

x1
Y1

x 21
y 4

(2(3)-2y.f.4x) ]

=[ -(4(-2)+3x+2y)
4 =6 -2y +4x+8-3x-2y

O=x-4y+10

x=2+t
t x-2 --+Eq.1
y =5-3t--+Eq.2

Substitute Eq.1 in Eq.2:


y=5-3(x-2)
y=5-3x+6
y=-3x+11

By inspection, m -3

y =4t+6 --+Eq.1
x=t+1
t x -1--+ Eq.2

Substitute Eq.2 in Eq.1:

x2 x3 x1

2 Y1 Y2

_ 1,-2
2--

Y3

2 -2,

Y1

(-2(4)+2y+3x)]
4=
[

-(3(2)+4x-2y)
4 =-8+2y+3x -6-4x+2y
O=-x+4y-16
O=x-4y-18

Suggested answer Is choice D.

y 4(x-1)+6
y 4x-4+6
y4x+2
By lnapectlon.

=.!1 x1

2 '3 .4 y 3

-+-=1
a b

=_:12

x3
Y3

x2
Y2

2 Y1

m 4

27 4 100 1 Solved Problems in Engineering Mathematics (2nd Edition) by Tiong & Rojas

X y
-+-=1

Given line--+ 2x-3y +6 =0

Atx

=0:

2x-3y+6=0
2(0)-3y +6= 0
y=2

y
-+-=1
-2 1
-x +2y = 2
x-2y+2 = 0
X

Topics

Mon

0
0
0

[I]

At y =0:

Tue

2x-3y+6=0
2x-3(0)+6 = 0
X =-3
Thus the x-intercept (a) is 3 units and the
y-intercept (b) is 2 units.

By inspection, the x and y intercepts are:


a= 1; b = -2

y
-+-=1

Notes

X y
-+-=1

A = 3 sq. units

By inspection, the x and y intercepts are:


a= -2; b = 1
Using the intercept form:

Fri

Solutions

1
1
A =-ab =-(3)(2)

Thu

Problems

Using the intercept form:

VVed

Analytic Geometry
Rectangular Coordinates System
Distance Between Two Points
Slope of Line
Angle Between Two Lines
Distance Between a Point & a Line
Distance Between 2 Parallel Lines
Division of Line Segment
Area of Polygons by Coordinates
Equations of Lines
Conic Sections or Conics
General Equation of Conics
Geometric Properties of Conics
Equations of Circles

Sat

-2

2x-y=2
2x-y-2=0
... ., '<

* .,., . . , .. "''

,,

'

,.

,.

... "' "'

,, '

"

i ' < ~ "' .,

'\'"'.
" .. ' .., ' ,, ~ '

}.

T
T

' .., ., <. '

" ' " " '

<. "'' "'

~ , ""'

",',

276 100 1 Sol'ved Problems in Engineering Mathematics {2nd Edition) by Tiong & Rojas

-~~

.<"'

,,.

~,,~<

'"~"~~".>"<

Topics

D
D
[I] D
D D
D D
D ~
Mon

Tue

,,.,_.

'

.,.

~.

{ ' o. '

,, ""

'

,.,,

Theory

Wed

Problems

Thu

Solutions

Fri

Notes

Sat

1, General equations:

What is a Parabola?
A parabola is a locus of a point which
moves so that it is always equidistant to a
fix,ed point called focus and it a fixed
straight line called directrix.
y
l

Parabolas
- General & Standard Equations
- Eccentricity & Latus Rectum
Ellipses
- General & Standard Equations
- Major & Minor Axis
- Eccentricity & Latus Rectum
Hyperbolas
- General & Standard Equations
- Transverse & Conjugate Axis
- Eccentricity & Latus Rectum
Polar Coordinates
-Radius Vector
- Polar Angle
Relation Between Rectangular
and Polar Coordinates

A. Axis parallel to the y-axis:

----t----t------~x

directrix

Ax2 + Ox + Ey + F

=0

B. Axis parallel to the x-axis:

<"!"'""'"'

"<

whQre

= distance from vertex V to


focus F

ci
I

di~;(;IIICi' fiOIII fHlillllo diWc!riX


fo If d1 .I dill "

. ----x

278 1001 Solved Problems in Engineering Mathematics (2nd Edition) by Tiong & Rojas

Opens down

C/ + Ox + Ey + F 0

Day 12 -An(llytic Geometry (Parabola, Ellipse & Hyperbola) 279


When the equation given is a general
equation rather than standard equation,
the vertex V(h,k) of the parabola and its
focal h;mgth or focal radius "a" can be
calculated by converting the general
equation to standard using the process
known as completing the square.

B. Axis parallel to the y-axis:


Opens upward

2. Standard equations:

--------~~~~--~

Ve1iex (V) at origin (0,0)

A. Axis along x-axis:

The following formulas can be obtained

Opens to the rrght

. x2 =-4ay

~~------~r----.

Vertex (V) at (h,k)


2

h = _E_-_4_C_F

A. Axis parallel to the x-axis:

y =4ax,
---+------~

For axis horizontal: C/ + Dx +

k = -E

2C

4CD

(x.., h)2

Opens to the right

Ey + F = 0
-D
a= 4G

=4a(y - k)
A~ + Dx + Ey + F = 0

For axis vertical:

y
Opens downward

h= -0
2A

Opens to the left

k::

..

""4AF

4AE

-E

a== 4A

y
X

What is an Ellipse?
---4---------+----~

y 2 "'~4ax
------~~---~

(y-k) 2 =4a(x-h)

Opens to the ~eft

(x- h)2

An ellipse is a locus of a point which


moves so that the sum of its distance to
the fixed points (foci) is constant and is
equal to the length of the major axis (2a).

=-4a(y- k)

P(x~

._

B. Axis along y-axis:


The eccentricity of the parabol;. is the
ratio of the distance to the focus to the
distance to the directrix.

Opens up

y
X

e=.!.

Since f = d, then:

(y- k)2 ""-4a(x- h)

x~ ':'<lay

e=1
The latus rectum of a parabola is a line
that passes through the focus and
perpendicular to the axis of the conic.

LR=4a

directrices

1. General Equation:

Ax2 +Cy2 +Ox +Ey +F =0

281 1001 Solved Problems in Engineering Mathematics (2nci Edition) by Tiong & Rojas

Nole: d1 + d2 2a. The major axis


the distance from V1 to V2.

=2a, is

Major axis is vertical:


y

Day 12 -Analytic Geometzy(P~ill:>oJ~Ellipse &_!ln>_er:l:>ola) 281

(x-ht +(y-kf = 1
b2
a2

Since a < 0 and c < a, then the


eccentricity of an ellipse is always less
than 1.

When the point is located along the minor


axis as shown in the following figure:.

e <1'
The eccentricity of .an ellipse is the ratio
of the distance to the focus to the distance
to the directrix.

P(x,y)
X

The latus rectum of an ellipse is a line


that passes through the focus and
perpendicular to the axis of the conic.

~[

LJ

x2

y2

z+z=1
b a

2w2

b +C~ =a

Center, C at (h,k)

.Et-_J:.d

Major axis is horizontal:

Whe'l the point P(x, y) is the minor axis:

will a is always greater than "b".

If D & E 0, center is at the origin (0,0). If


either 0 or E, of both 0 & E "' 0, the r..enter
is at (h,k).

2. Standard Equations:

~ri=~

directrices

The relationship between a, b and c is

----~r---~--~-----------x

where: a = semi-major axis


b = semi-minor axis
When the equatiOn given is a general
equation rathertl'lan standard equation,
the center (h,k) of an ellipse and its focal
length "c" can be calculated by converting
the general equation to standard using the
process known as completing the
square.

The following formulas can be obtained:

Ce!lter, C at (0,0)
Mljor axis is horizontal:

General equation:

(x~bf + (Y-:f =1
a2

Ax'- + Cy2 + Ox + Ey + F =0

a
D

Major axis is vertical:


--~----------.----------r---x

directrices

h::~

2A

k=

-a:
2G

c=.Ja2 -..tr

e=Q
Iff= c, then eccentricity of an ellipse, e is
2
x2
-+y
a2 b2 = 1

. c

e=-a

What is a Hyperbola?
A hyperbola is a locus of a point which
moves so that the difference of the
distanGes to the fixed points (foci) is
constant and is equal to the length of the
transverse axis (2a) .

Day 12 -Analytic Geometry (Parabola, Ellipse & Hyperbola) 283

282 100 1 .Solved Problems in Engineering Mathematics (2nd Edition) by Tiong & Rojas
directrices

asymptot'

Transverse axis vertical:

Transverse axis

2
X - 2 =1
-

i_

Transverse axis

a2 b

Transverse axis vertical:


I

.,

)I('

I Y6

...

Transverse axis

-- -~-- _... _ :

)I{

a i

Conjugate axis

~-7'

Transverse axis

Ax

Gy2 +OX,+ Ey + F 0'

Transverse axis is the axis that passes


through the foci, vertices and the center of
the hyperbola while the conjugate axis is
the one that is perpendicular to the
transverse axis.

Length of the transverse axis

= 2a

Length of the conjugate axis

= 2b = 2.../A

Length of the conjugate axis = 2b = 2.JC

. 2
X

a2-b2='1

where: A and C are the numerical


coefficients (absolute value) of x2
and y2 , respectively.

The eccentricity of a hyperbola is the


ratio of the distance to the focus to the
distance to the directrix.
y

Center, C is at (h,k)
Also, the relationship between a, b and c is
Transverse axis horizontal:

Transverse

b
I

VI

"*'

I Y6

2. Standard equations:

.--

Center, C at (0,0) .
Transverse axis horizontal:

a
c

where: A and C are the numerical


coefficients (absolute value) of x2
and y2, respectively.

Transverse axis

Also, the relationship between a, band cis


B. Transverse axis -vertical:

---t-----------+---~x

= 2.../A

= 2a = 2.JC

(y-k)2- (x-h)2 =1
a2
b2

E!-2 + b,~: =42


Length of the transverse axis

f< ~.-r'\1\
A.:~ + o"~+
eY:+ F=v
".
.~Y

1. General equations:

A Transverse axis - horizontal

--+-------~~----~x

b
I

VI

)I("

I 116

'

.. X
(x-h)2 _(y-k)2 =1
a2
b2

.
Conjugate axis

I a .
.., ~

c
e=a

or

e=oa

Since a > c and D > a, then the


eccentricity of a hyperbola is always
greater than 1.

e >1

Day 12 - Analyti(: QE!Qrnetry (Parabola, Ellipse & Hyperbola) 28J;

Z84 . 100 1 Solved Problems in Engineering Mathematics (2nd Edition) by Tiong & Rojas
The latus rectum of an ellipse is a line
that passes through the focus and
perpendicular to the axis of the conic.

Proceed to the next page for your 12th


test. Detach and use the answer sheet
provided at the last part of this book. Use
pencil number 2 in shading your answer.
GOODLUCKJ

(r.e)
(x,y)

y
X

QI:ribia:
Did you know that... the oldest known
example of numeration using place value
is not the Roman nor the Arabic
numeration but the Babylonian or
Mesopotamians' sexagesimal system of
numeration which dates back to the 2nd
millennium B. C. ! This system of
numeration still survives today (i.e. 1 hour
= 60 minutes and 1 minute = 60 seconds,
and 1 degree 60 minutes)

b
Polar angle is sometimes called the
vectoral angle, the argument, the
amplitude, or the azimuth of a point.

,;t.R=2~2-

I ....po e

. 'a'

where: a = semi-major axis


b = semi-minor axis
When .the equation given is a general
equation rather than standard equation,
the center (h,k) of a hyperbola can be
calculated by converting the general
equation to standard using the process
known as completing the square.

r
.
polar ax1s

~uott:

"Every new body of discovery is


mathematical in form, because there is no
other guidance we can have.

What is the relation between polar


coordinates and rectangular
coordinates?

- Charles DarWin

Relationship between polar coordinates


and rectangular coordinates:

x:rC0$6

y =rt~ne

The following formulas can be obtained:

h= -Q

2A

k=

-e

2t

What are Polar Coordinates?


Polar coordinates (r,e) refers to the
coordinates of a point in a system of
coordinates where the position of a point
is determined by the length of ray segment
(the radius vector) from a fixed origin (the
pole) and the angle (the polar angle) the
ray (the. vector) makes with a fixed line
(the polar axis).

r=Jx*~y2

Day 12- Analytic Geometry (Parabola, Ellipse & Hyperbola) 287


506: CE Board May 1:996
How far from the y-axis is the center of the
2
curve 2~ + 2y + 10 x - 6y - 55 0?

Topics

Parabolas
- General & Standard Equations
- Eccentricity & Latus Rectum
Ellipses
- General & Standard Equations
-Major & Minor Axes
- Eccentricity & Latus Rectum
Hyperbola
- General & Standard Equations
-Transverse & Conjugate Axes
- Eccentricity & Latus Rectum
Polar Coordinates
- Radius Vector
- Polar Angle
Relation Between Rectangular
and Polar Coordinates

Man

D
Tue

D D
[I] D
D D
Theory

Problems

Wed

Thu

Solutions

Fri

Notes

Sat

SOJ:: CE Board May 1:995


What is the radius of the circle x 2 + y2 - 6y
= 0?

A.
B.

2
3

A.
B.
C.
D.

502: CE Board November 1:995


What are the coordinates of the center of
the curve~+/- 2x- 4y- 31 = 0?

A.

(-1 ,-1)

B.

(-2.-2)
(1,2)
(2,1)

C.
D.

so~: A circle whose equation is ~ +

4x + 6y -'-23 = 0 has its center at:

A.
B.

(2,3)
(3,2)

D.

(-3,2)
(-2,-3)

504: ME Board April 1:998


What is the radius of a circle with the ff.
equation: ~- 6x + y2 - 4y- 12 = 0

c. 4
D.

C.

3.46

7
5
6

SOS: ECE Board April 1:998


The diameter of a circle described by 9~ +
9y2 = 16 is

A.
y2

C:

4/3
16/9
8/3

D.

B.

A.

-2.5

B.

-3.0
-2.75
-3.25

C.
D.

l-

B.
C.

D.

C.
D.

A.

(X - 2) + (y - 2)2 ::: 5

B.

(x - 2) 2 + (y + 2) 2 = 25
2
2
(X - 2) + (y + 2) = 5
2
2
(x- 2) + (y- 2) = 25

C.
D.

A.
B.
C.
D.

A.

A.
B.
C.
D.

x + / + 8x - 1Oy - 12 = 0
~+
+ax- 10y + 12 = 0
~ + / + 8x + 10y- 12::: 0
~+l-ax+ 10y + 12 o

51:4: ECE Board April 1:998


Find the value of k for which the equation
~ + y2 + 4x - 2y - k = 0 represents a point
circle.

2.1
2.3
2.5
2.7

509: ME Board Oetober J:996


2
The equation x + y2 - 4x + 2y - 20
describes:

SJ:~r Find the equation of the circle with


the center at (-4,-5} and tangent to the line
2x + 7y - 10 = 0.

7.07
7.77
8.07
7.87

508: CE Board November 1:99~


The shortest distance from A (3,8) to the
circle~+/+ 4x- 6y = 12 is equal to

A.
B.

25 1t
27 rr

SJ:2r Determine the equation of the circle


whose radius is 5, center on the line x = 2
and tangent to the line 3x - 4y + 11 = 0.

507: What is the distance between the


+ 2x + 4y - 3
centers of the circles x2 +
= o and x 2 +
ax - 6y + 7 = 0?

A.

C.
D.

=0

A circle of radius 5 centered at the


origin.
An ellipse centered at (2, -1 ).
A sphere centered at the origin.
A circle of radius 5 centered.at (2,-1 ).

B.

6
-6
D.. -5

C.

515: ECE Board April 1:999


3x2 + 2x- 5y + 7 = 0. Determine the curve.
A.
B.

Parabola
Ellipse
Circle
Hyperbola

uo: EE Board April 1:997.

C.
D.

The center of a circle is at (1, 1) and one


point on its circumference is (-1 ,.-3). Find
the other end of the diameter through (-1 ,3).

51:6: CE Board May 1:99~, CE Board


November 1:99~, ECE Board April

A.
B.
C.
D.

(2,4)
(3,5)
(3,6)
(1,3)

sur Find the area (in square units) of the


circle whose equation is~+ y2 = 6x- By.
A

20 rr
7/rr

1:994
The focus of the parabola y2 = 4x is at

A.
B.
C.
D.

(4,0)
(0,4)
(3,0)
(0,3)

51:7: CE Board November 1:994


Where is the vertex of the parabola ~
4(y- 2)?

__________ pay .~.:~nalylic Geornetry__(~ar~~~~Eilips~!_f:!.'[f~f::~E-~!<!L:~~~

288 100 1 Solved Problems in Engineering Mathematics (2nd Edition) by Tiong & Rojas

~~
~~
~~

SZ:JI ECE Board Aprll1998

8,

Find the equation of the axis of symmetry


of the function y =2x2 -7x + 5.

C.

D.

A.

Find the e~uation of the directrix of the


parabola y = 16x.

B.
C.
D.

7x + 4 = 0
4x + 7 0
4x-7 =0
x-2 = 0

What is the area enclosed by the curve 9x2


+ 25y 2 - 225 ::: 0?

directrix y =2. Find its equation.

x=2
x=~

x=4

A.

x=4

B.
C.
D.

5191 Given the equation of a parabola 3x


+ 2y2 - 4y + 7 0. Locate its vertex.

A.
B.
C.
D.

(5/3, 1)
(5/3, -1)
(-5/3, -1)
(-5/3, 1)

In the equation y
curve facing?

x2 + 12y -14x + 61 = 0
x2- 14y + 12x + 61 0
x2- 12x + 14y + 61 0
None of the above

= -x2 + x + 1, where is the

A.
B.
C.
D.

5Zftl ECE Board Noveaaber 1997


Compute the focal length and the length of
the latus rectum of the parabola y2 + 8x .
6y + 25 0.
2,8

4, 16

D.

1, 4

D.

c.

A.

Find the location of the focus of the


parabola y2 + 4x- 4y- 8 0.

c.

16,64

B.

x=-3
x= 3

D.

y=3

A.
B.

C.
D.

A
B.

=-8(x-

C.
D.

1). What is the equation of its directrix?

.J5

(2.5, -2)
(3,1)
(2,2)
(-2.5,-2)

n.3

2x - 4y 2 = 5
4x 2 + 3y 2 = 12
2
2x +
=3
2
x + 2y = 4

st

3
4
5
6

(3,-4)
(3,4)
(4,-3)
(3,5)

c. y =-3

533: EE Board October 1.997


Find the major axis of the ellipse x2 +
2x- By+ 1 = 0.

SZ8& ME Board October 199'7

A.

B.

The general equation of a eonic section is


given by the following equation: A>t! + Bxy
+ Cy2 + Ox + Ey + F 0. A curve maybe
identified as an ellipse by which ofthe
following cOndition&?

A.

B2 -4AC <0

I
11

2
10

D.

2
3

D.

4
5

lt99~

x2

y2

64

16

53$1 An ellipse with an eccentrir..:ity of 0.6!:i


and has one of its foci 2 units from the
center The length of the latus rectum is
nearest to

A.

3.5 units
3.8 units
4.2 units
3.2 units

B.
C.
D.

S361 An earth satellite has an apogee of


40,000 km and a perigee of 6,600 km.
Assuming the radius of the earth ns 6,4CO
km, what will be the eccentricity of the
elliptical path described by the satellite
with the center of the earth at one of the
foci?

A
B.

0.49

0.46

D.

0.52
0.56

S3jt: The equation 25x2 + 16y2 - 150 x +


128 y + 81 =0 has its center at

sa? Given a parabola ( y- 2 )2

5ZZI EE Board October 1997

A.
B.

'I

B.

20

~.8

D.

B.

c.

S3l.: The lengths of the major and minor


axes of an ellipse are 10m and 8 m,
respectively. Find the distance between
the foci.

B.

c.

A.
B.
C.

y2 - 11y + 11x- 60 = 0
y2 -11y + 14x- 60 = 0
y2 -14y + 11x + 60 0
None of the above

5z:ll CE Board ala,- 1995

C.
D.

50.2

D.

What is the length of the length of the latus


rectum of the curve x2 20y?

.J20

~1

B.

5301 ECE Board Aprill.998


Point P(x,y) moves with a distance from
point (0, 1) one-half of its distance from line
y = 4. The equation of its locus is

x-axis, vertex at (-1, 7) and one end ofthe


latus rectum at (-15/4, 3/2). Find its
equation.

Upward
Facing left
Facing right
Downward

A.

5Z5r A parabola has its axis parallel to the

5ZOI ME Board Aprll1997

A.
B.
C.
D.

S:l9: CE Board November 1.994

5Z4& A parabola has its focus at (7. 4) and

S34: CE Board May

The length of the latus rectum for \lh':


ellipse - --- , 1 IS equal to.

~~

518: ECE Board Aprll1994, ECE


Board Aprll1999

2
8 - 4AC = 0
8 2 - 4AC > 0
8 2 - 4AC = 1

S~7:

4/ ~

ECE Board Aprill998

The major axis of the elliptical path in


which the earth moves around the sun is
approximately 186,000,000 miles and the
eccentricity of the ellipse is 1/60.
Determine the apogee of the earth.

B.

C.
D.

93,000,000 miles
91,450,000 miles
94,335, iOO miles
94,550,000 miles

S38: CE Board November :199%


The earth's orb1t is an ellipse with the sun
at one of the foci. If the farthest distance of
the sun from the earth is 105 . 50 million km
and the nearest distance of the sun from
the earth is 78.25 million km, find the
eccentricity of the ellipse.

290
A.
B.

c.
D.

roo l Solved Problems in Engineering Mathematics (2nd Edition) by Tiong & Rojas
0.15
0.25
0.35
0.45

539: An ellipse with center at the origin


has a length of major axis 20 units. If the
distance from center of ellipse to its focus
is 5, what is the equation of its directrix?
A.

B:

X= 18
X= 20

C.
D.

X=
X=

15
16

540: What is the length of the latus rectum


of 4J2! + 9l + Bx - 32 = 0?
A.
B.

c.
D.

2.5
2.7
2.3
2.9

541: EE Board Odober 1993


4J2! - / ='16 is the equation of a/an
A.
B.
C.
D.

parabola
hyperbola
circle
ellipse

542: EE Board October .1993


Find the eccentricity of the curve 9J2! - 4l
-36x +By= 4
A.

1.00
1.~

1.~

D.

1.ro

A.

B.
C.
D.

3
4
5

sso: ME Board April1997

545: CE Board May 1996


What is the equation of the asymptote of

xz
9

C.
D.

B.

C.
D.

D.

4J2!+/+16=0
4J2! +
16 = 0
,(! - 4/- 16 = 0
2
4x
= 16

l-l

D.

,(! +
J2! + /
,(! + y2
2
x +

+z
+ z2
+ z2
2
+z

+ 6x - 4y - 8z
+ 6x - 4y - 8z
+ 6x- 4y + 8z
+ 6x- 4y + Bz

= 36
=7
=6
= 36

r - 8 cos e = o
r - 6 cos e =
r- 12 cos e = o
r- 4 cos e = o

5531 ME Board October 1996


What are the x and y coordinates of the
focus of the conic section described by the
following equation? (Angle e corresponds
to a right triangle with adjacent side x,
opposite side y and the hypotenuse r.)
rsin 2
cos

a=

A.
B.
C.
D.

and P2 {5,1,-4).
A.

11

J11

f12

The semi-transverse axis of the hyperbola


x2 Yz
.

c.

D.

5441 EE Board. October 1994

B.

How far from the x-axis is the focus F of


the hyperbola J2! - 2l + 4x + 4y + 4 = 0?

D.

A.

5J2! - 4y 2 + 20x + 16y - 16 = 0


5J2!- 4y2 + 20x- 16y- 16 =
5J2!-4y2 -20x+16y+16=0
5J2! + 4y2 - 20x + 16y- 16 =

12

B.

10.5

548: Find the distance between P1(6,-2,-3)

B.

c.

D.

B.
C.
D.

Find the polar equation of the circle, if its


center is at (4,0) and the radius 4.

with vertices at (-4,2) and (0,2) and foci at


(~5.2) and (1 ,2).

B.
C.

JjQ1

10

5521 EE Board April1997

5471 Find the equation of the hyperbola

A.

C.

A.
B.
C.
D.

Find the equation of the hyperbola whose


asymptotes are y = 2x and which passes
through (5/2, 3).
A.

at (-3,2,4) and of radius 6 units is

546: EE Board A.pril1994

x + 5y - 2z = 9; 3x - 2y + z

= 3 and x + y +

A.

z = 2 is at
A.

B.
C.

D.

(1/4, 0)
(0, rr12)
(0,0)
(-1/2,0)

sS4: Find the polar equation of the


circle of radius 3 units and center a.t
(3,0).

549: The point of intersection of the planes

B.
C.

(2, 1-1)
(2,0,-1)
(-1,1,-1)
(-1,2,1)

D.

---=11S
9 4

Given the polar equation r = 5 sin a.


Determine the rectangular coordinates
(x,y) of a point in the curve when a is 30.

A.
B.

551: The equation of a sphere with center

2x- 3y = 0
3x- 2y = 0
2x- y = 0
2x + y = 0

c.

4.5
3.4
2.7
2.1

yz
4

the hyperbola - - - = 1 ?
A.
B.

555: EE Board October 1997

What is the radius of the sphere center at


the origin that passes the point 8,1 ,6?

543t CE Beard November 1995

A.

Day 12 -Analytic Geometry{Parabola, Ellipse & Hyperbola) 291

r=3cose
r = 3 sine
r=6cose
r =9 sine

(2.17, 1.25)
(3.08, 1.5)
(2.51, 4.12)
(6,3)

Day 12 -Analytic Geometry (Parabola, ElliiJse &Hyperbola) 293

ED

x 2 +y 2 -6y=O

0
0
0
0
0

tvlon
Tue

0
0

Theory

Problems

Solutions

Notes

Wed

Thu

Fri

~
Sat

Topics

--

Parabolas
- General & Standard Equations
- Eccentricity & Latus Rectum
Ellipses
-General & Standard Equations
- Major & Minor Axes
- Eccentricity & Latus Rectum
Hyperbola
-General & Standard Equations
- Transverse & Conjugate Axes
- Eccentricity & Latus Rectum
Polar Coordinates
- Radius Vector
- Polar Angle
Relation Between Rectangular
ahd Polar Coordinates

x2 +y 2 -6x-4y-12=0

By completing square:
By completing square:

x 2 +(y-3) 2 =0+(3) 2
(x-3) 2 +(y-2) 2 =12+(3) 2 +(2) 2

x2 +(y-3)2 =(3)2

(x-3) 2 +(y-2) 2 =25


Standard equation with center at (h,k):

(X- h) 2 + (y- k) 2 = ,.Z

By inspection, r

(x-3) 2 +(y-2) 2 =(5) 2


Standard equation: (x- h) 2 + (y- k) 2 = r"

=3

By inspection, r = 5.

mJ

x 2 +y 2 -2x-4y-31=0

9x 2 +9y 2 =16

By completing square:

16

x2 +y2 = -

(x-1) 2 +(y-2) 2 =31+(1) 2 +(2) 2

(X -1 ) 2 + (y- 2) 2 = 36

x +y2=(;r
2

Stanaaro equation: (x- h) + (y- k) = r"

ANSWER KEY

501. B 516. l\
502.C 517.8
503. D 5"18. 0
504.C 519. D
505.C 520. 0
506.A 521. B
507.A 522.C
508.A 523.C
509. D 524.A
510. B 525. c
511 c 526.A
512. B
527. 8
5i3. c 528.A
514. D 529.A

531. D

546. D

532.A

547. A

515.A

545./\

530. B

533. c
534. c
535. A
536. D
537. D
538.A
539. B
540. B
541. B
542. A

By inspection, h = 1 and k
center is at (1 ,2)

RATING

548. B
549.A
550. c
551. 8
552.A
553.A
554.C
555.A

c:J 46-55

c::J
0
0

= 2,

thus the

Topnotcher

33-45 Passer

0-26 Failed

By completing square:

If FAILED, repeat the test.

(x + 2) 2 + (y + 3) 2 = 23 + (2) 2 + ( 3 )2
(x + 2) 2 + (y + 3) 2 = 36
Standard equation: (x- h) 2 + (y- k) 2 = r"

543.
544. B

By inspection, h = - 2 and k
center it at (-2, -3).

By inspection, r =- and d = 2r =-

x 2 +y 2 +4x+6y-23=0

27-32 Conditional

Standard equation with center at (0,0):

x +~l=r"

= - 3, thus the

2x 2 +2y 2 +10x-6y-55=0
2
x
+5x-3y-27.5=0

+l

By completing square:

(x+2.5) 2 +(y-1.5) 2 =27.5+(2.5) 2


+(1.5 )2
(x+2.5) 2 +(y-1.5) 2 =36
Standard equation: (x - h) 2 + (Y - k) 2 = r" /

'I

294 1001 Solved Problems in Engineering Mathematics (2nd Edition) by Tiong & Rojas

= - 2.5 and

By inspection, h

Day 12- Analytic Geometry (Parabola, Ellipse & Hyperbola) 295

x =d-r

k = 1.5
C2(4,3)

y-axis

=7.071-5

X=

2.071

x 2 +y 2 -6x+8y=O

Ill

By completing square:
(x-3) 2 +(y+4) 2 =(3) 2 +(4) 2
(x-3) 2 +(y+4) 2 =25or(5) 2

x 2 +y 2 -4x+2y-20=0
x2 + y2

C,(-1,-2)

4x + 2y = 20

By inspection, the radius is 5.

By completing square:
Note: The distance of the center of the
circle from they-axis is equal to h.
Thus, the answer is 2.5 unit length.

x 2 +y 2 +2x+4y-3=0
x 2 +y 2 +2x+4y=3

By completing square:
(x+1) 2 +(y+2) 2 =3+(1) 2 +(2) 2
(X + 1 ) 2 + (y + 2)

=8

By inspection, the center of the first circle


is at C,(-1, -2).

d = ~(x2 -x,)2 +(y2- y,)2

(x-2) 2 +(y+1) 2 =20+(2) 2 +(1)

d = ~(4 + 1)2 + (3 + 2)2

(x-2) 2 +(y+1) 2 =25or(5) 2

d=7.071

By inspection, the center is at C(2, -1)


and the radius, r = 5 .

x 2 +y 2 +4x-6y=12

By completing square:

P,(-1 ,-3)

+y -8x-6y+7 =0
x 2 +y 2 -8x-6y=-7

Using midpoint formula:

x = x1 +x2
2
1 = -1 + x2
2

By completing square:

(x-4) 2 +(y-3) 2 =-7+(4) 2 +(3) 2

= 18

By inspection, the center of the second


circle is at C2(4, 3).
Using distance formula to solve .for the
distance between c1 & c2.:

= n(5)2

. A = 25n square units

By inspecJion, the center is at (-2, 3) and


the radius, r = 5.

(3,8)

A= nr 2

(x + 2) 2 + (y- 3) 2 = 12 + (2) 2 + (3) 2


(x + 2) 2 + (y- 3) 2 = 25 or (5) 2

(X- 4 ) 2 + (y- 3)

X2

Solving for distance between (-2,3) and


(3,8):

=3

Try circle 1:

-d= A(x1)+B(y1)+C

~A2 +B2
Note: d is negative since the point is
below the line.

y = y, +y2
2

Note: There are two possible circles

1= - 3 +Y2

d=v(x2 -x1) +(y2-Y1)

d = ~(3 + 2)2 + (8- 3)2

Y2 =5

d = 7.071
Thus, the point is at (3,5)
Let: x = shortest distance

't

Given line: 3x -4y + 11 = 0


A=3;B=-4;C=11
Center of circle 1: (2, y)
X1

= 2; Y1 = Y

!~!!'"J.99J_._'?}v~d I:r.!?~~_in Engi~!l_l9 M~pematics (2nd Edi!ionl_ey Tiong ~ Rojas


Substitute:

Solving for the equation of the circle with


center at (-4, -5):

~ ::l(2) + H>(y) + 11

-5-

-~~;)2-~(-4?~-

(x-hf +(y-k) 2 =r 2
(x+4) 2 +(y+5) 2 =7.28 2

6-4y+11
-5 ;: ,,,_,
____ _

x2 +8x+16+y 2 +10y+25=53
x2 +y 2 +8x+10y-12=0

5
y =-2

SCJiving for the equation of the circle with


center at (2, -2):

1111

{x- h) 2 + (y- k) 2 ""r 2

x2 +y 2 +4x-2y-k=O

(X- 2f + (y + 2) 2
(X- 2)

::5 2

whenrO.x~5

Given line : 2x + 7y -1 0 = 0
A=2;B=7;C=-10
Center of circle: (-4,-5)
x1 =4;y 1 :::-5
Solving for the distance from the center of
the circle to the line:
r = j\(x1) + B(y1) + C

JA2 +fil ~-~2

=(-)7.28

& Hyperbola) 297

y2 = 4ax

By inspection 4a = 16, then a= 4.

3x + 2y2 - 4y + 7 = 0
2

3
2

7
2

y -2y+-x+-=O
By completing square:

~-

By inspection, r = .Jk + 5 .

Note: For a point circle, r = 0.

Standard equation: (x- h)2

+ ( 1) 2

(....rk+5f

(x+2) +(y-1) 2 =k+5 or

::-~X .. .?. +(1 )2


2

0 =.Jk +5
k:: -5

Note:
..
If A = C, the conic is a circle.
If A 'I C but the same sign, the conic

is an ellipse.

If A and C have opposite signs, the


conic is a hyperbola.
If either A or C is zero, the conic is a
"
parabola.
Given equation: 3x 2 + 2x- 5y + 7 = 0
By inspection C = 0, thus the curve is a
paratJola.

(y -1)

= 4a(y- k)
where: h and k are the coordinates of the
vertex
By inspection h = 0 and k = 2, thus the
vertex is at (0,2)

3 5
:::::--x-2

Given equation: x2 = 4(y - 2)

Y -2y=--x-2 2

Thus, the focus is at (4,0)

(x + 2) + (y -1 ) 2 = k + (2) 2

General equation of a conic section:


Ax 2 + cy2 + Dx + Ey + F = 0

+\/\<') +(7)

Standard equation:

(y -1 )2

--...:

r= 2(-4)+7(-5)-10

Given equation:/= 16x

By completing square:

[2x+ 2'-t~ 1o= EJ


~-t whenx=O,y"/.43

~.

Ellips~

By inspection, the equation of the directrix


of the given parabola is x = -4.

x +y +4x-2y=::k

+ (y + ;?) 2 = 25

Note: Since this equation is in the


chOices, there is no need to get the
equation of the second circle.

Ill

Day 12- Analytic Geometry (Parabola,

2:: -%(X+~)

Standard equation: (y -kf = -4a(x -h)


By inspection, h = - 5/3 and k = 1, thus
the vertex is at (-5/3, 1)

y=-x 2 +x+1

x - x = -y + 1

Given equation: y2 = 16x


Standard equation:

y2 = 4ax

By inspection 4a = 16, thus a= 4.

By completing square:
(x -1/2l = -y + 1+ (1/2)2
=-y+5/4
(x-1/2)2 =-1(y-5/4)

~-+u
=

{X..f1)2 t4~(ywk~

(x.-h)-z

=: -4~{y4<+

By comparison to the standard equations,


the given parabola is facing downward!

Day 12- Analytic Gee>rn~try (f'ara):)o@L~se & Hyperbol~ 299

298 1001 Solved Problems in Engineering Mathematics (2nd Edition) by Tiong & Rojas

Standard equatian: x = 4ay


Length of latus rectum = 4a

= 20

Ill

x 2 -14x + 49 = -12y -12

=r+~

By inspection, 4a

2
(X- 7) = -4(3)(y + 1)

(x-~r =t-%+(~r

Given equation: x 2 = 20y

Focal length = a = 2
Length of latus rectum = 4a = 8

Substitute:

By completing square:

x 2 -14x+12y+61=0

16

(x--~r =i(Y+~)
Standard equation: (x- h)2 = 4a(y- k)

(y- 2)2 = -8(x -1)


Standard equation: (y- k)2 = -4a(x- h)

V(-1,7)

By inspection, the vertex is at (1, 2).


Also 4a = 8, thus a = 2 .

By inspection the vertex is at (7/4, -9/8)

-/ +4x -_4y -8 = 0
y2 - 4y = -4x + 8
Solving for a:

By completing square:

15
11
a=--1=-

(y- 2)2 = -4x + 8 + (2)2

= -4x +12
(y- 2)2 = -4(x- 3)

V(7/4,-9/8)

714

Standard equation: (y- k)2

Refer to the figure, the axis of symmetry


is,

x=7
4

V(3,2)

Standard equation: (y- k)2

(y- 7)

4x-7 =0

y 2 -14y+11x+60=0

y 2 + 8x - 6y + 25 = 0
y

(y- 3)2

By inspection, 2a = 6 and a

2x 2 -7x
2

=y-5
y 5

x --x=--2
2 2

=3

For an ellipse, the discriminant must be


less than zero (B2 - 4AC < 0).

9x 2 + 25y2 = 225

x2

6y = -8x - 25

y2

-+-=1
25 9
x2
y2
--+-=1
(5)2 (3)2

= -8x- 25 + (3)2
= -8x -16

x2

(y-3)2 =-8(x+2)

By inspection, a
Standard equation: (y- k)2

= 5 and b = 3

= -4a(x- h)
A= 1tab = 1t(5)(3)

IJy inspection,

y2

Standard equation: 2 + 2 = 1
a b

The coordinates of the vertex is at (7,-1).


Standard equation: (x - h)2 = -4a(y- k)

Refer to the figure, the equation of the


directrix is, x = 3.

Note: This is an equation of an ellipse

By completing square:

Thus, the focus is at (2,2).

y =2x 2 -7x+5

=-4( ~1 }x + 1)

y 2 -14y + 49 = -11 X -11

= -4a(x- h)

Substitute:

= -4a(x- h)

By inspection the vertex is at (3, 2).


Also 4a = 4, thus a = 1.

4a = 8 thus, a = 2

A= 47.12 square units

'
Day 12 -Analytic Geometry (Parabola, Ellipse & Hyperbola) 301
300 1001 Solved Problems in Engineering Mathematics (2"d Edition) by Tiong & Rojas

mt

(x-3)2 (y+4)2 _
--+----1
16
25

P(x,y)

cb "'? (0,1)

By inspection, h = 3 and k
center is at (3, -4 ).

= - 4,

c =ea
2=0.65a

thus the

a= 3.0769
a-c

b = Fa~ = )3.0769
2(2.338) 2
2b 2
LR=--=---a
3.0769
LR =3.55

x + 4y2 - 2x- By + 1 = 0
x 2 - 2x + 4(y 2 - 2y) = -1

By completing square:

x + y2 - 2y + 1 = ..!. (16 - 8y +

l)

4x 2 + 4y2- By+ 4 = 16- By+ y2


4x 2 +3y2 = 12

1)2

---+(y-1)2

:::

2.33B

Assuming the radius of earth and sun to


be very small compared to a:

"'

=1

Satellite

a=5
b=4

40,000

c=1a -b =15 2 -4 2 =3
2

Distance between foci= 2c = 2(3) = 6

Ill

12,800

2a = 40,000 + 12,800 + 6,600


a=29.700

(x- 3)2 + (y + 4 )2 = 1
64
16

c =a- (6.600 +6,400)


c ::29,700- (6,600 + 6,400)

ml

25x + 16y2 -150x + ~ 2By + 81 = 0


2

Standard equation: (x- h) + (y- k) _ 1


a2
~-

16,700

a+c=105.5~Eq.1

29,700

a-c =78.25 ~ Eq.2

e =0.56

2
a = 64 ~a= 8;

25(x- 3f + 16(y + 4)2 = -81 + 25(3)2


+16(4)2

Length of latus rectum:

=400

e=-=--

By inspection:

By completing square:

25(x- 3)2 + 16(y + 4)2

b2 = 16 ~ b = 4

ml
2a = 186,000,000

a" 93,000,000

LR = LR = 2b _ 2(16)

a -

a+c

c=16,700

25(x -6x) + 16(y + 8y) = -81

a
Earth

Length of major axis = 2a = 2(2) = 4

Solving for c:

= 93,000,000 + 1,550,000
Apogee= 94,550,000 miles

By inspection, a2 = 4 thus, a = 2

2b=B

Apogee =a+c

6,600

Standard equation: (x-h)2 + (y-k)2 a2


~-1

2a =10

c = ea
1

c = -(93,000,000)
60
c = 1,550,000

mt

2
2
(X -1? + 4(y -1 ) = -1 + ( 1) + 4(1)2
2
2
(X -1 ) + 4(y -1 ) = 4
(X

e=-

(xf + (y -1? = ..!.(4- y)2


2

Solving for b:

2
J(x-O/+(y-1) =-i(4-y)

e=-

d2 =-d1

Earth

2
2
k)- 1
_
Standard equation: (x- 2h) + (y-b
a2

d1

..,

--a-- = 4
j

Add Eq.1 and Eq.2:


2a = ~05.5 + 78.25

a =91.875

a-c

Day 12- AnalyticQeometry (Parabola, Ellipse &

302 100 1 Solved Problems in Engineering Mathematics (2nd Edition) by Tiong & Rojas
Substitute a in Eq.2:

By inspection, a2

"Standard equation: (x- h)


(y- k)

a2 +~=1

c=105.5-91.875
c=13.625

By inspection, a = 9 thus, a

=4

thus, a = 2 .

Also,b 2 =9 thus, b = 3.

2b - 2(4)
LR=a- 3

ml

=.,ff+2 =1.73

=3 .

Also, b = 4 thus, b = 2 .

c - 13.625 = 0'.148

zjb

= 2.667

a
-~--,,-s<.,~

)>

x-axis

Ill
Solving for c:

General equation of a conic section:

Refer to the figure:

Ax +Cy 2 +Dx +Ey +F=O

-d

+d

2a=20
a=10

e=-=a d
(10)2

= 5d

=../4+9 =3.605

Solving for e:

Let:
D1 = distance of focus 1 to the x-axis
D2 = distance of focus 2 to the x-axis

3.605
e- -c--a
2

D2 = c -1

D1 =1+c=1+1.73=2.73

e=1.8

x=20

X
i_=1
-g-4

x 2 + 4x- 2(y 2 - 2y) = -4

(x + 2)2 - 2(y -1 )2 = -4 + (2)2 - 2( 1)2


(X + 2)2 - 2(y -1 )2 = -2
2

9x -4y -36x+8y = 4
2

(x+2) 2
---+(y-1) 2 =1

9(x - 4x)- 4(y - 2y) = 4


2

4(x 2 + 2x) + 9y 2 = 32

>

By completing square:
2

4(x + 1) + 9y 2 = 32 + 4( 1)2
2

4(x + 1) + 9y 2

'(x +1)2

=36

y2
--+-=1

(y -1 )2 - (x + 2)2 = 1

9(x- 2) - 4(y -1 )2 = 4 + 9(2)2 - 4( 1 )2


9(x- 2) - 4(y -1 )2 = 36

_ (y

Standard equation (y- k)

-1l = 1
9

Standard equation: (x- h) _ (y- k)

a2

a2

b = 4 thus, b = 2

=1

I
,j

'

2 thus, b =

J2 .

L:1
-g 4
X

(x- h)2

-~=1

By inspection, the center of the hyperbola


isat(-2,1); a2 =1thus,a=1;

---b-2-

=3 ;

By inspection, a = 9 thus, a

Length of semi-transverse axis: a = 3

By completing square:

(X- 2)

x2 y2
Standard equation: 2 - 2 = 1
a b

By completing square:

4x2 + 9y + 8x - 32 = 0

= 1.73-1 = 0.73

rm

Ill

By inspection A = 4 and C = -1 , and since


A and C have opposite signs, the curve is
a hyperbola.

By inspection the equation of the directrix


is,

Note:

If A = C, the conic is a circle.


If A '1- C but the same sign, the conic

is an ellipse.

If A and C have opposite signs, the


conic is a hyperbola.
If either A or C is zero, the conic is a

parabola.

x2 - 2y2 + 4x + 4y + 4 = 0

d=20

c=~a 2 +b 2

Given equation: 4x 2 - y 2 = 16

a 2 =cd

303

Solving for c:

c = ~a 2 + b 2

e =;- 91.875

Hyperbo~

x2 y2
Standard equation: 2 - 2 = 1
a b
By inspection, a2 = 9 thus, a = 3 ;
b

=4 thus, b = 2

fI

304 1001 Solved Problems in Engineering Mathematics (2nil Edition) by Tiong & Rojas

Day 12 -Analytic Geometry (Parabola, F.1l!Ese & H.:meEola) 305.

Thus, the equation is,


x2

y2

---=1
22 42
x2

y2

P2 (5,1,-4)--~x 2

at (0,0): r 2

r2=x2+y2+z2
r2 "'(8)2 + (1 )2 + (6)2

x + y + z = 2---+ Eq.3

= x 2 + y2 + z 2

Substitute the coordinates of the given


point to the standard equation:

3x- 2y + z"' 3---+ Eq.2

2x +3y =0

Thus the point is {2, 1, -1)

Standard equation of a sphere with center

x + 5y- 2z 9 ...-+ Eq.1

2x-3y = 0 or

x=2

d=Ji1

r 2 =101
Subtract Eq.3 from Eq.1:

Standard equation of the ~symptotes of a


hyperbola with center at (0,0): y

=E.a x

By inspection, the coordinates of the


center is at (-2,2); a = 2 and c = 3.

By inspection:

E.a = 2 ---+ b = 2a

4y -3z = 7 -l- Eq.4

b = ~c 2 - a 2 = ~3 2 -2 2
b = -J5

Multiply Eq.3 by 3:
3x + 3y + 3z

Standard equation: (x- 2h)2 -(y- -k)2


=1
a
b2

=1

(x+2)

2 2

(y-2) _
(

(x+2f

(y-2)

4y-3(%-%v)=7

------1
4
5
2
5(x + 2) - 4(y- 2)2 20

25
9
- -2 - =2 1
4a

a=2

b =2a =2(2)

=-

Substitute to the standard equation:

(x + 3}2 +(y -2)2 + (z

9 15
4y--+-y=7

=
2
2
5(x + 4x + 4)- 4(y - 4y + 4) =20

2 2
8y - 9 + 15y = 14
y=1

5x + 20x +20-4y 2 +16y -16 = 20

3 5

5x -4y2 + 20x +16y -16 = 0

z=---(1)=-1
2 2

b=4

-4Y =62

x2 +6x+9+y2 -4y +4 + z2 -8z +16:::: 36

Substitute in Eq.4:

-!5)2

(5/2)2 _K=1

As given the center is at (-3, 2, 4), thus


3, k = 2 and I = 4 and the radius is 6.
h

3 5
z=----y

------1
22

16 = 4a

5y+2z=3

=r2

where: (h. k, I)= coordinates ofthe center

(3x + 3y + 3z)- (3x- 2y + z) = 6- 3

Substitute b = 2a and the coordinates of


point (5/2, 3) to the standard equation:

(2a)

=6 ---+ Eq.S

2
2
( y-k) _
(X h)
- - 2-.. - - - 2 - - 1

(x- h)2 + (y- k}2 + (z -1)2

Subtract Eq.2 from Eq.5:

Substitute:
xz y2
Standard equation: 2 - 2
a b

p:..fi'01

(x + 5y -2z)-(x + y + z) 9-2

Given equation of asymptotes: y = 2x

4a

d = J(5-6) + (1 + 2) + ( -4 +3)

2
y=-x
3

d=v\~z-x1) +(y2-y,) +(z2-z1)

b
y=-x

=5;y 2 =1;z 2 =-4

~--2

Ill

y in Eq.3:

x+(1)+(-1)=2

P,(6,-2,-3)..-+x 1 =6;y 1 =-2;z 1 =-3

----=1
4 16
4x2 -y 2 =16
Equation of the asymptotes of a
hyperbola with center at (0,0):

Substitute z and

x2 +y 2 +z2 +6x-4y-8z 7

Standard equation: (x - h )2 + (y - k)2

=r2

Substitute coordinates of the center and


radius as given:

-4)2 +(y -0)2 = 4 2


2
x - Bx + 16 + y2 16
(X

II

'
I

:~

306 lOO(Solved Problems in Engineering Mathematics (2nd Edition) by Tiong & Rojas
x 2 + y 2 = 8x ~ Rectangular equation

ml
. Standard equation: (x- h) 2 + (y- k) 2 = r 2
Substitute coordinates of the center and
ra~ius as given:

Topics

D
D
D D
l_j D
D D

(x- 3)2 + (y- 0) 2 = 32


Refer to the triangle:

y~

x2+y2=r2
x =rcos9
Substitute:

Refer to the triangle:

x2 + y2

y~
x.

x + y2 =6x

r =8cos9

ra = 6()1_ cos 9)

r-8cos9=0

r sin 2 e = cos e

Tue

r'a = 8()1_ cos 9)

Refer to the triangle:

=r2

x=rcos9

x2 +y 2 =8x

Mon

x2 - 6x + 9 + y2 =9
x 2 + y2 =6x

r = 6cos9

y~
X

r(~J =(7)

Refer to the triangle:


x=rcos9

y =r sine

Parabolas
- General & Standard Equations
- Eccentricity & Latus fjectum
Ellipses
-General & Standard Equations
- Major & Minor Axes
- Eccentricity & Latus Rectum
Hyperbola
... General & Standard Equations
-Transverse & Conjugate Axes
- Eccentricity & Latus Rectum
Polar Coordinates
- Radius Vector
- Polar Angle
Relation Between Rectangular
and Polar Coordinates

y~

Theory

Wed

Problems

Thu

Solutions

Fri

Notes

~
Sat

.x

y2 =x

Given: r = 5 sin 9 and 9 = 30

Note: This equation is a parabola with


vertex at (0,0) and opening to the right.

Substitute:
x=rcose

Standard equation:

y2 = 4ax

By inspection, 4a = 1 thus, a = 1/4

x = (5 sin e)cos e = 5 sin 30 cos 30;


X=

2.17

y =r sine
y = ( 5 sine) sine = 5 sin 2 30
y =1.25
Thus, the point is at (2.17, 1.25).

Thus, the focus is at (1/4, 0).

'

308 . 100 1 Solved Problems in Engineering Mathematics (2nd Edition) by Tiong & Rojas

Topics

[QJ
Mon

0
0

Tue

Theory

<";.<, ,,,

0
0
0

Problems

Solutions

Notes

Wed

Thu

0
Fri

Sat

What is Calculus?

What,is Differential Calculus?

The term "Calculus" was derived from a


Latin word "calx" which means "stone"
and from a Greek word "chalis" which
means "limestone".

Differential Calculus is a branch of


mathematics which deals with derivatives
and limits.

In 1684, a German mathematician and


philosopher Gottfried Wilhelm von
Leibniz published his early work on
calculus, while an English astronomer,
physicist and mathematician Isaac
Newton made an early study on the
subject in 1665 but did not published his
work until 1704. These two
rn<t!hematicians are now considered as the
founders of today's calculus.

<

Differential Calculus
Limits
Theorems of Limits
One-Sid~d Limits
Continuity
Special Limits
Derivatives
-Algebraic Function
-Exponential Functions
- Logarithmic Functions
-Trigonometric Functions
- Inverse Trigonometric
Functions
- Hyperbolic Functions
- Inverse Hyperbolic Functions

1111 ,;ulljed Calculus is divided into four


,,,,.,~.;

lldllldy, differential calculus,

inftqra! calculus, differential equations


.l!rtl c.llculus of v<Jriiltioro.

What is a Limit?
Let f be a function that is defined on an
open interval containing a, but possibly not
defined at a itself. Let L be a real number.
The statement
limf(x) = L

x->a

defines the limit of the function f(x) at a


point a. L is the value that the function has
as the point a is approached.
What are the Theorems of Limits?

lhu followin(J are important theorems and


pr<>Jll'tlr'; 11! lrr;~rt'; con:oirl.-:rinq thr:
fiiJJ(lWIII<J

310 100 ~ Solved Problems in Engineering Mathematics (2na Edition) by T1ong & Rojas
lim f(x) = L

and

lim g(x) = K

What are One-Sided Limits?

1. If lim f(x) exists then it is unique.


X->8

lim[f(x)+g(x')] = limf(x)+ limg(x)


X-Joa

, X-J-3

X-+3

Suppose f is a function such that it is not


defined for all values of x. Rather, it is
defined in such a way that it "jumps" from
one y value to the next instead of smoothly
going from one y value to the next.

3.

x-Joa

lim f(x) = f(a)


x--+a

The following are the three conditions


being satisfied:

1. f(a) exists, that is, f is defined at a,


lim log. x = +oo

X-Joa

x->0

2.

lim f(x) = ~i~ f(x) =!::


x->ag(x) limg(x) k
X->8
1

6. lim--=~
x->a

7.

What is a Derivative?

3. the two numbers are equal.

X-+8

lim[f(x)g(x)] = LK
X->8

5.

lim[f(x)j"

x ...... a

The figure above shows that y


defined for all values of x.

=f(x) is not

Continuity test at a point x

X-+3+

9.

What are Special Limits?


1.

lim sinx = 1
x-+0

= lim[f(x)J" fern>
x~~a

lim ex"

2.
The fi9ure above shows that y = f(x)
"jumps" from a positive value to a negative
value.

= c lim x" = ca"

11.

lim
X-----+a

'1fW = \Jnllim f(x)


X-+a

If f(x)::; h(x)::; g(x) for all x in an open


interval containing a, except possibly
at a, and if lim f(x) = L = lim g(x) then
X-Joa

limh(x) = L

(1 + ~)"
n =e

3.

n-+oo

4.

lim{h n)n = e

lim

X-+3

n-+0

3.

Likewise, the statement lim f(x) = L


X->8
means that as x af?proaches "a" from the
left hand side or from negative infinity, the
function f has the. limit L.

5.

de =O
dx
d
du dv
-(U+V)=-+dx
dx dx
d
dv. du
-(uv)=U-+Vdx
.
dx
dx
du
dv

~(~) = vdX -udx

d n
n-1 du
- u =nu dx
dx
du

6.

lim log)'= +oo

8.

X HOO

lim log. x =

x->a

if and only if lim, f(x) ~ lim f(x)


X >il

~o

2vu

7.

X--+-oo

limf(x) = L

~JU = d~
dx

lim ax= 0

v2

dx v

lim ax= +oo


X-HOC

Iff is defined in <:~n open interval containing


a, except possible at a, then

X ><I

4.

5. For a > 1, then:

X--+8

2.

x approaches "a" from the right or from


positive infinity, the function f has the limit

R.

= f(a)

when n is positive integer

12

Algebraic Functions:

lim 1- cosx = 0

x-Joa+

10. Iff is a polynomial function then


X-->8

A.

The statement lim f(x) = R means that as

X---4-8

lim f(x)

The following are the derivatives of the


different functions:

1.

~i~[cf(x)] = c[~~f(x)J
X--78

X-+3-

The derivative of a function expresses its


rate of change with respect to an
independent variable. The first derivative
of a function is the slope of the tangent
line to that curve defined by the function.

(K ot 0)

X--+0

8.

= a:

lim F(x) = lim F(x) = F(a)

(K ot 0)

g(x)

lim f(x) exists,


x--+a

lim[f(x)g(x)] = limf(x)-limg(x)
X-Jo3

lim IQg. x = -CJJ

X->+oo

----------4r-----------X-43

lim ax= +oo


X->-oo

lim[f(x)- g(x)] = L- K
X->8

4.

lim ax= 0
X--++

lim[f(x)-g(x)]=limf(x)-limg(x)

x-+a

A function f is continuous at a point a if

lim[f(x) + g(x)] = L + K
X->8

6. For 0 < a < 2, then:

What is a Continuity?

X-+a

X-Joa

2.

Day_l3 -Differential Calculus (Limits anci Derivatives) 311

-CJJ

9.

d (u)

dx

c =c1 dxdu

du
d c c dx
dxu=lT

3~2

B.

1001 Solved Problems in Engineering Mathematics (2nd Edition) by Tiong & Rojas
Exponential Functions:

dx

dx

F.

10. _E_(e") =e" du


dx
_ dx
C.

Hyperbolic Functions: .
26.

~(sinhu) = coshu du

logarithmic Functions:

27.

~(cosnu) = sinhu~~

11

d
log du
. -(log )
a edx
.u =-~
u

28.

12

d
log
du
. -(log )
10 e dx
1ou =
.u dx

30.

du
d
-. dx(lnu)=dx

31.

13

Proceed to the next page for your 13th


test Detach and use the answer sheet
provided at the last part of this book. Use
pencil number 2 in shading your answer.

d
-1
-1
du
25. -(esc u ) = - - = dx
uN -1 dx

-~(a")= a" Ina~-~

9.

Day 13- Differential Calculus (Limits and Derivatives} 313

29.

dx

GOOD LUCK I

dx

m:ribia:

dx
dx
d
2 du
--,(tanhu)=sech u-dx
.
dx
d
2 du
-(cothu) = -csch u dx
dx
d
du
-(sechu) = -sechutanhudx
.dx
d
du
-(cschu) =-cschucothudx
dx

Did you know that. .. the most proved


theorem in Mathematics is the
Pythagorean Theorem which has more
than 370 different proofs! All the 370
different proofs are found in the book
entitled "The Pythagorean Proposition"
which was published in 1940

c!euote:
"The mathematician does not study pure
mathematics because it is useful; he
studies it because he delights in it and he
delights in it because it is beautiful."

D.

G.

32.

d .
du
14, -(s1nu) =cosudx
dx
15.

16.

H.
18.
19.
E.

Inverse Hyperbolic Functions:

Trigonometric Functions:

d (COSU ) =- -SinU-
. du
dx
dx
d
du
2
-(tanu) =sec u-dx
dx
d

2 du
--(cotu) =-esc u dx
dx
d
du
-(secu) = secutanudx
dx
d
du
--(cscu) = -cscucotu-dx
dx
-

Inverse Trigonometric Functions:


20.

~(sin-1 u) = _1__ du

21.

du
-~(cos- 1 u) =- c--1
. -u2 dx

dx

~(arcsinhu) = ~
dx

- Henri Poincare

u2 + 1

d
1
33. -{arccoshu) = ~; Jxl > 1
2
dx
u -1
d
1
34. -(arctanhu) =--
lxl < 1
dx

1- u2 '
d
1
35. -(arccothu) =- ;
lxl > 1
dx
1- u2
d
1
36. -(arcsechu) = - - - ; O<u<1
dx
x~1-x2
d
1
37. -{arccschu) = ~; x
dx
JxJ 1+ x2

*0

~1-u2 dx

dx

22.

"1/l1
d
_1 _ _ _1___ du
dx (tan u)- 1+ u2 dx

23.

d~ (cor-1 u) = t+ u2 dx

24.

:x (sec-1 u) = uf:l1

--1

'""'

du

du

a;
_1

:111

Day 13- Differential Calculus (Limits and Derivatives) 315


5&o: EE Board April :1995
1- cosx
Evaluate:

A.
B.

Topics
Differential Calculus
limits
Theorems of Limits
One-Sided Limits
Continuity .
Special Limits
Derivatives
- Algebraic Functions
- Exponential Functions
- Logarithmic Functions
-Trigonometric Functions
- Inverse Trigonometric
Functions
- Hyperbolic Functions
- Inverse Hyperbolic Functions

Mon

D
Tue

D D
D
D D
D D
Theory

Problems

Soi~Jons

Notes

Wed
Thu

Fri

Evaluate: Lim

X--+1

1/5

B.
C.
D.

2/5
3/5
4/5

x +3x-4

2
-1/2

A.
B.
C.

D.

1/7

Undefined
Infinity

1
0

D.

Infinite

(x-4)

------o:'-----'2

Undefined

3/5

B.

4
6

cos (x!- + 2)

B.
C.
D.

B.

ECE Board November :199:1

3x + 6
3x- 3

6x- 3
6x + 3

C.
D.

4x{x2 + 1)
4xlog10 e
x2 + 1
log e(x)(x!- + 1}
2x(x!- + 1)

24

Differentiate (x2 + 2)

B.

26
28
30

A.

Evaluate:

Lim (2- x)
X--~

x-2

A.

C.
D.

D.

5&9: EE Board October :1997

559: ECE Board April :199~


x2 -4
Evaluate: M = Lim - -

C.

A.

C.
D.

X--+2

2x cos (x!- + 2)

- cos (x!- + 2) cot (x2 + 2)


2x sec (x2 + 2 ) tan (x!- + 2)

A.

Infinity
Zero

x-4

0
1
8
16

(x - x -12}

A
B.

,I

5&8: EE Board October :1997


Differentiate y = log 1o (x!- + 1)2

Evaluate: Lim (x!- + 3x - 4)


X--+1

x -16

II.,

5&&: EE Board October :1997


Differentiate y = sec (x!- + 2).

A.

56:&: EE Board Octuber :1994


.
3x 4 - 2x 2 + 7
Evaluate: Lrm ---,3, - - - 5x +x-3
X-+oc

5&~:

1r

5&7: CE Board November :1994

t.

A.
B.
C.

-ex sin x!ex (cos x!- - 2x sin x!-)


ex cos x!- - 2x sin x!-2xex sin x

What is the derivative with respect to x of


(x + 1)3 - x3 ?

5&4: ECE Board November :1994

557: ECE Board April :1998


Evaluate: Lim
x--+ 4

C.
D.

1/2

D.

A.
B.

A.
B.
C.
D.

B.

5&:1: ME Board October :1997

558: ME Board April 1998

X--+4

Compute the following limit:


X+4
Limit:
X-+oc x-4

Sat

Evaluate the Lim

_x_2_

C.

D.

55&: CE Board November :1997


x2 -1

Lim
X--+0

5&5: EE Board October :1997


Differentiate y = ex' cos -?-.

!I

B.
C.
D.

ez.
e'l/Jr

B.

tan~

C.
D.

112

(x 2 +2)1/2
2
X

(x 2 +2)1'2
2x
(x 2 + 2)1'2
(x!- + 2)3/2

570: EE Board October :1997


If y = (f + 2)2 and t = x 112 determine dy .
dx

Day 13- Differential Calculus (Limits anciJJerivatives) 317

316 1001 Solved Problems in Engineering Mathematics (2nd Edition) byTiong & Rojas

A.
B.

c..
D.

3
2

B.

2x 2 +2x
3
2(x + 2)
x5'2 +x1'2

57:1: ME Board April :1.997


What is the first derivative of the
expression (xy)x = e?

4
(1-16x2 )o5

c.
D.

B.

--4

C.

(1- 4x2)o.s

D.

4
(1- 4x2 )o5

B.

0
X

D.

(1+1nxy).
X

-y (1-lnxy)
x2

A.
B.

B.

(x + 1)2 _ (x + 1)3
X
X
4(x + 1)2 _ 2(x + 1)3
X

c.

D.

2(x + 1)3

(x + 1)3

--x---r
3(X+ 1)
1)
--x----xz
(X+

Find the derivative with respect to x the


function .J2- 3x 2

576: ECE Board November :199:1

-2x 2
.J2-3x2

B.
C.

-3x

Differentiate the equation: y =_x_


X+1
x2+2x

A.

(x+1t

.J2-3x2
-3x 2

B.

.J2-3X2

c.

3x

D.

D.

.J2-3x2
573: EE Board April :1.995
Find y' if y = arc sin cos x

A.
B.
C.
D.

-1
-2
1
2

A.

D.

--4
(1-16x 2 ) 0 5

X+1
2x

A.

10x- 5
6x-10
3x + 10
3~-5x.

A.

B.
C.
D.

A
B

3x2 -5
8

c;
ll

A.
B.
C.
D.

2
-1
-1/2

-2

587: CE Board May :1.998


Find the slope of the curve ~ +
10y + 5 = 0 at point (1 ,0).

A
B.

-0.1463
-0.1538
-0.1654
-0.1768

2.21
-4.94

1~

58fn ECE Board November :1998


Find the slope of x2 y = 8 at the point (2,2).

C.
D.

1/5
2/5
1/4

Find the slope of the tangent to the curve,


y = 2x- ~ + x3 at (0,2).

C.
D.

1
2
3
4

589: ECE Board April :1999


.Find the coordinates of the vertex of the
parabola y = ~ - 4x + 1 by making use of
the fact that at the vertex, the slope of the
tangent is zero.

3.25

/21

l- 6x +

588: CE Board May :1996

A.
B.

~~

~1~

S8:Jz EE Board October :1997


If y = 4 cos x + sin 2x, what is the slope of
the curve when x 2 radians?

1~

B.
~

B.
C.
D.

64~

A
C.

A.

578: CE Board November :1993


Find the second derivative of y by implicit
differentiation from the equation 4~ + 8y2
= 36.

585: ECE Board November :199:1


Give the slope of the curve at the point

2 sin (x2 + 2) c~s (~ + 2)


-2 sin (~ + 2) cos (~ + 2)
8x sin (~ + 2) cos (~ + 2)
-8x sin (x2 + 2) cos (~ + 2)

D.

2x + 1 at x = 1.

x3

582: CE Board May :1996


Find the slope of the ellipse ~ + 4y2- 1Ox
- 16y + 5 = 0 at the point where
y = 2 + 8 5 and X = 7.

2x 2
X+1

577: CE Board November :1995


The derivative with respect to x of2 cos
(~ + 2) is

A.
B.
C.

574: CE Board May :1997


Find the derivative of arc cos 4x.

1/2
1/3
1/4.

(1,1): y=--2x+1

58:1: ME Board April :1998


Given the function f(x) = x to the 3rd power
- 6x + 2. Find the first derivative at
X= 2.

B.
C.

0.

C.
D.

D.

y2-5
y2
xy- 5y
2xy

A.

580: ME Board October :1997


Find the second derivative of x 3 - 5~ + x =

A.
B.

572: ME Board April <:f98

A.

3
Find the derivative of (x + 1)

A.

c. -y

curve y = x 3

32 xy
16 3
-y

575: CE Board November :1.996

C.

-y

584: ECE Board November :199:1


Find the slope of the line tangent to the

579: ME Board April :1998


Find the partial derivatives with respect to
x of the function xy2 - 5y + 6.

A.

4y

A.

(2,-3)

:Jls

1001 Solved Problems in Engineering Mathematfcs (2nd Edition) by Tiong & Rojas

B.

(3,-2)
(-1 ,-3)
D. ' (-2,-3)

595: ECE Board November 1.996

c.

Find the radius of curvature at any


point in the curve y + In cos x = 0.

590: ECE Board April 1:999


Find the equation-of the normal to
. 5 at the point (2, 1).
A.
B.
C.
D.

x? + y2 =

A. cos X
8. 1.5707
C. secx
D._ 1

......,.,._,_____

~
lv1on

y = 2x
2y
2x + 3y = 3
X+ y = 1
X=

What is the equation of the normal to the


curve x? + y2 = 25 at (4,3)?

Tue

A.

B.
C.
D.

,,I

..

5x + 3y = 0
3x-4y = 0
3x + 4y = 0
!}x- 3y = 0

59%: EE Board April :1997


Locate the points
X: e

y = f(x)
A.
B.
C.
D.

of inflection of the curve

J3
2 J2
-2 J2
2 J3

-2

i.;

Theory

Wed

Problems

Thu

Solutions

Fri

Notes

Sat

ANSWER KEY
556. B
557. D
558.
559.
560. B
561. A
562.
563.A
564. B
565. B

points.

B.
C.
D.

c
c

(2,18) & (-2,-14)


(2,18)&(2,-14)
(-2,18) & (2,-14)
(-2,18) & (-2,14)

5941 CE Board November :1997


Find the radius of curvature of a parabola

566.C
567. D
568. B
569. B
570.C
571.
572. B
573.A
574.A
575. D

576.A
577. D
578. B
579. B
580. B
581. A
582. D
583. B
584.A
585. D

RATING
:,"

586. D
587. B
588. 8
589.A
590. B
591. B
592.
593.A
594.A
595.C

";'

22.36
25.78
20.33
15.42

units
units
units
units

~~

c:J 34-40
c:J 24-33

Topnotcher
Passer

20-23 Conditional
0-19 Failed

If FAILED, repeat the test.

y2 - 4x = 0 at point (4,4).
A.
B.
C.
D.

Topics
______

.\

593: ECE Board November :199:1


In the curve 2 + 12x- x 3 , find the critical

A.

Differential Calculus
Limits
Theorems of Limits
One-Sided Limits
Continuity
Special Limits
Derivatives
- Algebraic Functions
-Exponential Functions
- Logarithmic Functions
- Trigonometric Functions
- Inverse Trigonometric
Functions
-Hyperbolic Functions
- Inverse Hyperbolic Functions

D
D D
D D
D
D D

59:11 CE Board May :1995

~,

'

-~

320 1001 Solved Problems in Engineering Mathematics (2nd Edition} by Tiong & Rojas

x2 -1
.
L1m -..--::-2
x-.1

Apply L'Hospital's rule & substitute x = 4:


2x
.
L 1m=-= 2 x

x-->>

lim= 2(4) =8
Substitute x = 1:
Lim=

(1) ~1

(1)1 +3( ).
1 4

=0 .indetenninate

Note: Using L'Hospital's rule, differentiate


separately the numerator and
denominator and substitute the value of
limit to the variable
Lim=

x-2

-5

x = 2:

.I

x-+o

Lim=(2~1)1an.'!~ =1"',

indete1minate

5x +X -3

= oo:
.

Lim= 3(oo)4- 2(oo)2 + 7.


5(oo)3 +oo--3
'1\:

-~

-x-2

00

Apply L'Hospital's rule & substitute x

_1tx (In

2
..: ln(2- x)

lim == .~. indeterminate

~r_,

.
1-cosO 0 . d t
.
L1m = - -2- =- 1n e ermmate
(0)

In lim= tan

tanJI!I
2

tan n:x

= oo:

12x 3 - 4x
Lim = - 15x2 +

(2- x))

2
.
ln(2--x)
in Lim ::: ------

cot2:.~
2

Lim= 2x-1

:..!
7

sinx
.
Llm=-2x
Lim = sin 2(0) = ~ indeterminate
2(0)

.
12(oo ) - 4( oo) = 00 indeterminate
Ltm == 15( 00 )2 + 1
oo

Note:

Apply L'Hospital's rule again:

d(lnu) = - and d(tan u) =-esc u du


u

36x 2 -4
Lim=

Apply L'Hospital's rule again:


2

Lim=

Lim x -16
X-+4

Substitute x ::: 1:

In Ltm =In (2-x)

Take In on both sides:

Apply L'Hospital's rule & substitute x = 0:

Lim= 2 (4 )-1

X-+1

Substitute x

Apply L'Hospital's rule & substitute x = 4:

Ill
Lim (2 _ X)tan 1t~

x->oo

Lim 1-cosx

4-4

x = oo:

. 3x 4 - 2x 2 + 7
ltm --~-
3

lim =2(2}=4

(4~ -4-12 = 0 indeterminate

oo-4 oc.

Lim=2=1
1

Substitute x = 4:

Lim =(4)2 + 3(4)- 4


Lim= 24

Substitute x = 2:

.
2x
L1m=-= 2x

x-4

Substitute x = 4:

x = co:

Apply L'Hospital's rule & substitute

Apply L'Hospital's rule & substitute

Ll m2- - x-+4 x -x-12

Lim=

x-+2

and Deriv!!_tives) 321

X->4

oo+4 co 'd
.
=---=-,
m etermmate,

. X2 -4
Llm.--

2x
2x+3
2(1)+3

.
L tm

~Limits

lim x2 + 3x- 4

x-4

Substitute

.
(2)2 - 4 0 . d t
.
L1m = - - =- m e ermma,e
2-2

Lim=~-2

. x+4
Ltm - -

x +3X --4

Day 13- Differential Calculus

Lim =

COSX

Lim= cosO

---2-=21

Substitute x = 4;

l-

36( oo
4 _ '!!._ indeterminate
30( oo)- - <X> '

Apply again L'Hospital's rule:


ltm '-

4 2 16
Lim= ( ) "
=.Q. indeterminate
4-4

l2x

30

72(co)
.:: .. --'~ oo

30

Apply L'Hospital's rule:

-1

30x

X -4

du

-1

lnlim=---.:._x_ =

csc2 n;(~)
-1

:n:

1t

2
In Lim= 2/n

2-1

-csc2~~)(~)

322. 1OOJ Solved Problems in Engineering Mathematics (2nd Edition) by Tiong & Rojas
Take exponential on both sides:
eln Lim

= e2/n

!I

,
y=

where:
u = (x 2

Note: d(uv) = udv + vdu

+ 1)2

du = 2(x 2 + 1)(2x) = 4x(x 2 -t1)


2

where:
u =ex; du = ex; v = cos x 2 ; dv = -2x sinx2
y' = ex(-2x sin x 2)+ cos x 2 ex
y'= ex(cosx 2 -2xsin x 2)

y' =log1o e

4x(x +
[ ( x2 +

In (xy)x = In e
x In (xy) = 1

ml

Note: d (uv) = udv + vdu

y =cos-14x

IB

xy' = - y - y In xy
xy'

=- y(1 +In xy)


X

Y =~2-3x

where: n=112;u=x 2 +2;du=2x


y' = ~(>.<z + 2 )112-1( 2 x)

Note:

y' = x(x2 + 2f112

= nu"-1 du
2

y' = 3(x + 1) (1)- 3{x) (1)


y' = 3(x + 1 )2 - 3(x)2

-4

y = (x + 1)

du

dJU = 2JU

Note: d(

=---X -

y' = 3(x 2 + 2x + 1)- 3x 2


y'= ~ +6x+3-}~

y = [(X112 )2 + 2

y'=6x+3

Y = (x +2)2

v::: x;

_J

vdu-udv

v2

v 2 = x 2 ; dv = 1
2

,_3\(x+1)

y-

sin 1cosx

wtww u

~)

y' = x(3)(x+1) -(x+1)


x2

Nterl(sin-

y' = 2(x +2)

y'=-~2-3x 2

u = (x + 1)3 ; du = 3(x+1 )

-3x

Substitute Eq.1 in Eq.2:

where:

y' = 2~2 -3x 2

t=x 112 ~Eq.1


y = (tz + 2)2 ~ Eq.2

u):::

du = 4

where: u = 2-3x 2 ; du =- 6x

~1-16x 2

v = (1-16x2

-4

y'=

y' = -y (1 +In xy)

-{)x

y = (x + 1)3 - x3
Note: d(u)"

where: u = 4x; u2 =16x

xy'+y+ylnxy=O

Note: d(u)" = nu"1 du

d{ cos-1u) = ~1- u2

\ [ x\:y ]+In xy(1) = 0

y' = 4x log10 e
x 2 +1

-du

1}

y = sec(x + 2)

y' = 2x sec (x 2 + 2) tan (x 2 + 2)

~sin 2 x

Sin X

y' = -1

Note:

Y= (xz + 2)1/2

= sec (x 2 + 2) tan (x 2 + 2)(2x)

-sin x

=--

Differentiating both sides:

1)]

where: u = x 2 + 2; du = 2x

~1- cos

Take in on both sides:

Ill

Note: d sec u = sec u tan u du

-sin x

, -sin x
y=-.-

Note: d(log10 u) = log10 e ( duu)

y =ex cos x 2

y'

Day 13- Qifferential Calculus (Limits and Derivatives) 323.

(xy)x =e.

Y = log1o (x2 + 1)2

Lim= e2 /n,

cosx; u2

=cos 2 x;

2
(

x + 1)

y------2-

v1- u2

du =-sin x

- -2 x

x2,

, _ 3(x + 1)

(x+1)

(1)

324 1001 Solved Problems in Engineering Mathematics (2nd Edition) by Tiong & Rojas

J
2
+ 2)sin{x + 2) J

! (

y' = -4x[ 2cos( x + 2)sin( x + 2)


y' =-ax[ cos( x

x2
y= x+1

Note: d(~) = vdu -.udv

v2

xy

!(

xy

4x 2 +8y 2 = 36

x2 + 2y 2 = 9 ~ Eq.1

where:
u.,x2 ; du = 2x

Differentiate both sides:

v = x + 1; v 2 = (x + 1)2 ; dv = 1
2

(x+1)2

2x 2 + 2x- x 2

2y

, x2 +2x

Take second derivative:

Y = -(x_+_1-)
2

y" =

5y + 6) = y (1) + 0

5y + 6) = y

y' = 4 (-sin x) +cos 2x (2)


y' = 2 cos 2x - 4 sin x
At x = 2 radians

-M y(1~;

1 00

=3x2 - 1Ox + 1

y' = 2 cos 2[ 2( :

xy']

~ Eq.3

y = 2cos (x + 2)

y' =-4.94

rm.

Differentiate and substitute x

e = 1+cos 26

Note: cos2
.
S1mplifying:
2

y-x( -x

y"=-~

2y

J]

.,

y2

= 2:

y'=6

Differentiate and substitute x = 1:

1!1

y' = 3x 2 -2.
y' = 3(1)2 -2

y+ X

Y = ~[ 1+ cos i x +2)]

y"=-i[ /y2]

Y 1 +cos 2(x 2 + 2)

y = 1 + cos (2x 2 + 4)

2l

Note: d(cosu) = -s.inu du


where: u =2x 2 + 4; du

= 4x

y' = -4x sin 2(x 2 + 2)


Note: sin 29 = 2 cos. e sin

2y

.9
y"=--34y

As given: y = 2 + 8 5 = 4.828, x = 7
Differentiate & substitute x = 7 & y = 4.28

Thus, the slope of the line is equal to 1.

2x + 8yy' -1 0 -16y' = 0

y"= 2y2 +x2


4y3
Substitute Eq.1 in

y' =-sin (2x + 4)(4x)

y'(8y -16) = 10- 2x

y":

, 10-2x
y=-8y-16

I
I

'-

10-2(7)
y- 8(4.828)-16
y' = -0.1768

y' =1
Note: Since the line is tangent to the
curve at x = 1, then the slope of the line is
the same as the slope of the curve at the
given point.

Note: slope = y'

y" = __1_. [2y2 + x2

Y=x3 -2x +1
Note: slope = y'

y' =3(2)2 -6

x2 +4y2-10x -16y +5 = 0

Y = 2cos (x + 2)

) ]-4 sin [ 2( 1:0")]

y' = 2 cos 229.183-4 sin 114.591"

y = x3 -6x +x

Substitute Eq.2 in Eq.3:

Differentiate:

y' = 3x 2 -6
2

Calculus (Limits and Derivatives) 325

y = 4 cos x +sin 2x

y" = 6x-10

-X
y'=-~Eq.2

y' = .=_--=~(X+ 1)2

y'

-2x
y'=4y

(x+1) 2

13.-_Q_if!~enti<ll

y = x 3 -5x 2 + x

2x,+4yy' = 0
2

y' = (x + 1)(2x)- x ( 1) =2x + 2x- x

Day

x3

y=--2x+1

Note: slope = y'


Differentiate and substitute x = 1:

iir'

,I
!. {

Day 13- Differential Calculus {l.ixnits a.11d Derivatives) 327

326 1001 Solved Problems in Engineering Mathematics (2"d Edition) by Tiong & Rojas
Note: slope

= _!(3x 2 )- 2

y'

Differentiate and subsfih.ite x c,.O.

y'::: ~(3)(1 )2 - 2
4

y' = 2 - 2x + 3x 2

y'=-1

y' = 2- 2(0) + 3(0)

y'

Note: Since the line is normal to the curve


at the given point, the slope (m2) of the
line is equal the negative reciprocal of the
slope of the given curve.

Note:
d(uv) = udv + vdu

1
1 - 1
mz = - m1 = - -2 - 2

y' = x 2 (ex)+ex(2x)

x y =8
8

y=x2

y-y1 =m(x-x 1)

y" = x 2 ex + 4xex + 2ex

Thus, the slope of the line is equal to 2.

2y-2 = x-2

Differentiate and substitute x = 2:

1
2

x3

- -4)(4) -4(1)(2) = -4.J8


x2(1)
2
x=

Differentiate and y' = 0:

Let: m1 = slope of the given curve


m2 = slope of the normal line

y' = 2x-4

Differentiate and substitute x = 4 & y = 3:

0=2x-4

Ill

2x +2yy' = 0

x=2

x +f-6x+10y+5=0

Differentiate and substitute

x = 1 & y = 0.

y'(2y + 10) = 6- 2x
6-2x
'- 6-2(1)
y- 2(0)+10

y'=~
5

4~~

y-y1 =m(x-x1 )

-10

y-3=-(x-4)

Let: m1 = slope of the given curve

4y -12 = 3x -12
Differentiate and substitute x = 2 & y = 1:

3x-4y=O

2x +2yy' =0
,

1m

y =--=-y
1

~
~
3

Y=x 1 ex

y' =-2
ml co . ;:o

-42.J2

=-2.J2

Substitute the values of x to the given


equation to solve for the values of y:
y =(-2+J2)2 e(Z+J2) =0.19or
y=(-2-J2) 2 e(Z-J2) =0.38

Using point slope form:

x2 + Yz = 1

y' = 2y +10

mz=--=--=m1
-4/3 4

Thus, the vertex is at (2, -3)

2x + 2yy' - 6 + 1Oy' + 0 = 0

=-- =-y 3

4
m1 =-.3
1

y = (2)2 - 4(2) + 1 =- 3

= y'

Y =2x- x2 + x

Substitute x = 2 to the given equation:

Note: slope

By quadratic formula:

x2 + y2 = 25

16

y' =-2

O=x 2 +4x+2

Note: slope = y'

y=-(2)3

O=x 2 ~ +4x~ +2~

x=2y

Y = x 2 - 4x + 1

, _ S(-2x) _ 16
y ------

At point of inflection, y" = 0:

y-1=-(x-2)

Note: slope = y'

x4

Note: Since the line is tangent to the


curve at (0,2), then the slope of the line is
the same as the slope of the curve at the
given point.

d(eu) = eudu

y" = x 2 (ex) + ex(2x) + ex(2) + 2x(e')

~2

Using point slope form:

'

= y'

Note: From the choices, only values of


the x-coordinates are given. Thus the
suggested answer is choice C.

Y= 2+12x-x 3

Note: Critical points are points wherein


the slope of the curve is zero.

y' = 12 -3x 2

0 = 12 -3x 2

x2

=4

X=

/
328 . 100 1 Solved Prob1Efms i!l Engineering Mathemati~_:C! Edition) by Tiong & Rojas
Substitute the values of~ to the general
equation to solve for the\values of y:
y = 2 + 12(2)- (2)3

=18 or

y =2+12(-2)-(-2)3 =-14

&

12

R:: [ 1+(yiJ

Topia

jy''l -

Thus, the points are (2, 18) and (-2, 14).

y+ln

Ill

~~

'{

=0
y =-In cos X

COSX

Note: d(ln u) = \]u


u

R =I1+(y')2r2

ly''l-

where: u = cosx; du =-sin x

l-4x =P

y=-=2y y
.. _ y{O)- 2y' _ 2y'
y . ---y2
y2

y"y2 =

Mon

D
rue

,:'!'

,,:{I

...

Wed

D
D D

Problems

Thu

Solutions

Fri

y" =sec2 x

Substitute y' and y" to solve for R:

12

-2(~)

0'
...

~~~

y, =- (-sinx)
- - =tanx
cosx

2yy'-4=0

RJ1+(tan4J

4
. y"--- y3
Substitute y = 4 to solve for y' andy":

Sat

sec 2 ;

Note: 1+ tan 2 x = sec 2 x


2

Differential Calculus
Limits
Theorems of Limits
One-Sided Limits
Continuity
Special Limits
Derivatives
-Algebraic Functions
-Exponential Functions
- Logarithmic Functions
- Trigonometric Functions
- Inverse Trigonometric
Functions
- Hyperbolic Functions
- Inverse Hyperbolic Functions

3/2

R= ( sec x)

y::3::3:::.!
4

. sec2 x

y" = --~ :: ___i_ = _...!_


y

(4)

16

R::: sec x

., < ~

sec2 x
R=secx

'.,

->'

j<- 0< X

, ,< ":

"{ >

;,,.

0.

'>< o

'i-

~ ~

> .' "l '(

<

~-..;,:" ~ ~

>

<

<, ;<

~ .0 ~

'" "'

, <

>

x.

,< 0

"0 :

0 {, "<,

~ 1-: ,,. '

''

_., .,.

"-<: .< _."


--;. " ' "

'< ..... ,-<

< -<> "-

tv.;

V X 0

< '', ~

o < '

o'

' .... 0 '

'> "< " ' :

:!

'<"

~ !"

'

""'

:,,..,x

> )

> 1o .; k ~"' ~

0.0 'A

[<

_.,.

<I ,. '

Substitute y' and y" to solve for R:

_Hi}T

R----

> "(

.0 X

~ "< )' '~'._, ~ ., ~

t:. .

1
16
R =22.36

T
T
'

~ o" <'

, ...

,:,,

<

<

,->:

t ' >

t.

:~
Topics

Mon

[QJ
Tue

0
0

\1\fed

Theory

Thu

Problems

: < ,< <:<

Fri

Solut;ons

~ ~ :" (;- 9

Notes

"~ ::. , ;_

f
~ .; "

>. """'"

:<<,<';.< .~,.;

Sat

What are Maximum and Minimum


Values?

If a function f is defined on an interval!,


then:

1. f is increasing on I if f(x,) < f(x2)


whenever x,, x2 are in I and x1 <. x2.

/(!~1

Xi

!f(x2)

j
X;

X2

Decreasing function
3. f is constant if f(x,)

=f(x2) for every

x,, x2 in I.

f(x,)

.--t--

Maximum and Minimum Values


Local Maximum and Local
Minimum
Steps in Solving Maxima/Minima
Problems
Steps in Solving Time Rates
Problems
Relation Between Variables
Under the Condition of
Maxima/Minima

X2

Suppose f is defined on an open interval!


and cis a number in I, then:

Increasing function

;>

f is decreasing on I iff(x,) > f(x2)


wlwnever x,, X:> in I.

1. f( c) is a local maximum value if f(x)


:s: ~(c) for all x in I.
.,

2. f(c) is a local minimum value ifl(x)


> f(c) for all x in I.

Day 14.:.. Differential Calculus (Maxima-Minima & Time Rates) 333


332 l 00 1 Solved Problems in En

4
How to Solve Maxima and Minima
Problems?

1.
2.
3.
4.

The following are the steps in solving


problems involving maxima and minima:

Draw a figure when necessary.


Formulate equation.
Differentiate with respect to time.
Substitute the condition I instant
to the equation.

1. Draw a figure when necessary.


2. Determine which variabl~ (the

Important: Substitute the given values


only after differentiating.

dependent variable) is to be
maximized or minimized
Formulate equation
Reduce to one variable.
Differentiate
Equate to zero

What are some of the Relationships


between the Variables and the
Maxima/Minima Values?

3.

4.
5.
6.

In step no. 5, the formulas in finding


derivatives will become very useful. It is
recommended to memorize all the
formulas as mentioned in the previous
chapter.

Largest rectangle that can be


inscribed in a right triangle with the
sides of the rectangle parallel to
the legs of the triangle.

:I, . ~
!_

5.

The following are the relationships


between the variables under the condition
of maxima/minima:
Largest rectangle inscribed in
circle.

Largest rectangle that can be


inscribed in an ellipse.

X=

2.

Minimum point'
slope= 0

0
0

---~~~

no
fence
needed

X
X=2y

6.

Minimum Point
Inflection Point

Rectangle with given area and


minimum perimeter to be fenced
along 3 sides only.

The maximum rectangle is a square.

9.

-'----~

.fi

slope= 0
Maximum point

First
Derivative

Y=2

When the first derivative (slope) fs


equated to zero, it results to either
maximum point of minimum point.

I Maximum Point

Rectangle with given area but with


minimum perimeter.

Dy X=y

b
2
h.

X=-

1.

8.

Largest rectangle that can be


inscribed in semicircle.

How to Solve Time Rates Problems?


Another types of problems involving the
use of derivatives are the Time Rates
problems.

3.

.fi

7.

Largest rectangle that can be


inscribed in a triangle with one
side lying on the base of the
trjangle.

perimeter or maximum area.

~b=C=i

In solving a problem under time rates, the


following steps are to b~considered:

=Y

11. Maximum area with perimeter (P)


given.

Sector with given area but


minimum perimeter.

equilateral triangle

X=--

4.268

y
X

2
h
Y=-2

=45

r=JA
e = 2 rad

b
X=-

10. Right triangle with maximum

Largest area of a triangle with


given perimeter.

"l~/2
Second
Derivative
Negative
Positive

y=

Day 14 - DifferentialgJlJq_t11~_(Maxima-Minima & Time Rates)

334 100 l.So1v'

12. Maximum light a~ittance for a


rectangular win do surmounted
with an isosceles tri ngle.

16. Stiffest beam that can be cut from


a circuJar section of radius r.

19. Most efficient trapezoidal section.

~-

clock.

It is

\X

t<=

~----

y = x.../3
-

I....,

,..l'

X= 2y

h'

e is maximized,

X= ~Y1Y2

23. Parallelepiped with maximum


width at top = sum of sides
width at top = 2x

It

1\ :

20. Length of rigid beam that can pass


a perpendicular hallways.

18. largest rectangle that can be


inscribed in a given ellipse.

'
~

L=a+b

X=Y=Z

:. a cube

24. Open squar~ container with


maximum volume.

X=2bH

y=2a~

LJJ
.

...

14. Maximum length of line segment


tangent to an ellipse.

volume.

e = 12oo

"Strength is proportional to the product


of breadth (x) and square of the depth
(y)"
b

a,

.... "

......... .

~
.

L = ~I{ a2/3 + b2/3 )2

X=2y
's_u_rf=-a_c_e_a_r_e_a
X=
3

21. Minimum length of ladder/rod to be


extended from ground to a wall
with an intervening fence.

2~.

Location of single stake at ground


level to minimize length of wire.

15. Rectangle of maximum perimeter


inscribed in a circle of radius r.

stake

h2

h1

~ r}

x=y

base (not side)


from an elliptical section.

........

Best view means

Ym]Y
...........

17. Strongest beam that can be cut

a
rx/2

~q'T~.w&s:<t20t~~-,

--=
:

13. Maximum light admittance-for a


Norman wind

dl:J

of a regular hexagon.

width at top

~"V"~

22. Best possible view of a picture or

"Maximum capacity with minimum


perimeter"

"Stiffness is proportional to the product


of breadth (x) and cube of width Cv)."
X I

335

Aellipse

iC?'"'

_ 1t

,(\rectangle -

I
i-i

L=

~(a2/3 +b2/3t

dh1
x- -- h1 +h2

336. 10.0 l Solved Problems in Engineering Mathematics (2ild Edition) b~g & Rojas

26. least amount of material to be


used for a square base rectangular

30. least amount of material for a


given volume.

parallelepiped.

r=

Day 14 - Differential Calculus (Maxima-Minima & Time Rates) 337


,;,

'.-\.

34. Maximum volume of right


circular cylinder inscribed in a
sphere of radius r.

J2

=2y

=t'2{Volume)
31. Maximum v<,>lume of cone with a
given slant height.

27. Least amount of material to be


used for an open top cylindrical

.
-

-~1tf3
J27

c-

Proceed to the next page for your 14th

tank.

test. Detach and use the .answer sheet

h=~
.j3

provided at the last part of this book. Use


pencil number 2 in shading your answer.

e =tan-1 J2

GOOD LUCK!
~

m:ribia:

32. Volume of largest cone, Vc that can


be inscribed in a hemisphere.

r=h

28. Minimum cost for a given volume,

v.

1
vc =-V.h
2

clauote:

33. Largest cylinder that can be


inscribed in a cone.

r=f{:

Did you know that. .. the word "Algebra"


comes from an Arabic word "al-jabr''
meaning to transpose terms from one side
of an equation to the other! This was
introduced by a Persian mathematician, aiKhowarizmi in around 825 A.D.

"The most beautiful thing we can


experience is the mysterious. It is the
source of all true art and scienCe.
- Albert Einstein

-:\----------r

29. Ratio of the weight of heaviest


cylinder, We to the weight of the

r---

r.

circumscribing sphere, W,..

we-

w: ./3

Y=3

Day 14- Differential Calculus (Maxima-Minima & Ti1;11e Rates)

&oo: CE Boart~J~ay

1.'9~

Find ttie minimum amount of~sheet that


can be made into a closed cylindB_r having
a volume of 108 cu. inches in square
Inches.

Topics

Maximum and Minimum Values


Local Maximum and Local
Minimum
Steps in Solving Maxima/Minima
Problems
Steps in Solving Time Rates
Problems
Relation Between Variables
Under the Condition of
Maxima/Minima

Mon

[QJ
Tue

Theory

Problems

0
0

Solutions

Notes

Wed

0
0
0

Thu

Sat

59&: ECE Board April 1999

598: EE Board March 1.998

Find the minimL,Jm distance from the point


(4,2) to the parabola y 2 = 8x.

A triangle has variable sides x,y,z subject


to the constraint such that the perimeter is
fixed to 18 em. What is the maximum
possible area for the triangle?

A.

4/3

B.

2J2

A.

f3
D. 2/3

C.
D.

c.

597: EE Board April 1.990


The sum of two positive numbers is 50.
What are the numbers if their product is to
be the largest possible.
A.

~&~

B.
C.
D.

W&~

25&25

W&W

B.

15.59
18.71
17.15
14.03

cm 2
cm 2
2
cm
cm 2

599: EE Board October 1997


A farmer has enough money to build only
100 meters of fence. What are the
dimensions of the field he can enclose the
maximum area?
A.
B.

c.
D.

\.

25m x 25m
15mx35m
20mx30m
22.5 m x 27.5 m

C.
D.

1.22
1.64
2.44
2.68

&os: EE Board April 1.997

A.
B.
C.
D.

125.50
127.50
129.50
123.50

&OJ.I ME Board April 1.998


A box is to be constructed from a piece of
zinc 20 sq.in by cutting equal squares from
each corner and turning up the zinc to
form the side. What is the volume of the
largest box that can be so constructed?

A.
B.
C.

599.95
592.59
579.50
622.49

cu
cu
cu
cu

in.
in.
in.
in.

The cost of fuel in running a locomotive is


proportional to the square of the speed
and is $ 25 per hour for a speed of 25
miles per hour. Other costs amount to $
1QOper hour, regardless of the speed.
What is the speed which will make the cost
per mile a minimum?
A.

B.
C.
D.

40
55
50
45

&o&1 ME Board April199&

&0:&1 EE Board April 1.997

The cost C of a product is a function of the


quantity x of the product : C(x) = x2 - 4000
x + 50. Find the quantity for which the cost
is minimum.

A poster is to contain 300 (em square) of


printed matter with margins of 10 em at the
top and bottom and 5 em at each side.
Find the overall dimensions if the total
area of the poster is minimum.

A.
B.
C.
D.

A.

&071 An open top rectangular tank with

D.

Fri

A.
B.

339

B.

C.
D.

27.76
20.45
22.24
25.55

em,
em,
em,
em,

47.8
35.6
44.5
46.7

em
em
em
em

&031 CE Board No-vember 1.99&


A norman window is in the shape of a
rectangle surmounted by a semi-circle.
What is the ratio of the width of the
rectangle to the total h1eight so that it will
yield a window admitting the most light for
a given perimeter?

A.
B.

C.
D.

1
1./2
2
2/3

1000
1500
2000
3000

square bases is to have a volume of 10 cu.


m. The materials for its bottom are to cost
P 15 per square meter and that for the
sides, P6 per square meter. Find the most
economical dimensions for the tank.
A.
B.
C.
D.

1.5m x 1.5m x 4.4m


2m x 2m x 2.5m
4m x 4m x 0.6m
3m x3m x Um

&o8: ME Board October 199&


What is the maximum profit when the
profit-versus-production function is as
given below? P is profit and x is unit of
production.
8

6041 CE Board May 1.998


I Jetermine the diameter of a closed
;ylindrical tank having a volume of 11.3
cu. m to obtain minimum surface area.

p
A.
B.
C.

= 200,00- X- ( -11
-)
\X+ 1

285,000
200,000
250,000

Day 14- Differential Calculus (Maxima-Mi~ma & Time Rates) 341

340 1001 Solved Problems in Engineering Mathematics (2" 0 Edition) by Tiong & Rojas
D.

613: EE Board October 1993

305,000

609: EE Board October 1993


A boatman is at A which is 4.5 km from the
nearest point B on a straight shore ~M. He
wishes to reach in minimum time a point C
situated on the shore 9 km from B. How far
from C should he land if he can row at the
rate of 6 kph and can walk at the rate of
7.5 kph?
A.
B.
C.
D.

4.15 km
3.0 km
3.25 km
4.0 km

At any distance x from the source of light,


the j.r{tensity of illumination varies directly
a~he intensity of the source and inversely
the square of x. Suppose that there is a
1ght at A, and another at B, the one at B
having an intensity 8 times that of A The
distance AB is 4 m. At what point from A
-.Qil1 the line AB will the intensity of
mination be least?

A.
B.

C.

~-~

610: EE Board March 1998


A fencing is limited to 20 ft length. What is
the maximum rectangular area that can be
fenced in using two perpendicular corner
sides of an existing wall?

A wall "h" meters high is 2 m away from


the building. The shortest ladder that can
reach the building with one end resting on
the ground outside the wall is 6 m. How
high is the wall in meters?

A
B.
C.
D.

il

B.
C.
D.

2.34
2.24
2.44
2.14

A.
B.

C.
D.

A.
B.
C.
D.

A M%
B.

j
'I

A
B.

~%
~%
~%

D.

2 and 0.5
2 and 1

6:zo: CE Board November 1998

61.6: ECE Board April 1998

A.
B.

C.
D.

\.

0.64 m/min
0.56 m/min
0.75 m/min
0.45 m/min

3 and 2
3 and 1

A statue 3 m high is standing on a base of


4 m high. If an observer's eye is 1.5 m
above the ground, how far should he stand
from the base in order that the angle
subtended by the statue is a maximum.

h. What percent is the volume of the


largest cylinder which can be inscribed in
the cone to the volume of the cone?

m/s
m/s
m/s
m/s

Water is flowin~ into a conical cistern at


the rate of 8 m /min. If the height of the
inverted cone is 12 m and the radius of its
circular opening is 6 m. How fast is the
water level rising when the water is 4 m
deep?

C.

10 kph
13 kph
11 kph
12 kph

61%: ECE Board November 1998


Given a cone of diameter x and altitude of

C.
D.

The coordinates (x,y) in feet of a moving


particle P are given by x = cost- 1 and y =
2 sin t + 1, where t is the time in seconds.
At what extreme rates in fps is P moving
along the curve?

3.64
3.94
4.24
4.46

619: 'E Board October 1993

611: EE Board October 199%


6J.S: EE Board April 1997

crosses a street at 9 m/s at the instant that


a car approaching at a speed of 4 m/s is
12 m up the street. Find the rate of the
LRT train and the car separating one
second later.
A.

3.41 m
3.51.m
3.71 m
4.41 m

Water is pouring into a conical vessel 15


em deep and having a radius of 3:75 em
across the top. If the rate at which the
water rises is 2 em/sec, how fast is the
water flowing into the conical vessel when
the water is 4 em deep?
A.
B.
C.
D.

2.37
5.73
6.28
4.57

After t hours, there are t +

m /sec
m 3/sec
3
m /sec
3
m /sec

Jt

gallons in

the pool. At what rate is the water pouring


into the pool when t = 9 hours?
A.
B.
C.
D.

8 fps
8.25 fps
8.33 fps
8.67 fps

6181 An LRT train 6 m above the ground

614: CE Board May 1995

The cost per hour of running a motor boat


is proportional to the cube of the speed. At
what speed will the boat run against a
current of 8 km/hr in order to go a given
distance most economically?

Water is pouring into a swimming pool.

~1.15
m
1.33 m

C. 1.50 m
D. . 2~

120
100
C. 140
D. 190

6:u: ME Board October 1996

D.

A
B.

A.
B.

,i

Problem6171
A man walks across a bridge at the rate of
5 fps as a boat passes directly beneath
him at 10 fps. If the bridge is 10 feet above
the boat, how fast are the man and the
boat separating 1 second later?

7/6 gph
8/7 gph
6/5 gph
5/4 gph

6:Z:ZI A helicopter is rising vertically from


the ground at a constant rate of 4.5 meters
per second. When it is 75 m off the
ground, a jeep passed beneath the
helicopter traveling in a straight line at a
constant rate of 80 kph. Determine how
fast the distance between them changing
after 1 second.

A.

B.
C.
D.

12.34 m/s
11.10 m/s
10.32 m/s
9.85 m/s

6%31 ECE Board November 1991.


A balloon is released from the ground 100
meters from an observer. The balloon
rises directly upward at the rate of 4
meters per second. How fast is the balloon
receding from the observer 10 seconds
later?
A.
B.
C.
D.

1.68 m/sec
1.36 m/sec
1.55 m/sec
1.49 m/sec

'&24: ECE Board April 1998


A balloon is rising vertically over a point A
on the ground at the rate of 15 ft./sec. A
point B on the ground level with and 30 ft
'from A. When the balloon is 40 ft. from A,
at what rate is its distance from B
changing?

A.
B.
C.
D.

13ft /s
,15ft/s
12ft /s
10ft/s

342 1001 Solved Problems in Engineering Mathematics (2nd Edition) by Tiong & Rojas
&25: CE Board May 1997
Car A moves due East at 30 kph at the
same instant car B. is moving S 30 E, with
a speed of 60 kph. The distance from A to
B is 30 km. Find how fast is the di~tance
between them separating after one hour.

&29: ECE Board November 1998


What is the allowable error in measuring
the edge of the cube that is intended to
hold 8 cu. m., if the error of the computed
volume is not to exceed 0.03 cu. m?
A.

A.
B.
C.

D.

36
38
40
45

B.

kph
kph
kph
kph

C.
D.

&2&: CE Board November 199&


A car starting at 12:00 noon travels west at
a speed of 30 kph. Another car starting
from the same point at 2:00 P:M. travels ..
north at 45 kph. Find how (in kph) fast the
two are separating at 4:00P.M.?

&30: EE Board October 1993


A standard cell has an emf "E" of 1.2 volts.
lfthe resistance "R" of the circuit is
hm/sec, at
increasing at the rate of 0.0
what rate is the current "I" hanging at the
instant when the resist ce is 6 ohms?
Assume Ohm's law E IR.

A.

A.

B.

c.
D.

49
51
53
55

B.
C.
D.

Topics

0.002
0.003
0.0025
0.001

-0.002 amp/s
0.004 amp/se
-0.001 ampdsec
0.003 amp/~c

D
Mon

&Q]
Tue

D D
D D
D
D D

I:
,J
!~

~~

&27: CE Board May 199&


Two railroad tracks are perpendicular to
each other. At 12:00 P.M. there is a train
at each track approaching the crossing at
50 kph, one being 100 km and the other
150 km away from the crossing. How fast
in kph is the distance between the two
trains changing at 4:00P.M.?

A.
B.
C.
D.

&28: CE Board May 1995


Water is running into a hemispherical bowl
having a radius of 10 em at a constant rate
3
of 3 cm /min. When the water is x em.
deep, the water level is rising at the rate of
0.0149 em/min. What is the value of x?
3

B.
C.
D.

2
4
5

Wed

Problems

Thu

Solutions

Fri

Notes

Sat

ANSWER KEY

67.08
68.08
69.08
70.08

A.

Theory

11
I

'

'i!!
t

"1.

.,
1
'
11

'>

596. B
597.C
598.A
599.A
600.A
601. B
602.C
603.A
604.C
605.C

606.C
607. B
608. B
609. B
610. B
611. D
612. A
613.C
614. B
615. D

c
c

616.
617.
618. A
619.A
620.C
621. A
622.
623. D
624.C
625. D

-----.~---

Maximum and Minimum Values


Local Maximum and Local
Minimum
Steps in Solving Maxima/Minima
Problems
Steps in Solving Time Rates
Problems
Relation Between Variables
Under the Condition of
Maxima/Minima

RATING

626. B
627.A
628.C
629.
630.C

c:J
c:J
c:J
c:J

30-35 Topnotcher
21-29 Passer
18-20 Conditional
0-17 Failed

If FAILED, repeat the test

344 1001 Solved Problems in Engineering Mathematics (2nd Edition) by Tiong & Rojas

'I

2=L
32
y3=64
(4,2)

'

1
l

y=4

Day 14- Differential Calculus (Maxima-M~-~'rilne Rates) 345

Thus, the size of the field is 25 m x 25 m.

Note: For maximum area, the triangle


must be an equilateral triangle.

Substitute y = 4 in Eq.2:

' )2
(4 =2
y=-a

Substitute x

=2 and y =4 in Eq.1:

d=J<'X~4) +(y-2r ~Eq.1

d=J8 =~4(2)

y2 =8x

y2

A=.!x 2sin9
A= i<6rsineo

Ill

Substitute Eq.2 in Eq.1:

K~ -r
Note:

x and y = two positive numbers


P = product of x and y

+(y-2f

:t

x+y=50
y =50-x ~Eq.1
P=xy ~Eq.2

f-4)()+~(1)
2./( ~ -4J +(y-2r

2(f-4 )(
o- .
o=(

Substitute Eq.1 in Eq.2:


P = x(50-x)
P=50x-x 2
Differentiate:
dP
-=50-2x
dx
0=50-2x
x=25

)+2(y -2)(1)

f: -r

+(y-2f

Substitute x = 25. in Eq.1:

~ -4)( 2i)+<y-2)(1)

y3
0=--y+y-2
32
'

,,

'

':'J

Differentiate:

Substitute Eq.1 in Eq.2:

A=15.59cm2

Let:

du
d(-!U) =2Tu

d'= 2(

A= 2xrh + 2(nr2} ~ Eq.2

d=2../2

x=--~Eq.2

108 = nr2h
_ 108. E
h --~ q. 1
xr2

3x=18
x=6

d = ~(2- 4) +(4--.:-2)2

V=nr2h

2x+2y=100
y=50-x ~Eq.1
A=xy~Eq.2

Substitute Eq.1 in Eq.2:

=2\~( ~~~ )+2(nr

216
A=-+2xr2 ~Eq.3

Differentiate:
dA _ -216 +4xr
dr
r2

--

-216 +4xr
0=-2r
216 =4xr

r2
_r3 =216

A= x(50-x)
A=50x-x2
Differentiate:

4x
r=2.58 in
Substitute r in Eq.3:

dA =50-2x
dx
0=50-2x
x=25

y=50-25
y=25

Substitute x

Thus, the numbers are 25 and 25.

y = 50-25
y=25

=25 in Eq.1:

216
>2
A = - + 2n(2.58,
2.58
A= 125.544 in2

2
}

.
Day 14- Pifferential Calc.ulus (Maxima-Minima & Time Rates) 347

346. 100 i Solved Problems in Engineering Mathematics (2nd Edition) by Tiong & Rojas

Ill

o J:

Ill

10

20

...'
f

20 +y

5.'

:.1''1

'.

). --::

10

'

:5

II

A =.::_d2h

2x

10 +X
A=(10+x)(20+y)

A= 200+10y + 20x + xy--+ Eq.1


,,.,

'!.

20-2x
V = (20- 2x)(20- 2x)(x)
V

=(20- 2x)2 (x)

~j

Substitute Eq.2 in Eq.1:

V =(400-80x +4x 2 )(x)


V = 400x- 80x 2 + 4x 3 --+ Eq.1

'

Differentiate:

=200+1o(_3 ~ 0 )+20x+ \( 3~0 )

,'~i

A= 500 +

dV
2
- = 400-160x +12x
dx
0 = 400 -160x +12x 2

3000

dA
dx

=0-~+20
x2

nx2 --+ Eq.2

Substitute Eq.1 in Eq.2:

i
II

Differentiate:

0 =- 3000 +20
x2

x = 10 in (absurd)

A=7tdh+2(
A= 1td(

x 2 = 150

x =3.33in

x =12.24cm

2
A = 45.2 + 1td

v = 400(3.33)- 80(3.33)2 +4(3.33)3


= 592.59 in3

Solving for h:

Thus, the dimension of the poster is,


-~

= total area

'

(1 0 + 12.24 )(20 + 24.5)


or 22.24 em by 44.5 em

Ratio= 2X
h
Ratio= 1

Differentiate:
dA = -45.2 +.::_(2d)
dd
d2
2

1t
d= 2.432 in

y=0.14P

300 =24.5cm
y = 12.24

y = 0.5P- 2.57(0.14P)

Substitute x = 12.24 in Eq.2:

:~;)+2(~ )d2

3 45.2
d=~

x=0.14P

Substitute x = 3.33 in Eq.1:

~d2 )

-45.2 + d
0=-2- 1t.

dA
-=P-7.14x
dx
O=P-7.14x

3000 =20
x2

3x =10

=area with the picture

A = y(2x) +

4x=40

Let: A
Ap

Substitute Eq.1 in Eq.2:

A= Px -5.14x 2 +.::_x 2
2
A = Px- 3.57x 2

,li~

~d2 )--+ Eq.2

y = 0.5P- 2.57x--+ Eq.1

A= 2x(0.5P- 2.57x)+ 2x2

Differentiate:

(4x -40)(3x -1 0) = 0

P-2X-1tX
y= .., 2

}~

+ 20x--? Eq.3

By factoring:

A= 1tdh + 2(

P = 2x + 2y + 1tX

l'

4
45.2
Eq. 1
d -----?
1td2

1
P = 2x +2y +-(27tx)

AP =xy
300 = xy
300
y=-. -?Eq.2

11.3 =.::.d2h

Let: P = perimeter

Let: C = total cost per hour


N speed in miles per hour

=~=
x+y

2(0.14'R_)
0.14'R_+0.14"R:

C = fuel cost + others


C = kN2 100--+ Eq.1

Day 14 -

348 100 i Solved Problems in Engineering Mathematics (2nd Edition) by Tiong & Rojas

C = 15x2 +'24xh--+ Eq.1

25 =k(25)

V=x 2h

k=25

h=

/1

10

\'~

~Eq.2
2
X

Substitute Eq.2 in Eq.1:

Let: x = total cost per mile

C:; 15x2 + 24x(:~)

_ 6
-7.5

75
J<4.5)2 +(9-.x)2 =-- (9-x)

~(4.5)

= 199,999.457

=200,000

11
)

0.371 + 1

(9-x)2

x=3km

I
y

L-'

4000x + 50

dC
-=2x-4000
dx

x+y=20

30

A= xy

= 2 in Eq.2:
Let: T = total time needed
T = time to row + time to walk

10 =2.5m
=(2)2

Differenti.ate:
-1.1
1)7( +1) )

dP _
1.1
1 8
dx - - - ( X +

Let: C = total cost

(X

Differentiate:

F-5) +(9-4
2

P=200,000-x- ( - 1.1 ) -+Eq.1


x+1

A= x(20-x)
A= 20x-x 2

T=-+6 7.5
T=

Ill

x=2000

~Eq.2

Substitute Eq.1 in Eq.2:

~~

2x =4000

y=20-x~Eq.1

Thus, the dimension of the tank is,


2 mx2 m x2.5 m

0=2x~4000

4.5

x=2m

= 36

9-x =6

3 __
_
240

Ill

0.5625(9- x)2 = 20.25

240 =30x

X -

+ (9- x) = 1.25(9- x)
(1.25)2(9- x)2 = (4.5)2 + (9- x)2

=0.371

Substitute x in Eq.1:

Substitute x

~
it
'll

dC = 30x- 240
x2
dx
240
0=30x--
x2

N=50mph

C=x

'i

9-x

X+ 1 = ~8(1.1 )

P=200,000-0.371-(

Differentiate:

0 = _!_+ (-100)
25
N2
2
N =2500

~;,!lll1

75

~(4,5f +(9-x)2

- - - + -2-

6~(45) 2 +(9-x)2

+ 1)9 = 8( 1.1 )8

C=15x2 + 240

_!_N2 +100
x= 25
N
N 100
x=-+25
N
dx _ 1 (-100)
25

(X

C=_!_N2 +100
25

x = total cost per hour


. miles
speed I n - hour

-1-8(~)
(~)
x +1
(x + 1)2

0 = -1 + 8(1.1 )8
(X+ 1)9

10= x 2h

Substitute k in Eq.1:

dN

0=

(Maxima-Minima & Time Rates) 349

~-(~9=-x:fo)== + _1 = 0

C = x2 (15)+ 4(xh)(6)

Fuel cost =kN2

Differ~ntial Calculus

dA =20-2x
dx
0 = 20-2x

+ 7.5

du

Note:

Differentiate:

d( .JU) =2.JU

=10

Substitute x = 10 in Eq.1:

dT -(

11

<.ix-- 6')

y = 20 -10 =10ft
2(9-x){-1)

2J(4s)2 +(9-x)2

+7.5

Substitute x and y in Eq.2:


A= (10)(10) = 100 ft2

'
350 100 1 SOlved Problems in Engineering Mathematics (2"d Edition) by Tiong & Rojas

Ill
Let: C
V

=C(OSt per hour


=speed of the motorboat

c, =total cost

=nr2h-+ Eq.1

Ratio=:!___= n(-;..,

~=-r

Ratio= 0.44

h _ RH-Hr
where: k = proportionality constant
_ distanee = ~ -+ Eq.2
t- speed
V-8

--R~-+

E
q. 2

Substitute Eq.1 and Eq.2 in Eq.3:

H
V ._
-nr 2H -nr- R

B
,,~
- l':'l. ~

4-x

E4

)o

E = intensity of illumination

A & B = luminous intensities of the two


light sources respectively.
k = proportionality constant

dC1 _ (V- 8)(kS3V 2)- kSV 3 (1- 0)


dV (V -8)2
= (V- 8)(kS3V 2)- kSV 3
0
(V -8)2

kSV3_ = (V- 8)(3R&_~)


V =3V-24
2V=24

2
dV
-=2
3nr H
dr
nrH ---

x2

=2~)(

x2

Differentiate:

2
r=-R
3

dE= -kA(2x) + (-8kA)(2)(4-x)(1)


dx

R_

Substitute r and h in Eq.1:

r(

2
V =n(_i.)R H

27

R
Let:
R radius of the cone
H = height of the cone
r = radius of the inscribed cylinder
h = height of the inscribed cylinder
V = volume of cylinder

x4

(4-x)4

= -kA(2x) + (8kA)(2)( 4- x}
4
4
x
(4-x)

k1\('2_\)- (8k1\}('2_)~
-~--

V=n(~R iH)

x3

(4x)"-

8
--3
(4-x)

2
Vc .!.nR
H
3

dL

-hcose

-2(-sine)
sin 2 e
cos 2e
= -hcose + -2(-sine)
0
sin 2 e
cos2 e
hcose 2(sine)
sin 2 e = cos2 e

-=~+-~~.:..

de

h = 2(sin e)
cos 3 e
h =2tan3 e-+ Eq.2

(4- x)3 = 8

--;(3"

Substitute L

J(4~xr =~

.) ..

Let: Vc = volume of the cone

_2_
cose

Differentiate:

h = 'R_H- H(2/3 'R_) = _!H

L=x+y
h
2
L=--+---+Eq.1
sine cose

(4- x)2

Substitute r in Eq.2:

v =12 kph

COSO=~

(4-x)2

E = kA + k(8A)

R-

x= _h
sine

Y=

E=kA+~

0 = 2nrH- 3nr2H
3\r"-H_

sine=~

Let:

Differentiate:

kV S
c,=-V-8

,..
- 2 (RH-Hr)
V -nr
--R

c,=kv (~)
V-8

A
~~
- ""' ~

Substitute Eq.2 in Eq.1:

c1 = Ct -+ Eq.3
3

)~

~7t~

vc

By ratio and proportion:

H H-h
Hr =RH-Rh

C = kV 3 -+ Eq.1

Day 14 ~Differential Calculus (Maxima-Minima & Time Rates) 351

4-x =

4 _ x =2x
x. l.JJ m

=6 and h =2 tan 3 e in Eq.1:


3

6 = 2tan e +-2-.
sine
3

cose

= 2tan ecos8+2sine
6
sinecose
6sinecose = 2tan 3 ecos0+ 2sine

(Maxima-Minima~ Ti.~-R.~tes)

Day 14- Differential Calculus


3

6sinecos9 = 2( sine ) ccis9 + 2sin9


cose

du

(x2 + 13.75 )3- 3x(2x)

=------------

(x2 +13.75t

s2 = 125(1)2 + 1oo

Substitute:

SubstituteS::: 15 in Eq.2:

. 9 ) +1
3cos9= ( ~.

de

du
Cix"'1;u 2

cos a

sin2 9 + cos2 9

Substitute 9 In Eq.2:

55

tan(a+9)=

tan a+tan 9 5.5


=1-tanatane
x
2.5 t 9
x+
an
2.5t 9
1--an

3.-108

5.5

/ ----
_/I

dt

dy =2cost
dt

dy =2(1)= 2
dt
Thus, the extreme rates are 2 and 1.

L--~"
__;1=1ot /~

-X

~-osition of the boat

V1 = 10

l_ v~fter I seconds

'~

1
9 = tan- (x2 +;31x3.75)

Jl

:11

S '= S/ + S/ + 10

:/ oc(10!)2 +{5tl +10 2

du
Note: d(tan-1 u) = 1+ u2

s7

2 +-1-:-3-.7-5'
u =-x"""

if=

9x

=(12-4ti" +(9t)2 +6 2

..

(x 2 + 13.75)'

Differentiate both .sides with respect to

2s( ~~}=194t(*)-s6(*) +O
dS 194t-96
= - - - - - - _, Eq.2
dt
2S

Substitute 1 = 1 in Eq.1:

125t2 + 100---+ Eq.1

I )ltferentiate both sides with respect tot:

where:

3x

-----

S2

S 2 = 97f - 96t + 180 __,. Eq. 1

13.75

3x
tan9= x2+13.75

y =2sint+1

V1 = 4 [POl.sitton of the c~r


. after t seconds

10

X+

.--1 . \.. _ _ _

s = 144 - em + 1st2 + 81t 2 + 36

13.75)
tane ( x+-.-x- =3

~~ is maximum ifcos t = 1

tane =

;;;::..-----------'---

S2=?L
.

,;/sz= 9t

s,=~t N2-4t

of!he ~~a:J
r.PoSiiic)n
aftertseconds

v2 =5

=-x

16

.:.:::.o

.~1

dx=-(-1)=1

Note:

=13.75

..J

~:---~

13.75
2.5 + xtane = 5.5 ---tane

~~ is maximum if sin t = - 1

X=

h=2.24 m

Note:

if;:/
lb&

)3_ -

2x 2 = x 2 + 13.75

h = 2tan46.1.

dt

osition ~t th$ ii-ai~ .

r:P_.r

l_ aftert seconds

(x + 13.75
~x(2x)
0 =--------(x2+13."15f
2

9=46.1.

dx
.
-=-s1nt

m1

O=du

cos3 9 = 0.333

x = cost-1

dt-=-15

0'~-:;-:;-_-;:l

3cos3 9=1

12~(!L "" 8.33 fps

dS

du

2
cos9

Substitute t = 1 in Eq.1:

s = 15

sin9r
6cos9=2 ( - +2
cos a

3cos9=

353

:f(\~ ) 250t {1) + 0


d:;

I )~it

dt

"

,.

s 2 = 97(1)2 -- 96(1) + 180


s = 13_45
Substitute S = 15 in Eq.2:

dS '194(1)- 96 = 3.64 m/s

>I q 7

d! = --2(13A's)

t:

354 1001 Solved Problems in Engineering Mathematics (2nd Edition) by Tiong & Rojas

V=

R=6

Day 14- Differential Calculus (Maxima-Minima & Time Rates) 355

rm

nr h ~ Eq. 1

By ratio and proportion:

V = 80

15

1000 m
1 'km.

s2: 4.5!

'

V=in(%r h

r
-=12 h
h

V = __!_h 3 ~ Eq.2
48

r=-~t::q.2

Differentiate both sides with respect to

t:

dV = 3n h2 dh
dt
48
dt

Substitute Eq.2 in Eq.1:

s 2 = 1oo2 +(S1 )2

v1 = 22.22

75

s1 = 22.22t

1 '( h ) h
V=3n2

= 2 and h = 4:

s 2 = 493. 728t2 + 5625 + 675t + 20.25t 2


s 2 =513.978t2 + 675t + 5625 ~ Eq.1

dV- 3n (4)2(2)

V=~h 3

dt- 48

Differentiate both sides with respect to

dV = 6.28 cm3/s
dt

12

dV = 3n h2 dh
dt
12
dt
Substitute dV/dt = 8 and h = 4:

t:

Differentiate both sides with respect to

dt

2s(

t:

~~) =2(513.978t)oo +67500


dS
dt

Let: Q = rate of discharge in gph

= 513.978t+337.5 ~E

Substitute t

Differentiate both Sides with respect to

dQ

dt=

dS
dt

= 15t ~Eq. 2

s
Substitute t =10 in Eq.1:
s 2 =1o,oooo+16(10)2

t:

_
2

=1 in Eq.1:

s2 =513.978(1)2 + 675(1) +5625

Substitute t = 10 and S = 107.7 in Eq.2:


dS

dt =

16(1 0) = 1.485 m/s


107.7

1m

2Ji.

Substitute t = 1 and S

s1 = 15t
= 82.547

in Eq.2:

R=3.75

H-1~ J-~-,

t "~

Substitute t

=9 hours:

dQ

-=1+--=1+dt
2.[9
6

dQ

- = - gph
dt
6

t:

)=0+32too

s =82.547 m

dh = 0.64 m/min
dt

1m

= 1o,ooo + 16t2 ~ Eq.1

8=107.7 m

O=t+Jt

3n
2 dh
8=12(4)

2s( ~~

s 2 = (S 1 ) 2 +(75+S 2 )2
2

Substitute dh/dt

S =100 2 +(4t)2

Differentiate both sides with respect to

S = (22.22t)2 + (75 + 4.5t)2

s1 = 4t

~ V2=4.5

~Eq.2

By ratio and proportion:

V1 = 22.22 m/sec

Substitute Eq.2 n Eq.1:

nr 2h ~ Eq.1

r=

V=

1 hf:..
3600 sec

'km_ X

hi:..

1
.

3.75 r
--=-

1m

dS

513.978(1) + 337.5

-~;

dt
dS
dt

82.547
2

10.315 m/sec

S = 30 2 + (S, )2
S

s2

= 30~ + (15t)2

=goo+ 22st2 ~ Eq.1

356 lO<U Solved Problems in Engineering Mathematics (2"d Edition) by Tiong & Rojas
Differentiate both sides with respect to t

2s(

~~)=0+4501~

Differentiate both sides with respect to

Day 14- Differential Calculus (N.[a_xirna-Minim(l & Time Rates) 357

t:

Substitute t = 2 hours (from 2 PM to 4


PM) in Eq.1:

2S(~T-)=5400t~ -2700~
' ,~1

dS = 225t ~ Eq. 2
s
dt

dS
dt

When S1 = 40 ft,

2700t -1350

40 = 15t

,;~.,l ,i

q. 2

= 2925(2) + 3600(2) + 3600

Substitute S = 111.8 and t = 4 in Eq.2:


dS
dt

= 1 in Eq 1:

rm

dS =51 k h
p
dt

t = 2.667 sec
SubstituteS = 30 and t = 1 ir. Eq.2:
Substitute t

=2.667 in Eq.1:

s =50ft

dt

;I

dS =45 k h
dt
p

Substitute t
dS

m.l

dS= 2700(1)-1350
dt
30

s 2 = 9oo + 225(2.667)2

:c

2.667 and S = 50 in Eq.2:

225(2.66 7 ) = 12 ft/s
50

VA= 50

~~

rm

100

1tX2

V=-(3r-x)

.r

f,,

50t-150

6.
U"1

ml

iO

4ltt"'-vs=60
B

$
A~

SA = 30t

60

-~

~ SA=30t 9
A'a.:~,--~-

1tX2

>::>s

--

V=-(3(10)-x)

-:

1tX3

Vs= 50

V=10nx 2 - -

0
0

(/)

-vA=3o

::

Vs=45

"<!"
II

5000( 4) -12500
111.8

dS = 67.084 k h
dt
p

dS = 2925(2)+1800
dt
150

= 2700(1f - 2700(1) +goo

25, 000( 4) + 32, 500

S = 150 km

S = 30 km

S = 111.8 km

Substitute S = 150 and t = 2 in Eq.2:

Substitute t

s 1 = 15t

8 2 = 5, 000( 4 )2

Differentiate both sides with respect to t


Ss =50!

Position of carA'i

at2~~-j

=(50t-100) +(50t-150)

=2500t2 -1 OOOOt + 10000 + 2500t2

dV = ( 2onx _ 3nx
dt
3

Jdxdt

-dV = ( 20nx -nx 2 )dx


- ~ Eq.1

dt

dt

-1 5000t + 22, 500


s2 = (60 + SA)2 +Ss2
2

s = (60 + 30t) + (45t)

By cosine law:
2

s 2 = 5000t2 -

I lifferentiate both sides with respect to

s2 = 36oo + 360at + 9Dat2 + 202st2

s = (30t) + (60t- 30)

"'

il

2925t + 3600t + 3600 -~ Eq.1


2

-2(30t)(60t- 30)cos60
s 2 = 9ooe + (60t- 30)2 - 30t(6ot- 30)
s 2 =9ooe + 360at 2 - 360at + 9Do
-1800f + 900t
s 2 =21om 2

21oot + 9oo ~ Eq.1

Differentiate both sides with respect to

2s(

~~) = 2(2925t)~ + 3600~1J,


.!:!_~ "' _2925t + 1800 ~ Eq.2
dt

t:

/S( ~~)

"

dS
dt

~;

Substitute dV/dt = 3 and dx/dt

25000t + 32,500 ~ Eq.1

t:

3 = (20nx -nx 2 )(0.0149)

2(5000t)~ -25000~
5000t -12500 ~ Eq.2

--s

:;utdihr\(; I= 4 hours (from 12 PM to 4


Ill lq t

I'M)

= 0.0149 in

Eq.1:

201.342 = 20nx -nx 2


64 = 20x- x2
x

20x + 64 = 0

(x-4)(x-16)=0

x = 4 em or
x = 16 em

358 1001 Solved Pcoblems In Englnee<lng

M~

0CL_.don) byTiong & Rojos

V = x 3 ~Eq.1

Topics

Differentiate both sides with respect to its


individual variable:

dV = 3x dx ~ Eq.2
2

Mon

Substitute V = 8 in Eq.1:

Tue

8= x3

D D
D D
D D
D

x=2
Substitute dV = 0.03 and x = 2 in Eq..2:
0.03 = 3(2)2 dx
dx =0.0025

.~

il
~~

'"'

E=IR
1.2 = IR ~ Eq.1

Differentiate both sides with respect to its


individual variable:
f\lote: d(uv) = udv +'vdu

o~{:)+R(~!)~Eq.2
Substitute R = 6 in Eq.1:
1.2=1(6)
1=0.2
Substitute I = 0.2, dR/dt = 0.03 and
R = 6 in Eq.2:
0 = (0.2)(0.03) +

JJ

.~

(6{ ~~)

dl
- = -0.001 amp/sec
dt

il

li

T
T
T
T

Theory

Wed

Problems

Thu

Solutions

Fri

Notes

Sat

Maximum and Minimum Values


Local Maximum and Local
Minimum
Steps in Solving Maxima/Minima
Problems
Steps in Solving Time Rates
Problems
Relation Between Variables
Under the Condition of
Maxima/Minima

,.;..r,.,

,,

1'opics

0
0

,.~.

/'

.
T.
,T
.....

;~ ',;. > S

.,.~+.!"\

~ . <

<~=~"~<'<"''~

'

'

> , < '"-" I

\> ' ( >

Mon

Tue

l~J

. !.

Theory

VVecl

Problems

Thu

Solutions

Fri

Notes

Sat

[] []
D []
[] 0

t'llj
i'

~hat

integral Calculus
Definite and Indefinite Integrals
Fundamental Theorem of Calculus
Basic Integrals Formulas
- Exponential, Logarithmic,
rrigonometric, Hyperbolic,
Inverse Trigonometric, By Parts,
Trigonometric Substitution,
Wallis Formula, etc.
Applications
- Area, Centroid, Arc Length
Surface Area, Volume, Work
Moment of Inertia, etc.
Propositions of Pappus
Hooke's law
Multiple Integrals

is Integra! Calwlus?

Example:
Integral Calculus is the branch of
calculus which deals with functions to be
integrated. Integration is the reverse
process of differentiation. The Junction to
be integrated is referred to as the
integrand while the result of an integration
is called integraL

lhe integra! symbol or sign

Example:

II

dongated S denoting sum (Latin: summa),


was introduced by Leibniz, who named
mtegral calculus as calculus
smnmatorius.

i!

What are Definite and Indefinite

P<'luuk' tnlegral is an integral that is


"' ~fuwd ly IIH ltm'f v; dtt"!~ a and b of the
Huf<p 'IHil 'IIi \l.llf.liil<'

Indefinite integral is an integral with no


. restrictions imposed on its independent
variables. It is also called antiderivative or
primitive integral.
Jt(x)dx

is an

lnte.m~l.l~

.,..

Jt(x)dx

What is the Fundamental Theorem of


Calculus?
Suppose f is a continuous function in a
closed interval [a, b], then '

F'(x) = f(x) for ali x in [a, b).


The fundamental theorem of calculus
S!<iles U-;at the definite integral is:

Day 15- Integral Calculus 363

362 1001 Solved Problems in Engineering Mathematics (2"d Edition) by Tiong & Rojas
b

Jt(x)dx = F(b)- F(a)

~u~---

19. Jsecut
What are the Integrals of the Different
Functions?
The following are the integrals arranged
according to its functions:
A.

Basic Integrals:

1.

fdu=U+C

2.

Jadu =au +C

3.

u"du=--+C
f
n+1

(n ,, -1)

du

4.

B.

Exponential & Logarithmic


Functions:

u=lnu+C

20. lcscucotudu = -cscu+C

41. fsinh udu = ~sinh2u -iu+C

21 I Jsin 2 udu = .!u- .!sin2u + C


2
4

42. Jcosh 2 udu =.! sinh2u + !u + C


2
4

!2_) Jcos

udu =

iu

Using a vertical differential strip:

+ ~sin2u + C

43. Jtanh 2 udu = u- tanh u + C

23. Jtan2 udu =tan u- u + C

44. Jcoth 2 udu = u- cothu + C

24. Jcoeudu=-cotu-u+C

F.

Trigonometric Substitution:

D.

44.

J~a 2 -u 2 du

let : u = a sin e

45.

J~a 2 +u2 du

let : u = a tan e

Inverse Trigonometric Functions:

25. Jsin-'; udu = u sin- 1 u + ~ + C

46.

J~u -a du

27. Jtan- 1 udu=Utan- 1 u-Jn~ +C

G.

Integration By Parts:

5.

feudu = eu + c

6.

au
audu=-+C
f
Ina

7.

Jue"du = e"(u -1) + C

= u cot- 1 u +In J1 + u2 + C

Jlnudu = ulnu-u+C

9.

f~=lnllnui+C
ulnu

30. Jcsc- udu = ucsc- u + lnlu + -Ju 2


1

E.

Hyperbolic Functions:

31. fsinh udu =cosh u + C

12. Jcosudu=sinu+C

34. Jcothudu=lnlsinhui+C

13. Jtanudu=ln!secu!+C

35. Jsec hudu = tan- 1(sinh u) + C

16. Jcscudu = ln!cscu- cotul + C

47. Judv

uv- Jvdu

-11

Wallis Formula: Formulated by John


Wallis.

2
48. fsinm ecos" ede =

A= Jxdy

[(m -1)(m - 3) .. 1or2][(n -1)(n- ~)1or2)

'-'----'---'----~-'-'-----'---"a

(m + n)(m + n- 2)(m + n- 4)1or2

B. Polar coordinates:

32. Jcosh udu =sinh u + C


33. Jtanh udu = In !cosh ul + C

15. Jsecudu = ln!secu + tanul +C

Using a horizontal differential strip:

H.

+C

11. fsinudu=-COSU+C

14. Jcotudu=inlsinui+C

let : u = a sec e

Trigonometric Functions:

A= Jydx

29. Jsec- udu = usec- u -lnlu + N~~


1

+C
8.

26. Jcos- 1 udu =U cos- 1 u- ~1- u2 + C

28. Jcot- udu

17. Jsec2 udu=tanu+C

40. fcschucothudu = -cschu+C

A. Rectangular coordinates:

un+1

C.

39. fsechutanhudu = -sechu+C

18. Jcsc 2 udu = -cotu+C

36.

Jcschudu=lnltanh~~+C

37. Jsech 2udu=tanhu+C


38. Jcsch 2 udu = -coth u + C

where: a = ~ if both m and n are even

a = 1 if otherv:ise
How to Find an Area using Integration?
One import::>nt use of definite integrals is
the determination of area between two
curves with given equations. This is done
by using. a differential area (horizontal or
vertical strips for rectangular coordinates
of a differential triangle for polar
c:o'oldlll<Jtes).

A=

r2
hd9

"\

_ _ _ _ _ _ _ _ _ _ _ _ _ _ _ _ _ _ _ _ _..:.D::.:a::.yL'.:.:lS::___-..::In:::t:.:::eSE~Calculus~

364 1001 Solved Problems in Engineering Mathematics (2nd Edition) by Tiong & Rojas
,tlow to Find the Location of the
Centroid using Integration?

The Second Proposition of Pappus


states as follows:

How to Find the Surface Area of a Solid


Figure by Integration?

The centroid is the center of mass of a


given figure. The figure below illustrates on
how to fin\l the location of the centroid of
an arc .. bounded by curves of given
equations.

"If an area is rotated about an axis, it will


generate a volume equal to the product of
the area and the circumference described
its centroid."

The surface area of a solid generated by


revolving a curve about a certain axis may
be calculated using the first proposition of
Pappus.

The First Proposition of Pappus states


as follows:

e.T;:tt{!tttl :~::18

centroid
"If an arc is rotated about an axis, it will
generate a surface area equal to the
product of the length of the arc and the
circumference described its centroid."

.....,

V ':" J2nyxdx

B. Using a horizontal differential strip


(Ring Method):

y
0

-~s,

~
f\
;d

_ fdAX
X;-

11

_ fdAr

Y=

or

S= J~1+(~~Jdx
x,
f 1+(ddy )2 dy

y,.

S=

Yt

A= JJ1+(~~J

>......!_.........

A. Using a vertical differential strip


(Shell Method):

The figure formed by rotating the


horizontal strip about the vertical axis is a
cir.cular ring or washer.

where: S =length of arc


d = distance from centroid to the
axis of rotation

(xz,Yz)

V= fdA 2nd

If the area is not given or cannot be


computed easily, the following will be used
rn finding the volume of the solid:

A= S 2nd

----4-------------------------~~

or

where : d = distance from centroid to axis


of rotation

\::I

ds

~dy
dx

V =A 2nd

- _2
A

How to Find the Arc Length of a Graph?

:
:
''

----~x

How to Find the Volume of a Solid


Figure by Integration?
The volume of a solid generated by
revolving 11 curve about a certain axis may
be calculated using thesecond proposition
of Pappus.

+~
dY~
j'
:

dx 2nd

,.,,.,,,

' ' .c'

"t"" .

v.

...
-

J x/ - x/ ~y

. V = n(

f
1'

I IH~ tr~ 1 ure formed by rotating the vertical


.trrp ;tlmut the veriical axis is a hollow
'vlindrical shell

366., 100 I Solved Problems in Engineering Mathematics (2"d Edition) by Tiong & Rojas
How to Solve for the Work Done by
Integration?

What are multiple integrals?

Work is defined as the force multiplied by


--~ce traversed. If the force i.s
constant throughout the distance, then
work is simply the product of the applied
force and the distance traveled. However if
the applied force varies with the distance,
then the best solution is to apply

integration.

An integral in which the integrand is


integrated twice is a double integral.
Example:

fJf(x)dx

Topics

0
0

If integrated thrice, the integral is called


triple integral.

lvlon

Example:

ffJf(x)dx

Tue

Work= fFdX

=force expressed in terms of x

where: F

Example:

What is Hooke's Law?


Hooke's Law, named after the English
mathematician and scientist. Robert
Hooke (1635- 1703) is stated as follows:
'Within elastic limit, the force required to
stre~ch (or compress) a spring is
proportional to the change in length."

F = kx
where: k

GOOD LUCK I

'{[;ribia:

0
0

Solutions

Notes

0
0
0

Thu

Fri

Sat

b:U: ECE Board April 1:999


What is the integral of (3t- 1)3 dt?

Did you know that... the propositions of


Pappus was inspired by two fruits, namely
apple and lemon! Pappus imagined a
circle cut by a line not passing though its
center and rotate the area of the major
part about the said line and produced a
solid which looks like an apple while the
remaining minor area when rotated about
the same line forms what is known as a
lemon.

~
Wed

ll.
('>.

12
1

--(3!-4) +C
12
1
)4 +C
~(3t-1

(>.J:Z:

~(3t-1)

= fdAy

'y = JdA-(~Y
2,

(() 10

"It is truth very certain, that when it is not in


our power to determine what is true, we
ought to follow what is most probable."

1\

:> \/:'

I\

IJ:>

(.

lrr :\
lrr :

II

~sin( 2x 2 + 7 )+C

B.

~cos( 2x 2 + 7 )+C

C.

( sine ) +C
2
4 ( x +7 )

D.

sin( 2x + 7 )+C

+C

ECE Board November 1:998

~uote:

- Rene D~scartes

A.
4

ll.

633: ECE Board November :1998,


ME Board April :1998
Integrate x cos ( 2x' + 7 ) dx.

1 ( 3t-1) 4 +C
--

I valuate the integral of dx I (x +2) from -6

---------L--+-----------~----~

lx

Problems

Functions of more than one variable may


be integrated with respect to one variable
at a time while the other variables are held
constant, reversing the process of partial
differentiation.

=spring constant

How to Solve for the Moment of Inertia


by Integration?

Theory

f J- Jf(x)dx"

Proceed to the next page for your 15th


test. Detach and use the answer sheet
provided at the last part of this book. Use
pencil number 2 in shading your answer.

For spring:

An integral in which the integrand is


integrated to n times is n-fold iterated
integral.

jlnt;gral Calcul~
Definite and Indefinite Integrals
Fundamental Theorem of Calculus
Basic Integrals Formulas
- Exponential, Logarithmic,
Trigonometric, Hyperbolic,
Inverse Trigonometric, By Parts,
Trigonometric Substitution,
Wallis Formula, etc.
Applications
-Area, Centroid, Arc length
Surface Area, Volume, Work
Moment of Inertia, etc.
Propositions of Pappus
Hooke's Law
Multiple Integrals

&34: ME Board April :1995,


ME Board April 1:997
Integrate: (7x 3 + 4x2 ) dx.

7x 3 4x 2
----.+-+C

368 l 00 1 Solved Problem~i.Il.~~gineering Math~matig~Jznct l~c!itio~!~~~ Rojas


Day 15-:. Integral Calculus 369

B.
C.
D.

6o39: CE Board May 11.?9"1

7x 4 4x 2 ,
-+-+C
4
5
7x 4 4x 3
-+---+C
4
3
4

6.

C.

12

Evaluate fx(x-5) dx

5'

P..

4x

7x --+C
2

63SI CE Board NoveJuber


5

1!9'9~
3

D.

0.456
0.556
0.656
0.756

8.
C.
D.

A.
Eva Iuate

D.

'I

B.

xdx

O.IJ11
B. . 0.022

63&: CE Board Noyember 1.'11'94

A.
B.

0.333

0.233

C.

0.433

D.

0.533

i2

D.

Evaluate the integral of sin

c.
D.

-2 .fi. COS X + C

A.

B.

x dx from 0 to

D.

7t

2n
17

3n
32
5n

C.
D.

0.423
0.293
0.923
0.329

A:
B.
C.
D.

765
12n
81

EE Board March 1998

0..278

B.
C.
D.

0.336

Evauae
I t

1.-'~

0.252
0.305

&43: ECE Boaad November l:9JJ:


2

Evaluatethe integral Jcos ydy

'II

t,,

A.

2.0

B.

49.7

C.
D.

3.0

21og10 edx

sin2y

A.

'-+-..--+C

B.

y+::>cosy+C

6481 CE Board May :1995


What is the integral of cos 2x e

ln(eexpx+1)-x+C
'
In ( e exp x - 1) + x + C
In ( e exp x + 1} square - x + C

1
1/2
0
1/3

c.

10
20
30

D.

40

A.

B.

5.12

sin2x

32

In ( e exp x - 1) square + x + C

65:.1& CE Board November 1'996


Evaluate the integral of {3x2 + 9y2) dx dy if
the interior limits has an upper limit of y
and a lower limit of 0, and whose outer
limit has an upper limit of 2 and lower limit
ofO.

f- - - - -

A.

C.
D.

10

3x+4

A.

sin x + C
secx+C
-sin x + C
cscx+C

Evaluate the double integral of r sin u dr


du, the limits of r is 0 and cos u and the
limits of u are 0 and pi.

B.

0
1
2
3

&471 EE Board October 1997

.
-1- w1th
respect to x an d

+c

+C

6511 EE Board April :1997

Evaluate the integral of In x dx, the limits


are 1 and e.

Integrate

ein2x

ein2x

Find the integra! of [ (e exp x - 1] divided


by [ e exp x + 1] dx
A.
B.

evaluate the result from x = 0 and x = 2.


32

C.
D.

6461 EE Board ApriJ :1997

_?:~~-

&38: ECE Board Aprilll997

B.

J(cos3J.\) dA

768

6421

A.

D.

a5n

B.

0.4
0.5

n/2.

2.fi.cos.!X+c

B.

~163

0.2

0.3

A.

27n

C.
A.
B.
C.

C.

C.

Find the integral of


sin x cos x dx if
lower limit = 0 and upper limit = 7[/2.

2
-2 .fi. COS X + C

~to~

637: CE Board May l'l99&

-2J2 COS-X +C

Evaluate the integral of cos x dx limits from

n/6

A.

D.

6501 EE Board April 1997

&41! ECE Board April 1'998


Evaluate

C.

-2 +C

ME Board October :1997


The integral of cos x with respect to x is

6451 ME Board October 1997

0.033
0.044

What is the integral of sin x dx if the


lower limit is 0 and the upper limit is n/2?

----~---

o (>r+ i)8

A.

C.
D.

6491 ME Board April 1995

n/2?

C.

y + sin 2y + C

ein2x

B.

Integrate the square root of ( 1 - cosx) dx.

640: CE Boa!.'d November ll'\\"'9&

0.0203
0.0307
0.041"7
0.0543

6441 ECE Board November 1998

What is the integral of sin x cos x dx 1f


the lower limit is zero and the upper limit is

A.
B.

y_+ sin2y +C

sin

dx?

6531 EE Board April :1996


x/21 2

Evaluate

e--+C
2

JJJzdz r dr
2

0 0 0

A.
B.

2/3
4/3

sinu du .

370 . lO<h SolveP, Problems in Engineering Mathematics (2"d Edition) by Tiong & Rojas
:.
D.

Day 15- Integral Calculus 371

659: ME Board April1.999

1/3
5/3

Find the area bounded by the parabolas


= 4y andy =4.

B.

x?:

D.

9 955
5.955
5.595

670: Find the coordinateP. of the centroid


of the plane area bounded by the parabola
y = 4 - x? and the x-axis.

654: EE Board April1993


Find the area of the region bounded by /
c: Sx and y = 2x.

A.
B.

21.33

A.

C.
D.

31.32
13.23

B.

C.
D.

1.22 sq.
1.33 sq.
1.44 sq.
1.55 sq.

units
units
units
units

Ms: CE Board November %994


What is the area bounded by the curve
= -9y and the line y + 1 = 0?

A.
B.

4 sq. units

C.
D.

5 sq. units
6 sq. units

x?

3 sq. units

D.

C.
D.

A.
B.

666: ME Board April1998


What is the area between y = 0, y = 3x?, x
= 0 and x = 2?

75
50
100
25

What is the area (in square units) bounded


by the curve y2 = 4x and .; = 4y?
A.

B.

5.33
6.67
7.33
8.67

",'li

square
square
square
square

units
units
units
units

the x-axis on the first quadrant.

A.

B.

c.
D.

8
24
12

x? =4y from x = -2 to x = 2.

D.

A.
B.
C.
D.

6&7: CE Board May 1995


What is the area bounded by the curve y2
= x and the line x -4 =0?

4.25
2.45
5.24
5.42

B.

2a

C.

D.

67::&1 Find the length pf arc of the parabola


4.2
4.6
4.9
5.2

units
units
units
units

a
a3

bounded by y = x2 and y = x.

ll.

Find the area in the first quadrant bounded


by the parabola
= 4x, x = 1 & x = 3

l>

units)generated by rotating the parabola


arc y = x? about the x-axis from x = 0 to x =

1.
A.
B.

C.
D.

5.33
4.98
5.73
4.73

674: CE Board May 1997

6&9: Locate the centroid of the plane area

t:

9.555

31/3
10
32/3

11

6&8: CE Board November 1.996


CE Board November 1998
Find the area of the curve ~ = a 2 cos 2e.

664: ME Board April 1999

A.

D.

~.~
~.~
~~~~
(~5.V~

B.
C.

C.
D.

7.67 sq. units


8.67 sq. units
9.67 sq. units
10.67 sq. units

What is the area bounded by the curve y


x3 , the x-axis and the line x = - 2 and x =
1?

A.

B.
C.

673: Find the surface area (in square

663: ME Board October :1997

Find the area bounded by the parabolas y


= 6x - .; and y = x2 - 2x. Note: The
parabolas intersect at points (0,0) and
(4,8).

11.7
4.7
9.7
10.7

D.

A.

A.
B.

Find the area enclosed by the curve x? +


By+ 16 = 0, the x-axis, they-axis and the
line x-4 = 0.

C.
D.

88/3
64/3
54/3
64/5

44/3
64/3
74/3
54/3

B.

C.

A.

B.

M8: EE Board April1997

A.
B.
C.
D.

6711 Locate the centroid of the plane area


bounded by the equation y2 = 4x: x = 1 and

66::&: CE Board May 1997

4.

C.
D.

A.

Find the area bounded by the line x - 2y +


10 = 0, the x-axis, they-axis and
X::: 10.

D.

Find the area bounded by the curve y = x?


+ 2, and the lines x = 0 and y = 0 and x =

B.

660: EE Board October 1997

c.

30/3
31/3
32/3
29/3

657: EE Board October 1997

A.

(0,1.6)
(0,2)
(1,0)

661: CE Board May 1996

C.

(0, 1)

B.

D.

What is the area (in square units) bounded


2
by the curve y
x and the line
X- 4 = 0?

B.

A.

c.

Mfn CE Board May 1995

A.

665: ECE Board April1998


Find the area (in sq. units) bounded by the
parabolas x? - 2y 0 and
.; + 2y- 8 = 0.

33.21

0.4 from the x-axis and 0.5 from theyaxis


0.5 from the x-axis and 0.4 from theyaxis
O.~i from the x-axis and 0.5 from they<~xis

0 IJ from the x-axis and 0.4 from theyil><b

The area enclosed by the ellipse

x2 y2
- +- =1 is revolved about the line x = 3.
9

What is the volume generated?

A.
-B.
C.
D.

355.3
360.1
370.3
365.1

675: CE Board May 1996


The area in the second quadrant of the
circle X: + y2 = 36 is revolved about the
line y + 10 0. What is the volume
generated?

372 100 l Solved Problems in Engineering Mathematics (2"d Edition) by Tiong & Rojas
A.

B.
C.
D.

2218.33
2228.83
2233.43
2208.53

680: CE Board November 1995


Find the moment ofinertia, with respect to
x-axis of the area bounded by the parabola
y2 = 4x and the line x = 1.

676: CE Board November 1995


2

The area bounded by the curve y


12x
and the line x = 3 is revolved about the
line x = 3. What is the volume generated?
A.

179

B.

181

C.
D.

183
185

A.
B.

C.

D.

Topics

2.03
2.13
2.33
2.53

D
D
Mon

Tue

D ~
D D
D
D D

677: CE Board November 1994


Given the area in the first quadrant
bounded by x2 8y, the line y -2 0 and
the y-axis. What is the volume generated
when the area is revolved about the line y

-2
A.
B.
C.
D.

=0?

28.41
27.32
25.83
26.81

678: Find the volume (in cubic units)


+ 6x
generated by rotating a circle x 2 +
+ 4y + 12 0 about they-axis.

C.

39.48
47.23
59.22

D.

62.11

A.
B.

53.26
52.26

D.

50.26

c. 51.26

Wed

Problems

Thu

Solutions

Fri

Notes

Sat

Definite and indefinite Integrals


Fundamental Theorem of Calculus
Basic Integrals Formulas
- Exponential, Logarithmic,
Trigonometric, Hyperbolic,
Inverse Trigonometric, By Parts,
Trigonometric Substitution,
Wallis Formula, etc.
Applications
Area, Centroid, Arc Length
Surface Area, Volume, Work
Moment of Inertia, etc.
Propositions of Pappus
Hooke's Law
Multiple Integrals

679 CE Board May 1!.99$


Given the area in the first quadrant by i!
8y, the line x = 4and the x-axis. What is
the volume generated by revolving this
area about the y-axis.

A.
B.

Theory

~tegral Calculus

631. A
632,D
633.A
634.C
635.C
636. D
637.A
638. D
639.A
640. B
641. B
642. D
643.A

ANSWER KEY
644.A 657.A
645. B 658. B
646. B 659.A
647.A 660.A
648.A 661. A
649.A 662. D
650. D 663.A
651. D 664. D
652. D 665. D
653.A 666.A
654. B 667. D
655. B 668. c
656.C 669.A

RATING

670. B
671. B
672. B
673,A
674.A
675. B
676. B
677. D
678.C
679. D
680. B

0
0
0
0

43-50 Topnotcher
30-42 Passer
25-29 Conditional
0-24 Failed
If FAILED, repeat the test.

Day 15- Integral Calculus 375

374 100.1 Solved Problems in Engineering Mathematics (2nd Edition) by Tiong & Rojas

IJI

Using Wallis formula: m = 5; n = 3; and


3

I(3t -1) dt =
=

i I<3t

i(

-1) (3dt)

~ Jc

(3t 1)4 +

12

nJ2
5

sin xcos

--B

Note:

--B

dx

(2:_)

sin 6 x dx = 5( 3 )( 1)
= Sn
6(4)(2)
2
32

Ju

du

= (5-1)(5-3)(3-1) (1)
8(6)(4}(2}_
=0.0417

=lnu

-x
7
7(X+1)

sin 5 x

ml

12

dv = (x-5)12dx; v =

Ixcos(2x + 7)dx

13

..:...(x_~-~-=-)-

=~(x-5f3 _ __!_

sin xcos x dx

ml

13

=12( (5 -1)(5- 3)(5 -1)(5- 3)) (1)

7x 4 4x 3
3 + 4x 2 )dx =
(7x
-+-+ C
}
4
3

10(8)(6)(4)(2)
=0.2

nJ2

sin x cos x) .dx

7(0 + 1)

4~[(t+1f6 -(0+1f6]

sin

x dx

13

= 0.456

l
fi

tl
ii

Jcos 3A dA
8

Let: u = 3A; du = 3dA, thus dA = du/3


Change limits: when A = 0, u = 0;
when A = rc/6, u = n/2
Substitute:
n/2

I
1

n/6

15

1[(6-st -(s-st]
-182

sin xcos x dx

=~(6-5)13 -~(5-5f3

=5; n =5 and

. 14]!:6

(x-5)
13
14

13

nl2

12

n/2

BIB

x(x-5f3 _ __!_ r(x-5f3dx


13
13 J

numbers

1m

Using Wallis formula: m

-6

7(1 + 1)7

2
Jx(x- 5f dx

5(3)(1)

a = 1 since both m and n are odd

=sin(2x + 7)+C

+..![~+1r ]

-1

Let:
u= x; du = dx

.rr/2

= Jcos(2x + 7)( 4xdx)

-X
1 [
11
-.---42 (x+1)--Bt

Note: Judv = uv- Jvdu

dx=~=0.533

=In(~) =In 2

r,

=0.022

n/2

J(

-X

sin x dx

2
Jx(x-5f dx

-7

- 7(x + 1)7 + 7 J(x + 1f dx

Using Wallis formula: m = 5; n = 0 and


a = 1 since m is an odd number

l-1o

= ln(-10 + 2)-ln(-6 + 2)
= ln(-8) -ln(-4)

-7

IIJ

-dx- = ln(x + 2) --B


X+2

= x[(X+1( ]- J(x+1( dx

n/2

-10

'

6
J<x + 1f xdx = uv- Jvctu

dx
X+2

c
.

rm

0
n/2

= __2_,(3t -1)4 +

-10

Using Wallis formula: m = 6; n = 0 and


a = rc/2, since m is an even number

a = 1, since both m & n are odd numbers

cos6 u du

n/2

.:!_

Let:

dv

Using Wallis formula: n = 8 and a


since n is an even number

x; du

xdx
(X+ 1)6 = (x + 1)--B xdx

n/2

1
cos6 u -du =-

=dx

(x t 1) 11 dx; v

cose u du = ..!( 7(5 )(3 )(1) \


3 8(6}(4)(2))

(X I

1)

1057t
2304

357t
768

=--=-

= rc/2,

(!:)
2

371 .1 00 l Solved Problems in Engineering Mathematics (2nd Edition) by Tiong & Rojas

rm

1 -dx-dx
-=3x+4 3 3x+4

1"'2

. X x/<4
d X::: Sin

COSX

.
=Sin

=0.305

Note: TCI2

Ill
2
Y)dy

=i
i

J(1+COS 2y)dy

Jdy +

Jcos 2y(2dy)

'Zi

J'nx dx

let: u =In x; du = dx ; dv
X

=dx; v =x

10

10

21og e dx _
X

dx
e +1

e + 1, thus e = u -1; du = e'dx


J du

"cosu

Jdx
X

Jrsinudrdu = J2"

du
(u-1)(u:_1+1)

~)d2x

i)+c
~ ~2..[2 cos(i)
=2J2(-cos

+C

= 21og10 e lnx1
1
=21og10 e (In 10 -In 1)

Jcos2x e5in 2x dx = Jein2x cos:tx dx

1\

I qu<Jte coefficients of u:

A11J

o
let: u = sin2x; du =cos 2x (2)

II

11 n
1

i "j(

cos u- o )sinu du

-i "

Jcos u(-sinu)du

=-..! cos
2
=

ul"

-~(cos3 7t- cos3 o)

1
=--(-1-1)
6
1

1 quate constant:

1=A(u-1)+Bu
1 =Au-A+Bu

=2

sinu du

f
f

1
A
8
-=-+u(u -1)
u u -1

"rr lcosu

10

.J2 Jsin ~

=2fi ~sin

= ln(e +1)2 - x +C

dx
e +1 = e(e +1)

10
-=
- - - 21og10 e -

=21n(ex +1)-X+C

dx

Ill

Substitute:

e -1
--dx = ln(e +1)
e +1

du
= u(u-1)

Thus, v.<:sin-='.11-cosx

=-ln(e+1)+x

Je +1

=1

r;;---

-ln(ex + 1) + ln(e)

= In( e~ + 1) +In( ex + 1)- X + c

= e(lne) -e- [ 1(1n1) -1]

sin(~)=P-~sx

=-In( ex+ 1) +In( ex+ 1-1)

-~esn2x + C

=In (ex +1)- - -

=xlnx-xj~

Ju-1
du

s-du
-u-+

-[ -ln(e +1)+x ]+c

I et: u =

J-b-cosxdx

= -lnu+ln(u-1)-

=~ Jesn 2x (2cos2x)dx

-1
edx
e +1 dx =-;.--+1-

I(,

"dx

fe

= (lnx)(x)- Jx( dxx)

f.J1-cosxdx =

Jcosx dx = sinx + C

Jrn x dx =uv- Jvdu

Ill

=90" and TCI4 = 45"

r;;

du
u(u-1)

Thus:

=j'_+ sin2y +C
2
4

Note:

51

7t ( . 7t)
2 - Sin 4

=0.293

..!2- ~s

Jcos2x e

n/4

=-ln(3x+4)
1
1
3
0
1
1
= rn[(3){2) + 4]- rn[(3)(0) + 4]

Jcos y dy =

Note: Jeudu = eu + C

1<12

Day 15 -Inteqral Calculus 377

fy

=3

0 0

r;

Jc3x + 9y

dx dy =

ly
+ 9y 2 x dy
0

....

378 1001 Solved Problems in Engineering Mathematics (2nd Edition) by Tiong & Rojas
2

Jx3 + 9y2xl: dy = Jv3 + 9y3) dy


0

rm

ID
y2 =8x ~Eq.1

J1oy d~

Day 15- Integral Calculus 379

x2

=-9y ~ Eq. 1

y + 1 =0
y = -1

y =2X ~Eq.2

~Eq.2

Substitute Eq.2 in Eq. 1:

= 10y412

10(2)4
=-4
=40

Substitute Eq.2 in Eq.1:

(2x)2 =8x

x2 = -9(-1)

4x 2 =ax
X=2
y = 2x = 2(2) =4

A=2 Jvdx

X=3

0
4

A=2 Jv'X dx

111

0
3

x/2 1 2

JJJzdz r dr sinu du

"-A= 2(x)

x/21~2

JJ~

0 0

JX

-3

0
2

JJ2r drsinu du
2

Jr3 11 sinu du

J-

xl2[13-03J
sinu du
3
0

=~

A=

J(~ -2x)dx

1"'2
0

=~( -cos%+cosoo)

Another Solution: Use the formula for


the area of a parabolic segment

-x

3
A= 2J8(2)a'2

A= 3_bh

A=li4X4)
3
A_ 32

-3

1m
2 12

(2)2 -[ 2J8~o)3'2 -(0)2

y 2 =X ~Eq.1

X= 4 -t Eq.2

Substitute Eq.2 in Eq.1:


A = 1.33 square units

-~~
y

4
y

12

A= Jydx
0

EE]
.

square units

A = 4 square units

2
2J8x3/2

(;r -x]'

=-(<~~3 -3]+((~3/-(-3)}

J8x2 2x 2
A=----

sinu du

-1Jdx

1-3

R~

= -[

2
,;-(-cosu)

2
3

~ YL) dx

0
2

x/2

=-

A =i(4)3'2
3
A = 32 square umts
.

-3

A= J<YP

x/2 3

=2

A= Jvdx

0 0

-3

=-J [-1-( ~J}x

Jr drsinu du

x/2 1

=2

0 0

2
r drsinu dl;J

22 02
;

x/2

A= J-ydx = J--<YL- Yp)dx

0 0 0

=ix3'21

. h .==_4 -r
II
.. . .o

. .

380 J..OO I Solved Problems in Engineering Mathematics (2"d Edition) by Tiong & Rojas
4

A=

J(x

Ill

+ 2) dx

A= J( 8x- 2x )dx

Day IS- Integral Calcull.!s 381


Substitute Eq.2 in Eq.1:

A=~

2x

= 8xz- 2x314

14

3 6

=4(4) 2

-~(4)3

43
A=( ) +2(3)
3
88
'
A=- square un1ts

10

II

rm

2 c- x2
vx - 4
x4
4X=16
64 = x3

=0

when x = O: y = 5
wheny=O:x=-10

rx=-~~

3
64
'
A = - square un1ts

y = 6x -x

2y

X=4
y=214
y=4
Thus, the parabolas will intersect at (0,0)
and (4,4).

Solving for the coordinates of the


intersection:

x2 - 6x = -y
(x-3f = -Y+9

x 2 = 4y

(x-3) =-1(y-9)

[X= 10J

10

A= Jydx

= 4(4)

X =4

By inspection, the vertex is at (3,9) and


the parabola is facing down.

1n 5+~}x
0

2110
.~ 5x + 2~2) o

y = x 2 -2x
x 2 -2x = y
(x-1)2 = y+1

(x -1)2 = 1(Y+ 1)

102
=5(10)+-

(0,0)

A=2 Jxdy

A = 75 square units

By inspection, the vertex is at (1,-1} and


the parabola is facing up.

=2

f<J4Ydy = 4 Jy 112 dy

4( ~:~~ J %(Y)3'2[
=

3
A= 21.33 square units

A= Jydx = J<Yupper- Yiower) dx


0

K(

6x - x ) - { x - 2x) dx

3
2
A =-(8)(4)

x2JX= 2;;~2 -4(:)[


=

:)dx

2(a+b)h

~(5+10)(10)

75 square units

ml
y'

b=8

'\!17

A= 21.33 square units

~c:J~
II

.0

Ill

h = 10

4(4)3/2 (4)3
=-3--12
A = 5.33 square units

Another Solution: Use the formula for


the area of a parabolic segment

A=~bh

Another Solution: Use the formula for


the area of a trapezoid

=~(4)3/2

A= Jydx= J<Yupper-Yiower)dx

Ill

4x

2/x ~ Eq.1

x2 =-By -16

4y

x2 =-8(y+2)

'lq ;>

By inspection, the vertex is at (0,-2) and


the parabola is facing down.

382 l 001 Solved Problems in Engineering Mathematics (2nd Edition) by Tiong & Rojas
By 1nspect1on the vertex is at (0,-4) and
the parabola 1s facing down.

1
3

A 2 = Jx dy
dx

x 2 = 2y ~ Eq.2

~[

Equate Eq.1 to Eq.2:

4
A 2 = 0.25

2(y-4)=2y

Atatal = JA1j + JA2j

Note:
x 2 + 8y+16 = 0
-By=

y=2

Atatal = 4.25 square units

dx

8 =4y

= 4 +0.25

A= J-ydx

By inspection, the vertex is at (0,0) and


the parabola is facing up.

o4'_ (1)4

Day 15- Integral Calculus 383

A= Jydx = J3x dx

= 2(2)

X =2

Ill

rhus, the two parabolas intersect at


points (2,2) and (-2,2).

x2 +16

x3
=3-

=x31:

x2
-y=-+2

(0,0)

=(2)3
A = 8 square units

Substitute:

A=

X~ +2}x

~____!_/

14

= 3~8) +2X 0
3

= (4)3 +2(4)
24
A= 10.67 square units

-2

~10
-2

04 -{-2)4
=----

X =4 ~Eq.2

A= Jydx

y = 2../x

Substitute Eq.2 in Eq.1:

2
2

A= Jydx = 2 J..fXdx

= ~[(3)3/2- (1)3/2

y2 =4

f(Yupper- Y1ower)dx

y=2

-2

= 2x3/2 = ~ x3'213
3/2
3
1

A1 = Jx dy

A1 =-4

y2 =X ~Eq.1

Y2 =4x

Iii

-2

~-

J[ 8 - 2x

::I [

x 2 +2y-8=0

B(2)

I(} tili(

; [ax

x2 = -2(y- 4) ~ Eq.1

J dx

A = 5.595 square units

x2 =-2y+8

x: ] dx

8 -2x2 -

J[

~ (2)

3
)

l8(-2)-

;qu;llf~ tlllll~l

~(-2) 3 )]

A= 2 Jydx

a
4

A= 2

J..fX dx
0

384 i 001 Solved }>robJems in Engineering Mathematics (2"d Edition) by Tiong & Rojas

2
A=

2(x)
~
2

= ~x.3121

4
0

_ _ _ _ _ _ _ _ _ _ _ _ _ _ _ _ _ _ _ __,D:::.a= 15- Integral Calculus. 38_5


1

Ax=

x2 =x

e
o
30"
45"

so
go
120"
150"
L1ao

Thus, the parabola and the line intersect


at points (0,0) and (1, 1).

1-

13

14

Jr

A= Jydx= J<YL-Yp)dx
0

dx

_tJ

= x2
2 31o
12 13

=-

f( y3/2- y2)dy

~:~~- y:[
5/2

Ax= Jydxx= J<YL -yp)dxx


0

2 2
) dx

8x 3 x5
=16x-3-+5 o
1

0.4

(332)(y

=a 2 [ sin(2)145.)- sin(2)(0)]

J4-

I hus, the coordinates of the centroid is


"' (0 ~i. 0.4)

= jt16- 8x2 + x4 )dx

45"
0

(1)5/2 (1)3
yoc------

(j

Jy dx

1.

A=a 2

a cos 20(2d9)

=a sin2ej

Ay=2 Jydx~=

0
1

2.(;) J
2

= f{Jy -y)dyy

45

. 2
2
32
A=-bh=-(4)(4)=-3
3
3

1\y= Jxdyy= J(xp-xddyy

= J(x -x2 )

A=-

de

(0,4)

Solving for the area of the parabolic


segment:

=--2 3

45

By inspection, the vertex is at (0,4) and


the parabola is facing down.

Thus the parabola intersects the x - axis


at points (-2,0) and.(2,0).

=-y+4

~
.(2,0)

de

x=0.5

45

45 2

at y = 0, x = 2

-X=---

r
a
0.707 a
0
i
i
i
0.707a
a
1

i Jr

f(x 2 -x 3 )dx

=-~3- :4[

Y=X =1

y = 4- x2

x2 =-1(y-4)

X=1

r 2 = a 2 cos2e

=2

f(x-x )dxx
1

co

Substitute Eq.2 in Eq.1:

=y~Eq.1

()

A =32
- square um'ts

A=4

Jydxx= J(YL -yp)dxx


0

y =X ~Eq.2

A =~(4)312
3

)=16(2)--8(~)3 + (~5
y=1.6

Thus, the centroid is at point (0, 1.6)

Day 15 -Integral Calculus 387

386 1001 Solved Problems in Engineering Mathematics (2nd Edition) by Tiong & Rojas

Change the limits:

r.
Ay=2. J(4x)dx
-

l=4X
atx=1,y=2

s=

-y

JJ4

45 ,.------;:2

dY) = 4x 2
( dx

+ 4tan 8 (2sec 8 d8)

0
45

3
4

Y=(0,0)

dy
-=2X
dx

X= 2; 8 = 45o

j(Y )=<1>2
-

dy = 2xdx

when, X::: 0; 8 = 0

=~( 4;2 J[

Thus; the parabola and the line intersect


at points (1,2) and (1 ,-2).

Y = x2

Substitute:

2
2
J4(1 + tan 8) (2sec 8 d8)

0
45

2
A = 27t JJ1 + 4x dx x

= J J4(sec 8) (2sec 8 d8)

Thus, the center is at point (3/5, 3/4).

0
1

0
45

0
45

2)3n

3
=4 J sec 8 d8)
(0,0)

4
45

I
2
s=2 fv1+(:~) dx

-c

4(~sec8tane +~ln(sec8 + tan8)l

2.

= J<2.JX )dx

dx

.( dy)2

x2

ldx =4

5/2

j(x)

S=2

3
5

x2
1+"4 dx=2

-(1+4(0))

312

A = 5.33 square units

ml
x2 y2
Given equation: - +- = 1

=1

By inspection: a = 3 and b =2

x =1, y =1

4+x2
-dx
4

=fJ4+x2 dx=J~(2)2 u2 dx
0

Jvdx"~=i Jldx

=~[(1+4(1)f'

12
2

2F?
21
J
J0

X=-

Ay=

3/2

~< 1 >s'2

when

Substitute:

= 27t(1+4x2)3'21

x2 y2
Standard equation: 2 + 2
a
b

Y= x2

=2()(612)

'4.6

dy =~
dx 2

312

~ln(sec45

=4y

2xdx =4dy

=2 Jx

-sec45tan45
4 1
= [:
+ tan45)

Ax= Jydxx

= 7t ( 1+4x

= J 2sec8 (2sec 8 d8)

Solving for the area:


1(2
1
4
A=-bh ) =-(4)(1)=2 3
3
3

2)1/2
21t ~(
(8x)dx
B
J 1 + 4x

Using the second proposition of Pappus:

Jds 21td

V=A21td

Let:
x =2tane;

fJ--------2
~~)
I

dx == 2sec2 e de

1I

II

dx 21tX

= ( 7t8b). 27td

=(~)(3)(2)(21t)(3)
V = 355.3 cubic units

388 . 1001 Solved Problems in Engineering Mathematics (2nd Edition) by Tiong & Rojas

Day

V=(~nr2 )-2n(;: +10)

Solving for y

Using the disk method:

x. b V
.
.

~~!+:.3~x ~

'" '" V~y I

=n Jy

dx

By inspection, d = 3.

dx

0
4

Ay= JdAy/2

C(-3,-2)

(0,0)

V =; 222B.91B cubic units

Using the second proposition of Pappur.:


2

=n f(YL- YP )

dx

V = A 2nd

= nr2 2nd

2]2

41(

=Jydxy/2

=n gl2- ~

=n(1)2 (2n)(3)

dx

=~ Jldx

x2
=n

X
= n(

c
r

=~ J<r2 -x2)dx
0

=~(rx-~;)[
(Y)

X3/2

312

dx

XS/2

V=A2nd

~nr )- 2n(y+ 10J

312

= 4n[ (2.Ji2)(3)

-~.Ji2(3)5 12 J

10

X=

4, y = 2

Thus, the parabola <:~nd the line intersect


at point (4,2).

(0,0)

(x + 3) 2 + (y + 2) 2

=1

I
~:

ll
~

~dy

r2

By inspection, h = -3 and k = -2, thus the

y = 2, x 4

=By

At

(4)5 ]

Standard equation: (x- h)2 + (y- k)2 =

x 2 =By

x2

(x+3) 2 +(y+2) 2 =-12+(3) 2 +(2) 2

V = 1B0.955 cubic units

At

x- 2(3) + 64(5)

By completing square:

.Q

Using the second proposition of Pappus:

xs

x 2 +y 2 +6x+4y+12=0

=4n[3J12--J12-3/2
5/2 ]1

By inspection, r = 6

V =(

.J12x

x3

dx

=4n J(3.J12x 112 -

Given equation: x + y 2 = 36

x4
64

= 59.217 cubic units

v = 26.B08 cubic units

3n

=n [ 4(4 )- 2(3) + 64(5)

=4n J(3- x)( .fi2X) dx

4r
y=-

(4)3

~x'?\(y )=; (! r'~)

V+P3-x)y dx]

Using the shell method:

r J
=21 ( r -3

4-

Calculus 389

y-axis

46
=..!.(n)(6)2 (2n)( ( ) +10)
4
. 3n

1
4nr

1~- Integral

Substitute r = 6:

Using the ring method:

conter is at (-3,-2). Also the radius of the


ctrclc is equal to 1.

V=

nJ( XL2 0

Xp

)dy

390 100 1 Solved Problems in Engineering Mathematics (2nd Edition) by Tiong & Rojas
2

=1t K(4)2 -( JSY)2J dy

2
IX= 2 jy (1-

0
2

~ )dy

=2Xy2- :4}y

= 1t J(16-8y) dy

Topics

=1t(16y-

y3

)[

= 1t[16(2)-4(2) ]

{-4-

y5
4(5)

Man

)0

= 2((2)3 - (2)5)

V = 50.265 cubic units

lx

Tue

20

D [Q]
D D

=2.13

l=4X

Theory

Wed

Problems

Thu

D D

At X= 1, X= 2

Solutions

Fri

Notes

Thus, the parabola and the line intersect


at points (1 ,2) and (1 .~2).

' ~ ~ ., ' " ~

tT
'

eJ

..

Sat

Integral Calculus
Definite and Indefinite Integrals
Fundamental Theorem of Calculus
Basic Integrals Formulas
- Exponential, Logarithmic,
Trigonometric, Hyperbolic,
Inverse Trigonometric, By Parts,
Trigonometric Substitution,
Wallis Formula, etc.
Applications
-Area, Centroid, Arc Length
Surface Area, Volume, Work
Moment of Inertia, etc.
Propositions of Pappus
Hooke's Law
Multiple Integrals

'

'

lx

=2 JldA
0
2

ll

= 2 Jy (xdy)
0
2

<<'>

lx = 2 Jy (xL- Xp) dy
0

"~<'r~oi,.-.

~:<-#0".~:.'<><

"'~''"'")'."<,~

~.

~"'

!?;1'<-~""''~h~-'"'".Cd'''

~,f.f<~~1<'<'<<'<'>~~&~'i" .... OO<'i""ii<.''i""'"""""'<

' " ,. ><-"'-< "

"+ .... .,. " " ~ ~ .- . "'. "-" <

392 l GO 1 Solved Problems in Engineering Mathematics (2nd Edition) by Tiong & Rojas

.;h.

<;-.

Topics

.,.,.,.

D
D
D

~-

~l1on

Tue

Theory
'_.,;

" ~ ..;

,, ' .......

"'

'

~ .:!"

,. ' "

,, <-. "

-'" ".(>

<

<-->"'~~

~*~

~<0<~'~"''""-

.. ~ / " '

D lQJ
D []
D D

'

'" '

'

'"'''~,.

,.,~

~"~-:%>>".*"<''<'

...

,<(

''

<,

.....,.':'""' ......

..<.,...-~~-;,""~

~. . :t~~~~~

' v

,.

\;<'

..,

VVed

4>:15:

Problems

Thu

Solutions

Fri

Notes

Sat

Differential Equations
Types of Differential Equations
Orders of Differential Equations
Degrees of Differential Equations
Types of Solutions of Differential
Equations
Solutions to First-Order Differential
Equations
Applications to First--Order DE
(Population Growth, Radioactive
Decay, Continuous Compound
Interest, Flow, Cooling and Heating,
Newton's Second Law of Motion,
Geometric problems, Orthogonal
Trajectory)

What is a Differential Equation?


Example:
differential equation is an equation that
' ontains one or more terms involving
1lr !rivatives of one variable (the dependent
v.1riable, y) with respect to another
v.u iable (the independent variable, x).

1\

I xamples:

(-<:!_~~J2Jd2y2 J3 + x3 ~ 0
3

l dx

dx

c. (x - 1) y" + xy' + y
(j

ilx

ily

its solution y = J2xdx = x + C , where Cis


an arbitrary constant.

Whatl!_re the Types

a. (2xy +x 2 ) dx + 2y dy = 0

b.

The differential equation dy = 2x has for


dx

ax
y-=
(fz

=0

~f

Differential

Equations?

1.

Ordinary Differential Equation- an


equation containing only one
independent variable, thus
having only ordinary derivatives in
the equation.

llu :;olul1ons of differential equations are


lttll<.llow; and not just numbers like the
dq' Ill ;tic: r" 111< 1l1oll'

Examples a, b and cat the left of this


page are examples of ordinary
differential equations

394 1001 Solved Problems in Engineering Mathematics (2nd Edition) by Tiong & Rojas
2.

Partial Differential Equation - an


equation containing two or more
independent variables, thus having
partial derivatives in the equation.

Example: x3 - 3y =ewhere: C =arbitrary constant

2.

Exampled in the previous page is an


example of partial differential
equation.

Ill

II

Example: 2xy + i

- 2=0

The order of a DE is the order of the


highest ordered derivative which
appears in the equation.

1.

dy = 2x2 + 5x + 3
dx

Variable separable type

jP(x)dx +

2.
d3y = 3x2 + 6x + 2
dx 3

Examples:
3

d2 + (d
x-Y
..1...) =15
2
dx

= 15

What are the Types of Solutions of


Differential Equations?
1.

ef(1-n)P(X)dx dx

What are the Applications of First-Order


Differential Equations?

1. Population growth problems.

F(x,y) = JN(x, y)ay + k 2

lQ(y)dy-~:c.

dP =kP
dt

F(x,y) = jM(x, y)ax + k1

where:

Test for exactness:

OM

Homogeneous type

oN

ay""' ax

=0

where:
dP =rate of change of the population
dt
P =number of inhabitants at any time
t
k = constant of proportionality

2. Radioactive decay problems


4.

General solution:

The degree of a differential equation


whose terms are polynomials in the
derivatives is defined as the highest
power of the highest order derivative.

f<1-n) Q(X)

Standard form:

What is a Degree of a Differential


Equation?

J + (dd~ )

y1-n -- ef(1-n)P(x)dx
.
1

The Bernoulli Equation is named after the


brothers Jakob (1654 -1705) and Johann
Bernoulli (1667,... 1748).

F(x, y) = C

fVl(x,y)dx +N(x,y)dy

d2
x ( dx;

General solution:

General solution:

.
=5x 2 + 1Ox + 3

dx

Exact typf!

General solution:

M(x,.y)dx + N(x, y}dy ""0

P(x)dx +Q(y)dy =0

d2
Second Order: - {
dx

Second Degree:

3.

Standard form:

Examples:

First Degree:

Let: f(x,y) = M(x,y) dx + N(x,y) dy


f(rx, ry) = r" f(x,y)

Standard form:
What are the Solutions to First Order
Differential Equations?

Third Order:

Test for homogeneity of degree n:

Particular solution - the solution


that has no arbitrary constant.

What Is an Order of a Differential


Equation?

First Order:

Day 16- Differential Equations 395

General solution- the solution has


at least one arbitrary constant.

Linear type

dQ""kQ
dt

Standard form:

Substitute y = vx or x vy and
the resulting DE becomes a
variable separable type

dy + y P(x)"" Q(x)
dx
General solution:

Note: First-order differential equations are


not always separable. Homogeneous
differential equations, however, may .be
transformed into separable equations by
the substitution of a variable.
An expression is said to be homogeneous
if all terms have the same degree. The
term "homogeneous" is also used to
indicate that the right-hand member of a
linear differential equation is 0

Ja<x)(Lf.) dx + c

y(i.f.) =

where: .1. f. = .10tegra!"mg factor = eJP(x)dx


Note: Integrating factor is also known as
Euler's multiplier.
5.

Bernoulli's equation type


Standard form:

dy
dx

y P(x)

y" Q(x)

where:
dQ
- = rate of change of the substance
dt
Q =amount of the substance present
at any timet
k = constant of proportionality
3. Continuous compound interest
problems

dP =rP
dt
where:
dP
- = rate of change of the account
dt
P = money present in the account at
a~timer
/
r =nominal rate of interest per year

396 1001 Solved Problems in Engineering Mathematics (2"d Edition) by Tiong & Rojas
4. Flow problems
dQ
dt

. - rate of loss
rate of gatn

wnere:
Q concentration of the mixture at
any timet
dQ
rate of change of the
dt
concentration of the mixture

~I

Topics

where:
( dy)
dx 1

- =

~
II

(:~)! =-(:;)g

D
D
D D
liJ ~
D D
D D

=slope of the orthogonal

Mon

trajectory
( dy)
dx 9

=slope of the given family of

Tue

curves

5. Cooling and heating problems


dT
dt

=k(T -t s )

dT =k(t -T)
dt
$

l3r

1:'/r

Cooling process

Heating process

where:
dT
rate of change of the body temp.
dt
T temperature of the body at any
timet
t. surrounding temperature

- =
=
=

6.

Newton's second law of motion


dV
F=mdt
where:
F =force
m = mass of the body

~~ = rate of change of velocity

( dx)
1
dy 9 = ( dy)
dx 9

Proceed to the next page for your 16th


test. Detach and use the answer sheet
provided at the last part of this book. Use
pencil number 2 in shading your answer.

'~i

il,

GOOD LUCK!

\lrribia:
Did you know that. .. the fame and
influence of Pythagoras spans for
centuries! Pythagoras even appears in
Shakespear's Merchant of Venice, when
reference is inade to the society's belief in
the transmigration of souls.
~uote:

"The most painful thing about mathematics


is how far away you are from being able to
use it after you have learned it."
- James Newman

Geometrical problems (Orthogonal


trajectory)
A curve which intersects all curves of
a given family at the same angles is
referred to as a trajectory; if the
intersection is at right angle, the curve
is called orthogonal trajectory.

Theory

Wed

Problems

Thu

Solutions

Fri

Notes

Sat

t.8u Determine the order and degree of


1111~

differential equation

ld_1_ ) -

c. 2r dx == (J?- + 1> dy
D.

y dx + (2x- 3y) dy = 0

d4 + 5x
2x _r
4
dy

xy

dx

=0 .

Fourth order, first degree


Third order, first degree
First order, fourth degree
First order, third degree

1\

II
<.

1,

(,,.zz Which of the following equations is


.111

7.

Differential Equations
Types of Differential Equations
Orders of 'oifferential Equations
Degrees of Differential Equations
Types of Solutions of Differential
Equations
Solutions to First-Order
Differential Equations
Application to First Order DE
.(Population Growth, Radioactive
Decay, Continuous Compound
Interest, Flow, Cooling and
Heating, Newton's Second Law
of Motion, Geometric Problems,
Orthogonal Trajectory)

684: ECE Board April1998


2
The equation y = ex is the general
solution of:

y'=

72y

B. -y'=

y2x

A.

oxacl DE?

1\
II
II

(x + 1) dx - xy dy = 0
x dy + (3x- 2y) dx = 0
Jxy dx + (2 + J?-) dy = 0
x1 y dy - y dx = 0

Which of the following equations is a


" 111. rhiP ~~eparable DE?

1.1-1,

11

(x 1 x"y)dy=(2x+xy2)dx

II

( .

y) d X

?y dy - 0

C.

Y- _1_
2x

D.

y'

2y

685: EE Board Mareh 1998


Solve the differential equation: x (y - 1) dx
+ (x + 1) IJy = 0. If y = 2 when x = 1,
/
determine y when x = 2.

398 100 I Solved Problems in En


A.
B.
C.

D.

1.80
1.48
1.55
1.63

Mathematics

~I

I~

If dy = x 2dx; what is'the equation of yin


terms of x if the curve passes through
(1 '1 )?

A.

x 2 - 3y + 3 = 0

B.

x3 -

y(k)=4-k

B.

y(k)=20+5k

Ill

C.

x + 3y + 2 = 0

-~

D.

2y+X 3 +2=0

687: ECE BoaJ'd November 1998


Find the equation of the curve at every
point of which the tangent line has 'a slope
of2x.

B.

X=-y 2 +C
y =-X 2 +C

c.

y = y2 +C

D.

2
X= y +C

A.

cosy =
cos y =
cosy=
cos y =

In (c cos x)
In (c sin x)
-In (c sin x)
- In (c cos x)

M9: EE Board Octeber 1997

B.
C.

D.

{y-~x 2 +y 2 )

A.

~x2 +y2 +y =C

B.

~x2 +y2 +y =C

C.

~x+y +Y =C

D.

~x 2 -y +y =C

dx-xdy=O

+
+

y=0
y=0

+ y =0
+

y=0

y = C (sec x +tan x)
y = C (sec x- tan x)

C.

y = C sec x tan x
y = C (sec2 x tan x)

D.

Solve xy' (2y- 1) =


A.

In
In
In
In

(xy)
(xy)
(xy)
(xy)

y (1 - x)

= 2 (x- y) + C
= x - 2y + C
= 2y - x + C
= x + 2y + C

Solve (x + y) dy = (x- y) dx~

x4
x 2y = -+C

D.

x3
y=-+C

+c

4
4

A.

88.60

B.
C.

95.32
92.16
90.72

D.

A.

~x-~y=O

B.

~y-~x=O

C.

~X+~y=O

D.

~X+~y=O

702: The population of a country doubles


in 50 years. How many years will it be five
times as much? Assume that the rate of
increase is proportional to the number of
inhabitants.
A.
B.
C.
D.

100 years
116 years
120 years
98 years

b98: CE Board May 199&


What is the differential equation of the
family of parabolas having their vertices at
the origin and their foci on the x-axis.
2xdx- ydy = 0

B.

xdy + ydx = 0

2ydx- xdy = 0

[)

dy
--X=O
dx

70:J: Radium decomposes at a rate


proportional to the amount present. If half
of the original amount disappears after
1000 years, what is the percentc>ge lost in
100 years?
A.
B.
C.
D.

6.70%
4.50%
5.36%
4.30%

694& EE Board Aprll1996

695& EE Board April 1996

What is the solution of the first order


differential equation y(k+1) y(k) + 5.

C.

A.

A.
B.

D.

690& ME Board April1996

xy=

(y") - xy" + y' = 0


y" - xyy' = 0
xy" - (y') 3 - y' = 0
(y') 3 + (y") 2 + xy = 0

Radium decomposes at a rate proportional


to the amount at any instant. In 100 years,
100 mg of radium decomposes to 96 mg.
How many mg will be left after 100years?

x4

B.

A.
B.
C.
D.

701: EE Board April1997

Find the general solution of y' = y sec x

C.

x 2 -2y-1 = 0
2x 2 +2y-2 =0

x3
xy 2 =-+C

A.

Find the differential equations of the family


of lines passing through the origin.

69:J& EE Board October 1995

B.

3x 2 +2y-3 =0
2y + x 2 -1 = 0

(x- 1) y" - xy'


(x + 1) y" - xy'
(x- 1) y" + xy'
(x + 1) y" + xy'

= x2.

b971 CE Board May 199"/

692& ECE Board November 1994

D.

Solve the differential equation dy -xdx = 0,


if the curve passes through (1,0)?

A.

y(k) =C...: k , where Cis constant


The solution is non-existent for real
values ofy

691& EE Board Aprll1995

A.
B.
C.

Solve (cos x cosy- cot x) dx- sin x sin'y


dy = 0
x
x
x
x

dy +'i..
dx x

Find the differential equation whose


general solution is y =.C1x + C 2ex.

688& ECE Board Aprll1995

A. sin
B. sin
C. sin
D. sin

D.

Solve

3y + 2 = 0

696: Solve the linear equation:

A.

C.
68&: EE Board October 1997

Day 16- Differential Equatl.ons 399

~+f=C

A.
B.
C.

~- + 2xy + f = C

D,

X: -

~- 2xy2xy +

i
l

=C
=C

C.99: CE Board November 1995


Determine the differential equation of the
family of lines passing through (h, k).

704: ECE Board November 1998.


Find the equation of the family of
orthogonal trajectories of the system of
parabolas y 2 = 2x + C .

1\.

(y-k)dx-(x-h)dy=O
A.

y = ce-

II.

(y-h)+(y-k)=_'j_
dx

B.

(x-h)dx-(y-k)dy=O

C.

I)

(x+h)dx-(y-k)dy=O

D.

y = Ce 2 "
y = ce
y = ce-2

7oor Determine the differential equation of


llu: l;tmily of circles with center on the
y

~IXIS

400 l 001 Solved Problems in Engineering Mathematics (2nd Edition) by Tiong & Rojas

It

II

70S: According to Newton's law of


cooling, the rate at which a substance
cools in air is directly proportional to the
difference between the temperature of the
substance and that of air. If the
temperature of the air is 30 and the
substance cools from 1oo to 70 in 15
minutes, how long will it take to cool 1oo
to so?

709: ME Board April1998


If the nominal interest rate is 3%, how
much is P5,000 worth in 10 years in a
continuously compounded account?

A.
B.
C.
D.

A.
B.
C.
D.

33.59 min.
43.50 min
35.39 min
45.30 min

706: An object falls from rest in a medium


offering a resistance. The velocity of the
object before the object reaches the
ground is given by the differential equation
dV/dt + V/10 = 32, ft/sec. What is the
velocity of the object one second after it
falls?
A.
B.
C.
D.

40.54
38.65
30.45
34.12

A.
B.
C.
D.

171.24 lbs
124.111bs
143.25 lbs
105.121bs

Topics

P5,750
P6,750
P7,500
P6,350

Mon

D
Tue

71:0: ME Board Oetober 1997


A nominal interest of 3% compounded
continuously is given on the account.What
is the accumulated amount of P10,000
after 10 years?

D D

A.
B.
C.
D.

Problems

P13,620.1 0
P13,500.10
P13,650.20
P13,498.60

A.
B.
C.
D.

15.45 kg.
19.53 kg.
12.62 kg.
20.62 kg.

708. A tank initially holds 100 gallons of


salt solution in which 50 lbs of salt has
been dissolved. A pipe fills the tank with
brine at the rate of 3 gpm, containing 2 lbs
of dissolved salt per gallon. Assuming that
the mixture is kept uniform. by stirring, a
drain pipe draws out of the tank the
mixture at 2 gpm. Fi.nd the amount of salt
in the tank at the end of 30 minutes.

'L.
..

Wed

(g
Thu

liJ D
Fri

Solutions

D D

707: In a tank are 100 liters ofbrine


containing 50 kg. total of dissolved salt.
Pure water is allowed to run into the tank
at the rate of 3 liters a minute. Brine runs
out of the tank at the rate of 2 liters a
minute. The instantaneous concentration
in the tank is kept uniform by stirring. How
much salt is in the tank at the end of one
hour?

Theory

Notes

Sat

ANSWER KEY
681. A
682.C
683.C
684.C
685.C
686.8
687.
688.8
689.C
690. B

II

691.A 701.
692. A 702. 8
693.A 703. A
694. D 704.A
695.C 705.A
696.B 706.
697.B 707.8
698.A 7.08.A
699.A 709:8
700.
710. D

Differential Equations
Types of Differential Equations
Orders of Differential Equations
Degrees of Differential Equations
Types of Solutions of Differential
Equations
Solutions to First-Order
Differential Equations
Application to First Order DE
(Population Growth, Radioactive
Decay, Continuous Compound
Interest, Flow, Cooling and
Heating, Newton's Second Law
of Motion, Geometric Problems,
Orthogonal Trajectory)

RATING

c::J 25-30 Topnotcher

c:J 1&-24

Passer

c:J 15-17 Conditional


0 0-:Lif failed
If fAILED, repeat the test.

402 I 00 I" Solved Problems in Engineering Mathematics (2"d Edition) by Tiong & Rojas

Ill

x(y - 1) dx + (x + 1) dy = 0
xdx +~:cO
X+1 y-1

Fourth order, since the highest derivative


in the equation is 4th derivative. First
degree, since the exponent or power of
the 4th derivative is 1.

I
I

'

1 -) dx+--=0
dy
1--

X+1
Integrate:

ED

dx-

A DE whose equation isM( x,y )dx +


N( x,y )dy = 0 is exact if aM= aN
ay Ox
Note: From the choices, choice (c) is an
exact DE.
2xydx + ( 2 + x

a( 2 + xz) =2X

a(2xy)
--=2X

ay

Ox

X - In (X + 1} + In (y - 1} = C
atx=1;y=2
-ln(1 +1) +ln(2-1)=C

c = 0.307

y = 1.55

1m

2y =
N = _sin x sin Y

a( -sin X Sin Y) = -COSX sin Y

ax

ax

x2

2y -1

J(cosxcosy-cotx)Ox.+k 1 =C

y(k + 1) = 20 + 5(k + 1)
y(k + 1) = 20 + 5k + 5

~1nxcosy-lnsinx+k 1

C=3.

=C

Eq. 1

y(k+1)=(20+5k)+5

J(-sinxsiny)8y+k 2 =C
Thus the solution is,

x3

2
3
3
3y = x + 2
3y + 2 = 0

sinxcosy+k 2 =C

x3 -

Differentiate:

yz

xz
y' =-y-= y_
2xy
2x

Eq. 2

I ry comparing equations 1 and 1, k 1 = 0


.u1d k2 = - In sin x
1ill IS, the solution is,

Ell

Since, the resulting equation is the same


to the given DE, then the assumed
equation is the solution to the DE.

1m
( y-

~x 2 + y2 }

dx- xdy = 0

sinxcosy -lnsinx = C

slope = dy = 2x
dx

sinxcosy = lnsinx + C

y2 = 2~yy'
2.

y(k+1) = y(k)+5

-sinx(-cosy)+k 2 = C

Y=-+-

y 2 =ex

=0

Assume the solution has the form: y(k) =


20 + 5k

Ill

x2 -1

Solve y(k + 1) using the assumed


solution:

atx=1;y=1

lxz2+ 1)dx = ( )dy

0 = x(2yy ')-

x2 1
y=---

ay

,,xact DE.

Thus the solution is,

il(cosxcos Y-cot x) = -cosxsin y

1 = (1)3 +

(cos x cosy- cot x)dx- sin x sin y dy = 0

ay

x3

x = 1; y = 0

o'=l.!t+c
2

Note: aM = aN , then the given DE is an

= Jx dx

y= -+C

2ydx
(x 2 + 1)dy
2
y(x + 1) = y(x 2 + 1)

c=

at

1
C=--

dy=~dx
fdy

2y dx = (x2 + 1)dy

xz

y = x2 +C

Jdy = Jxdx

Y=-+C

2x 2
y=-+C

M=cosxcosy-cotx
at x = 2; y =?
2- In (2 + 1) + In (y- 1) = 0.307
In ( y - 1) = - 0.594

dy- xdx = 0
dy = xdx

mJ
Note: From the choices, choice (c) is a
variable separable DE

=2 Jxdx

y _ 1 = e0.594

A DE is a variable separable if the


coefficient of dx shall be functions of x
only and the coefficient of dy shall be
functions of y only.

J!!'-111

Integrating:

Jdy

f~+
f~=
fo
X+1
y-1

N=2+~

M = 2xy

dy
.
-=2X
dx
dy = 2xdx
Integrating:

y-1

dy = 0

Day 16- Differential Equations 403

Since tangent, the slope of the unknown


curve is equal to the slope
the line

of

''"''' c; 'Inc
~IIIXCOSY

Ill('"""')

By inspection, this equation is a


homogeneous DE, since the coefficient of
the dx and dy are of the same degree
(degree 1).

404 1O"o I Solved Problems in Engineering Mathematics (2nd Edition) by Tiong & Rojas
Let: y = vx; dy = vdx + xdv

Substitute (3) in (2):


y' = c1 + y"
c1 = y'- y"

Substitute:
vx dx-

,J;.z + v

-~dx-xdv=O

I
~

'

dx+J~=O

JX

,j1+V 2

Let: v = tan 9 ; dv = sec e de


Jdx +
X

-7 Eq. 4

x dx- x(vdx + xdv) = 0

=0

f sec e de
2

J1+ tan 2 e

Substitute (3) in ( 1):

y =C1x + y"

In[ x(sece + tane)] = C


x(sece +tan e)= C

(x+y)dy=(x-.y)dx

-7 Eq. 5

Let: y

= vx;

x(vdx + xdv) + vx(vdx + xdv) = xdx - vxdx

"'

xdx (2v + 0

dx+ (1+v)dv =O
2

V +2V

-1

In y = In (sec x +tan x) + c

dx + _! J(2 + 2v)dv = 0
J x
2 v 2 + 2v -1

=C

1
In x + - In

sec x +tan x = C , where: ec

1..

-eJP(x)dx

ef~

i.f. =

=x
2

y(x) = Jx (x)dx + C

=c

xy=

- 1) + ~ dv ( 1 + v) = 0

Jdy = Jsecxdx

.f _

vxdx + x2dv + v 2 xdx + vx2dv = xdx - vxdx

dy = ysecx
dx

(x) = ~

Substitute:

y' = y sec x

secx + tanx

i.f.

(x - 1)y" - xy' + y = 0

In

= .! ; Q

Equations 405

i.f. = elnx

Substitute:

where: P(x)

dy = vdx + xd

Substitute (4) in (5):


y = (y'- y")x + y" = y'x- y"x. + y"

f~ + fsec e de = o
lnx + ln(sece +tan e)= C

1m

By inspection, this equation is a


homogeneous DE, since the coefficient of
the dx and dy are of the same degree
(degree 1).

2 2

vx dx- ,J1 + v 2 xdx- vxdx- x 2 dv

Da~J:.)if!erential

x4

+c

(0 + 2v - 1) = c
m

= 'f._;

where: m =slope (constant)

Multiply 2 on both sides:


where: ec = C
Refer to the triangle, substitute values of
sec e & tan e to the solution:

b>:'

= C1 + C2e
y" =C2 e
y'

= k,

where: k = 2c

2 Jdy -

r-; -

(0 + 2v - 1) =C, where C =ek

(0,0)

+ 2xy - x 2 = C
2

2xy - y 2 = C

Note: C is an arbirary constant which can

2y + x + C

=in x + In y

ln(xy) = 2y + x + C

xdy- ydx = 0

+ Jdx =c

2y -In y -lnx + x = C

IP!!IP.II
liiliilll

2
x ('i._ + Y -1J = C
x
x
l

rxx

Differentiating:
xdy -ydx =

xz

';ubstitute v = 'j_ :

x,J1 + v +v = C

ID

xy'(2y -1) = y(1- x)

= y(1- X)
dx
2(2y -1)dy = y(1- x)dx
2Y - 1dx - 1 - x dx =0

~xz +Y2 + Y= C

1)

In[~ (0 + 2v -'1)] = k
X dy (2y - 1)

xJ1+(~r +~=C

(0 + 2v -

Jln x + In

y = C1x + C2e

=C (sec x +tan x)

1", placed in either side of the equal sign.

Y2 = 4ax

[D ~~ +(~)y=xz

4a=

Differentiating:
2

-7 Eq. 1
-?>Eq.2

-7 Eq. 3

Note: C is an arbirary constant which c.an


be placed in either side of the equal sign.

x(2Y9Y)- y dx =
l~olo

v ( r )

This equation is a linear DE:


Jo(x)(i.f.) dx + C

-7 Gen. Eq.

xz
2xydy -

dx = 0

2xdy-ydx

=0

406 l 00 1 Solved Problems in Engineering Mathematics (2"d Edition) by Tiong & Rojas

II

ll

If

sdg

dQ =kQ
dt

In Q = kt + C

elny ::: e-X+C ::: e-XeC

dQ =kQ
dt

Let: Q = amount of substance present

= Jkdt

In Q = kt + C

J~ = fkdt

II

Day 16- Differential Equations 407

Gen. Eq.

Let: T = temperature of the body

when t
~

= 0; Q = Oo

Differentiating:
(x- h)dy- (y- k)dx =

= k(O) + C
C =In Oo

0
(x -hf
(x- h)dy- (y- k)dx = 0

when t= 0; Q = 100 mg
In 100 = k(O) + C
c = 4.60517

In 96 = k(100) + 4.60517
k =- 0.0004082

(y- k)dx- (x- h)dy = 0

when t = 1000 years; Q

In Q = -0.000693(100) +In Oo
elnQ = e-0,0693+1nClg
Q = Oo e0.0693
Q = 0.933 Oo

when t = 200; Q = ?
In Q = -0.0004082(200) + 4.60517
Q = 92.16 mg

C(O,k)

Let: P = number of population


dP =kP
dt

r:

>( + (y- kf

= Jkdt

In P = kt' + C

::: r"

~ Gen. Eq.

%lost= Oo - 0 93300 x100%


Oo
%lost= 6.7%

=k(O) + C

2x + 2(y- k)y' = 0

C =In Po

+yy' - ky' ::: 0

when t = 50 years; P = 2Po

k= -+y
y'
Differentiating:
y'(1)-xy"+ '=O
(y')2
y

In 2Po = k(50) + In Po
k = 0.0138629
when t

= ? Q = 5Po
In 5Po = 0.0138629(!) +In Po
t = 116 years

y'- xy" + (y') 3 = 0


xy" - (yf - y'

=0

ml
Let: Q

= amount of radium present

Gen. Eq.

= 1oo

In( 100 - 30) = k(O) + C


c = 4.24849
when t =15 min; T

=70

In (70- 30) = k(15) + 4.24849


k =- 0.0373
when t = ?; T = so

t = 33.. 59 min.
y 2 = 2x+C

when t = 0; P = Po
In Po

when t = 0; T

In (so - 30) = - o.0373(t) + 4.24849

Differentiating:

Differentiating:

_dt_= fkdt
J T -30
In (T - 30) = kt + C

= 0.5 Oo

In 0.5Qo = k(1000) +In Oo


k :::- 0.000693
when t = 100 years; Q =?

=100; Q =96 mg

when t

dT = k(T- 30)
dt

Gen. Eq.
In Oo

y-k
m = - - ; , where: m =slope (constant)
x-h

y::: ce-x' where: c =ec

2y dy = 2
dx

(:~l =~

+~=32
10

10dV + V = 320
dt

10f~=
320-V

Note: The slope of the orthogonal


trajectories is given by:
dy
1
(dx)
dx=-(dy) =-dye
dx

dV
dt

- 10 In (320- V)

=t + C

fdt
~

Gen. Eq.

whent = 0; V = 0 (at rest)


- 10 In (320- 0) = 0 + C
c =- 57.6832

:;ubstituting:
dy
dx = -y
ry =- Jdx

In y =- x + c

when t = 1 sec; V = ?
- 10 In (320- V) = 1-57.6832
In (320- V) = 5.66832
320 _ v = es.sss32
V = 30.45 ft/sec

409 l ~0 1 Solved Problems in Engineering Mathematics (2"d Edition) by Tiong & Rojas

Let: Q

Note: This is a linear DE

=amount of salt in the mixture

100 + (3 - 2)t

= 100 + t =volume of

_.9__ :::: concentratiQn of salt

I
Ill
,!1

when t = 10 years; P =?
In P

a(i.f.) = JA(t)(i.f.)dt+C 7 Formula for

i.f. =

r:

dP = 0.03P
dt

efP<tld!

100+t

d~ = rate of gain dt

rate of loss

2
A(t) = 6
where: P(t) = - 100 + t '
.f
I ..

~9=0-2(-a-)
dt
100+t

- e

'f
1. =

fP(I)dt .

when

i.f.

In a= -In (100 + t)2 + C 7 Gen. Eq.

J-2too+t

In 10000 = 0.03(0) + C

c = 9.21034
when t

=(1 00 + t)2

. .i.
~

Substituting:
when t

=0; a =50 kg.


In 50

I~)~

= - In ( 100+ 1)2 + C

c:: 13.12236
when t

= 60 min; a = ?

In Q =~In (100 + 60)2 + 13.12236

c
3
Q(100 + t)2 = 2(100 + t) 3 + c
when t = 0; Q

= 2(100)3 + c

C=-1.5x106
when t = 30 min;

Let: a = amount of salt in the mixture


100 + (3- 2)t = 100 + t =volume of
mixture at any
timet

_a_
=concentration of salt
100+t
= rate of gain - rate of loss

da
(-a-)
-=3,2)-2
dt

dQ

= 50
50(100) 2

a(100 + t)2 = 6 (100 + t)3 +

a= 19.53 kg.

dt + Q

a(100 + t) = J6(100 + t) dt +

Ina= 2.972

~~

a(100 + 30)

a =?

= 2(100 + 30)3 - 1.5 X 1()6

=171.241bs.

r:

dP =0.03P
dt

In P

=0.03 Jdt

=0.03t + C

when t = 0; P = 5,000

100+t

t =0; P =10,000

i.f. =eln(100+tl'

In a= 2 In (100 + t) + C

2 ) 6
100 + t =

=0.03 Jdt

In P = 0.03t + C

i.f. =e21n(tOo+tl

dt
+C
fada= - f1oo+t

= 0.03(10) + 8.51719

p::; 6,749.30

the general
solution of a
linear DE

mixture at any
timet

It

Day IS-Differential Equations 409

In 5000 = 0.03(0) + C

c = 8.51719

l!i
~

r
~,,,.
l

i~

I
l''
?cl

l
t

= 10 years; P =?
In P = 0.03(10) + 9.21034

p = 13,498.60

412 1001 So1:ved Problems in Engineering Mathematics (2nd Edition) by Tiong & Rojas

. ' "' ~,
,,,~' '"'~*''<-A V

, ...
"'"' 'o",. ~

ll

N.

:,0 _.

'A~'(

<",-

~~"'"'''""'"~

;. <, ... ~ ~

;. {'

~"'""'"' ~.,.,.,.,: .. ,... , ,~ ... :;...,,.,,

' ~' ' ~

"

0+

"~'"""'"'0~'~>.~~,...,~

'!<"'"

'do ,: ~ "' ,,. "<> '< '<>

'<

<-

>.

* -:<

>'

~'-l~,.,..,

< ,;

~ ~

~ '

..

.,

< "

..I<

Topics

..... .,~f

D
D
D
D D
D ~
D D

< , '

~.-~.,~

r~-..

....... ,."

I
' . , ... .+

0V-"

'

Mon

< "'' ~ "'~

..

1:

Tue

~II

~ .< ,, ~

~ ~

., " w " "' "'"' '

~ ~

"'
"'"~

">!'

....,..,.~,~..,:;.
~

"

-~~

If "'" ~ ~ o '"" '. " ~ .,. ~ . ?- ~ '

''<o-<.<->1- ....'%*'!""~

~~~~

~~,

* "'., . ., ~ 1~ .&. '" ~

...

'<-

"''<-.jj~1<

-~ ~ ~ ~

~-~/,.,,;.,..,,,.~ "'~

~ '' ''i ~ ~ ~ ,_. "'

1'

-.,;"<

'! .,..

.,

,.,.,

~ ~ ~ "' o ,, ,

%<!>

*'0'000.

"'

~ >$(

'1(0

* *'-" ,.

;<- i:-

'"*"*

. ::. ....
-'>-,jI'"I!-,Y~O~'R,i:t>:

;; ,1<

~ ..'-<<>"

..

/ .;_ *'

>" < >! ~ ~ 'r 'II' ... ~

'.~\<

.o .<~ ~" ~.,.. .. ,.. : ; . , , . ~

'**""

/~

;,

~~

"

;,

> .->

> -:; ~

0"

'~~~%%-0'"'"-''"'""<>:~-~

+,~"* ~'~ .;..,..,.*~''"'~ '\o)~"~".<'

'~ ~ >: ~-.,~ <"~C

., :t

............. ~

::.. ~.;;: .. ~ ~:i.-<),,.>~,s.-lr-i

..

+. .p

;io.<-'"'><,~'1<*!,-:i<>-:;vP<<~<o$':~,'':<

~<)-yo':,._~41/"~~-lt~t<~~,P.

.,: ,..

'""<: .... ,.. ,..

~, .... ,,._.>r~"'""'~"""'"'~

~ "!"

<

'<1.'<-'J<'~.y

;-,~ o.. ~'""Y. ~ ~ ~o "'""'

<

"<

'v > '".). ~ "'

',.~

':

Theory

Wed

Problems

Thu

Solutions

Fri

Notes

Sat

i:
..~

Su~cessive integral powers of i or j

What is a Complex Number?


By definition, a complex number is any
number expressible in the standard form
a + bi or a + jb.

>

The value "i" or "j" is the imaginary unit or


number. It is equal to the

J=-1 .

Example:
Find the value of x of the equation
0.
' "'" ,,

" 1> .

"'" , '

" ~

:'i<"'ff.x-

~ .; ,.. '

' ., .... '" .. '<- ~ '

..... -~ " .....

i< ;. ~ '

'(.""''''
.

x? + .1 =

Simplifying we get x :::


Thus the solutions are i and ..,. i.

''~'~-~ ;-;~ ~
~~-1

=1

j2 =- 1
= jl j2 ) =- j
j4 j2(
1
j5 = j3( j2 ) = j
j6 j4( j2 )
1
f i5 (
j
j8 j4 ( t) = 1

= l) =
= == f)==
... etc

J=-1

'

Advanced Engineering Math


Complex Numbers
Forms of Complex Number
Operation of Complex Numbers
Matrices
Operation of Matrices
(Addition, Subtraction, and
Multiplication)
Transpose Matrix
Cofactor of Entry of a Matrix
Cofactor Matrix
Inverse Matrix
Determinants
Properties of Determinants
Laplace Transforms
Laplace Transforms of Functions

What are the Different Forms of


Complex Numbers?
a. Rectangular form
z=a+jb
where:
a= real part
b = imagina;y part

Day 17 -AdvancedEngineering Math 415

414 1001 Solved Problems in Engineering Mathematics (2"d Edition) by Tiong & Rojas
b. Multiplication of complex numbers

b. Trigonometric form

z = r cos e + j r sin e

Arc sinh z =In (z .[zz + 1)


sino= ei" -e-iB
j2

Rule: Use the polar form

z=rcise

c. Polar form

Arc cosh z = In ( z ~)

coso= eio + e-iu

(r1 L91)(r2 L9 2 ) = r1r2 L(6 1+ 62 )

1 (1+Z)
Arc tanh z =-In
-2 1-z

c. Division of complex numbers

z= rLO

f
I
l

where:
r = modulus or absolute value
6 = argument or amplitude in
degrees

Note:

Rule: Use the polar form

What is a Matrix?

cote= case
sine

r1Le1 = 5_ L(91 -62)


r2

d. Complex number raised to exponent

Matrix is a rectangular array of real


numbers arranged in m rows and n
columns. The term "matrix" was introduced
by the English mathematician James
Joseph Sylvester (1814- 1897) in 1850.
The size of a matrix is determined by the
number of rows and columns. The
expression "m x n" is the dimension or
order of the matrix. If the matrix has only
one column, it is called a column matrix
and if it has only one row, it is called a row
matrix. The following is a 3 x 3 matrix or
square matrix (i.e. 3 rows and 3
columns).

sec e = _1
case

r2L6 2

Imaginary axis

tan 0 = sine
case

esc e = _1
sine

II

"n"
bl

'/

h. Hyperbolic functions

Rule: Use the polar form


Real axis

(rL6)"

=r" L (n6)

sin h0 = .ee -2--e


cos h6

e. n1h root of a complex number

.r""' -/a 2 +b 2

6=

tan 1 ~

(rLe )*

d. Exponential form
9

where:
e = argument in radians
O~erations

a. Addition and Subtraction of


complex numbers
(a1 + jb1) + (a2 + jb2) = (a1 + a2)
+ j (b, + b2)
(a1 + jb1) - (a2 + jbz)

"" rt L

e + k(36oo)

where:
k = 0, first root or principal root
k = 1, second root
k = 2, third root
k = n -1, n1h root

What are the Mathematical


of Complex Numbers?

= (a1 - a2)

Rule: Use the polar form

z = r ei

= e +e-o

f;

Logarithm of a complex number

Note

tan he=

~nh0

'

i.

sec he=

_1_
coshe

esc he=

. _1_
sinhe

Inverse trigonometric functions of


complex numbers

Rule: Use the trigonometric form


Arc sin z = - j In (iz ,)1- z2 )

In z = r efl = In r + In efl
In z =In r+ j e

Arccos z =- j In
Arctanz=

g. Exponential & trigonometric


functions of a complex number

+j(b1-bz)
ei 0 = cos

e + j sin e

e-ie =coso- j sino

j.

41
4
-2

coshO

cot he = cosh~
sin he

A= 2 1
[ 3 -2

(z~)

The first non-zero entry in a row of a


matrix is known as the leading entry or
the leading element. In the matrix above,
6 is the leading entry. The diagonal from
the upper left to the lower right is called
the principal diagonal or main diagonal
and all entries in the said diagonal are
called as diagonal entries.
If all entries in a matrix above the main
diagonal are zero, then it is said to be a
lower triangular matrix, and if all the
entries below the main diagonal are zero,
the matrix is referred to as the upper
diagonal matrix.

-i 1n(1+jz)
2

A= 2

1- jz

Inverse hyperbolic functi6ns of


complex numbers

~21

3 -2

Lower Triangular Matrix

Day 17 -Advanced Engineering Malh 417

416 100 i Solved Problems in Engineering Mathematics (2"d Edition) by Tiong & Rojas

6 5 4][0 01 -24

Example: Find A -

4 -3]~1
A=~
[

B= 0

'

Upper Triangular Matrix

,11

A-B= -12

Two matrices are said to be equal if and


only if there orders are equal and their
corresponding elements are equal.
Example: Matrices B and C are equal.

C=[3

2]
-1

What is the Sum of two matrices?


If A and B are two matrices of the same
order, the sum of A and B, denoted by A +
B, is the matrix for which each of its
elements is the sum of the corresponding
elements of A and B.

=[-15

-2 6 ]
8 -3

~=AB- 1
8

Solution:

Example: Find DC.

2 -3]
[
~- ~1

=[-4-1 1
3]
2 5

A+B=[9 5 7]
2 8 -7

4(-4)+(-1)(-1)
[
1(-4)+5(-1)

2(-4)+(-3)(-1) 2(1)+(-3)(2) 2(3)+(-3)1


4(1)+(-1)(2)
1(1)+5(2)

-5 -4 -9]
[

DC= -15

-9

L2

If matrix A is reflected in its main diagonal,


so that ali rows become columns and all
columns become rows without changing
their relative order of entries in the rows
and columns, the result is a transpose

11 28

matrix, AT.
Example:

4(3)+(-1)(
1(3)+5(5

A_ [-4-1 21 53]

I
-~
~.

What

--4 -1

AT= [

~ ~

ts a Cofactor of an Enta:y of a

.~atrix?

A cofactor of an entry of a matrix is the


same as the cofactor of the sam~ entry in
the determinant of the matrix and thus, is
defined only for square matrices.
S1gn conventions:

[:

:]

+
- +]
- + [+ - +

1 xample: Find the co-factor of 6 in the


lollowing matrix.

A=[~-~~

2]
1

The determinant of A is:

~~

l=

(1)(1)- (2)(2) =- 3

Thus, the cofactor of6 is - 3:

What is a Cofactor Matrix?

What is a Transpose Matrix?

D is a 3 x 2 matrix and C is a 2 x 3 matrix.


The product DC can be obtained because
the number of columns of D (2 columns) is
equal to the number of rows of C (2 rows).
The product DC will be a 3 x 3 matrix_
DC=

If A and B are matrices having the same


order, then the difference of A and B,
denoted by A - B, is defined as
A- B =A+ (-B)

B.

A + B = [ 5 + 4 -2 + 7 6+1]
-1+3 8+0 -:-3-4

What Is the Difference of two matrices?

where 8' is called the 1nverse matrix


of matrix B

Supposed that A is a matrix of order m x p


and B is a matrix of order p x n, then the
product of A and B, denoted by AB, is the
n x n matrix for which the element in the ith
row and the jth column is the sum of the
products formed by multiplying each
element in the ith row of A by the
corresponding element in the jth column of-

D=

B =[4 7 1
7 0 4

A=+f1

det A:=

'

What is the Product of two matrices?

Example: Find A + B.

Division of matrices is done by multiplying


the numerator by the inverse matrix of the
denominator.

Let: A and B are matrices

-1 + -7 5
8
-8 -2

Diagonal Matrix

2J
2 -1

''',l,

A-B=[~ -3] [-4 -3]


0 -6]
[-5 6

0 0 -2

B =[3

=[~ ~]

.)~~

Solution:

6 0 0]
C= 0 1 0
[

[:
1:

What is the Division of Matrices?

B.

A cofactor matrix is formed by replacing


each element in' the given matrix by its
cofactor.
Example: Find cofactor matrix of A

A=[! ~J
Cofactor matrix of A= [

~ ~]

What is an lnvers~ Matrix?


Steps required to find for the Inverse
Matrix (say. A):

a. Form the cofactor matrix of matrix A


b. Fom1 the transpose matrix of the
cofactor matrix A
c. Evaluate the determinant of matrix A
d. Divide each element in the (matrix
cofactorf

Example: Find the inverse matrix of A.

2 0
A=i2 1 4

4 2 6

-2

Cofactor matrix A= 1-12

6'

-4

I ht~ .:quivalent matrix is:


(Cofactor matrix A)T

=I

-2
4
0

-3
-12

--3

418 .1001 Solved Problems in Engineering Mathematics (2"d Edition) by Tiong & Rojas

det A =

6. The value of the determinant is not


changed if a column is replaced by the
column plus a multiple of another
column. Similarly for rows.

1 2 0
1 4 I =6
4 2 6

I2

[:

[
l
~II

~- .

Day 17- Advanced Engineering Math 419


-2

:.

'

det A=

I5

.'. . . 1

-'~ ','

-5

7
-5 I=

0
11

A=~;; -~

-6

''"c

Determinant of a 2 x 2 matrix:
1-2
Thus , A- 1 = _.!_
4
6
0

-12
6
6

0
6
-3

det A= 1: :1

~,;

Determinant is a square matrix (i.e. the


number of rows= the number of column).
Every deierminant can be associated with
a real number.

;.'i

Determinant of a 3 x 3 matrix:

2. If two columns (or rows) of a


determinant are interchanged, the
value of the resulting determinant is
equal to the negative of the value of
the given determinant.

3. If two columns (or rows) of a


determinant are identical, the value of
the determinant is zero.
4. 'If the elements of a column (or row) of
a determinant are multiplied by k, the
value of the determinant is multiplied
by k.
5. If the elements of the jth column of a
determinant 0 are the sum aij + bij, then
0 is the sum of the determinants 0'
and 0" in which all the columns of 0, 0'
and 0: are the same except the jth;
furthermore, the jth column of 0' is aii, i
= 1, 2, 3, ... , n, and the jth column of 0"
is bij. I = 1, 2, 3, ... , n. Similarly for rows.

0
11

-5

5 0

-6

3 11

=1 d

e f
g h i

d e
g h

Determinant of a 4 x 4 matrix:

Using Pivotal Element method:


Example: Find det A.

2 -4

-1

~1 CD~2 2

I 3

-4

-1

-2

11

Using Modification method:

-4
3
1
2 -4

-1

@0
-1

det A= 200

What is a Laplace Transform?


The Laplace transform of a function f(t)
denoted by l,. [ f(t) ) is defined as a function
of a variable "s" by the integral:

Multiplying column 2 by 1 and add it


to column 1
Multiplying column 2 by 2 and add it
to column 3
Multiplying column 2 by -2 and add it
to column 4

4- (5)(2)

j 2 for 2"d column

where: t > 0 and s is any number (real or


complex)
laplace transform of some elementary
functions:

The new matrix becomes,

5
3

.........

.... ...

r----;--;~;--;~~--;~: and

="'

F(s) = L [ f(t) 1 Jf(t) e-"1dt

A=~~~ cb~: ~

2- (-4)(-1) 3- (-4)(-2) -1-(-4)(2)1


2
3- (2)(-1) -4- (2)(-2) -1- (2)(2) (1)::
1- (5)( -2)

51(-1)21

-3

= 5 [(-5)(-3)- (11 )(5)) (-1)

detA =

1-2- (5)(-1)

detA=(5)~-5

det A= 200

Set the encircled numbers to zero by

det A= (aei + bfg + cdh)- (gee+ hfa +idb)

2 for 2"d row


1 for 151 column

det A= 2(0)(-6) + (-5)(-5)(3) + 7(5)(11)[3(0)(7) + (11)(-5)(-2) + (-6)(5)(-5))

A=

a b c a b

A=

-2 -5

1,.1

det A =I d e
g h

a.

b.

det A

-5

_,,_,_,

a b c

What are the Properties of


Determinants?
1. If the rows of one determinant.are the
same as the columns of another, and
in the same order, the two
determinants are equal.

detA =ad- be

What is a Determinant?

-2

f
$

-2 -5
A=(1)j

0
11

-5

F(s)

1.

2.

3.

t"

4.

eat

5.

t" eat

6.

eat,sin kt

-6

5 0@
3 11

f(t)

i(-1)2 2

-6

Set the encircled number to zero by


multiplying column 1 by 1 and add it
to column 3

1
s
1

52
n!
sn+t
s=t:a
n!
-- )"+1
(s+ a
k
(s +a) 2 + k2

4ZG l 001 Solved Problems in Engineering Mathematics (2nd Edition) by Tiong & Roj!!
eat cos

s=t=a

kt

(s+a) +k

8.
9.

[II

sin at

s2 +~
s
s2 + a2

cos at

10. sinh at

a
s2 -a2

11. cosh at

s
~2 -a2

12. t sin at

2as
(s2 + a2 )2

13. t cos at

sz- a2
(s2 + az )2

14. sin (wt + 8)

s sin8+kcos8
~~

II

[
l
,111

15. cos (wt + 8)

~uote:
"Obvious is the most dangerous word in
mathematics."
- Eric Temple Bell

.I

I
\~;,;

x:

l~
tvlon

}
;:%:,

i:'

Theory

!~

, Problems

0
0

--52

GOOD LUCK!

Solutions

Notes

I
h

'

determinants dates back to the ancient


Chinese who use barnboo rods in
representing the coefficient of unknown
quantities, and gain acceptance when
introduced in Japan by the 1th century
greatest Japanese mathematician Seki
Kowa (1642- 1708)! Even if German
Mathematician Gottfred Wilhelm von

Leigniz (1646 -1716) and Swiss


mathematician Gabriel Cramer (17041752) gave their valuable contribution to
the subjeGt, it was Alexandre-Theopl1ile
Vandermonde (1735 -- 1796) ir1e one
regarded as ti">e formal founder of
determinant theory.

\Ved

Thu

~
Fri

Sat

7U: ECE Board April 1999


: ;1mplify the expression i1997 + i1999 , where i
11; an imaginary number.

r;,~-----~_,_,

Adva11ced Engineering Math


Complex Numbers
1
Forrr1 of Complex Numbers
Operation of Complex Numbers
Matrices
Operation of Matrices
(Addition, Subtraction, and
, Multiplication)
Transpose Matrix
Cofactor of Entry of a Matrix
Cofactor Matrix
Inverse Matrix
Determinants
Properties of Determinants
Laplace Transforms
Laplace Transforms of Functions

C.
0,

2i -1
1+ i

714: CE Board May 1:994

'O.Critiia:
Did you know that. .. the theory of

0
0
0

Tue

s sin8-kcose
+k2

Prm:eed to the next page for your 17th


test Detach and use the answer sheet
prCNlded at the last part of this book. Use
pencil number 2 in shading your answer.

Topics
__

I''

']

1\
ll

The expression 3 + 4i is a complex


number. Compute its absolute value.

-i
1+i

I>

1- i

711:: EE Board April1997


: >unplify: i29 + i21 + i

B.
C.
D.

5
6
7

715: EE Boal!'d October 1993


/\

3i

ll

1- i

I.

1+ i
:Ji

IJ

Write the polar form of the vector 3 + j4.

'/II J: u:~

Boatd April 1'997


3217
form a + bi the expression i

W1 111 111 tlw


1\.'i. I 1111

i\

1'-

II

I
I

6L:53.1

B,

10L:53.1

C,

5L:53.1

D.

8L:53.1

422 . 1ooi Solved Problems in Engineering Mathematics (2nd Edition) by Tiong & Rojas
1:t:t1 EE Board October 1997

71ft: ME Board April1997


Evaluate the value of

r-10

..J-7

B.

-JiO

C.

JjO
Jfi

D.

II!

[
l

l~

71.'7: EE Board April 199ft


2
Simplify (3- i) - 7(3- i) + 10
A.

- (3 + i)

B.

3+i

C.

3-i
-(3-i)

D.

718: EE Board April. 199ft


1200
If A= 40 ei
, B = 20L- 40, C = 26.46 +
jO, solve for A + B + C.

A
B.

35.1 L45
30.8L45
33.4L45

71.9: EE Board October 1997


What is 4i cube times 2i square?

A.

-8i

B.

8i

C.

-8

D.

-8f

7:tO: EE Board April1997


What is the simplified complex expression
of (4.33 + j2.5} square?

B.

c.
D.

12.5 + j21.65
20 + j20
15 + j20
21.65 + j12.5

7:t1: ECE Board November 1998


Find the value of (1 + i) 5 , where i is an
imaginary number.

A.
B.
C.
D.

A.

B.
C.
D.

;I

B.

1- i
-4(1+4)
1 +I
4(1 + i)

'i!

1.9 + j1.1
3.26- j2.1
2.87 + j2.1
2.25-j1.2

:~

C.

D.

221-91i

731.: EE Board April 1:999

169
21+52i

Evaluate cosh ( j~ ).

What is the quotient when 4 + 8i is divided


byi 3 ?
A.
B.

C.
D.

8-4i
8+4i
-8 +4i
-8-4i

If A = -2 -3i, and B ::;: 3 + 4i, what is

~?

A.
B.

13
-90+220i

C.
D.

169

What is the simplified expression of the


6+j2.5
complex number--- ?
3+j4

A.

- 0.32 + j 0.66

B.

1.12-j0.66
0.32 -.j0.66
-1.75+j1.03

C.
D.

A.

B.
C.
D.

A.
B.
C.

25
-18+i
25

D.

18+i
25

square
square
square
square

root
root
root
root

of 3 of 3 of 3 +
of 3 +

2i
i
i
2i

77.91 EE Board June 1:990

7:51 EE Board October 1997

50 + j35
F.1nd t h e quot'1ent of 8+ j5

.
. 4+3i
Rat1ona11ze ..:i .

A.

A.

7.47 L3Q

11 + 10i

2.47 L53

5
5+2i

730: EE Board March 1998

--

5
2 + 2i

. l'fy (2 + 3i)(5- i)
S Imp I
(3- 2i)2 .

B.
C.
D.

1.34 + j0.32
2.54 + j0.866
2.23 + j0.21
1.28 + j 0.98

734: EE Board October 1997


Evaluate the terms of a Fourier series 2
ei10nt + 2 e-i1oxt at t = 1.

A.
B.
C.

2 +j
2
4

D.

2 + j2

s1n x = x-

follows:

!2B = 2

A.

5- j5
-10+j10
10-j10
1b+j15

x3

xs

31 51
x2

+ ......

x4

COS X= 1--+-+ ......

Three vectors A, B and C are related as


at 180, A+ C =- 5 + j15,

C = conjugate of B. Find A.

7:t&: EE Board October 1.997

A.

7351 EE Board March 1998

D.

C.

0.5+j1.732
j0.866
j1.732
0.5 + j0.86S

Given the following series:

C.

--

A.
B.
C.
D.

6.47 L3
4.47 L3

1 + 2i

B.

D.

B.

Evaluate In (2 + j3).

Perform the operation: 4 (cos 60 + i sin


60) divided by 2 (cos 30 + i sin 30) in
rectangular coordinates.

25
-18-i

Evaluate tanh ( j~ ).

7331 EE Board April1999

77.8: EE Board Aprilt.997


18-i

0.707
1.41+j0.866
0.5 + j0.707
j0.707

737.1 EE Board Aprill.999

77.7: EE Board Aprill.99&

1111

7:t4: EE Board October 1997

13
-7 + 17i

7:31 ECE Board April1999

27.7L45

C.
D.

A.

.1:~

A.

''I

A: i

Ill

Find the principal 5th root of (50( cos 150 +


jsin 150) ].

Day 17 -Advanced Engineering Math 423

2!

4!

x2

x3

ex = 1 + X + 2f + 3! + ......
What relation can you draw from these
series?

A.

e,x = cosx + sinx

B.

eix =cosx+isinx

C.

e ix

=I COS X+ Sin X

D.

740: CE Board November :1996

iex =icosx+isinx

Compute the value of x by determinant.

'736: EE Board October 1997


One term of a Fourier series in cosir)e form
is 10 cos 40nt. Write it in exponential form.

r:

[:
l,

A.

B.

C.
D.

5 6 i40nt
5 ei407tt + 5 e-i407tt

10

e-i 40lt!

A.
B.

c.

10 6 i40rd

D.

c.
D.

1
-2

2
-1

3
-2

A.

4
2
5
0

B.
C.
D.

738: ECE Board November 199:1


Evaluate the determinant:

D.

D.

452

-1

-1

-1

-3

-4

-3

-4

-1~ -~1

B.

~-~ -~,

D.

3
0

c.

(1, 2, -1)
{-1,-2,1)

If A= 1-1

I,

what is the

I~ -~1

_,-20

the second row, third column element?

B.

-1~ ~I

I~ ~I

D.

-1

is multiplied by

~ ] ;; eqoal to mo, then ~trix [ ~ ] ;,

A.

B.

c. 0
D.

-2

747: CE Board November 1997


Given the matrix equation, solve for

01
-1

A.
B.
C.
D.

x and

--4,6
-4,2
~4,-

2
-4,-6

7481 EE Board A.pril1996


If matrix [

0 1 0

0"0
0 0
0 0
0 0

-1 2]

[ ~ ~ J[;J=[ ~ J

1
0

~ ~

}s

multiplied by

[~ }s

equal to zero, then matrix [; is

q
0

0 0 1
C.

1
2

y.

1 0 0
A.

41 ,what is the cofactor of

' 0 5 7

120 531

.,...

B.

2 3 1

-2

A.

C.

1
-2

743: EE Board October 1997.


1

cofactor with the first row, second column


element?

-28
16
52

(2, 1, -1)
(2, -1 , -1 )

489

B. 389
c. 326

If A= 1-2

-32

A.
B.

A.
A.

multiplied together, write the product.

Evaluate the determinant:

739: EE Board April1997

If matrix

Solve the equations by Cramer's Rule:

D.

14

:'il

F;ngineering Math 425

746: EE Board April 199ft

7441 EE Board October 1997

2x-y+3z=-3
3x + 3y- z 10
-x-y +z=-4

'110
-101
101
-110

0
4

;I

I~

'7421 EE Board Aprlll997

0 5 3

c.

3
2

D.

745; EE Board October 1997


If a 3 x 3 matrix and its inverse are

1 6 0
4 2 7

A.
B.

Given the equations:


x+y+z=2
3x "'- y- 2z = 4
5x - 2y + 3z = -7
Solve for y by determinants.

Evaluate the determinant:

,1111

10
14

-1

741: EE Board Aprll1997

737: EE Board April1997

A.
B.

X=lI

Advanc~cj

Day 17 -

424 . 100 1'Solved Problems in Engineering Mathematics (2nd Edition) by Tiong & Rojas

0 1 0
'
0 0

A.
B.
C.

D.

8
1

-4
0

749: EE Board October 1997


450
100
If A
16 7 3 and B 0 1
what
1 2 5
0 0 1

is A times B equal to?

4 0 0
A.

lo

0, 0 5

oj ,

426 l 00 i Solved Problems in Engineering Mathematics (2nd Edition) by Tiong & Rojas
0
B.

io

1 0 0

75::&: EE Board October :1997


3
1

Transpose the matrixl-2

6 7 0

c. is
[

~'

'~

A.

2 3 5
4 5 0

~'

[,

-1

9 4

D.

16

2
3

B.

2 5

. I-12 31 I + 2 Matnx. 1-11 21 I=

Matnx

Matrix 1-2
2

B.

Matrix 1-1
1

c.

Matrix

~-~

Matrix

I~

D.

4
2

[ -11
19

:]

C.
D.

[ -10
-19
[-11
-20

2
3

.~!
~1

~~~

-1

2
-1

.:. iI

';'~

A.

B.
2

0 -5

[!

~J

.C.
D.

756: EE Board April :1997

[ 2 e( exp -t ) - 4 e( exp - 3! ) 1
[ e( exp -2! ) + e( exp - 3! ) ]
[ e( exp -2t ) - e( exp - 3t ) 1

B.
C.

[ 2 e( exp -t) ][ 1-2 e( exp- 3t) 1

D.

757: EE Board Marc:h :1998


Determine the inverse laplace transform of
l(s) =

200
-

50s + 10625

2
0

A.

l(s)

-1

B.
C.

l(s) = 2e-251 cos100t

D.

l(s) = 2t e-251 cos 1OOt

The inverse laplace transform of s I [ (s


square) + (w square) 1is

-~ )
(~ ~)
(~ ~)
5
c~ -1 J
(-~

f
:~

A.

sin wt

B.

C.

(e exponent wt)
cos wt

D.

759: Find the inverse laplace transform of


2s- 18
.
-

754: EE Board April :1997


k divided by [ (s square)+ (k square)] is
the Inverse laplace transform of:

B.
C.
D.

as a funct1on of x.

2 cos x - sin 3x
2 cos 3x- 6 sin 3x
3 cos 2x- 2 sin 6x
6 cos x- 3 sin 2x

760: Determine the inverse laplace

cos kt
sin kt
( e exponent kt)

9] .
6

D.

1.0

_:]

755: EE Board April :1996,


EE Board April :1997

The. laplace transform of cos wt is

II

s 2 +9

A.
B.
C.

s I I (s squam)

= 2e-251 sin100t
l(s) = 2t e- 251 sin100t

758: EE Board April :1997

A.

s2

-2

G~)

D.

1-1

3
5

w I [ (s square)+ (w square)]
w I (s + w)
s I (s + w)

B.
C.

A.
.:,

753: Determine the inverse matrix of

Find the elements of the product of the two


matrices, matrix BC.

B.

't"

Day 17 -Advanced Engineering Math 427

Find the laplace transform of [ 21(s +1) ]- [


41(s + 3) 1.

Elements of matrix C =

A.

2
-1

11

-1

Elements of matrix B = [

8]
-5

-2
D.

75:1: CE Board Mzy :1996

[ 11
-20

C.

~I

0
-2
3

-1
0

11

750: EE Board April :1997

A.

2
-1
1

I0

-1

f
transormo,
~.~.

4s'l 8s

(w square) ]

1 I .
-e s1nht

1 e''I sinh!
7

1
4

C.

-e 1 cosht

D.

..!e21 cosht

:I

jl
~

Day 17 -Advanced EMi.nE!~il:lg Math 429

-1

i2=

[Ill

D
D
D D
D D

~II

[I
l

Mon

,,

Tue

,1111

Theory

Problems

Solutions

Wed

Thu

~
Fri

D D
Notes

Sat

ANSWER KEY

711. A
712.A
713.C
714. B
715.C
716. B
717. D
718.C
719. B
720.A
721. B
722.A
723. c

724. B
725.A
726.C
727. B
728.C
729. A
730. B
731. A
732.C
733. D
734.C
735. B
736. B

737.C
738. B
739.C
740. B
741. c
742.C
743. B
744. D
745.A
746.C
747.A
748. D
749. D

750. D
751. A
752. B
753. A
754. B
755. A
756. A
757.A
758. D
759. B
760.A

--

Advanced Engineering Math


Complex Numbers
Form of Complex Numbers
Operation of Complex Numbers
Matrices
Operation of Matrices
(Addition, Subtraction, and
Multiplication)
Transpose Matrix
Cofactor of Entry of a Matrix
Cofactor Matrix
Inverse Matrix
Determinants
Properties of Determinants
Laplace Transforms
Laplace Transforms of Functions

. 3

c:J
c:J

RATING
43-50 Topnotcher

c:J

If FAILED, repeat the test

=-I

Since

j427:::-

16 is exactly divisible by 4

i2 = -1
i3 =- i
i4 = 1

Note:

,,,l;
IIi

',

'

'

'

j17:::

Since

1996 is exactly divisible by 4,

''
,;,_'

:t

Substituting:

Since

f1996

= 1,

i1997

=i

i1998

=- 1

11999

=- i

.1997 + i1999
I

i16=-

If the exponent of" i " is exactly divisible


by 4, then the simplified equivalent of the
imaginary number is equal to 1.

Substituting:

j3217 - i427

Let r

+ i18 = i - (- i) + (- 1) = 2i - 1

= absolute value of the complex no.


(a + bi)

r= -/a

+ b2

Substituting:

r=

= i + (- i) =0

j{3)2 +(4)2

r =5

Ill
20 is exactly divisible tiy 4,
i20

= 1,

i21

=i

The polar form of the complex number, a


+ jb" is given by: z = r L6
where:

r= -la

+ b 2 and

e = tan-1 E.

Since 28 is exactly divisible by 4,


i28

= 1,

Substituting:

='1/32 + 4-

i29= i

Conditional

0-24 Failed

i426::: -

i16=

I~ ,

30-42 Passer

c:J 25-29

.. ::\

,'

i4 =1

Topics

r=5
substituting:

i29 + i21 + i ::: i + i + i = 3i

e = tan-1 .i
3

e =53.1

Since 32'16 is exa(.ily divisible by 4,


1,

i3216::::

i3217

=i

;nu;e 424 is exactly divisible by 4


i424

== 1'

i4?5 "'

Thus, the complex number is 5 L53.1 o

a
..J-io =JfO .[:1
'..J-10 = .Jffii

Day 17 -Advanced Engineering Math 431

430 I 00 I Solved Problems in Engineering Mathematics (2"d Edition) by Tiong & Rojas

H=.fi~

"--1 =.fii

( 4.33 + j2.5) = 4.33 2 + 2( 4.33)(j2.5)

+ l (2.5)

Substituting:

R
../-10 X R

../-10
IIH
[

../-10

~Ill

1~11

Iii

l,

'~~

../-10

x.fl
X R

(JiO i) {.fi i)
.fi0i

,,,

--.3-

( 4.33 + j2.5) = 18.749 + j21.65


+(-1)(6.25)

I
li

ltr.

..fi0(-1)

-.fiO

7 (3- i) + 10 = 9- 6i + i2

21 + 7i

+10
2

(3-i) -7(3-i)+10'=19-6i+(-1)-21+7i
2

(3-i) -7(3-i)+10=-3+i
2

(3-i) -7(3-i)+10=-(3-i)

a
Note: Convert all the complex number in
form

rectang:.~lar

A= 40 ei 120'
A= 40 .L120
A = - 20 + j 34.64
B = 20Z-40
B = 15.32- j12.855

= 26.46

A + B + C =- 20 + j 34.64 + 15.32
- j12.855 + 26.46

= 21.78 + j 21.785

Note:

= 30.8L45o

i2
i3

= -1
=- i

(2+3i)(5-i) = 13+13i(5+12i)
(3-2i) 2
5-12i 5+12i

Rationalizing:
-2-3i = -2-3i(3-4i)
3 + 4i
3 + 4i 3 - 4i

. 1 + i = 1.4142 .L45
(1 + i)5 = (1.4142 .L45 )5
(1 + i) 5 = (1.4142) 5 .L5(45)
(1 + i) 5 = 5.656 .L225
(1 + i)5 = - 4- 4i
(1 + i)5 =- 4(1 + i)

50 (cos 150 + j sin 150) = 50L150


~50L15oo = (50)115 .L150(1t5)

~50.L150 =2.1867.L30

~5QL150 = 1.893 + j1.093

(2+3i)(5-i)- 65+156i+65i+156(-1)
(3-2i) 2 - 25+60i-60i-144(-1)

-2-3i
3 +4i

(2 + 3i)(5- i) -7 + 17i
(3-2i) 2 =-1-3-

-18-i
25

--=--

~50.L150 = 1.9 + j1.1

tm
4 + 8i

-i.....

4 + 8i since i3 = - i

-~

~i3--

Rationalizing: Multiply the denominator


with its conjugate.

4+3 8i = 4+ 8i(i)
i

-i

4 +8i

4i+8i2

-1

4+3i
2-i

(2 + 3i)(5- i) _ 6s + 156i + 65i + 156i2


(3-2i)2
- 25+60i-60i-14M

-2- 3i
-6 + 8i - 9i + 12i2
3 + 4i = 9 - 12i + 12i - 16i2
-2-3i
-6+8i-9i+12(-1)
3+4i = 9-12i+12i.-16i(-")

liD

-.3-=--.2-

(4i 3)(2i 2)= (4)(- i)(2)(-1)


( 4i 3 )(2i 2 ) = 8i

Rationalizing:

-2-3i
3+4i

(3-2il =5-12i
(2+3i)(5-i) 13+13i
(3-2i)2
= 5-12i

liJI

( 4.33 + j2.5) = 12.5 + j21.65

(3-2i) =9-12i+4i2

= -8 + 4i

Note: (r.Le)" = r" .Lne

(3- i)

4 +8i

4+8i- 4i+8(-1)
i3 -(-1)

(2 + 3i)(5- i)- -91 + 221i


(3- 2i)2 169

6 + j2.5 = 6.5.L22.619
3+ j4
5.L53.13

Rationalizing:
4+3i_4+3i(2+i)
-2-i
2-i 2+i

~----

4 + 3i 8 + 4i + 6i + 3i 2
2=1= 4+2i-2i--,i 2

6 + i2 5 = 1.3.L- 30.5
3+ j4
6 +i 2 5 =1.12-"0.66
3+ j4
J

liD

4 + 3i 8 + 4i + 6i + 3( -1)
2=1 = 4 + 2i- 2i-(-1)

4(cos60 + i sin60) = 4L60


2(cos30 + isin30)
2L30

--=--

4 + 3i
2-i

4(cos60 + i sin60) = .L o
2 30
2(cos30 + isin30)

4+3i
1 2"
-=+
I
2 -i

4(cos60+i sin60) = 1.732 +i


2(cos30 + isin30)

5 + 10i

liD
(2 + 3i)(5

~~) = 10- 2i + 15i- 3i2

(2 + 3i)(5- i) = 13 + 13i

4(cos60 + i sin60) =
2(cos30 + isin30)

J3 + i

----------------::::D~a;:.!_..:1..:.7_
7_Advanced Eng~ering_!\iath__.43;'!,_

43Z l 00 1Solved Problems in Engineering Mathematics (Z"d Edition) by Tiong & Rojas

180")
eJ., + e- f.f = 2cos (1t x-;-

50+ j35
8 + j5

e1~ +ei! =1.4142


Therefore,

61.03L35

= 9:43L32
50+ !35 =6.47 L3"

. ) =~1.4142

cos h( 14

8+]5

cosh(H)

IIIII!
[

Hklllli

l:l o
[

Iill

l.
'llllllj

Ill

~=-2

tanh(.7t)
J- =
3

B
A=-28

10cos(40nt) =10 [ ~-~e

Substitute: t = 1

10cos(40nt) = 5 ei~Om + 5

=0.707

Note:. eJ9 + e-fo

c =-5+ j15

.,-2 (a + jb) + (a - jb)

=-5 + j15

-2a-2jb+a- jb = -5'+ j15

-a- jb =-5 + j15

e~- e -~
e'~ +e-~.

Euler'~
equat1ons

., +e-r,., = 2cos [1t x-7t180]


er,

eii +e-ig = 2cos60

e~+~-~=1

e~ + e-~

By inspection:

:/,~

=j2sin60"

tanh(ii) = j1.732

'I!

cosn(H) =e + er.

=2cos0 7

.:...ix3

x4

Euler's
equation

let: x

ix5

= In (2 + j3)

x =In (3.6 eious)

x -ln3.6 + lne1ooo

0:.1

,,

21

~~ 1~)( . . ~3 .~~),!
.. + 11 x-3!+5!+ ..... !
l

v
i sin x

D=le
i

1
3

e 1'

-=

2~osn

1
:

-2
-4

-~ ~-
2
-3

-3
-4

!li

eix = cos x + i sin x

rm ,
~~~

=-101

I hus,

0 5 3

0=6+0+0-0-35-72

=:1--~+---+ ... .

6
2

- (3)(4){6)]

X 6 .i

COS X

- (0)(2)(0) -- (5)(7)(1)

X4

1 6 0
4 2 7

D = [(1)(2)(3) + (6)(7){0) + (0)(4)(5)

................ ., .. .., ........................................

2 + j3 = 3.6 ejS6.Jx,:.

2 + j3 = 3.6ei0 98
Note: ejll + ejll

-')1(6

Convert (2 +j3) to polar form, then to


exponential form:
2 + j3 =3.6L56.3"

0:::5

+6! .......... .

111

=e +e
jg

- x2

- (4)(-2)(2)]

e =1+1X+-+-+-+2!
3!
4! 5!
3

D ::: - 4 - 12- 6 + 9 + 2 + 16

.1t)_j1.732
t an h( J- - - -

6 -i40.t

- (3)(-1)(3)- (1)(-2)(1)

)
(ix )
(ix )
(ix )
=1+. I X(ix+
--+-+--+-2!
3!
4! 5!
(ix)6
+6!
........ ..

,.

A=-10+j10

ei10)

Let x = ix

Therefore,

coshx

=2 (ei10x .+

.2

_J'l

-j40l<l

D=l-2 -1 -2
-2 -1
3
1
4
3
1
D = [(1)(-1)(4) + (2)(2)(3) + (3)(-2)(1)

x""4

e1

Therefore,

A =-2(5- j5)

7 Euler's

(
180")]
x =~ [ 2cosl107t
x -n-

e~ +e-~ =j1.732

a=5
-3b = 15
b=-5
a=5

=2 cos 0

equation

A+C = -5+ j15


-28 +

x =2 (ei1o~ + e-i10)

Note: efo + ej(l =2 cos a }


efo _ eiO = j 2 sin 0

= a - jb

j40.t

Let: x = 2 el10.t + 2 ei10.t

x = 2 ei10n(1) + 2 e-i1D!11

tanhx = e -e
e +e ;

X =1.28 + i0.98

121

~ == 2L180"

Let: B = a + jb; C

i'

1./

Using Pivotal element method: (Use the

second row,

tirst column element as the

pivot number).

7 Euler's equation

D = (1) -2-1(5)

14 _ 2i5)

3 _ 2(-1)
2-1(-1)

1--4- 3(5)

:..3- 3(-1)

~lilt

l.li
[

+'

-4- 3(3)

4
-7
-13 -19

D = (-1) l-7

3
-19 0

Ill!

1- 2(3) I
-3-1(3) (-1)2

-5

..,.6

3
0

-2

-1

0
8
4

X=l

X=(1)1

31

-2

-6

-2

-1

5
-4

ll

oG)

-2
X=

1'111

- (0)(-6)(4)- (-13)(-7)(5)]

-1

-4
3 -2

8
4

-1

..,.6

I! I

X= [(-2)(3)(-6) + (-1 )(-2)(4)


+ (-4)(8)(2)- (4)(3)(-4)

D = 326

a
X=l

-1

10

14

X=- 28

Ill

Multiply column 4 by -2 and add it to


column 1:
3(- 2) + 4 =- 2

x+y+z=2
3x-y -2z =4
5x - 2y + 3z = - 7
By Cramer's rule; y =

1(-2)+2=0
1(- 2) + 10 = 8
5(-

1(-2)+2=0

=I 3

2) + 14 = 4

Multiply column 4 by -2 and add it to


column 3:
3(- 2) + 2 =- 4

-1
5 -2

ci

-2
3

3
5

:f

-1

-2

'I~

,r

D = [(1)(-1)(3) + (1)(-2)(5) + (1)(3)(-2)


- (5)(-1)(1)- (-2)(-2)(1)- (3)(3)(1)]

D = [- 3-10-6 +5-4- 9]

Dy

=I 3
5

4
'--7

-2
3

3
5

4
-7

Dy = [(1 )(4)(3) + (2)(-2)(5) + (1 )(3)(-7)


- (5)(4)(1)- (-7)(-2)(1 )- (3)(3)(2)]

=12

D
12
y=-

-1
3

-1

~1

-1

-1

-3

-1

10

-1

-1

3
-1

Dz=l

I 32.

-1

I -1

.- 1

Dz

2
3
-4 I -1

= [(2)(3)(-4) + (-1)(10)(-1) + (-3)(3)(-1)


- (-1)(3)(-3)- (-1)(10)(2)
- (-4)(3)(-1)]

D = ((2)(3)(1) + (-1)(-1)H) + (3)(3)(-1)

Dz

- (-1 )(3)(3)- (1 )(-1 )(2)- (1 )(3)(-1 )]


D

=[6 -

1 - 9 + 9- 2 + 3)

=6

= [- 24 + 10 + 9-9 + 20 - 12]

Dz= -6
Dz

Z=o
-3

-1

-3

-1

10

~1

10

Z=-

-1

z = -1

Dx=l

-4
Dx

-4

-1

= ((-3)(3)(1) + (1)(-1)(-4) + (3)(10)(1)

-6
6

Therefore, the answer is ( 1, 2 - 1).

- (-4)(3)(3)- (-1)(-1)(-3)
"(1)(10)(1)]

ox= (- 9- 4 .. 30 + 36 + 3 + 1OJ= 6
Ox
x=o

X::

Dy=:

3~

4
0 5 7

A=l-12

Let: X.'= cofactor matrix of A

X=l~ ~~(-1)2+ 3

x=s
1

3 - 36 + 30 - 8 + 9]

= Dy

D = -27

5(- 2) + 4 =- 6

2x--y+3;t=-3
3x+3y-z= 10
-x-y+z:=-4

(.

1(-2)+0=-2

The new matrix becomes,

.
l
'

=[20 -

oJ

- (2)(-2)(-2)- (-6)(8)(-1)]
X = [36 + 8 - 64 + 48 - 8 - 48]

Dy

y=2

D= (-1) [-156 + 570 + 0-285 + 0-455]

D = (-1)(-326)

- (-1 )(10)(3)- (-4)(-1 )(2)

y=-27
y=3

'jg

= 81

-81

,Sf

.,

Dy

Dy = [(2)(10)(1) + (-3)(-1)(-1) + (3)(3)(-4)

I~

-2

= [12 ... 20-.21 20- 14 -18]

Y= Dy

..~

( -1)2+4

3 -2
2 -6

8
4

Dy

- (1 )(3)(-3)]

.;,

D = (-1) [(4)(3}(-13) + (5)(-6)(-19)


+ (-5)(-7)(0)- (-19)(3)(-5)

'~~

0
3

-4

_____ . ____ J:>ay 17 -Advanced Engineering Math 435

~ -~

434. 1001'-Solved Problems in Engineering Mathematics (2nd Edition) by Tiong & Rojas

2-3
10
3

-1

-4

X=-\~ ~I

312-3
~1

3
-1

10
-4

Ill

CD

3
A =1-2

-1

2
0

-1

436 1001 Solved Problems in Engineering Mathematics (2"d Edition) by Tiong & Rojas

X=
ll'lil
[

~~11

1~1

lll

~~~~

A (A"

A (A

-2Q -1o

X=-

(-1)1+2

''[

X=-1-20

-1

!!f

El

Note: T~e transpose of a given matrix is


formed by interchanging the rows and
columns.

'I'

[ _;

"!~~-

jl

~-

][;] = 0

A =j-2

1
-1

.li

A = 3 x 3 matrix
A" 1 = inverse of matrix A

Let:

'!ill

6- 3y + 2y = 0
y=6
x=2-6

= cofactor matrix of A

Let: x

Day 17-Advanced Engineering Math 437

=A ( ~)

= 1(unity or identity matrix)

Note: A unity matrix is a matrix whose


elements in the main diagonal are
all number 1.

1 0 0
1 0

1 0 0

6 7 31X

0 1 0
0 0 1

2 5

Unity matrix = 0
1

o o

El

[ 1 -1 2] [ z
2

-1

1-12

+2

1(x) + 1(y) = 2
x= 2-y

-7 Eq. 1

=0

-7 Eq.2

Substitute Eq. 1 in Eq 2:
3(2 - y) + 2y = 0

I I. .

2-2

-52]

= [ 0(3) + (-5)(4)

. =[ _;~

1+41
3+2

58

A-1

=_!ld
0 -c

rhus,
A_,=

5j

51

_1_19
1-92

A-1=

1(6) + 2(1)
0(13) + (-5)(1 )_

-b

- -2
2

= cos wt 7 Formula

S +W

2s-18

2s

18

s2 + 9

= S2 + 9 -

s2 + 9

5
-1

Note:

}
cos at = - 2- s +a 2
.
a
Formulas!
sm at=-2- s +a 2
2s -18
.
Thus, - 2 = 2 cos 3x-6sm 3x
s +9

By completing the square of the


denominator:

8s = 4 ( s 2

2s)

_4s2 - 8s = 4 ( s 2

2s + 1) - 4

4s2

ml

. kt
s1n

1m
1m

cos wt = - 2- -7 Formula
2

= - 2-k -2
s +k

-7 Formula

Note: e at

1\ n "''

s-1 a

2
s

-1]

1[

12

Note:
7 Formula

4
I

--=4 (s-1) -1

+W

=-1-

4s 2 -8s=4[(s-1)

I hw;, ;> e

= 2e-251 sin100t

-5
' 1

(-1) -2

~J

[!

..

. A = ac bd , 1ts 1nverse 1s g1ven by:


rnatnx

,
lit

100
)
(s+25)2 +(100)2

2s-18 _ 2 [ - s J- 6 [-3-J
52 + 9 s2 + 9
s2 + 32

Note: For a 2 x 2 matrix, say,

~-~ ~I

= -1 +2

1(3) + 2(4}

3(x) + 2(y)

O=l; ~I

1-.l

[~

-1

Solving the determinant of the given


matrix:

21=12
11
1
-1
3

-1

=0

0 =-1

=I 0

[ ~ ; J[;]=[ ~ J

- 2
-1

-7 Formula

....,.__
20_0~us, s2 -50s+ 10625
-

Ill

4 5 0
6 7 3
2 5

Th

(s+a) 2 +k 2

-2(

0=9-10

By inspection, since the resulting product


is zero then, x = y = z = 0.

A transpose = j1
2

Since matrix B is a unity matrix, then A x


B is equal to matrix A.

Note: e-t sinkt =

0
-1

By inspection, since the resulting product


is zero, x = y = 0.

4 5 0

<; I :~

e"1 sinh kt =

k2
(s-a) -k 2

1
1
"('h us -----, h\
=- eI s1n
' 4s~ 8s 4

+Formula

~ 1001 Solved Problems in Eng!n~ring MatheJU&tiCS (2"4 Edition)by Tiong & Rojas

.. '- .; ,.,..
~"' ......... .;, ... < ... "

t;

ltl~li~l

L~,

'~

-~

'

.v .;

("

~ 'f '!o ~ z. ,, a ..-

""'

*"" ...... ~ ..

~-

~~-

,<~-~~

* ~ W\.4

.;,. ,,

"~ >

< ." " ~ "lli

... "

~ 'i':1' ;f4 '<' "- ~ 'll \!

<':": ''

"'

~ ~

~~

'$

j(

~~

J . f,) 4

"

., ,,

'X '>: "

'

~ ~

-.. , "

" ..

.,..

,'?,.If.; !I' Pit

iJ.''li,

~'f ,*''to>;: "I',. *: ~-"> '=" 0<. it- !tcf.;. -~ ~ *' "' ~.,."" :->=:->;~ ~1 :.O..<o."' .,_._,..-:~"'

.,_ f..\o; ":1.

4; ~ ~ ., " ~

\o ! A'

~ < "' ~

,. v

> + ,. p" \

> ~

}~.

v, ' ,

<11>_,..,,.,,.

,:;g

Topics

';~

0
0

.~~.
'11

~ i' 1 .~ '>. ~ ~ ~ 't ~

X :

"t " ( !"-"" 1-" '' ~: 9' ~ -#- 4 ~

<>

"'<~''<~.{'<~

H<t

~ '<

II {'

'o'

~ .< ' ~ '

k >l" f:

f<'"' 'f J" "<" ..,_. f- ' #

,. '

~".-~~".'!)< ".r,

;,,

.,

,. { '"' " .

;!-l!<+.~~.'>e,o>~~ ~":<_'*:1>.~

{lj Y.."/l'i!

~ ~ ~

.<.><~<-~~'-\">t/'>~1'Wox.~~"<

..<~t-~.}~-~

1' _..,. 'I "'/ {, '< : :'>

"' .s- ~ .;_ ? '< " ~ ( < ~- ~ "' ~ ~ "' ,. 6

,:.,. <

1{~~..,~~,,_,.~

l",'<)',~.!tfc~;....,.,t:Yt"-<1}

'i< ' ~

(t

< ' ~ .,_"'

,v

~ ~

<

f>"i'lt""'"' I><"~"

111111111!11

r~'
~-~~I

i< 11 "' "

_,.

<

--~~>!<

.1' ..

..o - <,

"<'

'<!- 4,

!"".'*'~ '-* ,. _,.,. y~ ."-It"- .if" ,<

&,.'~

~ \o,..., "',,.'<II"'-~

~ ~,S. ~ ~ .<~ \' i<c <" 4.' ~ ,"> I

< ;o ..o,

.~ i->

'-<" '-o l'

;l

,. /A.,,.

.~ ..... 'J it--~ '

,f

i,t~:

Tue

~ ~ :"~; ~.,'<

.''

~,.'"'!'(i;.f'j,:t.1t'\""'"'.!>.~!!P.;S.f..s.'oJt:O '~

O:"f'..j. ,..,.._

"'"' ,. .

' yx,"..,

Mon

>'

"'>

j.

,,,.-i--';~..;,
\<-

t-~ >~-"'-""'":o><'l}

f;":<i-1'

~~ Jt.., ,."~.;, ~

-..:-:;:> _;; ~

;,. <),

l<'.-'. fi- "'*"} 'f :<l1r1J~

~<I<

<I'*.*"'' >10

-t~ ~,~:.,. ~

"l' ~ ., ~...,.<of ;".0

1;- <1:

~ ), .< ~

i "'

~.~f..,~,.,;..

~ <\:.~<t

"?;" =i <'4< .o:t

y:~l,!--..1, i .Jib-~~

*-i'-*#-~,.;,.1<! +"~J,.,"t-.,41.~

~ ! ~ ~

Si'

< ,..,..-.,:..

~ .'-, ~

-tf.

.,;," .<.

~\~

;.!

'v" ' ~

'II"'<! -i<

'( ~< ~ ><.

,_.;.~.'k/,, ~..) <

~I

o'

! ....

'.i r ~F.,.-" l ~ ,-~" :>-:<..< "

,.

. . '* *''

<

~ """:'~.

'~" ~ >t~

"' ....

o>r ,;
;>. 'i' 1 .,.. .,

' ..... -~ .,...,,..,. "'~ ~~f" ..,'to,..,."

..,. ..... +f.~~ .;ci><.,~ ~ ~ ~<,""


~~-~ .. ~-;:,{.~1""'

"

'~

~:,.

,<>'..

4, .... ~ ~- &

" *"*

'r

~-.,_,:

-:>

\P ., ,..

r(i. <'.'~,j,.

1'"'" .t ""

~ ~

~--.:;.

''"

1>, " .., ~- ;.

;o

~ ''~.4~'

>

., '".,; ('

l"'>

~"'."'

.!<l~<r<

Y<'"" .. .'l~><>,'"~

< .,

.., .. "" """

' ~'1

0
0
0

Problems

I'

Solutions

..

~:<-"'-<-i

' ' ..

'<-'""

Nctes

41 ,,

"'''i

'\>

-~

.......

,~-~""<!<"'1"""~f"'1

.....

~' ~ '"~

}! ... .;. ,. ~

< f"

,,;~ . .<"C'>"'i"'~ .....


~ .......... ~*+of*:~"'

;.;.

''-:""~;':.;i"l~ .....\.ll-.<:~~'"""''""'"""*"'"'

*,.

.~..,

"

~'

'-"lf.'(-1<'<:

""+'<'!'l'lf,~1><l1.1i>01A-<:~... ,..~~~-"*'':'

..... *'t- ., *''"

;:"~.).. <.><"' ~ ,.,., ~

>~1 "i'>"'.''-l<f,:<-

~.>.<...d.,~-)!

,,.,;;.,...;,,:0,~*':'-~.'-'~~.,

' * "' ... '- , .... +. ;


.:,..~

Theory

'~li<'"~

'1..<~"'."" ,..... ..,..~-"} **~;.~~

!'>) -..'",.;

.. <(!>

1 'f'

"'f" .f.:~ ,. "'"" ~ . ,,, .., " ~ ,.. -<... '{ .. ~, ... i

,o,

'~.:....,.'".~

'#

"")<,.<,.,..l.,,

" "~ ,;, ..: ~ "' .., 1 ~ ,.:,. ....... '~ .; " " .... ~ ~ .. _, " ... , ;-."' -~ ..

.,_:.:<1. ...... ....

""'~. :! , ,1-->. ;;~ .... io

:" '\< :<

r"~~

~ ... -..., . .,.~ ......i> ... <"<~.""

'<

'"".~

"":"''""'<I' ,./.of""-"' {.""

!( /i'-$'

<:t-'<>".f!-A',.. j.>

~ ~ ~ 0~

. .<-

.. ,...,.__ :,_,..,. ~ol<'t 1l-..;. ,:,~-"' ~ v. i+ ;-~1 ~ -~* 1<-:<1!., ... ~ ' " .,.

'.~, V~ >

., ".;

~>''

> ', <

* .<

~ ~- ,., . '

... A-~

1f

.<-

,_,,t.,\"'-"'-'"\~.*l$-<t<:\~~~le11'>~><,.f<,"YI'!'*.-t-io<1!->:).o~\'";t

''>'

"O!" " .f!! ... ,..

-~,

'""~"-->',J-0.'. .~

-'"'*f+~,o:> ..... .. , \ .............. ;t-.......

:/<'.,/,~ ,.. ,"

-;,, ... ~.,~~~..~.,~.,"'-.''~"';.<>.,

"V <1-f<

~...,. ~~~~''"'

,;,. ,.,.

" "!' ..

J .,.; il< ~ , _,.,.

.. ,)

o' ,..,.. e ... ._.~'->~-"-

~!"!-.:'"'"'"""""""'\'""'''".'"'~'<1",<"~"<".

'"""'~,.. .......

.~".?I*"''~:.<,...~ .,..t,-<i.<J'<>t

<l-',!<"'<'

=t/>f ;<t / ~,;. 'i

~f<yo ........ ~

r--.

1o-<r..i1: 1<

~<t !.1""" ""':.~ "'.~ ~ *'!'.~

"">;< <>

i>"<"i~tr-~<".'<~o.r':(,.,

' "' ,, ~ "" ' ~ .... ~ " ... \-. .., " '

Y,

"'''""

~!

<'ic~f<.,.,

~-"~'>t.ff<~jo."''~~-~""''"'

*' ( '" ;..,. ,,

't ...

i\'.,.,,., "ff< ~ ~ .' <1 ~

:i'\~.,;..,,.,.11!-"*"'!"

~ f.",~'t

t;~-r.~~

"""~.~"'*

'!I

"'.., ~"' ~, ' .;; .,.. ~ . ,~ ~~ t '\"' ., , "

..,.,_. :

4. :j.,':.<~

,J

~ :J>~ 'ft. t>.~~-lt- ,o::"..,_l.!;,l.,. o1 ~:<~'< l!"4.l!~ <fi -"'.+ .'.{~~

'P- ,..,.

i '..,.';:..,....,"

~.>'~.,.

~1i~;<u~~'"'"'

oY

>{,''<to";..;..:,..,

,. ..

=~~>~.~t;.'>+.~

' "''

<; $ ~ <!" ~ '.

:<~

~ "'" ~ $<

.j "'

'

"

~ ~

'

! ."' ~ if- ~ , '

~~ ~'t ,, ...,:~ .. ,,. .,.. >:"1'>,~ ~'

~ ~ ., ,.""), ~~l

> ..., ' ... ~ ... .., ""'<_,

~ 1",. ' ' ~

' ,. '. ~, '>"


' ~ ., ,;. "'*"

, ,.

Physics
Vector and Scalar Quantities
Classifications of Vectors
Speed and Velocity
Distance and Displacem~nt
Acceleration
Laws of Motion
Forces
Law of Universal Gravitation
Work, Energy and Power
Law of Conservation of Energy
Momentum and Impulse
Law of Conservation of Momentum
Gas Laws
Properties of Fluids
Archimedes Principle

I
~

Wed

Thu

0
Fri

[g
Sa!

What are Vector and Scalar Quantities?


Vector quanities are quantities whose
measurement is specified by magnitude
;md direction. The following are e~amples
of vector quantities: Weight, momentum,
torque, velocity, displacement,
<~cceleration, electric field intensity, etc.
Scalar quantities are those quantities
which have only magnitudes. The following
.~,,~examples of scalar quantities: Speed:
11\ass, volume, energy, length,
I "nperature, pressure, voltage, time etc.
1he term "scalar'' comes from Latin "scala"
wl11ch means "a ladder".
v.~ctor 1s the line whose length indicates
''' ~;aft' fhP maqnitude of the vector
'l'l<~rJtrty and whose direction indicates the
"" ''"'"" nl ltw quant1ty The term "vector"
l.ol111 "vthr"" wl11chrw~ans

'"'"'"""'II
'It'

I ;lily''

Yl/hat are the_Ciassifications


Vectors?

1.

2.

3.

91

Free vector- is one whose action is


not confined to or
associated with a
unique line in space.
Sliding vector- is one for which a
unique 1ine in space
must be maintained
along which the
quantity acts.
Fixed vector- is one for which a
unique point of
application is specified
and therefore the
vector occupies a
particular position in
space.

44Z. 1001 Solved Problems in Engineering Mathematics (2"0 Edition) by Tiong & Rojas

t'lllllb~

lt1m1~~~~

What is the difference between a Speed


and Velocity?

Newton's Second Law:


(The law of acceleration)

Slug is the mass to which a force of one


pound will give an acceleration of one foot
per second per second.

Speed is defined as the distance per unit


time. Speed is a scalar quantity.

"Whenever a net (resultant) force acts on


a body, it produces an acceleration in the
direction of the resultant force that is
directly proportional to the resultant force
and inversely proportional to the mass of
the body."

Gram force is one-thousandth the pull of


the earth upon a standard kilogram at a
place where g has a value of 980.665
cm/s 2

Velocity is defined as the displacement


per unit tim~. Velocity is a vector quantity.

t""

What is the difference between a


Distance and Displacement?

l.,,

Distance is a length from one point to


another usually measured in a straight
line. It is a scalar quantity.

a-lii~ It

'~~

Dayl8- Physics 443

Dis!)lacement is the change in position,


specified by a length and a direction.
Displacement is a vector quantity.

What is a Frictional Force?

Instantaneous acceleration is the time


rate of change of velocity.
Uniformly accelerated motion is defined
as the motion in a straight line in which the
direction is always the same and the
speed changes at a constant rate.

Frictional force a force acting on the body


whenever it moves while in contact with
another body. This force always opposes
the direction of the motion. The frictional
force is proportional to the normal force
and is directed parallel to the surface.

Newton's Third Law:


(The law of reaction)
For every force that acts on one body
there is a second force equal in magnitude
but opposite in direction that acts upon
another body."

Newton's First Law:


(The law of inertia)
There is no change in the motion of a
body unless an unbalanced external force
is acting upon it."
Inertia is the property of the body by virtue
of which a resultant force is required to
change its motion.

Centrifugal
Force

Let Fe = centripetal force.


For Sl System:

F=J..i.N
2

Fe"' tnVr
-r-.

What is a Force?

where: J..l. = coefficient of friction

Force is a push or a pull that one body


exerts on another. This includes
gravitational, electrostatic, magnetic and
contact influences.

Coefficient of kinetic friction is the ratio


of the frictional force to the perpendicular
Ioree pressing the two surfaces together

Constant forces are forces that do not


vary with time. External forces are those
actions of other bodies on a rigid body
while those forces that hold together parts
of a rigid body are' called internal forces.

J..l.k

"''N

Coefficient of static friction is the ratio of


the limiting frictional force to the normal
Ioree.

Weight (of a body) is the resultant


gravitational force acting on the body due
to all other bodies in spac.e. It is always a
vertical force acting downward.

Fmax

J..I.,=N

For English System:

J:n'Vr...
Fe=~

Dyne (dyn) is the force that will give to a


mass of one gram an acceleration of one
centimeter per second per second.
Poundal is the force that will give to a
mass of one pound an acceleration of one
foot per second per second.

~
~:

~' '

......I

What is a Centripetal Force?


Centripetal force is the force (real force)
on the body towards the center of rotation
when the body is moving around a curved
path.
Centrifugal force is the force (apparent
lor cc) on the body directed away from the
': ~~~ll~r of rotation when the body is moving
uorrrHI a curve path.

lbf

9c.

where:
m = mass in Ibm
Vr tangential velocity in ft/sec
r = radius of curvature in ft

gc = 32.2
Newton (N) is the force that will give to a
mass of one kilogram an acceleration of
one meter per second per second.

Newtons

where.
m =mass in kg
Vr = tangential velocity in m/s
.r = radius of curvature in m

What are the Laws of Motion?


There are three Laws of Motion which are
commonly called as "Newton's Laws of
Motion".

.......

F=ma

What is an Acceleration?
Acceleration is the change of velocity per
unit time.

Ibm- ft
sec 2 -lbf

Normally, the centrifugal force is equal in


magnitude with the centripetal force.

What is the Law of Universal


Gravitation?
The Law of Universal Gravitation is also
known as Newton's Universal Law of
Gravitation is stated as follows:

i!

444 100 i Solved Problems in Engineering Mathematics (2nd Edition)


"Every particle in the universe attracts
every other particle with a force that is
directly proportional to the product of the
masses of the two particles and inversely
proportional to the square of the distance
between their centers of mass."

Day 18- Physics 445

tiy Tiong & R~


What is the Law of Conservation of
Energy?

Energy is also defined as the ability to do


work. Energy is a scalar quantity.

The Law of Conservation of Energy is


stated as follows:

The typical units for energy are joules,


calories and BTU (British Thermal Unit).
The unit BTU and calorie are used for
thermal energy.

"Energy can neither be created nor


destroyed; it merely changes from one
form to another."

11.111111

Calorie is defined as the amount of heat


required to raise the temperature of one
gram of water 1C.

lllillltlllf
1~1~111~

l'lllklli

l~~~~

'~~

Transformation of Kinetic & Potential


Energy:

m1

mz
Gm 1mz

F=~

British Thermal Unit (BTU) is defined as


the amount of heat required to raise the
temperature of one pound of water 11=.

What is Work?
Work refers to the process of changing the
energy of a particle, body or system. Work
is a scalar quantity. The typical units of
work are joules, foot-pound and inchpound.

fngh :=; ~tnv

EP = mgh

The unit "joule" is equivalent to the units of


N m and kg m 2/s 2 . This unit was named
in honor of the English Physicist, James
Prescott Joule (1835- 1889).

where:
W = weight of body
m = mass of body
g = gravitational acceleration
g = 9.81 m/s 2
h = height of body

Mathematically, work is defined as the


product of force and the displacement in
the direction of the force.

Kinetic ener~:IY is the energy in motion.

W = Force x distance
What is Energy?
Energy is the property of the body or
system of bodies by virtue of which work
can be done.

or

= J2gh

Transformation of Work & Kinetic


Energy:
Work

= Kinetic Energy

Fs=imv

1
Ek =-mv 2

where:
m = mass of body
v = velocity of the body

"If there is no net external force acting


upon a system of bodies, the momentum
of the system does not change."

\11

,,

;~~

Impulse is the product of the force and the


time during which it acts. Impulse is equal
to the change in momentum.
Impulse= FAt.
Impulse= p 2

P1

Impulse= mvrmat -mvinitiat

What is Power?

'

EP =Wh

The law of conservation of momentum is


stated as follows:

What is Impulse?

What are Two Types of-Energy? .

Potential energy is also known as the


energy of position or configuration or
gravitational energy. This type of energy is
decreases as the elevation of the body
decreases and increases as the elevation
of the body increases. Normally, the lost in
potential energy is converted into heat or
kinetic energy.

What is the Law of Conservation of


Momentum?

Potential Energy = Kinetic Energy

Energy is classified either a potential


energy or a kinetic energy.
where: G = gravitational constant
G = 6.673 x 1o 11 N. m 2fkg 2
G = 6.673 X 1oB cm 3 /~. s 2
G = 3.436 X 10'8 lbf-ft /slug 2
G = 3.320 x 10'11 lbf-ft2/lbm 2
G = 3.436 X 1oB ft 4/lbf-sec4

where:
m = mass of the body
v = velocity of the body

Power is the time rate of doing work or the


amount of work done per unit time. Power
is a scalar quantity.

P=Y!_

The typical units for power are watts, ftlbf/sec and horsepower.

1 watt = 1 joule per second


1 hp = 746 watts

where:
F =force
At = change in time
P2 = final momentum
P1 = initial momentum

What are the Two Types of Collisions?


Elastic collision is a collision of two
bodies in which kinetic energy as well as
momentum is conserved.
Inelastic collision is a collision of two
bodies in which only the momentum is
conserve but not the kinetic energy.

What is the Coefficient of Restitution?


What is Mom~ntum?
Momentum is the product of the mass and
velocity of a body. Momentum is a vector
quantity.

c~

nw

Coefficient of restitution is the negative


ratio of the relative velocity after collision
to the relative velocity before collision.

e'= __vz,..- '~~2s _ ~ -v2v1A-

'~~1e

'~~1a /vtA

446 100 1 Solved Problems in Engineering Mathematics (2"d Edition)


If e = 1, the collision is perfectly elastic
while if e = 0, the collision is completely
Inelastic.

by Tiong & Rojas

Day 18- Physics 447

What are the Properties of Fluids?

Density is the mass per unit volume. This


is expressed mathematically as

Fluid
displaced

What are the Gas Laws?

IIIII
(
1111001111

1~11.!11

Boyle's Liiw:
"If the temperature remains constant, the
product of the pressure and volume is
constant." This was named after l~ish
physicist Robert Boyle (1627- H)91):

:11\l'~

'i~~i

Weight density is the weight per unit


volume. The relationship between density
and weight density is expressed in the
following equation.

.PV.:;:k

l.,,,

,,,

P=v

D= W
Charles' Laws:
"If the volume of a confined gas is
constant, the pressure is directly
proportional to the absolute temperature."

.Pl ~fz.

T1 -:-r2
"If the pressure of a confined gas is
constant, the volume is directly
propl)rtional to the absolute temperature."

D=pg

<i\sp~cific
9f'aVi~
, '.I
.

. density

BF=pV.

where:

.,

"' . density ofwater 1

The values for the density of water are as


follows:

v: ~.

BF

Specific gravity is the density of the


substance relative to that ofwater. This is
also known as relative density. Water is
considered the standard substance which
has a maximum density at 4c.

I
IJ;

II

General Gas Law:


The combination of the Boyle's Law and
the Charles' Uiws may be regarded as the
general gas law. This is expressed
mathematically as

PV::::nRT

where:

n=
n

or

Pi\'1 P.Na

Tt

= Tt

BF"'W

62.4 pounds I ft3


1000 kg I m 3
9.81 kNim 3
9810 Nlrn 3
1 gram/cc

Note: The buoyant force is always acting


at the centroid (center of gravity) of
the submerged volume.

The specific gravity of water at densed


condition (4"C) is exactly 1.0

Proceed to .the next page for your18th


test. Detach and use the answer sheet
provided at the last part of this book. Use
pencil number 2 in shading your answer.

What is the Archimedes Principle?

GOOD LUCK I

The Archimedes' Principle is stated as


follows:

mass .of the gas


atomic mass of the gas
R = universal gas constant

"When a body is immersed (partially or


wholly) in a fluid, it is subjected to an
upward force (buoyant force) which is
equal to the weight of the fluid displaced,~
This was discovered by Greek
mathematician and inventor, Archim~des
(287- 212 B.C.).

Also by equilibrium, the buoyant force is


equal to the weight of the body, thus

j,

.....!=~

1'1 T'z

Vs = volume submerged
p = density of the fluid

,
:s_!

it!

~:t,
"i.!

\!rtibia:
Did you know that. .. the most difficult
problem in mathematics is the "Fermat's
Last Theorem! The search for the proof of
this theorem begun right after Fermat's
death in 1665 and remained an unsolved
theorem through centuries of hopeless
search. A British-born professor in
mathematics at Princeton University,
Andrew Wiles brought an end to the
search of the proof in 1995, i.e. 330 years
later. Because of this, Fermat's Last
Theorem was regarded as the Mount
Everest of Mathematics.

~uote:
"With me everything turns into
mathematics."
- Rene Descartes

Day 18- Physics 449.

II

'\J

Topics

lii!IMI

[]

~llllil~l

LOlli

Physics
Vector and Scalar Quantities
Classifications of Vectors
Speed and Velocity
Distance and Displacement
Acceleration
Laws of Motion
Forces
Law of Universal Gravitation
Work, Energy and Power
Law of Conservation of Energy
Momentum and Impulse
Law of Conservation of Momentum
Gas Law
Properties of Fluids
Archimedes Principle

Mon

lli>'i~lijl

.(

~IIIWi~ll

Tue

0 0
[Q] 0
0 0

'""

Theory

Problems

Wed
Thu

Solutions

Fri

[J ~
Notes

Sat

761: ME Board October 1994


The weight of a mass of 10 kg at a location

7&3: ME Board October 1994,


MEBoardApril1998

where the acceleration of gravity is 9.77


m/s 2 is

The mass of air in a room 3 m x 5 m x 20


m is known to be 350 kg. Find its density.

A.
B.
C.
D.

A.
B.
C.
D.

79.7 N
77.9 N
97."7 N
9T7 N

7&:&: ME Board AprU :1998


How much does a 30 Ibm weigh on the
moon? Gravitational accelerati.on in the
2
moon is 5.47 ft/s 2 and in earth is 32.2 ftls .

A.
B.

c.
D.

2.0 lbf
3.21bf
3.41bf
5.0961bf

;~

7651 ME Board October 1997,


ME Board April t998
100 g of water is mixed to 150 g of alcohol

C.
D.

(p = 790 kgfm 3 ). Calculate the specific


vol.ume of the solution, assuming that it is
mixed completely.

7701 EE Board October 1996


A 10 g block slides with a velocity of 20

3
0.82 cm /g
0.88 cm 3/g
3
063 cm /g
3
1.20 cm /g

A.
B.
C.
D.

A.

100 g of water is mixed to 150 g of alcohol


(p 790 kg/m 3 ). Calculate the specific
gravity of the total mixture.

B.
C.
D.

A.

'771t ME Board April 1.997


A 60 ton rail car moving at 1 mile/hr is

B.

c.
D.

1.862
0.963
0.286
0.862

767a ME Board ApriiJ.996


The specific gravity of mercury relative to
water is -13.55. What is the specific weight
of mercu~y? The specific weight of water is
62.4 lbf/ft .
.

'

I
~

.:
,'.r

1.167
1.176
1.617
1.716

A.

B.
C.
D.

102.3 kN/m
3
132.9 kN/m
3
150.9 kN/m
3
82.2 kN/m

7&81 ECE Board November 1998

A 16 gram mass is moving at 30 em/sec


while a 4 gram mass is moving in an
opposite direction at 50 em/sec. They
collide head on and stick together. Their
velocity after collision is

A.
B.
C.
D.

0.14 m/s
0.21 m/s
0.07 m/s
0.28 m/s

D.

coupled to a second stationary rail car. If


the velocity of the two cars after coupling
is lftls {in the original direction of motion)
and the coupling is completed in 0.5
second, what is the average impulsive
force on the 60 ton rail car?

A.
B.
C.
D.

50!bf
3500 !bf
1200 lbf

60 lbf

7721 ME Board April :1:997


What momentum does a 40 Ibm projectile
posses if the projectile is moving at 420
mph?

A.
B.
C.
D.

24,640 lbf-sec
16,860 lbf-sec
765 lbf-sec
523.6 lbf-sec

7691 ME Board October 1996

.~~

A 60 ton rail car moving at 1 mile/hr is


instantaneously coupled to a stationary 40
ton rail car. What is the speed of the
coupled cars?

A.
B.
C.

I,,

I\

the density of the block?

988 kg/m 3
1255 kg/m 3
3
2550 kg/m
BOO kg/m 3

15 cm/s
10 cm/s
25 cm/s
5cm/s

773; ME Board Aprii199S,


ME Board April1998
A 10-kg block is raised vertically 3 meters. :
What is the change in potential energy?
Answer in Sl units closest to:

7&4: ME Board April1996


An iron block weighs 5 N and has a
volume of 200 cubic centimeters. What is

A.
B.
C.

r;m/s on a smooth level surface and makes


a collision with a 30 g block moving in the
opposite direction with a velocity of 10
cm/s. If the collision is perfectly elastic,
what is the velocity of the 30 g block after
the collision?

766: ME Board October 1997

.. I

0.6 mph
0.4 mph

ll

0 88 mph
1 mph

D.

320 J
350 kg-m 2/s2
294 J
350 N~m

4\50 l 00 l Solved Problems in Engineering Mathematics (2"d Edition) by Tiong & Rojas

1111~11

~~ll!illlll

lif;illlll
(

11~11~11

l~'""

'

,111

"174: ECE Board AptriiD 1.997

779: ME Board October l'.991J

An aircraft engine develops a forward


thrust of i 5,000 N. If the gross mass of the
aircraft is 100 tons, what horsepower does
the engine develop if it is flying at 1000
l\ph?

What is the water pressure if manometer is


0.6 m Hg? Mercury is 13.6 times heavier
than water.

A.
B.

c.
D.

150,000
5585
5400
3108

'il

A.
B.
C.
D.

B.

27.4 kPa
47.2 kPa
79.97 kPa
72.4 kPa

3.81 kCal
32 BTU
2300 Cal
9.41 kJ

A.

'1.63 X 10 hp
3.16 X 107 hp
3.61 X 107 hp
1.36x107 hp

A force of 200 lbf acts on a block at an


angle of 28 with respect to horizontal. The
block is pushed 2 feet horizontally. What is
the work done by this force?

C.
D.

320
480
540
215

~:~

'~.

C.
D.

78:1: EE. Board October 1994


Assuming the barometer reads 760 mm
Hg, what is the absolute pressure for 900
mm Hg gauge?

A.

B.
C.
D.

74.213kPa
221.24 kPa
48 kPa
358 kPa

J
J
J
J

7"1&1z What average force is necessary to


stop a .45 caliber bullet of mass of 15
grams and speed of 300 m/s as it
penetrates a block to a distance of 5 em?

78:&: CE Board May 1.994


A barometer reads 760 mm Hg and a
pressure gage attached to a tank reads
850 cmof oil (sp.gr. 0.80). What is the
absolute pressure in the tank in kPa?

A
B.
C.
D.

168.1 kPa
186.1 kPa
118.6kPa
161.8kPa

783: EE Board April 199&


A sealed tank contains oxygen at 27C at
a pressure of 2 atm. If the temperature
increases to 100C, what will be the
pressure inside the tank?

A.
B.

12.5 kN
13.0 kN

C.

13.5 kN

A.

D.

12.0 kN

B.

4.92 atm
4.29 atm

2.49 atm
9.24 atm

784: ME Board April :1996


A volume of 400 cc of air is measured at a
pressure of 740 mm Hg abs and a
temperature of 18C. What will be the
volume at 760 mm Hg and 0C?

1\.

11

c.
IJ

I
~

376
326
356
366

cc
cc
cc
cc

34C
45C
60C
90C

78&: ME Board April1998


A transportation company specializes in
tile shipment of pressurized gaseous
rnaterials. An order is received for 100
l1ters of a particular gas at STP (32oF and
I atm). What minimum volume tank is
ru:cessary to transport the gas at 80F and
.r maximum pressure of 8 atm?

\~

~
"'i

788: ME Board Octol>er 1.997

A 10 Ibm object is acted upon by a 4.4 lbf


force. What acceleration in ft!s 2 does the
object possess?

A.
B.

C.
D.

A.
B.
C.
D.

785: EE Board October :1995


The pressure of the nitrogen gas
thermometer is 76 em at 0C. What is the
temperature of a liquid in which the bulb of
the thermometer is immersed when the
pressure is seen to be 87.7 em?

1,934.5 m.
3,508.4 m.
4,168:2 m.
2,647.7 m.

"Jt'li/1 ME Board Apnrifl lt998

A.
B.

'l'li\'

A mercury barometer at the base of Mt.


Makiling reads 654 mm and at the same
time another barometer at the top of the
mountain reads 480 mm. Assuming
specific weight of air to be constant at 12
3
N/m , what is the approximate height of
Mt. Makiling?

B.

C.
D.

C.
D.

780: EE Board October 199&

7'1&: ME Board October 1997 .


A rocket is moving through a vacuum. It
changes its velocity from 9020 fUsee to
5100 ft!sec in 48 seconds. How much
power is required to accomplished this if
the rocket's mass is 13,000 slugs?

A.

Day 18- Physics 451

-'{'

'1"1: EE B~J;ud October :1994


if a i 0 kg piece of copper falls 100 m, how
much heat might be produced?
A.
B.
C.
D.

,\\)\

A.
ll

<:
ll.

16 liters
141iters
10 liters
12 liters

'187x EE Board April1996


!\ 20 liter sample of gas exerts a pressure
',f 1 atm at 25C. If it is expanded into a 40
111"' vessel that is held at 100C, what will
1" rts final pressure?
1\

II

ll

0.50 atm
1 0 atrn
o :\15 atm
0 ti:~ atrn

12.4
10.0
14.2
13.0

789: A 50 kN truck traveling with a speed


of 50 kph hits a lamp post and is brought
to rest in 0.1 s. What is the average force
of the truck?

A.
B.
C.

D.

-408
-508
-608
-708

kN
kN
kN
kN

790: A tennis ball moving horizontally to


the left at 40 m/s hits a racket and
rebounds horizontally to the right at 30
m/s. If the mass of the ball is 100 grams,
find the impulse of the force (in kg-m/s)
exerted on the ball by the racket.

A
B.
C.

1
-1
7

D.

12

7911 Two steel balls of masses 500 kg


and 50 kg, respectively are placed with
their centers 0.5 m apart. The two balls
attract with a force of

. A.
B.
C.
D.

6.67 X 10"10 N
6.6'7 X 10"7 N
6.67 X 10-6 N
6.67 X 10"3 N

19:&: EE Board October 1.995


A 50 g mass hangs at the end of the
spring. When 20 grams more are added to
the end of the spring, it stretches 7 em
more. Find the spring constant.
A.

2.8

B.

2.9

C.

4.:3

D.

2.5

452 1001 Solved Problems in Engineering Mathematics (2nd Edition) by Tiong & Rojas ,
!l

793: EE Board April1996

A.

Determine the submerged depth of a cube


of steel 0.3 m on each side floating in
mercury. The SJJecific gravities of steel and
mercury are 7.8 and 13.6 respectively.

C.
D.

B.

3min.
1 min.
4min.
2min.

::1

',,

~t: I

798: ME Board April199&


A.
101!11
[

B.
C.

IIMilliil~ll

D.

~~~~Iii

7941 EE Board October 1995

L.,,l,ll

'

,1111

A.

A block of wood floats in water with 5 em


projecting above the water surface. When
placed in glycerine of specific gravity of
1.35, the block projects 7.5 em above the
liquid. Determine its specific gravity.
A.

B.
C.
D.

0.514
0.704
0.836
0.658

7951 EE Board October 1996


A solid cube material is 0."75 em on each
3
side. If it floats in oil of density 800 kg/m
with one-third ofthe block out of the oil.
What is the density of the material of the
cube?
A.
B.
C.
D.

533 kg/m
3
523 kg/m
3
513 kg/m
3
543 kg/m

79&1 CE Board November 1993


A hollow cyJinder 1 m in diameter and 2. m
high weighs 2825 N. How many kN of lead
weighing 110 kN/m 3 must be fastened to
the outside bottom of the cylinder to make
it float with 1.5 m submerged in water?

A.
B.

C.
D.

0
0

Mon

'~1~1111

Topi4:S

What is the power required to transfer


97,000 couiombs of charge through a
potential rise of 50 volts in one hour?

0.155 m.
0.165 m.
0.134 m.
0.172m.

8.5 kN
6.5 kN
10.5 kN
9.5 kN

7971 ME Board October 1995t


ME Board Oetober 1996
How long must a current of 5 amperes
pass through a 10-ohm resistor until a
charge of 1200 coulombs passes through?

C.

0.5kW
1.3kW
0.9kW

D.

2.8 kW

B.

Tue

.,:

799: EE Board October 199&

How much oil at 200C must be added to ~


50 grams of the same oil at 20C to heat it
to 70C?
A.
B.
C.
D.

B.
C.
D.

15.87c
10.3oc
8.65C
23.5TC

Wed

0
0

Solutions

Thu

Fri

~
Sat

soo: EE Board October 19'96

A.

Theory

Problems

12.39 grams
29.12grams
19.23 grams
23.91 grams

The temperature of three different liquids


are maintained at 15C, 2oc and 25c
respectively. When equal masses of the
,[1.
first two liquids are mixed, the final
r: 1
temperature is 18c and when equal
.
masses of the last two are mixed, the final ':
temperature is 24C. What temperature
will be achieved by mixing equal masses
of the first and the last liquid?

Notes

RATING

ANSWER KEY

761.
762. D
763.A
764.C
765. D
766. D
767. B
768.A
769.C
770. D

771. B 781. B
772. c
782. A
773.
783. c
774. B 784. D
775.C 785.A
776. D 786: B
777. B 787. D
778. c
788.
789. D
779. c
780. A .,790. C

Physics
Vector and Scalar Quantities
Classifications of Vectors
Speed and Velocity
Distance and Displacement
Acceleration
Laws of Motion
Forces
Law of Universal Gravitation
Work, Energy and Power
Law of Conservation of Energy
Momentum and Impulse
Law of Conservation of Momentum
Gas Law
Properties of Fluids
Archimedes Principle

791. c
792.A
793. D
794. D
795. A.
796.A
797.C
798. B
799:C
800.D

c:J
c:J

34-40 Topnotcher
26-33 Passer

c:J 20-25

Conditional

0-19 Failed
If FAILED, repeat the test.

454 1001 Solved Problenis in Engineering Mathematics (2nd Edition) by Tiong & Rojas

"

rm
[

1111~~"~1

ljlil(l'~*ijl

li;li~llll

"

IIDal

W=mg

mt = m1 + mz

W = (10 kg)(9.77 m/s 2 )

mt = 100 + 150

W= 97.7 N

mt =250

lr.J.i I

Day 18- Physics 455


Initial momentum = Final momentum

Note: Specific weight of water is 62.4


3
lbs/ft3 or 9.81 kN/m
(j)

Vt

30
32.2

(!)water

= mt.+ mz

Mass in earth = Mass in moon

Pt

P2

Note: Density of water (Pt) is 1000 kg/m


v
0.100 0.150
t = 1000 + 790

5.47

w = 5.096 lbf.

LOIIIIII

Vt = 2.8987 x 10" m x

C~m J
0

Vt = 289.87 cm 3

~IIIII
Volume of air is the same as the volume
of the room

v = 3(5)(20)

V = 300 rn 3
m

p=-

- 350 kg

P- 300 rn3
p

vt

289.87

m1

250

~.
I

..

...

..

''i~ '

mt = m1 + m2

m = 0.5096 kg.

1 = v2 ,_ v1,
Vt-V2

v1 -

= v2 ,_ v1,
20- (-10) = V2- (-v1 ')

,,,2

m
p=-

0.5096 kg
p=---0.0002 m 3
p = 2548 kg/m 3
Note: From the choices, the nearest
3
answer is 2550 kg/m

V = 0.14 m/s

Initial momentum= Final momentum

+ mN2 = (mt + m2)V

= 0.100 + 0.150

mN1

t
Vt

1000
= 2.8987

60(1) + 40(0) = (60 + 40)V

'790
X

10"4 m 3

p=-

0.250 kg
P- 2.8987x10c4 m3
p = 862.45 kg/m

-- -

v2

m1

m2
~----

____,_

V2'

~).Q't.t.r.I)Nl l\'l'!>l'l'!l~
I --II
I'

taeJ

IV

=m(V1 -

V1 ')

F(0.5) = _?0( 2 000) (1.4667 -1)

Vt

V1'

V =~X~X5280 ft
1
hr 3600 s
1mi
v1 = 1.4667 ft/s
F(t)

V = 0.6 mph
Note: Since the second car is stationary,
its velocity (Vz) is zero.

sp. gr. = 862.45


1000
sp. gr. 0.862

-1oo = -10(3o..:.v2 ')+30V2


v2' = 5 cm/s

P2

Pwater

-> Eq. 1

-100 = -300 + 10V2 '+ 30V2 '

mN1 + m2V2 = (mt + m2)

+!':lz..

sp. gr. = _e__

Substitute Eq. 2 in Eq. 1

V = 0.0002 m3

v2

V1 = 30- V2

16(0.3) + 4(-0.5) = (16 + 4)V

Note: Density of water (pt) is 1000 kg/m

5 = m(9.81)

. v2 -v1
e=-V1-V2

Initial momentum'= Final momentum

mt = 0.100 + 0.150

=10(-V1') + 30W

Note: For a perfectly elastic collision,


coefficient of cestitution (e) is equal
to 1.

v2
-+-----

Vt == Vt + V2

W=mg

(~)
100 em

132.9 kN/m 3

m1 +m2

Pt

COm=

-v

= 1.2 cm 3/g

Vt::: mt

V = 200 cm x

13.55 (9.81)

mt

mt = 0.250 kg.

= 1. 167 kg/m 3

10(20) + 30(-10)

- 100 =- iOV1' + 30V2' 7 Eq. 1

COm=

Vt

v=--=--

m1V1 + m2V2 = mtV1 + m2V2

sp.gr. ::. __
m_

Vt= Vt +V2

'l,~!~llill

'

32.2
F := 3478.509 lbf

Note: From the choices, the nearest


answer is 3500 lbf.

V = 420 mix~x5280 ft
hr
3600 s
imi
V=616ft/s
Momentum

=mV

I'

!lf..,

456 lOO 1 Solved Problems in Engineering Mathematics (2nd Edition) by Tiong & Rojas

Momentum= 40(616)
Momentum

=24640 lbm-ft/s

24640
.
Momentum=-- = 765 lbf-sec
32.2

'

1,,
l!r
Jl,

b.KE =

~m(V0 2 - V 2 )

l.ili~~:~l

PE =mgh

t.KE =

= 10(9.81)(3)
PE =294 J

~(13000)[(9020}

'11~1~1~11

t.KE

L""'l'

PE

~H1111

-(5100n

!t/

=3.597x1 011 lb-ft

F(s) =!mv2

.:~

1
~

p = t.KE

hr

)i

1hp
P = 7 .49x1 09 lb-ftls x 550 lb-ftls

=15000(277.778)

P = 4166670 watt
1hp
)
p = 4166670 watt ( 746 watts

p = 5585 hp

F=13.5 kN

=(Sp.gr.Hg )( (I)Waler )h

p = 13.6{9.8)(0.6)
P = 79.97kPa

Paouom =Prop + roAi,h

w =rcose(s)
w

Ass~:~ming

no losses, the total heat equals


the initial PE of the copper.
cal
4.2 J
Q = 2335.714 cal

x--

1 BTU
'
Q =2335 714 calx - -

252 cal
Q =9.268 BTU

roHghb = roHgh, + ro Airh

h = 1934.5m

x9.81 ft/s

p2

= 100 + 273
P2 = 2.49 atm

Ill

PtVt

P2V2

740(400)
760V2
18 + 273 = 0 + 273
V2 =365.38 cc

Note: From the choices, the nearest


answer is 366 cc.

m
+ palm

W = 479.55 N-m

I 'ub

pgngo

w~480J

I ',~>

13.6(9.81)(0.9) + 13.6(9.81)(0.76)

"""'

221.47 kPa

iiii'IW<'I I'>

/74 /4 kPa

PtV1 = P2V2

T2

Note: V1 =V2
78
87.7
0-273 =--,=;

T2 =306.95 K
T2 = 306.95-273

13.6(9810)(0.654) = 13.6(9810)(0.48) + 12h


1kg
1m
.
-W = 3.53.179 lb-ft X 2.202 lb x
3.281
ft

T2

27 - 273

(sp.gr.)Hg(I)Walerhb = ( Sp.gr.)Hg (I)Waterhl + roAirh

=353.1791b- ft

Ntltu I rom the choices. the nearest

Note: From the choices, the nearest


answer is 2300 cal

T1
Note: V, = V2

Pressure (P) =Specific weight (ro) x


Height (h)

W = 200cos28 (2)

let: Q =heat

P1V1 = P2 V2

~=--,=;

Let: h = height of Mt. Makiling

PE = 9810 J

=168.1 kPa

P = {wH9 }h

=10(9.81)(100)

0=9810 J

~(0.015)(300)2

Pabs

F=13500N

P=1.36x10 7 hp

PE ::::mgh
PE

F(0.05) =

'.

;I;,

11
P = 3.597x10
48
P = 7 .49x1 0 9 lb-ft/s

1 km

...

~.

at

= 1000 kmx~x!OOO m

P=FV

Work done by retarding force = initial


kinetic energy of the bullet

'

3600s
V = 277.778 m/s

= Pgage + Palm

Pabs = 0.8(9.81)(8.5)+13.6{9.81)(0.76)

"-+
s
2

Pabs

v-

~t

llll!(!:iljljil

F-

p = b.K__

101!111

Dayl8- Phyf:!iCs 457

T2 = 34C

P1Vt

P2V2

~=--,=;
1(100)
8V2
32+460 = 80+460.
V2 =13.7 liters

v2 "'141iters

------------------------------------~ !8- Physic~ 459

458 .1 001 Solved Problems in Engineering Mathematics (2"d Edition) by Tiong & Rojas

Ill
= P2V2

P1V1

T1

lllll!limll

111!1~111~1

T2

'll~li~~ll

L_IIIMI

~"""

4.4 =

= ks

50= ks

W=BF

F = k(s + 7)

~-

( sp.gr)wood (Ah)

50+ 20 = k(s + 7)

:r

Impulse = P2 - P1
Impulse= 3- ( -4)

Eq. 1

( sp.gr. )\11/ood Pwater VTotal =

'i

Impulse = change in momentum

70

Impulse= 7 kg-m/s

-~

50
S=
-

P2 = 3 kg-m/s

5 em

P2 = mVz
p2 = (0.1)(30)

(__!Q_}a
32.2

-7 Hooke's Law

I" = tension in the spring due to


the load attached to it
k = spring constant
s elongation of the spring due
to the load attached to it

where

p1 = (0.1)(-40)

Newton's second law of


motion

a= 14.2 fUs

P1 = -A kg-m/s

I!'P.9!tl
lliiia
~

~:~

P1 =mV1

P2 (40)
25 + 273 1()0 + 273
P2 = 0.63 atm

F = ma

II

Momentum = mass x velocity

___!t?_QL =
~1111~~~~

= ks + 7k

(sp.gr.)Wood

-) Eq. 2

Pwater VDisplaced

= A(h- 5)
h-5
=h-

--+ Eq. 1

Substitute Eq. 1 in Eq. 2:

(50)

7() cckj-- + 7k

\ k

m1

7 em

k=28

m2

i!ll

Glycerine
sp.gr. = 1.35

Using the relationship between impulse


and momentum:
Impulse = change in momentum
F6t

=mV1 - mV0

50000 N
m = ----=~9.81
m = 5096.84 kg

Using the formula for universal


gravitation:

W=BF

.F = Gm1m2.
sz

(sp.gr. )VII:Jod rwater VT

where : F = force of attraction in N.


m1 and m2 = respective masses
of two particles in
kg.
s = distance between the centers
of the two particles in m.
G = gravitational constant

G = 6.67 x 10'11 N-m


V. = 50 km
0

x -~x 1000 m
3600 s
km

hr
V0 = 13.89 m/s
Substituting:
F(0.1)

=5096.84(0 -13.89)

F = -707951 N
F = -707.951 kN

kg2

"i:'

= !.:_35(h- 7.5)

(0.5)

F = 6.67 x 10-6 N

W=BF

( sp.gr. )Steel Pwater \/Total

=(sp.gr. )Hg

Equations 1 and 2:
2

7 81(9 81)(0 3) = l3.6(9.81)(0.3) d

d=0.172m

Substituting:

(6.67x10- 11 )(500)(50)

(spgr.),i\I'A:d (Atl) = (1.35)A(h-7)

BF

2fv""

Pwater VD

i .ci I\ ::':.: base area


h :-: hf~ioht
the VJood

-~=-_!?_ = ~~~-~5(h - 7.5)

h
h
11- 5 = 1.35h -- '10.125
1'1 = 14.64 em
Substitute in

1:

14.64-5
sp.gr.vvoor! = --14'.64.
.
:!S.l.LJi.,!l/ood

cc

0.658

-~ Eq. 2

Day 18- Physics 461

_.60 100 1 Solved Problems in Engineering. Mathematics (2nd Edition) by Tiong & Rojas

01111~1

illll~lll~il

BF2 = Pwat.,;vLead
BF2 = 9.81Vtaad

wlead = 11 ovlead

BF1 + BF2 = Wcy,inder + Wtoad


vlead = 0.0772 fT13

11~:~11~11

Heat gained = heat loss


mc 2 (24- 20) = mc3 (25- 24)
c 2 = 0.25c 3 ~ Eq. 2

wlead = 110(0.0772)

L" "~"'

W=BF
Pcube Vrotal = POit Voisplaced

l".,

Pcube(x

3
)

800(~x ){x

2
)

Peube(0.0075) = 8oo( ~ }o.oo75)


3

Peube = 533.33 kg/m3

WLaad = 8.5 kN

Substitute Eq. 2 in Eq. 1:


Q =It

3c1 =2(0.25c3 )

1200 = 5t
t = 240 sec

c 1 =0.1667c3

1 min
t = 240 sec x 60 sec

Heat gained = heat loss


mc1(t -15) = mc3 (25- t)

t=4 min

We

0.1667c3 (t -15) = c 3 {25- t)

Note: ampere = coulomb/sec.

0.1667t- 2.5 =25 -t

Ill

t =23.57C

Q::lt
97000 = 1(3600)
I =26.944 A

BF1 = Pwater Vo;splaced

*
*

2
BF1 = Pwatar( )d Y
2

BF1 = 9.81( )<1) (1.5)

BF1 =11.56 kN

Heat gained = heat loss


mc1(18 -15) = mc 2 (20 -18)
3c1 = 2c 2 ~ Eq. 1

11.56 + 9.81Vlead = 3.825 + 11 ovlead


1!1:!~1~11

Heat gained hE!at loss


50c(70- 20) = mc(200- 70)
m = 19.23 grams

P=EI
p = 50(26.944)
p = 1347.2 watts

1 kW
p = 1347.2 watts x 1000 watts
P=1.3472 kW

Heat = mc(at)

where: m = mass
c = specific heat
at change in temperature

fl
484 lOOI Solved Py>blems in Engineering Mathematics (aM. Edition) by Tiong & Roj..-

~I

' \'
~~~<S..t:>tt!.,:~i.ot~.Jif"t: ... ~.t1f.~~Jll-.~"<--~1>'>:"->- !.,.. ., . . . ,. .... ~ o;~:J'"*'.,~"l$fo.it ''""" ,..<-+"<! ........... ,, ~-..o.~:j! ~-~""
'"""'; '!') "' ...

,.,..lt:>ll. . . ..,_.:~#."'-..:~ CO<"J-,., .. *!'""'"'"*.. ..,"""'l"'":...="l'

~.:.-!1:, ., .,.._,.,'-'. 't'~"'


-t

~- 1> t ~ -~- ~--

...

;1:

.'If ... ,.-~~. *... :$'+~~. ~.+r:\.'V. ~ '"<tt-'i. 't--1'

t;,;. 1""" .. ~

'~'-"'1''l> r l'~..,,~ "'""'"'~ ~f '1< ~.,

L,,,,~,"

~ ~ ~" ~

f!- 4< "\' ''\

~ .<>

~"

,. !I ,

!f 111-

,. .,,

,_.,.

'* ~ ... ,:#' t

/.o. to ~- 4".";::4'. >" PJ; #' i ;.

<!I~..,~~,~.""'.,.:..._ i

tf

<y .. "

"<1.

1>.

}II

~ ~

t -so:

~-"!'~

t ;,. .,

-t: 1>- ~

0"

'{

:""

'!- ...

o " '",

<>

;. ~

"<'.

"'~"'<I"'"',<

~ .<. ~

t-' ,;

<

-"

'

"' f

;<;

~ ~

#>>! '(.

Jl.4f.-f-

.o~" .......

'l

jf.$.

.-~_. . . . +..<

-~

" < ...: >I '

-~

<.

.;,. ;

;: ., il' "'

,.. .,

~ ~

.. '0

~ ~

~ 'J ~ i< .~ ~ ~

~ ~>

:..~<!

"- " , . , ........ ,.. ,,:,."" .. , . . . , , .

~-)f..

:t<

....

,< ,;. :

"<

~>- ~ .' ~ r .; ..! >~ ~ ~ *

...... 4":., t"' .J ........

'"'*""' ~ ~ ~

'I'" of< J>l"f<?

*"" ,~ .. -~ "" {

~ ~

'<- "" <Y P

v" ! ,. '~""' ~ ,oc'~. ,~

'>I

~ ~

t .,

-~

' '!" ., 4

~ <- 1

v, \ .. -(< "< ..<

;.. ~. ~ 6 .,

~ .., 4

"'/'i'-ot.'ili<t 4-'.!i- .;;1 ~ ... ~'t'"'"

.-c ;' 4 _,. :I,

'f. ., ,.

t:

"'

-~

Topics
-~

-s.

Mon

"A'- v .,.. '

'Yo~

oii #. u. ~t ~~~i

:;_.,~~,.,:,..i'-'ll"V>If.~...-:.~-;:*-~>(.,..,.<ir"'""'~l%>'...,~.>t<

>lt

:~if

io!

~ ,1' . 'f4%.'*:~. """*"f..., <t. 1-* ~ ~ ~

4 .....

.~

~ ~- j * ~ .... k

,...,. ~"'>~"'~~'Ill',.~ :i\c1 ,r '!I' 'i<:~~~'*_...,..,.<!< ..,

[]

,,

o.

.... ,. "'".:"' .~ ..';~. .,

.~

Tue

~-~

:s
4

jl:o , ,

~ i<.'or. 'fi ,._..........,_ ..~,.,.,...

~--

i-,,i.,-~ ... ,....,..,.,.,,.,.._.,>!!! !f'*=*_Wolt-'1'# *''11!""~~:~..,.,,,:~--?< ~~ ~~.~~

..

~~~ t: ~ .... <!<*~. ~ ;.. . . ~~~ ~~.:~.,.,:it

*11'.:, , ."~>."' ~.f"#'# ~~.,..,._!it . . :~:....,. ,. if t;"::f. ~"" >ll'11>.fl!. ~~-:"='if~ ... ~ :o!-.'1-i<'f'~- ~'11-t;j>'~ . . . ""',. '~';;'" ll' ~.!J."t ,.,..,"'"*'If-~'""~
11.<

iic.:>t

~!~!-. .!~-~.~;- .....:".'\!-~ ~ ._,.,ifl!./0 .. W' ~.:; ~- '*:t,li'loi 't<'i ~.fie-if*-~ t.,:..-..(t ~ i4 """""~ j> "l ... ~ ~=%~

**

ot

.~<'f,;. ... ,.;.,;':4.,: ,..,...

"!*-~. ~ .Jt~:~,t!~~~~:"t il"+'(l!t:ilt>ft...:f'+*"'~~>< 1<~ 1,~--*''" .!J'~;..>,S.,~.t... ~ih~'* ot .... *:'H ~::lol.4-,;i.~.:~,
<t.~.;jrt:4J.!'}~ -+:*'~:'t'~ ~..~$~..,,;.:. ".~.~,~:j, ~~* .<>.+ ~~(_>:-'!' ~*'*~ $<~ ~"t~"1'.'"' ~.ll'y'f'-~\0: ",."" v.".o.,.. ~.,..

. ...
~'*

.W:i!.:'-'""11-llt ..i<

... A\.

~ 4, 11:~ ~ l!t il,>'!< ::..k.4\''l!.~'.fl\o <i '!>*,!.:~\.flo ik'\1' ~ +.M, ',l>,~t'l!' "*~.-;

~-i~;C'ti."f:fl>l/.w ... ..::'!<: . ~ ,fc.<fo!~ * ,..-t- ~

t ,;,

i;.1,'.,:'

,;~.it4:> .<1!;:.0,;,.. ....*- k.4o .;:""''". ""~::t-'ii-P.. .. 4: :-:. ~ t ..l!'~'~.~

,'f< <!:'r,.

<e:t .~ ~

":~<',. ~~~~ < "''*, :t- if, ~ .:..ts '-."" " ~

.
1>.!("'!"........

11' ...

,.,.,.""'1'

t :.,

4 .. '" .,

..J,.. .., ~

....i,."' "'>f .

'

"'"*:*

...

4! ..

~ ~

'!-

:< ....... li< ..

"'"' '.,._

~ <.<

.,.. '!'

'

,:~-; ~ * ;." ..;.,~'<

,.,$<-~{.~.IIi ,AI~~ '8-~ .~ ill'~;.., J< f; ~ to' o -t ~ i

;,

~"' ill")<::;,'*..,, <I ~,."'}''*"it. i< ~.,i>. ~

~ : .~ ~~~ ::- ~
'

4:\:.

:rr.olo <4

~,,.., .t;'<f""':'.""'"'"~"': "~"". ,.-,.,~~.~ ".~ "f~,~)" ......\.,..., ..... 1-,..~.,

~ <4-~.~.i '~
#,

A'

,.,

'I

.. ~"':'"'"""

.1;

~.,..~'~~~~ ....;to,., "'"'~.r."t.*'i~ ... -1'*."!', :it 'l>~_,....,l~.,,. .'11..... "'+;..;,_,., ~-'. 4 ~'> .,,,. ~

n.p '#

1..- :..;.~ .;.:~~"\ ......IIO;fl.i. ~~,,... Y:~,.i'~.:.. .... _~<~.A-~~,1' ~ ~ ,_ >Ji,~, ..o: si.,'#. .... ~u:?><"'.:f:":o$',.. "~ ~ ~

...

...,

* *',... .~ *

.' .s. ~ i

Proi}lr,ms

Thu

[_]

~-=]

Solutions

Fri

LJ

[=]

Notes

~ ~ * ~. ,.,~,

Sal.

#;

, .<

.~

"')'"'"

,,

~
~

.., .,.

....

'o: .. . .,

~ .">
.

,.,-~

..

'"'."~'"

+<~ ,... ~ .. '!".""'"' ": *.1'>'~, ... !

'4~ ~.t ~ ~-~."': _._ =<~ .~ .i ~ -;-.~ ~ ~

, ....

*."' :.: .. .:c: .. ~: ~ "- '* ": ...... -"'"' ~ -. .,. '*""

~!t'-.*<ii,.\!f! =.;,.t .;..,,. ,-;w:i>.lo ~, .,:*"~:~!" *"*k~':(-;f,ii,.jj~#+f'* "':.~,,.,;~i.*:rilt':,.-~ ~ !.' *.*t ll.:or. 1!1:,.,_ ~,. *~ ~i',., ..\ ')<;~ ..."~
'!<~ !ti "*'. lt}~>'*!~:t (<i<t-f. .. ~ ~. ~.~ &:~.~~ ....~:$' ~.~.~t..t:,.t --6'. i'!ji

Wed

"~ ~ "~"' -~>.+ .... .~

<. :'

* "''* >~< ~ ~

,.;., '!:~~. ~ ,.,.~ "":t ~t;'l' p,. ~"" "'~-,. ~ '*' l'f'"''-"' .. ~ ~ . ~ 1.<: ~.~.'-', *'~.~~i ~-"f.,.~.; .If.~"""...~ ..:.~ 4o.fl '11

'<

1>11

Theory

[J []

~: tf~ ~-~.~ ... t~)l'ti '*"'!i!',;!l;,..,.,11'.4.'~\f'-4-~ <'f~-~~:*:}'1-.'*'"'i'k-~lil "'~.:41-''~ ..... 'i * ~'? !j.-;,: ~.;'~~...'* 'l ~ ~i. .~ 4<
'\'!

;j,~ ':ri:.li: ~: )': .j:~.'* ~ :a. ~-'II ,.;,...:'~ 4 ii. ~ i*..~

~hat .!iL~!!_eerin.!l Mec~anics?

C.

Mechanics is the branch of physics that


considers the action of forces on bodies or
fluids that are both at rest and in motion.

What .11re the

Engineering Mechanics is the branch of


that applies the principles of
mechanics to any design that must take
111to account the effect of forces.

~ngineering

4: ........... "'!;.'l",if.*'~'*'*''*.~.i'*

il:f.,.~<'\' ~'*'*:~If illi1*;. ... ,.."~~' .,.:.~ "'>";ll''i~

.r:); ,;\'~,"'
.4:

~ ~-\1. ~ ;!< ,. ... '* ~.., ~ :.-.~


i)-

.,,,,:..;,.*'""""" ~

'<+<- ,,

.~> ."~',"'

("''"'"'

,.~

t~
~.,;vo. ~ ~ ~t ,."" ~-"'~ .. . , lfl:1t 1

~ .. ~-.;.;,

,, .;

"<;<;i<

,*."' *J:~ .,..,.......~~:- ._. >ll.:i.~""'r.~ ~ i ~; ~ .. -.~.

~ ~ :,.,;:~....:~:.. ~'"'"*i>!... n.,:'*

1
"'"'"'""""?<,i:>;V.'>''>V i>

~-., ~.:+ ..

; . ,. '""""f.<~- "'~;-.';"-<-:..o..o ,._,.,.,.,., .6oi> "'"' ~ ~-"'"" ~ ~"' ' ,:, .. " ~ ... -~ "''"" "'~ "~*-"'.,."' ,.,.,~"' "'"'-"':"-~ ';,"".~'... ~~a.,~.,
..t "'

"'~

' l': "'

... .,

..; .f, "

i .. ""' ..

~""

"'

.~

"' v,

~ ~ ~ ~

,.... "

~ :' ~

1< ... _,. .... .,.

""<I< "" ~!''I

' ' ,

"' '" ..,

~ "' .~....... <I< ,,

I he branches of mechanics are the


lllnwing:

condition, the forces or vectors are


transformed into a force polygon. For
equilibrium, the force polygon must
close.

F1
I\

,,

Statics. It deals with bodies in the


;;tate of rest.

F2
Dy11arnir.s or Kinetics. It deals with
l11><ilc'; 111

"

fo!

G1aphical condition: Under this

4:

,<;;, !0\ ;,""

.'

are the Branches of Mechanics?

~'*if~~:,., 'f'" *-j>'*"'"' ... "'.";. ... ..:t'lt.t..-t .... :o;*''*"'!t~tift'll:;t-it

'lloi..:"'~ J. ~ ~.!14v;....,;t10<e).f!>.11~-:t:...ji,.. ~ <11'':1,"" ~ lri'~'t,!l,IJ'~'II!~':'\> *'4'.:..... ~-.,.:~-(~~:*.;:~"' :;.. .-.~ .. >~,'Wit:':it-:'f.t-:.

CQ~s

A body is in equilibrium if it satisfies the


following conditions.

1.
Wh_~t

lio'~~.)l..;t~~* ............... .<t

Kinematics. It refers to the study of


motion without reference to the forces
which caus1~s the motion,

~guilibr_!!!m1

'.'~~~"*'"' ~-.'41t<,<lll~,.. .,~jt:.>l'.): j!.'l;'.,t.li'.+~~ ... ,fl.'.t ~~ ~:;~.# *'~ "'.""""~"'~~:.. ?~,.:,:,.it ~~ . ~lto;ft-0!' -;,.,i-tf: *~~.*~?.1-.~ ""~if.
; "';JI>

Engineering Mechanics
Branches of Mechanic..s
Conditions for Equilibrium
Friction
Parabolic Cable
Catenary
Centrolda! Moment of Inertia
Moment of Inertia with respect to
the base
Transfer Formula
Mass Moment of Inertia

--.....1

.4<,..,. , ~~}! t.~jliti'.~.*.:J>'l!>.~.-. Jff><-"':t~ ~<~. "': "'<11 _.,,.~;,.""$., ..:,.'-~>::,~ 0..'1!' '*' ""~ ~.'~)<'*.><-* i;t,,:., #:.~-* >t-.,~,. ot-~ ><::

~ffi,'-~<j'> ~<t:t'..,. <h*-*>,-1.} '* ~ ~.'41''f-'-"""''4rlii.

\'"""

"'

+ "'~ .,_,.,._., .. _~_.., ~~ _.-~

..,.'$' .... ., .><>.; .t,il- ~'!')> ... """' ~,ij.'fk


~ *)P>

........-......, -"-"

.:-

rnotion under the action of

; '( ... '

(!11<'1 ;'
?,. ........

~.,.~ .... ..r.~<#oif"'''~.~.,q..~ ~:.;''~"'.l+ '~--~

, .. ,.

, I

< " ' .. ~ ,'

f''' _,.

\ F.l

466 1001 S0lved Problems in Engineering Mathematics (2nd Edition) by Tiong & Rojas

N~

F2

Force Polygon

2.

where:

Directional condition: If three or


more non-parallel fqrces or vectors
are in equilibrium, then they must be
concurrent
F1

c) Length of parabolic cable, S:

F=~N
tan~=

Day 19- Engineering Mec;:hanics (Statics) 467

S =L + 8d2 32d4
3L - 5L3

f.1

= length of parabolic cable


=sag
= span or distance between
supports
w = unit weight or load per unit
length
T = maximum tension (usually at
the support)
H = minimum tension (usually at
the lowest point of the cable)

where: S
d
L

A cable is analyzed as a parabolic cable


when the loading is uniformly distributed
horizontally throughout the cable.

111 1 ~~~

F3

3.

Analytical condition: If forces or


vectors are in equilibrium, then 'it must
satisfy the three static equations;
namely

}:F, =0

- i

LFy=O

LMx =0

0=0

JTIITffiUTl.!!J

S =length
L =span

Analyzing half of the

A cable is analyzed as a catenary when


the loading is distributed along and
throughout the cable. The word "catenary"
comes from the latin word which means
"chain". It is a graph of the equation y::::
cosh x.

What is Friction?
!......

Friction is defined as the limited amount


of resistance to sliding between the
surfaces of two bodies in contact

y
ltU'Jio

U2

---

a) Tension at the lowest point, H:

Span, L
L=2x

X= c.ln

E.

A.

H= (j)t:

x-axis

Distance from the x-axis to a point on


the catenary, y:

Y2 = 8 2 +c2

ad

S =L + 8d2 32d4
3L - 5L3

If the cable has uneven supports, the


formulas to be used are the same, only
that all unequal dimensions will now
contain subscripts 1 and 2. For example,
the distance from the origin to the left
support is now taken as x1 rather than x,
and xz for its distance to the right support,
and so on.

What are the Centroidal Moment of


Inertia of Common Figures?
Moment of inertia is also called the
second moment of area.
Centroidal moment of inertia is the
moment of inertia of the figure with respect
to an axis passing through the centroid.

A.
II

Rectangle

Maximum Tension, T:
Like a parabolic cable, the maximum
tension in a catenary occurs at the
support

Centroid

hll

01

T=-my

Length, S

b) Tension at the support, T:

S+ Y

length= 2S

y-axis

D.

What is a Catenary?

cabl~;:

_..,.._

Minimum Tension, H:
Like a parabolic cable the minimum
tension in a catenary occurs at the
lowest point
H=wc

N = normal force
F = frictional force
R = resultant force
ll = coefficient of friction
cp = angle of friction

What is a Parabolic Cable?


Fz

C.

~I

1
~

468. 100l'Solved Problems in Engineering Mathematics (2nd Edition) by Tiong & Rojas

What are the Moment of Inertia of


Common Figures with respect to the
Base?

. b~h .
...........
. Y -.1'2.

i)h3

ilx=f2

'~

Day 19- Engineering Mechanics (Statics) 469

~.

where: m = mass of sphere


r = radius of sphere

i
~

<muote:
"How happy the lot of the mathematician.
He is judged solely by his peers, and the
standard is so high that no colleague or
rival can ever win a reputation he does not
deserve."
-W. H. Auden

lit

B.

Spherical shell

A.
B.
llii!IWI
[

Triangle

1~??:~

*I Will~~~
i,llllllllil~l

l~!~lli~l~i~l

..

L." ~'" '

\,,,.,.,

'~ ..

C.

Rectangle

' JD.
-,.

.-b-aXIS
.

....._:

~--.,

bh3

t=bh$

B.

Circle

3 .

--~----~---1---- X

,:':"

t
).

TOaA
.

36

1=_g_mr 2

.I

4 64

lte

,rab3

'4

a.,,:,

mrtbia:

= distance from the centroid

Did you know that ... the integral sign

to the axis
area of the figure

I' _'lrt!~b

r--;r

where: a = length of semi-major axis


b = length of semi-minor axis

A.

and elongated S denoting sum (Latin for


"summa"), was introduced by Gottfried
Wilhelm Leibniz, who named integral
calculus "calculus summatorious" !
... the definite integral which is defined as
the integral between two values of an
independent variables is also known as
"Riemann Integral" after the German
mathematician Georg Friedrich Bernhard
Riemann (1826- 1866)!

What are the M~ss Moment of Inertia of


Common Solids?

,1

GOOD LUCK I

L=l8 +Ad2

>

Proceed to the next page for your 19th


test. Detach and use the answer sheet
provided at the last part of this book. Use
pencil number 2 in shading your answer.

For composite figures and for axis not at


the centroid nor at the base, moment of
inertia may be calculated using the
transfer formula, which is as follows;

....

~
1::: 1 mr

where: m = mass of sphere


r = radius of sphere

1tr

where: d

rt(O.
............
'' ,::/1-.}~A

1rt)"
Jx.=-.::~

Ellipse

Cylinder

b-axis

where: r = radius of circle


D = diameter of circle

D.

C.

... ....

'X "''')'

where: m = mass of sptrere


r = radius of sphere

Sphere

..

,l:~:~~yf

Day 19- Engineering Mechanics (Statics) 471

:i

A.

~~I

B.
C.
D.

Engineering Mechanics
Branches of Mechanics
Conditions for Equilibrium
Friction
Parabolic Cable
Catenary
Centroidal Moment of Inertia
Moment of Inertia with respect to
the base
Transfer Formula
Mass Moment of Inertia

~~~~II

Mon

tlln~~~~~

,.,qi<~il

14!~1~~~~

Tue

D D

L.""""''

l:. ,

Theory

Wed

D
Thu

o o

Problems

A.
B.

c.
D.

104.48
105.58
106.69
107.96

A.

B.
C.
D.

A.

B.
C.

c.
D.

so:sz What is the magnitude of the


resultant force of the two forces 200 N at
20 and 400 N at 144"?

332.5
323.5
313.5
233.5

N
N
N
N

8041 ECE Board November :1998


A load of 100 lb is hung from the middle of
a rope, which is stretched between two
rigid walls 30 ft. apart Due to the load, the
rope sags 4 feet in the middle. Determine
the tension in the rope.
A.

B.

D.

sos:

1651bs
1731bs
1941bs
1491bs

A boat moving at 12 kph is crossing


a river 500 m wide in which a current is
flowing at 4 kph. In what direction should
the boat head ifit is to reach a point on tho
other side of the river directly opposite its
starting point?

D.

121 kN
265 kN
211kN
450 kN

612.38 N
628.38 N
648.16N
654.12 N

809: EE Board October 1:99:!


A man can exert a maximum pull of 1000
N but wishes to lift a new stone door for
his cave weighing 20,000 N. If he uses a
lever, how much closer must the fulcrum
be to the stone than to his hand?
10
20
10
20

times
times
times
times

nearer
farther
farther
nearer

m
m
m

length. It carries a uniformly distributed


load including its own weight of 200 N/m
and a concentrated load of 100 N, 2
meters from the left end. Find the
reactions if reaction A is at the left end and
reaction B at the right end.

B.
C.
D.

RA = 81 0 N &
RA = 700 N &
RA =810 N &
RA = 700 N &

on: A beam

Rs
Rs
Rs
Rs

= 700
= 800
= 780
= 810

N
N

N
N

of span "x" meters with


uniform loading of "w" kilograms per meter
is supported at one end (A) and a distance
of 2m from the other end (B). Find the
reaction at support A.

A.

wx 2
2(x-2) kg

B.

wx(x-4)
2(x-2) kg

C.

wx(x-2)
2(x-2) kg

D.

A.
B.
C.
D.

2.48
3.24
3.43
4.21

8U: EE Board October 1:991


A simply supported beam is 5 meters in

A.

808: CE Board November 1:994


A 200 kg crate impends to slide down a
ramp inclined at an angle of 19.29 with
the horizontaL What is the frictional
resistance?

A.
B.
C.
D.

810: A beam rests on a fuicrum, 1.2 m


from one end. A weight of 350 kg is
suspended from this end causing the
beam to balance. If the weight of 350 kg is
suspended on the opposite end of the
beam, it is necessary to suspend a 1000
kg weight on the first end in order to effect
an even balance. Find the length of the
beam.

A.
B.
C.

88.67 kg
100 kg
70.71 kg
50 kg

807: ECE Board November 1998


A block weighing 500 kN rest on a ramp
inclined at 25 with the horizontaL The
force tending to move the block down the
ramp is _ _ _ .

D.

80:&1 ME Board October 1:996


Two forces of 20 units and 30 units act at
right a!)gle. What is the magnitude of the
resultant force?
36
42
40
44

C.

Sat

80J:r Three forces, 20 N, 30 N and 40 N


are in equilibrium. Find the largest angle
they make with each other.

B.
C.
D.

B.

D D
Notes

A.

A.

Fri

Solutions

downstream
upstream
downstream
upstream

80&: EE Board October 1997


A 100 kg weight rest on a 30 incline
plane. Neglecting friction, how much pull
must one exert to bring the weight up the
plane?

Topics

19.47
19.47"
18.43
18.43

wx
2(x-2) kg

47_? 100 r Solved Problems in Engine~~ing Mathematic_;U?n<~.~Tiong & Rojas

II

oi<llll~l

"ffi iilll~~l

Sll!U When one boy is sitting 12 m from


the center of a see-saw, another boy rnust
to sit on the other side 1.5 m from the
center to maintain an even balance.
However, when the first boy carries a(1
additional weight of 14 kg and sit 1.8 m
from the c,enter, the second boy must
move to 3 m from the center to balance.
Neglecting the weight of the see-saw, find
the weight of the heavier boy.

~~~.~ 11~~1JJI
.. ~~~~~1:!11

L.

.,lllMII

~~"'

A.

30 kg

B.
C.
D.

42 kg
34 kg
45 kg

8:14: CE Board November 199ft


A 40 kg block is resting on an inclined
plane making an angle of 20 from the
horizontal. If the coefficient of friction is
0.60, determine the force parallel to the
incline that must be applied to cause
impending motion down the plane.

A.
B.
C.
D.

82
77
87
72

815: EE Board Oc:tober :1997


A 250 ib block is initially at rest on a tlat
surface that js inclined at 30. If the
coefficient of kinetic friction 0.30 and the
coefficient of static friction is 0.40, find the
force required to start the block moving up
the plane.

A.
B.
C.
D.

1901b
2121b
125 lb
751b

8161 A 600 N block rests in a surface


inclined at 30. Determine the horizontal
force P required to prevent the block from
sliding down. Angle of friction between the
block and the inclined plane is 15.

A.
B.

160.75 N
198.55 N

C.
D.

164.60 N
190.45 N

Day 19- Engineering Mechanics (Statics) 473

tU7: ME Board March 1998

821: EE Board October 1991.

825: EE Board October 1993

Assume the three force vectors intersect at


a single point.
F1 ::: 4i + 2j + 5k
F2 = -2i + 7j -3k
F3 = 2i- j + 6k
What is the magnitude of the resultant
force vector, R?

A certain cable is suspended between two


supports at the same elevation and 500 ft
apart, the load is 500 lbs per horizontal
foot including the weight of the cable. The
sag of the cable is 30 ft. Calculate the total
length of the cable.

A copper cable is suspended between two


supports on the same level, spaced 600 m
apart. The cable hangs under the influence
of its own weight only. Under these
conditions, it is desired to calculate the
maximum sag (at the center of the span)
when the maximum stress in the material
2
is 1000 kg/cm . The cross-section of the
cable is 1. 77 sq. em. Weight of cable 1.6
kg/m. Use parabolic equation.

A.
A.
B.
C.
D.

B.
C.
D.

"14
12
13
15

503.76 ft.
502.76 ft
504.76 ft
501.76ft

822: EE Board April1994


1111:111: EE Board March :l998
Given the 3-dimensional vectors:
A = i (xy) + j (2yz) + k (3zx)
B = i (yz) + j (2zx) + k (3xy)
Determine the magnitude of the vector

sum lA +
A.
B.

C.
D.

Bl

at coordinates (3,2, 1)

32.92
29.92
27.20
24.73

A.
B.

C.

819: At what angle does the force F =


6.23i- 2.38j + 4.92 k N makes with the xaxis?
A.
B.

C.
D.

:39.2
40.2
41.3
42.2

=
=

14.73

D.

16.16

82:J: A cable carries a horizontal load of


20 kg/m. Neglecting its own weight, find
the maximum tension on the cable if the
distance between the supports is 100m
and the sag is 5 m.
A.

Assume the three force vectors intersect at


a single point.
F1 = i + 3j + 4k
F2 2i + 7j- k
F3 -i + 4j + 2k
What is the magnitude of the resultant
force vector, R?
15
13.23

D.

64.02 m
66.37 m
67.76 m
69.28m

A.
B.
C.
D.

B.
C.
D.

5099 kg
50591<g
5199 kg
5215 kg

824: CE Board May 1993


Determine the sag of a flexible wire cable
weighing 60 N/m over two frictionless
pulleys 100 m apart and carrying one 10
kN weight at each end. Assume the weight
of the cable to be uniformly distributed
horizontally. The cable extends 5 m
beyond each pulley to the point they are
attached to the weights.
1\
II

7.2 rn

I G rn

ll

/Am

r 11 rn

42.26 m
43.26m
44.26m
45.26 m

826s A cable weighing 0.4 kg/m and 800


m long is to be suspended with a sag of 80
m. Determine the maximum tension.

A.
B.

420 kg

D.

416 kg
400 kg

c.

414 kg

827: A cable weighing 60 N/m is


suspended between two supports on the
same level at 300 m apart. The sag is 60
m. Compute the distance of the lowest
point of the cable from the ground level.
A.

B.

s:zo: ME Board Oc::tober ll99&

A.
B.
C.

A cable supported at two points of same


level has a unit weight , ro of 0.02 kg per
meter of horizontal distance. The allowed
sag is 0.02 m and a maximum tension at
the lowest point of 1200 kg and a factor of
safety of 2. Calculate the allowable
spacing of the poles assuming a parabolic
cable.

C.
D,

205.5m
196.8m
200.5m
188.2 m

8:18: Find the location of the centroid of


the composite area consisting of a 10-inch
square surmounted by a semi-circle. The
centroid of a semicircle is located 4r/37t
above the base (diameter) of the semicircle of radius r.
A.

B.
C.
D.

6.0
6.2
6.4
7.0

inches from the


inches from the
inches from the
inches from the

bottom
bottom
bottom
bottom

47 4. 1001 solved Problems in Engineering Mathematics (2"d Edition) by Tiong & Rojas

~
'Ill

8%9: EE Board March 1998

il

Load

~I

d~ijj!I~I[IJ

' ltl~i~~l!ll
1111'~11~~~11

coordinate

ycoordinate

Kilowatt
load

1
2

3
4

1
2
2

100
180
200
120
150
200
180
100

111~11111~~111

L.,,,l""'"

6
7

~"'"""'

D.

Electrical loads are arranged on horizontal


x, y axes as follows:
X-

3
3
4

0
4
1
3

--

A.
B.
C.
D.

X= 2.000, y = 2.049
x=2.163,y=2.195
X= 1.854, y = 2.211
X= 2.146, y = 1.902

2304 in

833: EE Board March 1998


An isosceles triangle has a 10 em base
and a 10 em altitude. Determine the
moment of inertia of the triangular area
relative to a line parallel to the base and
through the upper vertex in cm 4 .

A.
B.
C.
D.

Topics

[QJ
tvlon

2750
3025
2500
2273

Tue

834: ECE Board Apri11999


What is the moment of inertia of a cylinder
of radius 5 m and mass of 5 Kg?
A.

B.
C.
D.

Theory

120 kg-m 2
80 kg-m 2
62.5 kg-m 2
.
2
72.5 kg-m

Problems.

830: A rectangle has a base of 3 em and


a height of 6 em. What is its second
moment of area (in cm 4 ) about an axis
through the center of gravity and parallel to
the base?
A.

B.
C.
D.

64
34
44
54

831: EE Board March 1998


A circle has a diameter of 20 ern.
Determine the moment of inertia of the
circular area relative to the axis
perpendicular to the area through the
center of the circle in cm 4 .

A.

B.
C.
D.

14,280
15,708
17,279
19,007

83Z: ME Boa,.d October 1993


The moment of inertia of a section 2" wide
x 2' 0" high about an axis 1'0" above the
bottom edge of the section is:

A.
B.
C.

1834 in 4
384 in 4
9214 in 4

835: ECE Board April :1998

Solutions

What is the inertia of a bowling ball (mass


= 0.5 kg) of radius 15 em rotating at an
angular speed of 10 rpm for 6 seconds?

A.
B.
C.
D.

0
0
0
0

Wed
Thu

Engineering Mechanics
Branches of Mechanics
Conditions for Equilibrium
Friction
Parabolic Cable
Catenary
Centroidal Moment of Inertia
'
Moment of Inertia with respect to
the base
Transfer Formula
Mass Moment of Inertia
'

'

Fri

Sat

Notes

0.001 kg-m
0.002 kg-m 2
0.005 kg-m 2
0.0045 kg-m 2

ANSWER KEY
801. A
802.A
803.A
804.
805. B
806. D
807.
808.
809. D
810.C

c
c

811. A
812. B
813. B
814.
815. B
816. A
817. B
818. B
819.
820.A

821.
822. D
823.A
824.
825. A
826.
827. B
828.D
829. A
830. D

c
c

RATING

831. B
832. D
833.
834.
835. D

c
c

c:J
c:J
c:J

30-35 Topnotcher
21-29 Passer
17-20 Conditional
0-16 Failed

If FAILED, repeat the test.

iI

476 100 1 Solved Problems in Engineering Mathematics (2nd Edition) by Tiong & Rojas

~,I

II

'II

For equilibrium, the force polygon must


close.

il

30ft

T L

Day 19- Engineering Mechanics (Statics) 477

- ..

F=P

P = Wsine

F = 200(9.81)sin19.29

P = 100sin30
p =50 kg

:E Finclined = 0

P =648.15 N

1m

Ill

II!IUI~~llill

tl'~ ~~~k4~11

40N

("l~lli11*1141il

a..~~~~~lli~~M

2Tcos9=100
2

L""'."'"

\.,.

,,,

1001bs

:EFv = 0

By cosine law:
2

( 40) = (20) + (30)


e 104.48

T=~

2(20)(30)cose

1!1

2cose

T=

-~0
'

100
2cos75.068
T=1941bs

-25! 500 kg

.G

t)

4 kph

LMFulcrum = 0

........

30

20000(x 2 ) = 1000(x 1 )

_20_0_0_0x~2

x1 = 1000

The component, P along the plane of the


force 500 kg is the force that tends to
move the block down the ramp.

R = J(20)2 + (30)2
R = 36 units
sinB=12
.e = 19.47"

II

P=Wsine
P = 500sin25
P=211 kN

=0

II

100 kg

x1 = 20x 2
Thus, the fulcrum must be placed 20
times nearer his hand than the stone
door.

:E Finclined

Ill

.-..-,

350 kg

We

F1=200N
By cosine law:

Let: We= weight of beam

R~ = 200 2 + 400 2 - 2(200)(400)cos56


R=332.5N

LMFulcrum =0

Ill
"15
tane=-

9=75.068

W8 (0.5x -1 .2) =350(1.2)

3o....--:
1OOsm
\ 30"!1 00 kg

Let:

P = component of the weight of


crate along the inclined

.p

Lf~ndlnod =0

..... -

F = frictional resistance

We =

420
7 Eq. 1
O.Sx-1,2

11

Day 19- Engineering Mechanics (Statics) 479

478 100 lSclved Problems in Engineermg Mathematics (2"a Edition) by Tiong & Ro~.
1000 kg

W8

'I

0.5x

ll

411111~!1~~

l,llil.l'lllll

M:
L

=0

wx

Fwenm

x-2

4 ~~li~~jl~

L,,,.,.,.,

~'""'

0.5x -1.2

WX (X ;

RA (x- 2) =

Equattng equations 1 and 2:

Substitute the value of N in Eq. 1:

p = 125 + 0.4(216.506)

y-axis
0.6N =40(9.81)sin20 + P

4)

P=0.6N-134.208

We

w,

!12m i

::J
p = 0.6(368.735)-134.208

L~=O

f . ::
RA

Re

=0

WA +14

11.8

2:Fx = 0
Pcos6+F=Wsin8

P = 87 Newtons

WB =0.8WA

::5 ~::;::::::: :~ 8

y-axis

Substituting the value of N in Eq. 1:

.3

"'{8 (1.5)- WA (1.2)

e =~oo

N = 368.735

EDI

W= 600 N

N = 40(9.81)cos20

1.5m
:
:: :~;; ~:;;
t: ::::::;; ::;

x=3.43 m

1m

~Eq.1

N=Wcos20

0.5x -1.2
0.5x -1.2
420 = 1200 - 350x + 440
350x =1200

P=212!b

LFY =0

wx(x- 4) kg
RA = 2(x-2

~= 1200-350(x-1.2)

N=Wcose
N = 216.506 lb

Ra

We= 1200-350(x-1.2) 7 Eq. 2

2:Fy = 0
N = 250cos30

!
(

RA i

350(x-; 1.2) + W8 (0.5x -1.2) = 1000(1.2)

fl

e = 20

,t

A?:!
U:U~ !1.
lt

---,

I
diM~~~

H:

x-1.2

w = 40 kg.

2- 2

350 kg

Pcos30 + tan15 (N) = 600sin30


W=250 lbs.

e =30

We

3.0m

Pcos6+f.1N=WsinEl

N=

300-0.866P
0.268

~Eq1

L;Fy = 0

t;;;;:~;;;:~~7~-;;;-:;w~d

2:MA =0
100(2) + 1500(2.5)- R8 (5) = 0
R8 =700 N

2:M8 =0

N = Wcos8+Psine

N = 600cos30+ Psin30

LMF=O

y-axis

W8 (3)- (WA + 14)(1.8) = 0

RA(5) -100(3) -1500(2.5) = 0


RA =810 N

2.4WA -1.8WA -25.2 = 0

Ill

RA(x -2) =

wx(~-2)

P = WsinEl+f.1N
Pcc125+0.4N

2:Fx =0
F = WsinEl+P
f.1N=WsinEl+P

Eq.2

P =Wsin6+F
P = 250sin30 +0.4N

LMs=O

Equating equations 1 and 2:

(0.8WA)(3)-(WA +14)(1.8)=0
WA =42kg

Ill

N = 519.6 + 0.5P

'-'F
L~
X = 0

Eq.1

Note: Since in the condition of the


problem, the block is to start moving, then
''~":tile cooftrcient of static friction.

300-0.866P =519.6+0.5P
0.268
P = 160.75 N!3wtons

480 100 l Solved Problems in Engineering Mathematics (2"d Edition) by Tiong & Rojas

11

r = ../6.232 +2.382 + 4.922


r=8.287

'Iii'

R=F1 +F2 +F3

I~

R = (4i+2j+5k)+(-2i+ 7j -3k)+(2i- j+6k)

II

R =4i+8j+8k

,111

~fUtlll~lllili~

~ lU~~ili~l\
lt:II~~Wil!l~l!

1-il~l ~~~il~ I

L. "~'" "

l"~'"'

+(Ak)

Ell

roe
H=--
8d
H= 20(100)
8(5)

IRI =J42 +82 +62

By cosine law:

5.465

1!11!11

= 8.287

Substitute the values


two vectors:

of x,y and z to the

A= i(xy)+ j(2yz)+k(3zx)
A= K3)(2)+ j(2)(2)(1) + k(3)(1)(3)
A=6i+4j+9k

(:) = 41.3

R=F1 +F2 +F3

B = K2)(1) + j(2)(1)(3) + k(3)(3)(2)


IRI =
A +B = (6+2)i +{4 +6)j+(9+18)k

IA+BI =J(A;t +(Ai +(AS

Ja

+10 +27

J22 +142 +52

Ell

3(~00)

11

'J~~axis

T2 =( ~!: +H2
l2)
12

17702"" [.1.6(:00lj +H 2

T=5099 kg

H =1703.67 kg
ro=60N/m

11

,..roe
8d

1703.67 "' .~~~00)

8d-

d"' 42.26

s = 500 + 8(30)2 -

+ 50002

T =10300 N

8d . 32d
S = L + - - -33L
5L

lA + Bl =29.88 units

4.92

J( 20(~00) J

T '= 10000 + !30N (5m)

Let: S = length of the cable

T=

IRI = 15 units

A+B =8i+10j+'27k

Note: This force is equivalent to the


tension (i) at each support.

~s~

IRI= J(A;) +(AJ) +(AS

B=2i+6j+18k

x 1.77 cm 2

F = 1'770 i<g

ii!

R =(i+3j+4k)+(2i+ 7j-k)+(-i+4j+2k)
R=2i+14j+5k

B = i(yz) + j(2zx) + k(3xy)

F=SA
. 1000 kg
F =--err?

T=~Lr +~

+ 6.23 2

- 2(8.287)(6.23)cose
cose = 0.7517339

liMill

S=A

H=5000 kg

d2 =r 2 + x 2 - 2rxcose

IRI = 12 units

L=69.28 m

d = J2.38 2 + 4.922
d-5.465

IRI=F~r~(Aj)

lA +BI =

----------------------------~D~a~y~i~9~-~E=n~gtn~~e~e~nn~~M~~tat~~

~tao
c

32(30)
5(500) 3

s = 504.76 ft

T2 =

(mL)

+H2

\ 2

103002 =[60(100)]
--2-. +H2
H=9853.42 N

T=roy

T = 0.4(130 +c)

roL
H=8d

e =H[~J
L2 =' 1200[8(0.02)]
2
0.02
L2 = 4800

H= ool2
8d
9853.42

2S =800

~ ~0(100)

d= 7.6

8dm

S =400 m
y2= 8 2 +c2
(80 + c)2 = 400 2 + c2
6400 +'2G + c 2 "'400 2 + c 2
Co,960 m

Eq. 1

48~ 100 l Solved Problems in Engineering Mathen1!ticsJ.?..::=..~iticm) by 'f!ong & ~5?1!!.

Substitute the value c in Eq. 1:

-~~

T = 0.4(80 + 960)

il

T=416 kg

I
.

x=150

Ar = 10

1 )( n: \1

+l:Z l4)10)

Substitute values:

Ar = 139.27 in 2

'---~

x=150

rm

'J2==10+

3n
12.12 in.

I=

let: J = polar moment of inertia


nd 4
J=-

Substitute Y1 and Y2 in Eq. 1:


139.27y = 100(!)) +

J"' n(20)

32

y = 7 inches from the bottom

, $

J = 15708 cm 4

S+y

X=Cin-C

S + (60 +C)

150=cln-~---

= _..!.._..!____.--"._e______, __ , _________.!!._.!.!.

bh 3

Ll +L2 +

->Eq. 1

lx=--

12

x~~O):_!OO(~+;;m:1)+1ax2'J+'~!~+;m;3)"':100:3~~~

lx =~(24[
12
lx =2304 in 4

1C0+100+2Xl+ 1:tfJ+ ito+ilJD+1lD+'!CO

Yz = 8 2 +c2

X=2

(60+c) 2 =S 2 +c 2
3600 + 120c + c 2 = S 2 + c 2

y =-

s == FGoo .; 120c

---'*' Eq. 2

'-p 1

:~.::..~~-~~---~-:.:;:.L~.:_:~~-:.:.:._r_:._-:~:_~l
L1 + L2 + L 3 ......L,

- 1~) + 10C(1l::.?Q:5~+E()\0)+ ~4)+20Q1l_+'l80(3) +.!_C0:2)


++2Xl+~+~+2Xl++~

....~,.:Jr.~. X1

Y-

Substitute Eq. 2 in Eq. 1:

150

y =2.049

= cln ../3600+120c + (60 +c}

By trial and error:

2
d =-h

h = 10 em

c = 196.8 m

l'l!'R
~

h=6

b = '10 em
)(

4r

10 in.

J':

ivn + Ari

br1 3

~lb

r.

_..

lx =i Xo +

oh

L-1'~-~--+-bh

Using transfer-axis moment formula:

':'_!'_

!Xl =co

-1-

.L\.cf

\ 1"-,

2:' ;uul:lil'

3.mr2
5

I= 3_(0.5)(0.15)2
5
I= 0.0045 kg-m 2

32

-~~ri!)

~)li-~~--JWA&~~ft!!t.,$1 L

I= 62.5 kg-m 2

1Xo =54m 4

5 in.

Y2::::

ix., = 3{6)3
3 - 3(6)(3)2

7 Eq. 1

wher-e:

60

1 2
1=-mr
2
I= _!(5)(5) 2

ArY =A1Y1 +

)>,

bh 3
1x.. =--bhd2
4

+ Asomi-cil'"...im

Ar =

Day 19- Engineering Mechanics (Statics) 4~3

-------

(2

,=,

-hi

3 )

(4 )l

1
2(10)(10)
3(10)

486 1001 Solved Problems in Engineering Mathematics (2nd Edition) by Tiong:_ & Rojas

'~I

I'

Topics

Mon

~
Tue
''"~~~~~~

Wed

Theory

Rectilinear Translation
- Horizontal Translation
-Vertical Translation
-Free Falling Body
Curvilinear Translation
- Projectile
-Rotation
D'Aiembert's Principle
Centrifugal Force
Banking of Highway Curve

Probien1s

Soluttons

[]
[]

Notes

Sat

Fri

:cctil!near translation is a motion in a


!llane or one dimension.

are l:h<"

i l1.e
11 ;m::,lalion:

Not this path

ca::::.::.-..:. . . . . ,....,:::::::}:::::::. .+::::~..


This path
(straight line)

of rectilinear
Top View

1.

2.
3.

Hori2:ontGI transiation
Vertical translation
C;Jiviline;;;,r

Vo

a=O
.............)!-

11 ., u nnl;1l tr;;n~;laiion (motion) is a straight


~~l~illl)l :llui'ifJ ~~

w1,.,,., dllH

..

jj

iliG

[ :,,,

ly'

'9jd;l'l/)

H 'f
1 1\111

hcni.;-:ont;]j
~

j,, {,
'!

~;\a;J;;;_

ri:

rr:~5'- :1nc,~tv

"

Elevation View

v
~

,,
'ii

488 1001 solved Problems in Enginee~~g Mathematics (2nd E@;ion) by Tiong & Rojas

1J

where:

IIWI~~II1

....

IIIIIN~~~~~~~~~

,~,

llll~ij/~11!

111~~111~1~

~'"'""

-~Vo

S=Vt

Vo = initial velocity (time = 0)


V = final velocity (time = t)
S ::: distance traveled after
elapsed time = t

.............

-~

,~

____:__J
s
The following formulas

~g

will be used:

x =V0 coset

: a

'f

So the time of flight, t is:

-~v

-~v
v

Since the ho'rizontal component of the


velocity is constant throughout the
projectile, then the Value Of X iS calculated
as follows:

Vo=O

If acceleration is constant, the initial


velocity, Vo is not equal to the final
velocity, V.

Vo

as it goes down and decreases as it goes


up.

V=Vr:;

II,,

Day 20- Engineering Mechanics (Dynamics) 489

t=~
V0 cos6

' 1
'/=2gt2
In the figure, the value y is obtained using
the formula:

The following formulas will be used:

~~~lil'
V""Vo:t:'ati

'""~~"

What Is a Curvilinear Translation?


Curvilinear translation (motion) is a motion
along a curve path either on the vertical
plane or on horizontal.plane. This type of
motion includes a trajectory or projectile,
which is a curve path on the vertical plane
and rotation on horizontal plane.

v
~J ""';.; 1:;~
, ""'':VIfSII'I:<>'""-l;l~ ,

. . '

,2

Substituting the value of the time of flight


"t" in the above equation:
2

V 7 =V0 ~ ;:J:2a$
V = V0 :tat

1
S ""V0 t:o-at2
'

where:

.2

Vo = initial velocity (time = 0)


V = final velocity (time = t)
S = distance traveled after
elapsed time = t
a = acceleration

Note: Use + if accelerating and - if


decelerating.

where:

Vo =initial velocity (time= 0)


V = final velocity (time = t)
S = distance traveled after
elapsed time = t
a = acceleration

Note: Use + if going down and - if going


up.
If the acceleration of the body is not given,
then it is presumed that the acceleration to
be used is the acceleration due to gravity
(gravity on earth).
g = 9.81 m/s2
g = 32.2 ftls2

What Is a Free Falllnp Body?


yvhat Is a Vertical Translation?
Vertical translation (motion) is a straight
motion along a vertical plane.
In this type of motion, acceleration cannot
be equal to zero since the velocity of a
body rnoving in vertical motion, increases

A body is said to be free falling if the initial


vertical velocity is zero. The acceleration
of a free falling body is due to gravity
which is equal to 9.81 m/s2 or 32.2 ftls 2 .

A projectile is a body which after being


given an initial velocity with an initial angle
of release is allowed to travel under the
action of gravity only. A projectile is a
trajectory which is a graph of a parabola.

(V cose )- 2 (V cose J
0

Simplifying the above equation will


produce the "General Equation of
Projectile".

....................

"-.. ' (x,y)

(O,O).x.
. . . . .

. O -X- +1 -X= "v,0 Sin


9

""'!!!lll!lllll!llii!I-.,4~W.flll\'!i~Y 1

j;,'., .......

J.!ii!i:t'#/Z;;;;pi

@ ~k

v.

Let:
and Vy be the horizontal component
and the vertical component, respectively of
the initial velocity, Vo.

kVo

tJilfrv.

v. = V

Vv

cose

=V0 sine

Since the variable "x" is in second degree


while the other variable "y" is in the first
degree, then the graph is a parabola.
Note: The vertical component of the
velocity decreases as it goes up and is
zero at maximum point of the projectile
and increases as it goes down, while the
horizontal component is constant.
If the initial point of the projectile is on the
same elevation as the point of impact, the
following formulas may be used:

Day 20- Engineering Mechanics (Dynamics) 491

490 1 Ob l Solved Problems in Engineering Mathematics (2nd Edition) by Tiong & Rojas

I(

r;
(

diM1 1 I!~IIII

Wh;;it is a Rotation?

Highest Point

11

Rotation is a motion along a circular path.

......
Vo .

Ymax

/'\e

F".J,p.~~'<'Wb~!\'t~v;'"'1J,'i~'"i11,-W('"&!~ '!:f,-"?'ry..'<'"""!-:;,.">~''7~"-"J1<_'-,,Y~

""-:oW>->"'

Initial Point

Maximum height, Ymax:

L. , . ., .,

~REF

.'\ "(
"

Let: S
V

or R:

''

g
If the initial point of the projectile is not on
the same elevation as the point of impact,
the following formulas for ran!=)e, R may be
used:
Highest Point

...:""
Vo

=linear distance in meters

or

REF::: W a
~~

._,L. _t.lt,. .,.,_,.,


Initial Point

R _ 2V0 2 sinecos(e + ~)
gcos2 p
Highest Point

Fe =F
g

f:C'lrC~0

v,,no

is the force thRt will


movinq in a circular o.8th

1''1

fmm U1e center.


~"'=ret

where: F

= force due to friction

Why is Banking of
necessary?

Hl!it~l_"!\'

go. !D'_~

In order for the weight of the carlo help


the frictional force resist l.he
force, banking of highway curve is
necessary. Once banked, a car can move
at higher speed due to bigger resistance of
the centrifugal force.

For variable anqulat aceeleration, the


following formulas must be used.

ui "'w02 2a6
"I

ro"" ro 0 at

9 ""ro0 t

rap view ot"


c1rcular track

D'Aiembert's principle is stated as follows:

gcos2 13

acceleration

::-c

s,.. wf!

~Q~eJ.mbe!f~

R "'~V,/ si~eoos(e-~)

If there is no force other than the force due


to friction between the tires of the car and
the pavement of the road to resist the
centrifugal force, then:

Impact Point

\e
-

Elevation view

REF ==ma

e = angular distance in radians


m = angular distance in rad/s
a ::: angular acceleration in rad/s2

!,\i.;;.-~themat.~cally,

= linear velocity in m/s


a ::: linear acceleration in m/s2

R"' Vo 9in2El

Fe~-

reverse effective force is equal to the


,,,nduGt of mass and accelertion.

i-

Range, Xmox

=weight of body

V = tangential velocity
r = radius of curvature
g = gravitational acceleration

..

=Range = R

where: W

. ...... ;,.. a

'T'~~ -"'"(~": -'>4\'

fjllii!III\WI

~ ....:

,. ;r:fe:<r;tiv force.

-....

...

rJ1i~~~~lfil!l

f... ,~lll~~~~~lll

"1"'
,'~

'{7

Impact Point
Xmax

force mentioned in D'Aiembert's


nrinciple is known as the reverse

"When the body is subjected to an


acceleration, there exists a force opposite
the direction of !he mo!ion and equal to lhA
product of mass and acceler~:.~tim;"

II'

The ideal angle of banking,


calculated l!Sing:

centr if11gal force


2

I,

1/V\11

'JI

tane"' v
gt

e may be

492

100 l Solved Problems in Engineering Mathematics (2"d Edition) by Tiorig & Rojas

1\,

For greater velocity without skidding,

tan(e + cl>), V
gr

I,

Topics
tancjl

dillllllll~lm

Utliljl11ill\lllll

,111 ..~1~~111
(

=ll

where: cjl = angle of friction


ll = coefficient of friction

Mon

IIIIIIIIO!I

~ .,.,1

Proceed to the next page for your 20th


test. Detach and use the answer sheet
provided at the last part of this book. Use
pencil number 2 in shading your answer.

[QJ

<4 ~~~~~1111 L'l

L.,..

Tue

D D

'QI;ribia:

~uote:
"He who loves practice without theory is
like the sailor who boards ship without a
rudder and compass and never knows
where he may be cast"

Theory

Wed

Problems

Thu

Solutions

Fri

D
D D

GOOD LUCK I

Did you know that ... about 500 B.C., th~


Pythagorean Brotherhood was originally
aware of the four regular polyhedrons and
considered them to represent the four
basic elements namely tetrahedron- "fire",
octah3dron- "air", hexahedron- "earth",
icosahedron - "water"! When the
Pythagoreans learned the existence of the
fifth regular polyhedron, dodecagon, they
considered it to represent the fifth element
- "universe".

D D
Notes

Sat

s:st: ME Board April 1.996


What is the acceleration of a body that
111creases in velocity from 20 m/s tp 40 m/s
111 3 seconds?
1\.
ll.
('

ll.

5.00 m/s2
6.67 m/s2
1.00 m/s2
8.00 m/s2

H7: ECE November 1.998

- Leonardo da Vinci

Rectilinear Translation
- Horizontal Translation
-Vertical Translation
- Free Falling Body
Curvilinear Translation
-Projectile
-Rotation
D'Aiembert's Principle
Centrifugal Force
Banking of Highway Curve

I low far does an automobile move while its


.peed increases uniformly from 15 kph to
1', kph in 20 seconds?

I\
II
I.

ll

185m
167 rn
200m
1l2m

838: CE Board November J.99fl


A train passing point A at a speed of 72
kph accelerates at 0. 75 m/s 2 for one
minute along a straight path then
decelerates at 1.0 m/s2 . How far in km
from point A will it be 2 minutes after
passing point A?

A
B.
C.
D.

3.60 km
4.65 km
6.49km
7.30 km

839: CE Board May J.99fl


From a speed of 75 kph, a car decelerates
at the rate of 500 m/min 2 along a straight
path. How far in meters will it travel in 45
seconds?

A.
B.
C.
D.

790.293
791.357
793.238
796.875

m
m
m
m

'

494 I 00 I Solved Pmblems in Engineering Mathe~ti~ (2"' Edition) by Tiong & -~


,~,

'I,
1('

840: CE Board November 1997


A train starting at initial velocity of 30 kph
travels a distance of 21 km in 18 minutes.
Determine the acceleration of the train at
this instant.
A.
B.
C.

0.0043
0.0206
0.0865
0.3820

D.

m/s 2
m/s 2
m/s 2
m/s 2

841: EE Board October 1996

-~'itJII~

An automobile moving at a constant


velocity of a 15 m/sec passes a gasoline
station. Two seconds later, another
automobile leaves the gasoline station and
accelerates at a constant rate of 2 m/sec2 .
How soon will the second automobile
overtake the first?

A.
B.

1.02 m/s 2
. rn/s 2
3.4 m/s 2
18.1 m/s 2

C.
D.
84~:

B.
C.
D.

building 40 meters tall will hit the-ground


with a velocity of:

50 m/sec
28m/sec
19.8 m/sec
30m/sec

847: ME Board April J:'JC):!


Using a powerful air gun, a steel ball is
shot vertically upward with a velocity of 80
meters per second, followed by another
shot after 5 seconds. Find the initial
velocity of the second ball in order to meet
the first ball 150 meters from the ground.
65.3 m/sec
45. 1 m/sec
56.2 m/sec
61.3m/sec

ME Board October 1!995

36
54
24
20

844: CE Board May 1998


Determine the velocity of progress with the

given equation: D = 20t +


seconds.
A.

84&: ME Board AprU :1995

A ball is dropped fro, n the roof of a

A
B.
C.
D_.

The distance a body travels is a function of


time and is given by x(t) = 18! + 9f. Find
its velocity at t = 2.
J.\.

4.52 s
4.42 s
5.61 s
2.45s

18.6-m/s

.ii
'J~

'

'il'

8491 lEE ha~rd October 199&


A ball is dropped from a height of 60
meters above the ground. How long does
it take to hit the ground?
A.
B.
C
D.

2.1
3.5
5.5
1.3

sec
sec
sec
sec

ilil:!OO: ECE Board April 1998


A baseball is thrown from a horizontal
plane following a parabolic path with an
initial velocity of 100 m/s at an angle of 30
above the horizontal. How far from the
throwing point will the ball attain its original
level?

A.
ll

n.

890m
883m
880 m
875m

~'ill!

If a particle position is given by the


expression x(t) = 3.4e- 5.4! meters, what
is the acceleration of the particle after t 5
seconds?

A.
B.
C.
D.

C.
D.

842: EE Board October :1996

A.

19.8 m/s
21.2 m/s
22.4 m/s

845: ECE Board April 1999


A ball is dropped from a building 100 m
high. If the mass of the ball is 10 gm after
what time will the ball strike the earth?

A.
B.

15.3 sec
16.8 sec
13.5sec
18.6 sec

C.
D.

B.
C.
D.

Day 20 - Engineering Mechanics (Dynamics) 495

~
t+1

when t = 4

848: EE Board O<.rto~r 1!99:1>


A ball is thrown vertically upward from the
ground and a student gazing out of the
window sees it moving upward pass him <Jt
5 m/sec. The window is 'I 0 n1 above the
ground. How high does the oa!l go above
the ground?
A.

15.25 m

B.
C.
D.

14.87 rn
9.97 m
11.30 m

ME Board April :11:991


/\ plane dropped a bomb at an elevation of
1000 meters from the ground intended to
hit the target at an elevation of 200 meters
from the ground. If the plane was flying at
:1 velocity of 300 km/hr, at hat distance
Irom the target must the bomb be dropped
lo hit the target Wind velocity and
o~tmospheric pressure to be disregarded.
.'\
11.

(;

1024.2 rn
1055.6 m
1075.5 m
"1064.2 m

83: ME ik,lard April :1995,


ME Board October 199&
A shot is fired at an angle of 45 with the
honzontal and a velocity of 300 fps.
Calculate, to the nearest value, the range
of the projectile.
A.
B.
C.
D.

932 yards
1200 yards
3500 yards
4000 yards

854: CE Board May :1.995


A projectile leaves a velocity of 50 m/s at
an angle of 30 with the horizontal. Find
the maximum height that it could reach.

A.
B.
C.

D.

31.86m
31.28m
30.63 m
30.12 m

1!155: ME Board October 1997


A shot is fired with an angle of.45o with the
horizontal with a velocity of 300 ft/s. Find
the maximum height and range that the
projectile can cover, respectively.

A
B.
C.
D.

800 ft, 1600 ft


923 ft, 3500 ft
700 ft, 2800 ft
1800 ft, 3000 ft

S$6: CE Board November :1.996>


A ball is thrown from a tower 30 m high
above the ground with a velocity of 300
m/s directed at 20 from the horizontal.
How long will the ball tlit the ground?

1.1',-;::e,: ME &11!1rd: O~"l:ooor :ll'Jil'\ll1


ll!o muzzle velocity of a projectile is 1500
Ips and the distance of the target is 10

''Illes. The angle of elevation of ihe gun


""1st be:
/\
II

21"59'
2:~ tl i'

Jl

/h"Hl'

?!lo:r:r

A.
. B.
C.
D.

s:n:

2'1.2s
22.2 s
23.2 s
24.2 s

In the last 2 seconds of NBA finals


featunng Chicago Bulls VS Utah Jazz, with
the latter ahead by 2 points with the former
at 94-92 count Bulls Michaei.Jordan
decides to snoot from a certain point on
the rainbow territory wt1ich counts 3 points
if converted. During the play, i(Jorcjan
releases the ball at 7 m from tne basket
a11d 2. i 5 m above the ground and an

I
Day 20 -

496 1001, Solved Problems in Engineering Mathematics (2nd Edition) by Tiong & Rojas

~II

I.

~
(

*MIIV\~1111~

rii.

ifi~IIII~Wl

inclination of 40" with the horizontal and


assuming no block was made by the
opponents, at what velocity will the ball be
given to cast the winning basket? The
basket is 10 feet from the ground.
A.
B.
C.
D.

8.57 m/s
8.86 m/s
9.03 m/s
9.27 m/s

~UIIIIill

~ .. ,.ur~l~~l.il~

L. ..,,,IJ

l ..,.:!

8581 CE Board May 1995


A projectile is fired with a muzzle velocity
of 300 m/s from a gun aimed upward at an
angle of 20" with the horizontal, from the
top of a building 30 m high above a level
ground. With what velocity will it hit the
ground in m/s?

A.
B.
C.
D.

298
299
300
301

m/s
m/s
m/s
m/s

8591 CE Board May :1.995


A stone is thrown upward at an angle of
30" with the horizontal. It lands 60 m
measured horizontally and 2 m below
measured vertically from its point of
release. Determine the initial velocity of
the stone in m/s.

A.
B.
C.
D.

22.35 m/s
23.35 m/s
24.35 m/s
25.35 m/s

8601 CE Board November :1.99Z


A wooden block having a weight of 50 N is
placed at a distance of 1.5 m from the
center of a circular platform rotating at a
speed of 2 radians per second. Determine
the minimum coefficient of friction of the
blocks so that it will not slide. Radius of the
circular platform is 3 m.

A.
B.

c.
D.

0.55
0.58
0.61
0.65

86!.1 ME Board October :199:1.


The flywheel of a puncher is to be brought
to a complete stop in 8 seconds from a
speed of 60 revolutions per minute.
Compute the number of turns the flywheel
will still make if its deceleration is uniform.

A.
B.
C.
D.

5 turns
3 turns
4 turns
6 turns

8621 ECE Board April :1998


What is the speeq .of a synchronous
earth's satellite-situated 4.5 x 10 7 m from
"the earth?

A.
B.
C.
D.

11,070.0 kph
12,000.0 kph
11,777.4 kph
12,070.2 kph

86~1

ECE Board November :1998


A rotating wheel has a radius of 2 feet and
6 inches. A point on the rim of the wheel
moves 30 feet in 2 sec. Find the angular
velocity ofthe wheel.

A.
B.
.C.
D.

2
4
5
6

rad/s
rad/s
rad/s
rad/s

8641 CE Bqard Novembeio :1997


A turbine started from rest to 180 rpm in 6
minutes at a constant acceleration. Find
the number of revolutions that it makes
within the elapsed time.

A.
B.
C.
D.

500
540
550
630

8651 A flywheel is 15 em in diameter


accelerates uniformly from rest to 500 rpm
in 20 seconds. What is its angular
acceleration?

A.
B.
C.
D.

2.62
3.45
3.95
4.42

rad/s2
rad/s2
rad/s2
rad/s2

~W1

:~1

;!)

866: ME Board April1991


A boy tied a 80 grams stone to a string
which he rotated to form a circular motion
with a diameter of 1000 mm. Compute for
the pull exerted on the string by the stone
if it got loose :eaving at a velocity of 25
m/sec.
A.
B.

120 N
100 N

C.

150 N

D.

135 N

867: EE Board April :1997


A man keeps a 1kg toy airplane flying
horizontally in acircle by holding onto a.
1.5 m long string attached to its wing tip.
The string is always in the plane of the
circular path. If the plane flies at 10 m/sec,
find the tension in the string.

A.
B.
C.

28N
15N
67N

D.

18 N

868t ME Board October :1.996


An automobile travels on a perfectly
horizontal, unbanked circular track of
radius R. The coefficient of friction
between the tires and track is 0.3. If the
car's velocity is 15 m/s, what is the
smallest radius it may travel without
skidding?

A.
B.

c.
D.

68.4 m
69.4 m
71.6 m
7-6.5 m

8691 CE Board November :1998


A hi-way curve has a super elevation of
7degrees. What is the radius of the curve
such that there will be no lateral pressure
between the tires and the roadway at a
speed of 40 mph?

A.
B.
C.
D.

265.71 m
438.34 m
345.34 m
330.78 m.

EngiJl~(!ring

Mechanics (Dynamics) 497

8'701 ME Board April 1998


Traffic travels at 65 mi/hr around a banked
highway curve with a radius of 3000 feet.
What banking angle is necessary such that
friction will not be required to resist the
centrifugal force?

A.
B.

c.

D.

3.2
2.5
5.4

1Po

8711 ECE Board April1999


Determine the arigle of the super elevation
for a 200 m hi-way curve so that there will
be no side thrust at a speed of 90 kph.

A.
B.
C.

19.17"
17.67
18.32

D.

20.11"

8'721 ECE Board April :1998


The inclination of ascend of a road having
a 8.25% grade is _ _ _.

A.
B.

4.72
4.27

c.

5.12"

D.

1.86

8731 ME Board April :1.996


A cyclist on a circular track of radius r
800 feet is traveling at 27 fps. His speed in
the tangential direction increases at the
2
rate of 3 fps What is the cyclist's total
acceleration?

A.
B.
C.
D.

2.8 fps2
3.1 fps2
3.8 fps2
4.2 fps 2

8'74r ME Board October 1997


A concrete hi--way curve with a radius of
500 ft is bar;~ked to give lateral pressure
equivalent to f = 0.15. For what coefficient
of friction will skidding impend for a speed
of60mph?

A
B.

c.

1-1. ~

0.360
Jl < 0.310
Jl > 0.310

~.

498_ 1001 Solved Problems in Engineering Mathematics (2nd Edition) by Tiong & Rojas

,,,
~~II

,('

~~

,.~.,,.J
',!
lllllllllliii!M~

D.

ll < 0.360

87'>: EE Board April1993


What force is necessary to accelerate a
. 30,000 pounds railway electric car a.t the
rate of 1.25 fUsec2 , if the force required to
overcome frictional resistance is 400
pounds?
A.
B.
C.
D

1565 pounds
1585 pounds
1595 pounds
1575 pounds

87&: ME Board October 1995


A car weighing 40 tons is switched to a 2
percent upgrade with a velocity of 30 mph.
If the train resistance is 10 lb/ton, how far
up the grade will it go?

A.
B.
C.
D.

1124 ft
2014 ft
1204 ft
1402 ft

on
on
on
on

A.
B.
C.
D.

19.63 feet
19.33 feet
18.33 feet
19.99 feet

...

(
'.11

',

880: ME Board April1997


A pick-up truck is traveling forward at 25
m/s. The bed is loaded with boxes whose
coefficient of friction with the bed is 0.4.
What is the shortest time that the truck can
be brought to a stop such that the boxes
do not shift?
A.
B.
C.
D.

2.35
4.75
5.45
6.37

Topics

D
Mon

~
Tue

s
s
s
s

D D
D D
[I] D

slope
slope
slope
slope

B.
C.
D.

4250 N
0.68 N
680 N
42.5 N

878: ME Board April1998


An elevator weighing 2,000 lb attains an
upward velocity of 16 fps in 4 seconds with
uniform acceleration. What is the tension
in the supporting cables?

A.
B.
C.
D.

2,150 lb
2,4951b
1,950 lb
2,250 lb

8791 ME Board April1998


A body w.eighing 40 lb starts from rest and
slides down a plane at an angle of 30 with
the horizontal for which the coefficient of
friction ll = 0.30.' How far will it move
during the third second?

Wed

Problems

Thu

Solutions

Fri

D D

877: EE Board April199&


A car moving at 70 km/hr has a mass of
1700 kg. What force is necessa~ to
decelerate it at a rate of 40 cm/s ?

A.

Theory

Rectilinear Translation
- Horizontal Translation
-Vertical Translation
-Free Falling Body
Curvilinear Translation
-Projectile
-Rotation
D'Aiembert's Principle
Centrifugal Force
Banking of Highway Curve

Sat

Notes

RATING

ANSWER KEY
836.B
837.B
838.B
839. D
840.B
841. B
842.B
843.B
844. B
845. A
846.B
847. D

848.D
849. B
850. B
851. D
852.C
853.A
854. A
855.C
856.A
857.C
858.D
859. D

860.C
861.
862.C
863.D
864.B
865.A
866. B
867.
868. D
869.A
870.C
871. B

872. A
873.8
874.B
875.A
876.
877.
878.D
879.B
880.D

c
c

c:J 3!-45 Topnotcher


c:J 27-37 Passer
c:J.22-2b Conditional

0-21 failed

If FAILED, repeat the test.

'l
500 .100 LSolved Problems in Engineering Mathematics (2"d Edition) by Tiong & Rojas

,,,

Ill

1~1

""''"""~!

s,

V = V0 +at
40 = 20 +a(3)

rm

S1 = 2.55 km

lj~lllll:~

L.MIIIIIIJ

l. .~~

Overtaking point ~!
t,= t2+2
-v,=15
a,= 0
;,

s,

V1 = V0 +at1
v, = 20 + 0. 75(60)

V = V0 + at -7 Eq. 1

V. = 15 km x 1000 m
0
hr
1 km
V0 = 4.167 m/s

x~
3600 s

S2 = Vot2 + 2at2

V = V0 +at
12.5 = 4.167 + a(20)
a = 0.41665 m/s2

s = 4.167(20) + ~(0.416665)(20) 2

S =167m

!
.ALNAA<J."'

S2 = 2100 m

1 ;:::

~~~

1 2
=V2 t 2 .+ -at
2 2

si = o(t 2) + i<2)(t2)

s = 2.55+2.1
S =4.65 km

s2 =

-7

Eq. 2

s, =S2
15t2 +30 = t/

60 min

t2 2

15t2 - 30 = 0

By quadratic formula:

l!!l!tl
liiiiiJI

1 2
S = V0 t+ --at

Vo-

v, -

s = 1250(

45

60

)+

45
60

t - 15J15 -4(1)(-30)
2
2(1)

~(500)(
)
2

t - 1518.574

S =796.875 m

tz

t 1 =1 min.
=1 min .
~-----------;~~------~

s,

s2

V. = 72 km x 1000 m x ~
0
hr
1 km
3600 s
V0 = 20 m/s

2-

Taking + sign:

V. = 30 km X 1000 m X _!.!!!:
0
hr
1 km
3600 sec
V0 = 8.333 m/s

2
t 2 = 16.8 seconds

.1

s = V t+-ae
2
0

1m

21000 =8.333(18){60) +ia[(18)(60)]

a =0.0206 m/s

ge

dx
v = - = 18+18t
dt

V=18+18(2)
V =54 m/s

1m
5
D=20t+t+1

dO

-5

V=-=20+-dt
(t + 1)2

.5

V=20--2
( 4 + 1)

V = 19.8 m/s

Note: Since the ball was dropped, initial


velocity of the ball is zero.

t2 = 15+18.574

1. 2
S1 = V0 t 1 + at1

=1st+

Equating equations 1. and 2:

x~

~->Gx

At timet= 2:

S=S 1 +S2

V. = 75 km x 1000m
0
hr
1 km
V0 = 1250 m/min

dV
a=--= 20.4t
dt
a= 20.4(5)
a= 102 m/s2

'

15t + 30____.---2
.
-7 Eq. 1

t/

sw1

S 2 = 2.1 km
.
S2

v = dx = 3(3.4)e- 5.4
dt
v = 1o.2e- 5.4
dV = 20.4t
dt

s, = V.t11
s, = 15(t2 +2)2
s

Ill

s = V0 t+-~ae

;;.;

s2

Solving for total distance:

Substituting values to Eq. 1:

-v2=0

:ul;.;w{~n';; .m.tMittiMM.: =;;~.

s2 = 65(60) + i(1)(60)2
V= 45 km X 1000m X~
hr
1 km
3600 s
V = 12.5 m/s

:l!!~Z.,X

t2

V1 = 65 m/s

I"'I!Wf!il~(l

""'""1

= (20)(60) + i<0.75)(60f

S1 = 2550 m

a= 6.67 m/s2

Day 20- Engineering Mechanics (Dynamics) 501

.l

1oo = o + i<9.81)e
X

1 2
h=V0 t+-gt

= 3.4t 3

t = 4.52 seconds
-

5.4t

l,,i

Day 20- Engineering Mechanics (Dynamics) 503

502 I 001 Solved Problems in Engineering Mathematics (2"d Edition) by Tiong & Rojas

-IIi I

I.!I

S:

.liM

h3 = V3t3 - 2gt3

V 2 = V0 2 +2gh
2

h1

;;0,;

''

,; t 2

= range of the projectile

V3=?
.

10

R=

0 =52 - 2(9.81)h
h=1.3 m

0 =802 - 2(9.81)h
h1 =326.2 m

1 2
h2 = v,t2 + 2gt2
326/2-150 = O(t2) + ~(9.81)t/

t2

=6 seconds

t3=~+t2-5

t3 =8.155+6-5
t3 =9.155 s

.. _ .
lt

~~"'!'!'l'~~~~

V/ = V0 2 -2gh

=8.1555 s

r~

i#~

Let: H = maximum height

;;

0 = 80-9.81t,

/.

~~=30o

'Vo=?

Vo

gx2
Y = xtane- 2Vo2cosze
(9.81)X 2
-800 = xtanoo - 2{83.33)2 cos2 oo

R = Vo sin2e
2
10(5280) = 1500 sin[2e]
32.2
e = 24.540

-..
,

.1. I 1&.4

2
sin 2 e
H=-"-o- 2g

H = 699 feet

~9=3~

H=31.86m

t = 3.5 seconds

...

2(9.81)

2
300 (sin 45
H = ---'-----''2(32.2)

Ill

60 = O(t) + ~(9.81)t 2

...

2
50 (sin30

H=-~---'--

X= 1064.2

1 2
h = V0 t +-gt

=100

.. Am

1000
3600
V0 = 83.33 m/s

V0 = 300 X

H=10+h
H=10+1.3
H =11.3 m

2
2
sin_
H=-"-o_
2g

V2 = V/ :...2gh

t,

32.2

BD
v, =50/

8001

t'

V1 = V0 -gt1

300 sin [ 2( 45)

-----:"~-'""'

R = 2795 ft x 1 yard
3 feet
R = 932 yards

Yl=

g
2

R = 2795 ft

Vo = 83.33
; -

6 .1
..

h3

4"':.

.:v1. = 5 ...i
;0;:

1002 sin[ 2(30)

9.81
R =833 m

V=O

R = V/sin2e

R = V/sin2e

R=

h 1

h2

T
: ...
t3 .~

!Vo=80

.1

Let: R

v3 = 61.3 m/s

V1 1= o

.
t1 ~
.

il

150 = V3 (9.155)-~(9.81)(9.155)2

= + 2(9.81)(40)
V = 28m/s

.:
:

iii. liiJ$$ q;;;

~
I

0 = 24032.4'

R = V0 sin2e
g
2

300 sin [ 2( 45)


R=--32.2
R = 2795 feet

Note: l'he closest answer from the


choices is 700 feet and 2800 ft.

504 .1 001 Solved Problems in Engineering Mathematics (2nd Edition) by Tiong & Rojas

1,1
1.11

S:

~ij

...

1111111~11

V=O

Let: t

.-

t1

.r

1:'vo

v.

...,.

E 1
q.

~-~~111U~

l~. :~

v.

H= o

t 1 = 10.46 seconds

v.y -2gH

0=(300sin20") -2(9.81)H
1 2
H = --gt 2
2

=-i(9.81)t2

t 2 = 10.747 seconds
Total time = 11 + 12

, .

t1

h = H-0.9

. t2
.

V2x =Vox

~ Eq. 3

v2x = 300cos20

Vo

ti/v'
(}\e

0...9

~2

2.15m

9.14- 0.066V02

x= 7 m

~i

83.54 -1.206V/ + 0.004V0


2
= 0.2025V (0.021V
0

0.9)

V=rro
= 1.5(2)

V=3m/s

wv2

Fe=gr

IJ

'II
II

Fe= 30.58 N

:II

LFtt.=O

l
2
=0.45v0.021V
0 -0.9

Squaring both sides:

:-----

~i

v2 = ~(v2S + (v2S

v2
3.05lm =10ft.

Fe

v2y

Fe= 50(3)2
9.81(1.5)

V2 =

9.14 = 0.066V0 + 0.45v0.021V~ - 0.9

o
= 40"

...

v2x = 281.9 m/s


t = t1 + 12

. 02
-9.14 = 0.066V0 + 0.45.j0.021V

V2 y2 = 0- 2(9.81)(30 + 536.59)
V2y = 105.43 m/s

"

Vo = 25.35 m/s

...

V2 y2 = V/- 2g(30 +H)

'

. .

0 = (300sin2of - 2(9.81)1-t

t2 = 0.45v'h

....................... ;,;.t-tj.:.;

H =536.59 m

h = 0.021V0 2 - 0.9
1 = 0.45J0.021V 2 -0.9

V=O

-2 = 60tan30o _ (9.81)(60)2
.
2(Vo )2 cos2 30

,12

V/ = V0 / -2gH

h = -i(9.81)t/

Total time= 10.46 + 10.747


Total time= 21.2 sec

1 2
h = -gt2
2

.;:.,

'*' ... : ..

....

30

300 sin 40"


H=----2(9.81)
H = 0.021V0

H= 536.595 m

. ,

. !<~.=-~~:. . . ..

e-

30 + 536.595

sin
2g

T .,
H

,.v

Eq. 2

_..

Vay /
~

t 1 = 0.066V0

o = 300sin2o - 9.81t1

V2 =

v1.=o

. .
t1/

0 = V0 sin40- 9.81t1

v =vov -gt

V0 =.9.03

V = VoY -gt1
V = V0 sin 0 - gt1

30

.l.

liiiiilll

.. . . . -~~-~. :. ~. :. ~. ~->:::
l Vo=?

83.54 -1.206V02 = -0.1 E

=V0 cos4ot

..

=0.004V04 -0.

t = 9.14

.....():-. .~. = 2~

L~,J

83.54 -1.206V0 2 + 0.004"

lf- = V0 coset
H

r:::::1

= total time (t) of the flight

Day ZO- Engineering Mechanics (Dynamics) 505

~(281.9) 2 + (v2S

=301 m/s

:1
'I

F -Fe =0

P- 30.58 = 0
F=30.58 N
F 30.58
Jl=-=-- N
50
J.1=0.61

:.II

il
'I

~
.

506 l 00 1 Solved Problems in Engineering Mathematics (2nd Edition) by Tiong & Rojas

Ill

!l,.:ll
1~111
,(',j

o> = 60 rev x 1 min


0

.ll!llll

rev
m0 = 1 sec

min

.,ill,~

"""'"""''Ill
lill ..

lllllii~J~

"''"I

V=rm
15= 2.5m
m =6 rad/s

60 sec

,'f

:f I

Ill

Ill
Note: The force exerted by the string is
equal to the centrifugal force

wv2

Fe=-m =m0 -at

0 = 1-a(8)

180=0-a(6)

re:s

1<1o1Ujll!ll'l~~l

a=0.125

,.~.,. ",1

m2 = mo 2 -2a9

a=30 - . 2

....

mtn

Fe

0 = (1)

"UiiM~II!

2
-

m2 = mo2 +2a9
2

1802 = (1) + 2(30)()

2(0.125)9

9 = 4 rev or 4 turns

V=rm
1 rev
2n rad
24 hr ., 1 rev
-5 rad

hr

3600 s

V = (4.5 X 107 )(7.27 X 10'5 )


V =3271 m/s
V=3271m x ~
s 1000 m
V =1.1777.4 kph

e =540 rev

3600 S
-;,:-

9.61r
r =265.41 m

=66.67 N

"' 67 N

Ill
w

V = 65 mi X 5260 ft X ~
hr
mile
3600 s
V = 95.33 ft/s

m =m0 -at

9= 5.40

l:FH :.Q

re:s

a =2.62

Fe= F = tJN

Ill
V= 90'k"'

l:Fv =0

Fe
V

= 25 m/s

F.e---- wv2
f.LW=

1000m X~
km
3600 s

V=25 m/s

v2

gr

s
V=-

hr

N=W
Fe= tJW

r =0.5"m

Ell

tane=g;
(25)2
tan9 = 9.6:1(200)

9 = 17.67,0

wv2
gr

0.3 = 152

V=30
2
V = 15 ft/s

6
r=. 2 +12

r = 2.5 ft

tan7 = 17.88

1.5

llii1l

52.36 =0- a(20)

gr

500 rev 1 min 2n rad


m=---x--x-min
60s
rev
m 52.36 r~d/s

m=--X--X--

m=7.27 x 10

tan~=~

Fe= mV2
r
Fe = 1(10)2

rev

V= 40mi x.5280ft X~ X~
hr
mile
3.218 ft 3600 s
V =17.88 m/s
'2

gr

m =m0 -at

Day 20- Engine4!rin9 Mechanics (Dynamics) . 507 .

Fe= WV2
~r

9.81r
r = 76.5 m

Ell
'i
i
I

I
'

508 100.1 Solved Problems in Engineering Mathematics (2nd Edition)


slope= tan9

1"11

tan9

~~~

2
8.53 + cjl =tan, , {88}
32.2(500)

=0.0825
e = 4.71

Day 20- Engineering ~-~cha~tics (Dynamics) 509

by Tiong & Rojas


V. = 50 mi X 5280 ft
0
1 mi
_ hr
V0 = 44 ft/s

....!...!!!:_

V = V0 +at

3600 s

16 = 0 +a(4)

Ill

L !'=inclined =0

J.l = tancjl

LFv =0

J.l = tan17.16

REF =;:F + Wsin9

J.l = 0.309

...,.,,I

I'

Fe= wv2
gr
Fe= mV2
r
Fe =ma

40(2000) a= 10(40) + 40(2000)sin1.146


a = 0.8049 ftls2

kz .............,..+a
'WJ-REF

LFH =0

v2

V2

i.JL

i&k&JL

0 = (44)

..,.

2(0,08049)8

Note: The nearest value from the choices


is 1,204 ft.

Wa F
p = ---+
g

T=W+ Wa
g
T = 2000 + 2000{4)
. 32.2
T = 2248.41b

+a

............

Ill

p = REF+F

T=W+REF

Note: The nearest value from the choices


is 2250 lbs.

=V02 -2aS

s = 1203 ft

F = ~00 lbs.

a='
r
27 2
a=----' 800
a, =0.911 ft/s

.............,..-a

30000 ( 1.25) + 100


p = 32.2

p = 1556Jbs

2
a =a/ +a/

LFH =0

Note: REF means revers~ effective force


by d'Aiemberts principle.

x 5280 ft x _h_r_

F=REF
F=ma
F = 1700(0.4)

a2 = (0.911)2 + (3) 2

Ell
V = 60 mi
hr
V =88 ft/s

Wa =F+Wsine
g

The coefficient of friction is less than


0.310

at

a= 3.1 f/s

a= 4 ft/s 2

cjl = 17.160

mi

..........

3600 s

-a

V=O

'\kf_Position of the
body after 3 s
LFinclined =0
Wsin9 =REF+ F
Wsin9

F=680N

Ill.

tane = 0.25

sin30 = ~ + 0.3cos30
32.2
a= 7.734 ft/s 2

Qf

tane = 0.02
9=1.146

Wsine = Wa + J.1(Wcos9)
g

e"' 8.53
tan (9 + $)::;: V

= Wa + J.LN

~EF

Let: V

=velocity after 2 seconds

S =distance traveled iii the third


secol)c:Ywith reference from
the position at the end of 2
seconds.

,I!

, I~
jl.'

510 1001 Solved Probtema in Engineering Mathematics (2'"1 Edition) by Tiong & Rojas

'[

''"~l,,l

v
v

~I

.11~11

[
r'
"1
~ ~~~~~u~
....

l.~,.,,l

l . . :!

V = V0 +at
= 0 + 7.734(2)
= 15.468

'l

',';: j

ftls

1 2
S =. Vt+-at
2

s = 15.468(1)+~(7~734)(11
s = 19.33 ft

s
w
REF

F
N

LFH=O

I!

F=REF

J1N=-a
g

J1W=-a

a= J19
a = 0.4(9.81)

R,~'<~~O'~'

(.'O~+

~<~

-''~<'(X~~'

~.~x.

a = 3.924 m/s2

~11"~:~''><;v;<;'

'

V=V0 -at
0 = 25- 3.924t
t =6.37 s

"<<'

(~''

~10,<.,.0:<'.~<~-<t>

~ ~

"' *

!<

<

x. "', .... ~

'4"'

<'<(><<~~x~v;,.,.,.;~<

"* ,, -'t * ,... "

~~'~<~-~.,-.,.~,.

<''>

.;,'to.<

>~

<v>~<~<O<->

<f'

;,

I'

< - '

... ,._.,,,,,;.., .... _'(!'~("'''+~<<'<-'.~'<-~,..-~


~~O<C<c>

...

<-~

~4~~"''

+<

><'('X''<~<<'>'>'<"<,<>'"<''"><<-

1:
'_,,,

.,,:~~.,..,.,,......

"" "

'

<

,.

Jt

,.,._,, ..

,..;,;y-..~"'~"'~\0'~'\+'<'*

< < < X

< ....

_<."

_,,(;;<,.<A

.<'

< .-'

<

~'""~~.

* oi:.,

<

~""""<?~-

'>

<

~ '

" '

'

.~

< .,- _,_ ",.

~ ' ..., ~ ..... ~

>

...

,<

'

<>0<-.~

<'! '<

< ,

........ .

<)' '-.

' ,.. "" ... .. ...

'

'

"<

~ ~.,. >

<t-

~ ~

0 ' \ ...

"' <

'j- ,.. o

512 :1001 SolVeclProblems in ~S Mathematica(a"'i Edition) by Tions & Roiaa

'

,,,~

1,1

.,

(' :~

.s

"'" <

~-

... +

J<

!<

:c

"' .,. "'

"<II

~- 1<"' ~.. "*

~-,. .. .,_

' ., .,.. ,.. *


.0, )<." 0 ~

,.

'""'~'

<'

,_ .;,

1111~111~11~

, e

'""'"'I

< "<f

.,. "'

4',.

'i' ., ,. ;

,. ~- ~

" ' 'f. ...

t<

,. .,. 11< "' ,... ,. "'

"'(' -~ !<!"

t:<

'

"

,. "

"'

*....
< <-

'f<

1' ~"'

'-' ~"

<' ~ <- :0

"' ., t tt " <t ,. c~ ~ ~

,.

~ ~ ~

-~-

~ ~

;. "' .,

-'

.0

10."'"

., .. .,. ..

"' "

... f "' .... ~ 0~0

-1< ~

.,. 0< ,f_

->'h

>!<

-t",. .. , '

It,.,

>1-

II-

: "'

~ ~

..

..

"

v ~ ... < ,. ""' "' ... "' ... " ., ~ ~

< ., ,. ~ "" """ .., ..,. . . . , -1' "'

<t .,

<'

.J ._. o!. " -~ ,. ~-:c.

< ~ "" ~ "" .;,. <t :~ "'."' ~ .. ~ -< "" ,,

:r. "'"

;,>,."' t

o>, 'I! .. "'"(< ..

~-

>' ..- "' "' .. ~ .,. .,. "' " ...

*' "' '"'. "'

~ ~ .0: 'I,""' ~

.,

.0

"

ol! ..

>

~ ~ ,.

1o ' <!- "

"< <fo

;..'<

~ ~ ~

>t ...... " " ........ " ... ~- ~ "'

" <- " '< > ~ ... '

..

~ ~ ~ ~

;.

< ~ " .,

'.., ..

A ..

\-

"''<

'

Topics

l< ,... " :

0
,_J

~ <_..a ,. '> J!. 'f ,}

'

X'>'~~'<! 't ,._

>

M ...

<"

Mon

<- t ., ~. ""."' "''t ."' ~ .$ .,.. ~ " < .., .,. .... ~ ., .. ~ ~- .., ~ .. ~

~--,

< .; ,;-. > :1;- ~ :" ~ !'

"' :"> ll' X.

HIIII'

'I ..

>f

..,.,,I

!J'

$.

><- ">

,>,

~~

ill:

-~ *.>'<.""-.$' 1<<i<1o

'I'

-~~ ~,.,~:-:.:.;/' ~

......~Mflllill

<'\

& ..... l1c <0 >'

.j).

~'

$!

. ii<#'f.WIIIi.t< ~;.,:~I!>~"" :t II{ .if.

~< ~4 ~ ..;.>'i'4<-} lli

~:,.,

ljl

<i-

f,.,... "''"'"' ~,o;,.


of',_,.._,.~,.,...

*' ~~1<>-.~i<d"'

"' "' .;>

;,.;,

"(:< .. y

~ ;,. ~

'

:,q.

+'>t"ot.ti.

~ ~:~~oo ~

1, 'il0

< I!'$

1 ,. : " ,. "-. ~ ""= '> .. ,.

.., <.

,.<&,,,~

+;:
'<-

<* 1!!.11<

'P '0

~ ~

~ ~

'< ;,.

)I

'f " ~."' -" '{'

>1 'l'

"'

lo.._o(.f< ~

J:<

'"

<; <> " 1f <'1 ; .. ', 4 J :1 ~ .

J_ '.,i "< ~

fl ot ~ ~4',.,; ,..,.

1'<&

<t,.

.~ i<J.-..4$. ""~ .0

~"" + ~ <\-." " ~ .... ~ ... ~. "" ~


)!<

. . . . . . -.< " ,...

\,>'<' >'-!.~ < ~")

'+'"' ~

"'i>JI! ""'"'

"' .: ..,.

...

... o'

>"'

1<

'f 1< >- '!I

;1-

.,.,.

O</

N'

t' ,.

,o/< ,><- ,

'

Tue

"', (.

:3- f'

>ol;

_f:

i;.,..,

,to';-~' ~4'., *'.)!>

[QJ
LJ L--l

:(<

~)< -p~ ~~--:~ ~:_4<,. ~

Theory

..

..

"'"'~""""I!"!;:., ,~,.,.

>9

.i<f! ~ "'"' 1{

Problems

1<

."' "'~ lf-t- "'<~ '" ~ ~


~ . '* <.:; -r ;

<.: '" ,. lO >f

tj'1f

<i"

~~':~

,.,....

.> >

'l'

...

~!!

10

"<

~ ~

-tY

~ _, ~"'' ~ 1< '11 ~ ,~ "'. "' ~


"

1<

_,...,..,. . . . ,.,

<- , "" '-.

~>I! .... ~ ~ ~

j' .,

* . . ~ "'"'~ . . .,..,., ~ ~ ~ + ": .. " ~

x'Jo .,

_,...._':

If "' lf!'

'~

1 >

>0 X.<!

'i! "

-A "

I '

,. ..., ~ ''

> "' ? "

Of

, ...

)<'

><

)1.

~ ~ ,. ,.

":~1-"'

4-- -$

; '<

~~

_,

~ ~ .~.

> ><"~--"."'.,.~

<. ,<-. \0 :

It Jo;

~ -;1 :,>.

.o< .4

.<

"4;

1"

"" <llo 'I>

-.

-1! -:> .<, .;,-: -'j

'., i<..::-'l>il<'il"** ,.,,.

~ ~

.. ;; "'

~ ~

.. ,. ..

l< A" "' "f ' -""'

,~

.,,:

"

~ ~ :"' .f!l o+,

"' .. "''

"<

M ......

">""

"''JII

"':i ~ ,., ~.

0!

t<l.,.,._.- . . .

~.

1>

,.

" ....

., ....

... "

~. ~ ~

' ii'

~,~.._

'

,_,

r.'

'f

.. !:.

~.

"

~ ~

... "'i

........ ~- "' "' ,.

.,

.,~

~ .<

,_

<k

-k ..... ><- ... ~

'

> -...

-1'. .. -6-

~ v >

o:~x-x<:'"'~~""">

<;; ,._ '>,"*

.->

">

'~!. '<'

.. -t '> , $ ,.

0."

....

"<' '

f!j 'f. &

<"< ,.

"0

~ ~

"' '<

4 ,. ~- 10

...

I< 'It ... f

#,.'11'.1<:., .....

<! ...

)1..~

<.* ~

-<- ;... < ~ , ,..~ ....... "". "" ~

... ,,. .... ;t. ... ~ ...,,..,.. .,,...

II<&~
~

0'

>.-;. 'o.

;~''''"'-

<!- ")- ..

<('

...

3> '< :,1- <f ..

-f

:,.

l ..

~ " ~ "'..-. >

...

'

...

.; " . ~

~''''"''

:..1 .. ", .. ~

..:

+. <-

.ji"

"*': \!

-i' '

~~~
,.

.. ,. It .;

......'to".,

- 't

_f..,."'

.. ~,.~ ...

,.

~ :'

:,_I,

is Simple Stress?

~>imple stress is defined as the force per


1111it area. The typical units of stress are
2
MPa, lbf/in and ksi (thousand of pounds
per square inch). The MPa is equivalent to
MN/m?. or N/mm 2

~ <""

o ~

,, .............. ~ "' '

11\[~t

* If"~ t-

j'>.,;. ... ~-% 4' if ~ ~~ 'f,. ~

1>. .,. ,.

~ ' ; ~ ...

"!

"

~ ~

\1' ...... <!-$-4. -t

~ of! ol! . . . .~-

.,. ,. ,; ...

w .... li" i.. 'li ~ ."' .. ,. .. r. ~ ;:

1.o:10

.-o.

~ ~ -4 'IF.~ 11-.::. -"' "' "

>. ~ ~ ~ ~. .~ ~ ~ 'f .,- ~ '"." ~ ........ ~ ' .,

<, ~ ~ ' "'

~~'-

perpendicular to the area resisting the


forces. Normal stress is also called as
bearing stress. Shear stress, r, is the
type of stress which is caused by forces
acting along or parallel to the area
resisting the forces. Shear stress is a!so
called as tangential stress.

<~:o>o,... ~;i>~'o

...; "t .... "-t .... r.

10 <. :(<- ... "

'< """' <!.

..., .. " ...

,_,,,..,.

'll' 00: ,. .,. ') '> <0: ?; \t

~ ~,

;,.:" .. :l1:t."O

oO< X

~..--~"4>~

... ..-

l- M ,_

""'J';'(.<t"~>l-'f:>;--~ "'""""'~X~

~ .. .,. ~ ~ ~ " '-! ~ "", ... :'!(: $.~

' '" ' ' :

o<-

~-~"''~""''<~~_,;.<.~~"'**'"''

"'"'

J!'

o0<

S<lt

:;trength of Ma'terials is the branch of


:;cience that deals with the elastic behavior
of load bearing engineering materials.
:;tren9th of Materials is also known as the
im1Ghanics of materials.

""-"~~rN'<''>--:=:."';"'"<

><

Notes

Wl1at is_ Stre!]!Lth of Materi~l!!.1.

.~. 1>"- '

' ,. >. "' , .... "" " ~ ~ ., ........ "" ~~ ~ ... ~ ~- < ~ ~ ~

Helical Spring

1<4 ~.

....

"" ' ~ " "' i ,. '

_,.. >1

Fri

*'h. o. ~ ~ * *~.. ~

Torsion

Thu

Solutions
'II

I" ...... ..;,.,"' ... .. ~..:. -\..

~ l- ~ " 4 4-1,. ~ ~ >' >. ~ ~.;. ~ ~

Wed

[] 0
0 [-]

"'""' *''*'!:,. "~".""'*-.'"' ...... ,.~ "'." ... ,.,..~,.,._~It"' ... ,.,. .... ., ... ~

~ .~ ~

;_. :

4 "

'* .,.~ "'* ,; _.. .t . ,.x- 11.11 .... 'if"'.""-.

~#"i'+ .;...,.\". ..,_., ~w.

.ii ~f-.~~ :t ~ ~.~ .... &

..... $.-It ....

'

~ "

.. -*' *'** ~*'"" -~ - (-.. .....,_-..:::.;J"~':,: ~Ill 'f'<ti :f.

1(:

"4

.>< ~ ~ ,. ~ lr-~ ij "' .,.,. / ...

.;. ,._,. ...

t."' 1!' .. ,. '* .. ~ .... . * .~ ~ o"' ........

'

~)<I<"

'!''II' <I

~ .., ~ ~<! ~ -~~~'!!~ ~~

Jii1r'*f< ..-b'<''\0"-t-';;- ":.,~ <t'lo

~ .,

.,...oli-

)0;

~- .;- :~ ~ 4< ,-( "' ~ ~' :t '!" :w.. '<.' {! ~ JR "' -:o- ~

"! <!'"

.,

>~ ~

.. _,..,,.._,..,_.,. ............. "" ... ~~-"'.fl:' . . ,.

'( .......

.,. ~ .,.~... 1! .... "'

.,. ""'. "' -!<.f' ..

<

I"< '" .,.,- <$o ;< <If

(: :.;

.1tl ,.,,:.-~lit~ f<':' of!.~~:,.,~.,;~~ ~'*:lt_-,;.,. . -+}- '@<.,.;:-><...,~'tis

')<.!fl.~ . . "" ,j\'\"'.t '(<';t' ~~II:!'*"''*, ~

......... ~ .,\0!

'!~ ~ ~

~ <~- ,.,.

ji<<t

..,m.,. *"'*~ * ~!'.*~~'

.""~< .,. , ",..

::C.

";_lit;.::..<':' ,. "':
i

1<J :<-

..-

~. ~ ~- 't> ~ ~ ~ ~ -~ ~

Strength of Materials
Simple Stress
Types of Normal Stress
Simple Strain
Hooke's Law
$tress .. strain Diagram
Thermal Stress
Thin-Walled Cylinders

Area

1he two basic types of stress depending


the orientation of the loaded area are
<~11ll:1l :,;iTess and shear stress .

'lll

1> "' .~u ~- ': ~ 11 ,. ~

Nonnal ~;ilxl.'s~. o, IS the type of stres::;


wh1< .II 1:. "" l';~:d I lV forct"S actin<)

...... y'l.. ~ II. ',, ,,

-~

Nonnal stress

p
fl""A

"I'

J
-------------------D~ay~21~txengthof~~

514 1001 Solved Problems in Engineering Mathematics (2nd Edition) by Tiong & Rojas

,,,1~1

to:;~p

1.11

},

(' ~

ilm;:

!~1111

(-"'...."1"l

JIWI<I'AI1UI

Shear stress

-~1111,1

. .,,I,!,
,.lol!ll!ll~ll

Ultimate stress (or ultimate strength)


refers to the highest ordinate in the stressstrain diagram.

where: G

't"""A

What are the Two Types of Normal


Stress?

Strain,

What

E=l

Is Stress-Strain Diagram?

Allowable stress is the maximum safe


stress which the material can carry.

II
1:,.
]

Factor of safety is the ratio of the ultimate


stress to allowable stress.

'I

D e

.."'

Mathematically,

0" OCE

Shearing strain is the angular change


between two perpendicular faces of a
differential element..

I,

Modulus of rigidity (G) -refers to the

I
I

modulus of elasticity in shear.

What .is..Tit~!!~~
Thermal stress is the stress on the
~ material caused by the intern<;~! forces due

cr=EE

Tensile Stress

Compressive Stress

What is a Simple Strain?


Strain refers to the elongation of the
material with respect to its original length
when subjected to a load. The typical units
of strain are mm/mm, inch/inch or no unit
at all. Strain may be expressed as a
percentage of the original length.

The constant of proportionality, E is the


modulus of elasticity of the material. It is
also known as the Young's modulus.
The Young's modulus was named after
Thomas Young who introduced this
constant of proportionality in 1807.
Also,

Eo

A ""'L
When the material is subjected to a
shearing stress, as shown in the figure on
the next page,

Stress

"Within elastic limit, the stress is


proportional to strain."
Area

material when loaded.

between the stress and the strain is known


as the stress-strain diagram. The
diagram below is the stress-strain
diagram.

Hooke's Law formulated by Robert Hooke


in 1678 is stated as follows:

Working stress is the actual stress of the

= shear modulus

A diagram that shows the relationship

What is Hooke's Law?

I'

Rapture strength is sometimes known as


the stress at failure.

1:i=GS

The two types of normal stress are the


tensile stress (forces away from the area)
and the compressive stress (forces
towards the area).

the materia!' even without any


corresponding increase of load.

to change in temperature. The change in


temperature can cause a change in length,
area and volume of the materia!. The
temperature deformation (linear) may be
Strain

A -7 proportionality limit
B -7 elastic limit
C -7 yield point
D -7 ultimate strength
E -7 rapture strength
F -7 actual rapture strength

calculated using

cr:::.~
.. ,:::=J,
-
S. ubje.c. ed.
.t.

.,.r..---~--

~.:.t.~mp. change
.E

...

li

~--------~~

o1t

Elastic limit refers to the stress beyond


which the material will not return to its
original shape when the load is removed.
fhe permanent deformation caused by
excessive stress is called permanent set.
Ylold point refers to the point where there
Ill an appreciable elongation or yielding of

'1!,

Br == aLt6.1')
where:

a = coefficient of linear expansion

:1~ :
I'

1111'

~~~~
1,
516 : 100 1 Solved Problems in Engineering Mathematics (2nd Edition) by Tiong & Rojas

'"1~1

1.11
~

:~1

'Miat is a Thin-Walled Cylinder?

What is Torsion?

A thin-walled cylinder is a cylinder under

Torsion refers to the twisting of solid or


hollow circular shafts.

pressure and has a thickness, t :::;


inner radius.

1~ of its

Day 21 -Strength of Materials 517


where: T = torque applied
L =length
J = polar moment of inertia
cross-section
G = modulus of rigidity

I
I

A. Shearing stress:

GOODLUCKI

-r:p

(-""'~1

'{ "'j

:::::,,

P =27tf T

where: T = torque applied


p = radial distance from the
center of cross-section
J = polar moment of inertia of
the cross-section

,. ..1111111

.,.,.,J,,
',In'

B. Maximum shearing stress:

'I!IJI!II"Ii!lfll'lll

The thin-walled cylinders will experience


tWI> different stresses, namely, tangential
stress and longitudinal stress.

Max.f::;~$f.
'J

where: P =power in watts


T = torque in N-m
f = frequency or' speed rps

What is a Helical Spring?


A spring which forms a helix is called a
helical spring.

A.

Maximum shearing stress:

where: r = radius of the cross-section


T = torque applied
J = polar moment of inerti~ of
the cross-section

A. Tangential Stress:
pD

16PR
(1
3

t""

rcd

C. Maximum shearing stress of:


where: p = pressure in N/m3
D = inside diameter in mm
t = thickness in !Tim

or

+..!!_)
4R

pO

Another term for tangential stress is


ciR:umferential stress, or hoop stress or
girth stress.
Note that the longitudinal stress is one-half
the value of th~ tangential stress.

"Mathematics is often defined as the


science of space and number ... not until
the recent resonance of computers and
mathematics that a more apt definition
became fully evident: mathematics is the
science of patterns!"

Maxit.~ 1&1

where: P = axial load


R = mean radius of helical
spring
d = diameter of rod/wire of
spring
m = ratio of the mean
diameter of the spring to
the mean diameter of the
spring rod or wire

>''"'\'

;_.'.~'"

B. Hollow shaft

where: p pressure in N/m


D = inside diameter in mm
t = thickness in mm

4m-4

Did you know that... QED, the


abbreviation of Quod Erat
Demonstrandum, Latin for "which was to
be demonstrated" was commonly used by

mathematicians to indicate that a


conclusion has been reached, was first
introduced by Euclid using its Greek
equivalent in the 3'd century B. C. !.

- Lynn Arthur Steen

/c

O'L=M

16PR(4m-1 + 0.615)
rcd 3

A. Solid shaft

. . .:-:;;r

B. Longitudinal Stress:

p:

~tibia:

~uote:

:;:2t

Proceed to the next page for your 21st


test. Detach and use the answer sheet
provided at the last part of this book. Use
pencil number 2 in shading your answer.

E. Transmit power, P

~~

CJT

of

Max. t= . ~~:;;P '"'i . t


#''

2R

il,l!

m= - = where: d = inner diameter of shaft


D = outer diameter of shaft
D. Angular deformation,

'!I',II

B.

Spring deformation:

9:

. e....;r:~
-Ji

1111111'111111

0 = 64PR4 n
Gd

where: n = number of turns


G = modulus of rigidity

Day 21 -Strength of Materials 519

,,,a

~,

1.1

(' It

'st

Topics

0
0

-'' '1

Mon

:::::1,
""'!iilllli

Strength of Materials
Simple Stress
Types of Normal Stress.
Simple Strain
Hooke's Law
Stress-Strain Diagram
Thermal Stress
Thin-Walled Cylinders
Torsion
He.lical Spring

. . .I,

Tue

,f'

Theory

-"""

Problems

0
0

Solutions

Notes

~
Wed

0
0
0

Thu

Fri

885: ME Board April 199ft


If the ultimate shear strength of a steel
plate is 42,000 psi, what force is
necessary to punch a 0.75-inch diameter
hole in a 0.625 inch thick plate?
A.

B.
C.
D.

What force is required to punch a 1/2-inch


hole on a 3/8 thick plate if the ultimate
shearing strength of the plate is 42,000
psi?
A.
B.
C.
D.

A.

B.
C.
D.

12.75 mm
12.. 57 mm
17.75 rnm

15.75 mm

88:&: ME Board April1998


A force of 10 N is applied to one end of a

10 inches diameter circular rod. Calculate


the stress.

A.
B.

0.20 kPa
0. '15 I<Pa

C.
D.

0.051{Pa
0.10 kPa

24.~0

24,620
24,960
24,740

B.
C.
D.

11.77 mm
13.18 mm
10.25 mm
12.6 mm

1!:

891: EE Board October I990


A water reservoir of 24m high and 12m in
diameter is to be completely filled with
water. Find the minimum thickness of the
reservoir plating if the stress is limited to
50 MPa.

887: A single bolt is used to lap joint two

steel bars together. Tensile force on the


bar is 20,000 N. Determine the diameter of
the bolt required if the allowable shearing
stress is 70 MPa?

B.
C.
D.

!\.
ll.

has an outer diameter of 200 mm and is


subjected to a force of 74 kN. Find the
thickness of the wall if the allowable
compressive stress is 10 MPa.

890: EE Board Apri11996


A cylindrical water tank is 8 m in diameter
and 12 m high. If the tank is to be
completely filled, determine the minimum
thickness of the tank plating if the stress is
limited to .40 MPa.

A.
88&: ME Board October 1995

4.09 in~hes
3.96 inches

24.5 mm
28 mm
21 mm
26mm

89:t: EE Board April 1995

Sat

88lU IECE Board November 1998


An iron column of annular cross-section

63,000
68,080
61,850
66,800

C.
D.

883: ME Board April 199ft


A steel tie rod on bridge must be made to
withstand a pull of 5000 lbs. Find the
diameter of the rod assuming a factor of
safety of 5 and ultimate stress of 64,000
psi.

A.
B.

C.
D.

0.75
0.71

0.84
0.79

884: EE Board October 1996


Determine the outside diameter of a hollow
steel tube that will carry a tensile load of
500 kN at a stress of 140 MPa. Assume
the wall thickness to be one-tenth of the
outside diameter.
111.3 mm
B. 109.7 mm
C. '113.7mm
D. 112.4 mm

A.

c.
ll

17 mm
18 mm
19 mm
20mm

The stress in a 90-cm diameter pipe


having a wall thickness of 9.5 em and
under a static head of 70 m of water is

A
888: EE Board October 199&
What is the stress in a thin-walled
:;pherical shell of diameter D and a wall
thickness t when subjected to internal
pressure p? .

!\

S = D/pt

II.

s = 4D/pt

c:

S = pD/4t
S = pD/t

()

889: ME Board April I998


ornpute the safe wall thickness of a 76.2
diameter steel tank. The tank is
.cci>wr.ted to 7.33 MPa pressure and the
.1:1 cn<Jterial has a yield stress of 215.4
Ml 'd !he factor of safety to use is 3.

Ill

!\
II

I I 1/ inches
I W) 111dws

B.
C.
D.

ill:

325 kPa
32.5 kPa
32.5 MPa
3.25 MPa

II
[,

893: ME Board October 1994


A cylindrical tank with 10 inches inside
diameter contains oxygen gas at 2,500 psi.
Calculate the required thickness in mm
under a stress o.f 28,000 psi.
A.
B.
C.
D.

11.44
11.34
10.60
10.30

894: ME Board April 1:995


A solid shaft 48.2 em long is used for a
transmi,ssion of mechanical power at a rate
of 37 kW running at 1760 rpm. The stress
is 8.13 MPa. Calculate the diameter.

II

'I~!
ll;

520 :100 l Solved Problems in Engineering Mathematics ~2nd Editio~:l_!~fl~q & Roj~

l~~l

IJil,

g
.....'"'J

" . '1
UHJIIill

A.
B.
C.

D.

r
---

A.

B.
C.
D.

1.512 x 10-s m4
1.215x10-am4
1.152x10-em4
1.125 x 1o-e m4

A.
B.
C.

D.

200 GPa
180.32 GPa
148.9 GPa
106.48 GPa

895: ME Board October 1995

9001 ME Beard April 1:997

90S: EE Board Aprii19C)6

What is the modulus of elasticity if the


stress is 44,000 psi and unit strain of
0.00105?

What power would a spindle 55 mm in


diameter transmit at 480 rpm. Stress
allowed for short shaft is 59 N/mm 2

A cylinder of diameter 1.0 em at 30C is to


be slid into a hole on a steel plate. The
hole has a diameter of 0.99970 em at
30C. To what temperature the plate must
be heated? Coefficient of linear expansion
for steel is 1.2 X 10" 5 em/" C.

A.

B.
C.

....,,J,

30mm
35mm
40mm
50mm

Day 21 -Strength of Materials 521

D.

41.905 X 106
42.300 x 106
41.202x106
43.101 X "106

8961 ME Board October :1995


A 2-inch solid shaft is driven by a 36-inch
gear and transmits power at i 20 rpm. If
the allowable shearing stress is 12 ksi,
what horsepower can be transmitted?
A.
8.
C.
D.

29.89
35.89
38.89
34.89

sen: ME BOard October :199/i


A hollow shaft has an inner diameter of
0.0:-15 m and an outer diameter" of 0.06 m.
Compute for the torque in N-m, if the
stress is not to exceed "120 MPa.
A.

B.
C.
D.

4500
4100
4300
4150

8<~JI!I$ ME Board October 1996


Compute the nominal shear stress at the
surface in MPa for a 40-mm diameter shalt
that transmits 750 kW at 1500 rpm. Axial
and bending loads are assumed negligible.

A.
B.

c.
0.

218
312
232
380

A.
B.
C.

D.

42.12 kW
50.61 kW
96.88 kW
39.21 kW

A.

B.
C.
D.

90: A 30-m long aluminum bar is


subjected to a tensile stress of 172 MPa.
Find the elongation if E = 69,116 MPa?

62C
65C
48C
55C

906: EE Board April :t99S


A.
B.
C.
D.

An iron steam pipe is 200 ft long at 0C.


What wi.ll its increase in length when
heated to 100C? Coefficient of linear
expansion is 10 X 1o-s ft/"C.

0.746 m
0.007 m
6.270 mm
7.46 em

90:&: EE Board October 1'996


A steel wire is 4.0 m long and 2 mm in
diameter. How much is it elongated by a
suspended body of mass 20 kg? Young's
modulus for~steel is 196,000 MPa.

A.
B.

c.
D.

0.18 ft
Q. 12ft
0.28 ft
0.20 ft

9071 ECE Board November 1996


A.

1.123 mm

B.

1.385 mm
1.374 mm
1.274 mm

C.
D.

A simple beam 10 m long carries a


concentrated load of 200 kN at the
midspan. What is the maximum moment of
the beam?

903: A steel wire is t} m long, hanging

A.
B.
C.
D.

vertically supports a load of 2000 N.


Neglecting the weight of the wire,
determine the required diameter if the
stress is not to exceed 140 MPa and the
total elongation is not to exceed 4 mm. E =
200,000 MPa.

908: ME Board Oc:tober 1993


A beam supported at both ends and
carrying a uniformly distributed load:

A.

3.4 mm

B.

4. 4 mm

A.

C.

4.26 mm
5.4 mm

B.

D.

899: ME Boa!l'd O~tobe~ 1lHS

904: A copper rolled wire 10 m long and

A hollow shaft has an inner diameter of


0.035 mandan oute.r diameter of 0.06 rn.
Determine the polar moment of inertia of
the hollow shaft.

1.5 mm diameter when supporting ,,.


weight of 350 N elongates 18.6 rnrn.
Compute the value of the Youn!ir:.
.modulus of this wire.

250 kN-m
500 kN-m
400 kN-m
100 kN-m

C.
D.

has its maximum bending moment at


the supports
has its maximum shear at the center
of the beam
has its maximum shear at the
supports
has uniform shear throughout the
length of the beam

909: A simply supported beam, 10 m long


carries a uniformly distributed load of 20
kN/m. What is the value of the maximum
shear of the beam due to this load?
A.
B.
C.
D.

250 kN
100 kN
1000 kN
500 kN

910: A simply supported beam, 10 m long


carries a uniformly distributed load of 20
kN/m. What is the value of the maximum
moment of the beam due to this load?
A.

B:
C.
D.

10,000 kN-m
5,000 kN-m
2,000 kN-m
250 kN-m

11

....

.
H
l~~ll

1!!!111
liiiiiill

= 0.2 m

p
cr =kA
where: k = factor of safety

,.

Topics

D
D

-"""

,......w~ll'~~

lvlon

...111l"
.,.,.,1116

f:

Day 21 -Strength of Materials 523

liiilll

'll".

r,\:
.

Tue

_.,,J,,

(g

_ ...Ill

Wed

Theory

D D
D
D D

64000 = (5)5000

Strength of Materials
Simple Stress
Types of Normal Stress
Simple Strain
Hooke's Law
Stress-Strain Diagram
Thermal Stress
Thin-Walled Cylinders
Torsion
Helical Spring

A
A= 0.3906 in2

A=~d
4

=75000

10 X 106

....

A= 0.0075 m2
A= ~0 2 -~d 2

4
4

~d
4

4
d = 0.71 in

liiiill

0.0075 = ~(0.2)

0.3906 =~d 2

Thu

Problems

Uli :

!1

cr=~

d=0.1745 m

Fri

Solutions

Notes

Solving fort:
D = d+2t

Sat

0.2=0.1745+2t

881. A
882. A
883. 6
884. D
885.C
886. D
887.
888.
889. 6
890.A

c
c

891. 6
892. D
893. 6
894. D
895.A
896.6
897. A
898. D
899. D
900.C

901. D
902.0
903. 6
904. D
905. D
906. D
907.6
908.C
909. 6
910. D

.....

RATING

ANSWER KEY

t = 12.75 mm

[:} 18-24 Passer

140 X 106 = 500000

d= 10

15-17 Conditiona.l

.
In

1ft X 1m
- . .
x 12 in 3.281 ft

d=0.254

[:} 0-14 Failed

If FAILED, repeat the test.

7t 2 7t (
A= -:0
- - D- 2t )2

=A

cr=

~d2

A =0.282702
10

cr=---

0.'00357 = 0.2-8270 2

~(0.254)2
4

D = 0.1124 m

cr

7t(
A= -D 2 - D-2(0.10) )2
7t

A
A =0.00357 m2
Note: t = 0.1 D

cr

cr=~
A

liiilill

[:} 25-30 Topnotcher

t = 0.01275 m

= 197.35 Pa

"' 0.2 kPa

D;, 112.4 mm

lill, 1'\11'
1

Day 21 - Strength of Materials SZS

524 100 l Solved Problems in Engineering Mathematics (2nd Edition) by Tiong & Rojas

'"'

t = 3.89 mm

-1''
.....,,,,

Ill

p = 61850 lb

l!tiB

liiiiilll

p = 9810(70)

cr,

p = yh

2cr1
p

2(40

.10

t = 0.4464 in x 2.54. em X10


mm
--

ln

where d = diame~er of the bolt


70 X 106 = 20000

~(d2)
d=0.019m
d=-19mm

Ill
pD

pD
crL = 7 Formula
4t

p = 2nfT
60

Note: The biggest pressure occurs at the


bottom of the tank

37000 = 2n(1760)T
60
T = 200.75 N-m

p = yh

7td3

8.13 X 10& = 16(200.75)

where: k = factor of safety

120 x 106 =

em

d=0.050 m

d=50 mm

2(.50 X 10 )

t = 0.028 m
t= 28 mm

16TD
n(D4 - d")
16T(0.06)
7t[(0.06)4 - (0.035)4

1!9!!11
liii6ia
p

= 27tfT
60

750000
.

27t(1500)T
60

16T
t=-

7td3

7td3

t = _23,...-54_4_0(,_12_,_}

crt=kpD
. 2t

t-

16T
t=-

2cr1

liiiiilll

33000(12)
p = 35.89 hp

T =4774.648 N-m

p = 235440 Pa
t = pD

lf!I!R

2nfT.
33000(12)

T =4500 N-m

1m

2t

p = 9810(24)

liiiiilll

t = 11.34 em
CJr =

I!

p = _2n~(1_2-'0)'-'--(1_8_84_9_.55.....!._)

28800- 2500(10)
2t

t = 11.77 mm

~(d2)

16
T
3
n(2)

T = 18849.551b-in

pD

t = 0.01177 m

p
CJ=--

686700(0.9}
2(0.095}

CJr = 2t

t = _1,.-17_72_0_,_(8-::'-;)
CJ = -

12000 =

Ill

t = pD

J!IP.II!W
Uilll

2t

cr1 = 3.25 MPa

p = 117720 Pa

p = 24740 lb

nd3

cr1 = 3252789.474 Pa

p = 9810(12}

p
42000 = n(0.5)(0.375)

E=41.905x106 Pa

= pD
I

Note: The biggest pressure occurs at the


bottom of the tank

II; I:

16T
t=-

cr

p
CJ=-

1::

liiiill

p = 686700 Pa

cr, = pD
2t

p
42000 = n(0.75)(0.625)

1.:"

E = 44000
0.00105

..,.

p = yh

1m

CJ = -

1111

E=~

Note: The biggest pressure occurs at the


bottom

t = 0.0389 m
-~

'

]I' :'

&

2t

Cl:
--~~~~~~-

=21

215.4 X 106 = 3(7.33 X 10 )(0.762)

'~~-:

.rill

Ill

pD

CJr
6

~,

t~~JOIIII

11.1

p
A

CJ=k-

111

I
I

t=

16(4774.648)
3

7t(0.04)
t=380 MPa

!,:Ill!'

..

SZ& :100 1 Solved Problems in Engineering Mathematics (2"d Edition) by Tiong & Roip

'"'"

'"
H

Ill

I)=

16T
7td3

t=-

59=~3
n(55)

=1927391.637 N-mm

T = 1927.39N- m

15 = 1.274 x 10-3m
15 = 1.274 mm

140

ID

d =0.00429 m
d=4.26mm

AE
p
aS=A

AE
where: P = weight of the body
P mg 20(9.81)
P 196.2 N

= =
=

2000(6)

d = 0.0044 m
d= 4.4 mm

15 = aL

E
8
. I)= (172 X 10 )(30)
69,116x106
8 =0.0746 m
&:7.46 m

Note: To be safe for both stress and


elongation, use d = 4.4 mm.

E = 106.48 MPa

ill

:':ll

10m

HI:

R2

L:Fv =0
R1 + R2 = 200
2R1 = 200
R1=100kN

Maximum shear= 100 kN

Refer to solution in Problem # 909:


Consider the half of the beam:

Consider the half of the beam:

max. moment= R1(2.5)

1:11111
.,

Iiiii

R1=100 kN

max. r:noment = (100)(2.5}


max. moment = 250 kN-m

:;- ::fc
1

max. moment= LMc


max. moment= R1(5)
max. moment== {100)(5)

E = 1.0648 x 1011

R1

max. moment= LMc

~::::::::.:

~(0.0015)2

Note: Maximum'tnoment.of a
symmetrically loaded system is at
midspan.

200kN

l>= PL
AE
350(10)
.....-0.0186 =
E

]I!

20(10)=200 kN

20(5)=100 kN

R1 + R:;i = 200
2R1 = 200
R1=100kN

~d2 (200000 X 108)

Substitute Eq. 2 in Eq. 1:

15 = PL

LFv =0

AE

0.004

. ;r~~ ; :=;;~;t

Note: Since the load is at midspan, then


the reaction at both supports must be
equal.

15 = PL

Eq.2

,I'!"1
'I

fi~J:P!Jj

t 1)

200kN

t'~:::.

Strength of Materials 527

Maximum shear of a symmetrically


ioaded system is equal to the reaction at
the supports.

01 = 10 X 10-6(200)(100 -0)
81 = 0.20 ft

106 = 2000
!:d2

-+ Eq. 1.

Or= aL(t 2

Ill

Considering limitations of elongation:

l5 = PL

t 1)

t 2 = 55C

196.2(4)
3.1416 X 10--6(196,000 X 108 )

=~ltfr

=aL(t2 -

0.0003 = 1.2 X 10'5 (1)(t 2 -30)

Considering limitations of stress:


p
y=-

60
p = 21t(480)(1927.39)
60
P=96.88 kW

81

A=3.1416x10.s m2

J=1.125x10-6 m4

ST = 1- 0.99970

81 = 0.0003 em

A = *(0.002)

Day 21

A=~d 2
4

J = _!t_(D4 - d4)
32
J= ;2[(0.06) -(0,035}

~~

Pa

max. moment

500 kN-m
l

I
11

'f'l
530 100 f Solved Problems in Engineering Mathematics (2nd Edition) by Tiong & Rojas

~I

' "'.'"'
q,l::

'

'

'!"

'

.,.

'*

< '

'""
~~~:~11',11

Topics

'

~ ~

., ~ '
~

,A<,.<.:>.<

'

< > " <. ' ,.... "',' ,>

<- '

< "

r-1

~ ,, ""' <\ <

.< <f "'"""

>'

'*' ,..... ,

< .> "

~. ~

,,

..,

N""

;-! '<

""~ ,... "

,, :

'

~....

~ "<'

~ ;..

~-'".~t~><'r'olc.~~

L_J
Tue

!" '0

'< ., '<: '>

::>

' <

O.~.j.oi~t*

'''''

>: ...

'

,; "'

"'"'".'-~ ''':~'

Theory
~ ~ ~>"' "'.~ "~

.,:..

~ ..... ~ "1:""' :.,.

1< ~:>

'< ,~ r, ~

>'

<'i- -~ ~ It- ~ "" "'.,.. # 4 .~.:~

'i' .. '

'<

~ '!' ';-.,. ~.

~"'

~ ~ ~

... "",1

...

A-,, . . , ~ *' ,.. ~ .,.

,<:<>- ~

..... -~ ... ,.,..

'='. >\' ~ 1

i 't . "

~ ,.. .. ,

;< '!

~ ~ ~

""""

'I' -t' :~ .,. -< oi' ,;

:""' ~- ,., "' ~

'<

,. .....

,, ,.

< .,...

i.

,; ,..

[--]
[-]

>t "!'""'

Solutiom;
~

< ........ ;, :,. :::. ~ ,, .,; ,.

<o

'l ,. ~~ 4 ~ ;

~ ' ,...-._. :!<- ,:

~ ~

':;' ... II'

___J

0 < It.

'

Jo

hu

Problems

;._.,

,. ...... ~ ;-;' :<

Wed

-1

,..,..,.,.,..,.,,.,.."!!~'~'~

t ,.

1'1 ~

1, "" ~ " .. ~

;< "' .., "['<;' .,

I(""

""'"'..'!",.,~-~.,..~

.<: ;;.-

<)-_.., . ,.,_ '

--,

[__J

Notes

:::.

[=]
Fri

l~]
Sat

~ ~ if; -4

<: ""

l ' _>.

*" ~

-~

., '

,. ~

.<.'d

~ ~ ~

.,

<

"y->' I

<

o-~:>"<<'<".,'f,o:_

1<

,; '"' .;. ,,. ' '

' .. >i

., "\>

< "

-~

...

-~ ~

..

"":- ._

~"""<jo.h<<i~:O..,.;.w

... -~1,.,,,..-,_.,.-,/<-";:<jo..'!'~~~
"' ' .... ~

+ ,, -.; <". '" ,..

'> ,

,. "' ~ ,, ) ( f.

~<v<-.~~o<.<'<-~~.<-_,._r.,{,

~%._

"' !II ........ .,,.,_,.,;

1;-

.;..~~-,,-~,.;->:

~-

t( ,_

--.

E~:onomics is a sc1ence which deals with


!he attainment of the maximum fulfilment
of society's unlimited demands for goods
and service.

V.1:!J!1Js Engineering

Economics?

..,~~-

i .. I>.;-_... < '- ,, .\ " ' ' ;.,

'!'

Engineering Economics is the branch of


t!conomics which deals with the
.1pplication of economics laws and theories
IIJVolving engineering and technical
r 11 ojects or equiprnents.
IIVIJ<l(i!_~_G_on~umer and Producea:s
! :.cJ_ot!~!!!l.ci_Services(

,.

', _.

!i

II

IIi

Compound Interest
Continuous Cornpounding
Nominal and Effective Rate of
Interest

-~ <1 ... "

.w.!at is E_@.!lOmics?
~

Engineering Economics
Consurner & Producer Goods
and Services
Necessity and Luxury
Different Market Situations
Demand
Supply
law of ~upply and Demand
Simple
-Ordinary Simple interest
- Exact Sirnple Interest

consumer goods and services refer to


1/w 1>1oducts or services that are directly
11:.d l1y pt~opl" to satisfy their wants.
I .. JIIlpl, .. ; :lit' food. dotltlrH), shc!ter or
lit liiH' l"IJ

Producer goods and services are those


that are used to produce the consumer
goods and services.

What.l.l.b!L.Q1fi~I!LIJCe betwemJ.

!'!feces*illY...!nr!_Lu~.!Y1

'

Goods and services are divided into two


types, namely necessity and luxury.
Necessity refers to the goods and
services that are required to support
human life. needs and activities.

Necessity product or staple product is


defined as any product that has an
income-elasticity of demand less than one.
This means that as income rises,
proportionately less income is 'spent on
such products. Examples qf necessity
products include basicfoodstuffs like
bread and rice, clothing, etc.

I,
' 'I~

"

;I

If!

'rT

Day_2_g- Engtneering Economics (Simple and_ Compoun<i_lnterest) 533

532 l 00 i Solved Problems in Engineering Mathematics (2nd Edition) by Tiong & Rojas

l,illt,,,
1.11.,1111

~.,,',,

'

jljll

Luxuries are those goods and services


that are desired by human and will be
acquired only after all the necessities have
been satisfied.

The price of any commodity or product will


depend largely on the market situation.
The following is a tabulation of the different
market situations:

r,,.

_.,.J

Luxury product is defined as any product


that has an income-elasticity of demand
greater than one. This means that as
income rises, proportionately more income
is spent on such products. Examples of
luxury products includes consumer
durables like electric appliances,
expensive cars, holidays and
entertainment, etc.
Necessities and luxuries are relative terms
because there are some goods and
services which may be considered by one
person as necessity but luxury to another
person.
For example, a man living in the
Metropolis finds a car as an absolute
necessity for him to be able to go to his
workplace and back to his home. If the
same person lived and worked in another
city, less populated with adequate means
of public transportation available, then a
car will become a luxury for him.

A.

Few seller and many buyers: The


bulk of market supply is in the hands
of a relatively few sellers who sell to
many small buyers.

B.

Homogeneous or differentiated
products: The products offered by
the suppliers may be identical or more
commonly, differentiated from each
other in one or more respects. These
differences may be of a physical
nature, involving functional features,
or may be purely "imaginary" in the
sense that artificial differences are
created through advertising ahd sales
promotion.

C.

Difficult market entry: High barriers


of entry which make it difficult for new
sellers to enter the market.

Monopoly is the opposite of perfect


competition. This market situation is
characterized by the following:
Perfect competition (also known as
atomistic competition) refers to the
market situation in which any given
product is supplied by a very large number
of vendors and there is no restriction
against additional vendors from entering
the market.
Perfect competition is a type of market
situation characterized by the following:

A.

What are the Different Market.


Situations?
The term "market" refers to the exchange
mechanism that brings together the sellers
and the buyers of a product, factor of
production or financial security. It may also
refer to the place or area in which buyers
and sellers exchange a weli-defined
commodity.

B.

Buyer or consumer is defined as the


basic consuming or demanding unit of a
commodity. It may be an individual
purchaser of a good or service, a
household (a group of individuals who
make joint purchasing decisions), or a
government.

C.

Seller is defined as an entity which makes


product, good or service available to buyer
or consumer in exchange of monetary
consideration.

Absence of all economic friction:


There is a total absence of economic
friction including transport cost from
one part of the market to another.

This market situation provides an


assurance of complete freedom on the
part of both the vendors and the buyers
though the latter benefits more from the
reduced prices brought about by the
competition while more and better services
are afforded by the vendors or players in
the industry .

1~11

..... 1111111

E.

D.

Many sellers and many buyers:


Since there is a large number of
selleos and a large number of buyers,
each seller and buyer will become
sufficiently small to be unable to
influence the price of the product
transacted.
Homogeneous prodm:ts: The
products offered by the competing
sellers are identical not only in
physical attributes but are aiso
regarded as identical by the buyers
who have no preference between the
products of various producers.
Free ma1ket-entry and exit: There
are no barriers to entry or
impediments to the exit of the existing
sellers.

Perfect information: All buye!s am!


ail sellers have complete inforrn''il.ion
on the prices being asked and otlemd
in all other parts of the mmlwt

One seller and many buyers: A


market comprised of a single supplier
selling to a multitude of small,
independently-acting buyers.

B.

Lack of substitute products: There


are no close substitutes for the
monopolist's product.

C.

Blockaded entry: Barriers to entry


are so severe that it is impossible for
other sellers to enter the market.

There exists a perfect monopoly if the


single vendor can prevent the entry of all
other vendors into the market. The
monopolist is in the position to set the
market price.

What is a Demand?
Demand is the need, want or desire for a
product backed by the money to purchase
it. In economic analysis, demand is always
based on "willingness and ability to pay"
for a product, not merely want or need for
the product.
The demand for a product is inversely
proportional to its selling price, i.e. as the
selling price is increased, there will be less
demand for the product; and as the selling
price is decreased, the demand will
increases. Figure 1 below illustrates the
relationship between price and demand.

A natural monopoly is a market situation


where economies of scale are so
significant that costs are only minimized
when the entire output of an industry is
- supplied by a single producer so that
supply costs are lower under monopoly
than under perfect competition and
oligopoly.
Oligopoly exists when there are so few
suppliers of a product or service that the
act1on of one will inevitably result in a
~imilar action by the other suppliers. This
tyrt~ of mark0t sJtuation is characterized by
till' loiiOWIII<J

Price

. .:
jp1

........................ !

6~

!P2
!

Figure 1

Demand

I':,I
'-~

.!1..Jgo-I Solved Problems in .Engineeri;l!~athematics (2r.d Editionl.PY 'I'iong & ~jas

'.,,,,,,
"~ ' "

!l.,l,

.lljil

rwh

Assuming a linear function for the


relationship between price and demand, it
shows that at point 1, the selling price, P,
is high, thus there is less demand for. the
product as compared to point 2, where 'the
demand, Dz is great because of lower
selling price, Pz.
What is a Supply?
Supply is the amount of a product made
available for sale.
If the selling price for a product is high,
more producers will be willing to work
harder and risk more capital in order to
reap more profit. However if the seiling
price for a product declines, capitalists will
not produce as much because of the
smaller profit they can obtain for their labor
and risk.
Therefore, the relationship between price
and supply is that they ,are directly
proportional, i.e. the bigger the selli11g
price., the more the supply; and the smaller
the selling press, the less is the supply.
Figure 2 below illustrates the price-supply
relationship.

Price

. . . . . . . . . . . . . .Szf/
.. / l

~1

p2

"Under conditions of perfect competition,


the price at which any given product will be
supplied and purchased is the price that
will result in the supply and !he demand
. being equal."
The relationship between price-supplydemand may be illustrated by combinirlfj
figures 1 and 2. For general application,
the curves for supply and demand are no
longer represented in linear function.

_ _

What is the Law of S!ill.P_tl and


D_emand?
The law of supply and demand is stated a~
follows:

When there is additional demand without


an additional supply, a new and higher
price is established as shown in Figure 5.
Price

~~'!.I

d
n=-

360

.....

The simple interest is:

...

tl....---;::-------

uUOOIV

=Demand

l
"' Demand
,.,! ------Units

Figure 3 illustrates the price-supplydemand relationship, showing equal


supply and demand at a given price.
When there is additional supply without an
additional demand, a new and lower price
is established as shown in Figure 4.

Previous
Supply

New
Supply

~~JJ

._ _ _ _ ,.

t---- -------Figure 4

Figure 5

What is an Interest?

Figure 3

Price

... Previous
Demand

Units

!'""'

r,t/~

Units

Ordinary simple interest is based on one


banker's year. One banker's year is
equivalent to 12 months of 30 days each.
Also, 1 banker's year = 360 days.
The value of n is:

Price I

~Supply

Figure 2

Day 22 - Engineering_J::<.:O_I\OIDi<:._@J!lj)l_e and Compound Interest) 535

Price

/
!pj

::f

Suppose a debtor loans money from a


creditor. The debtor must pay the creditor
the original amount loaned plus an
additional sum called interest. For the
debtor, interest is the payment for the use
of the borrowed capital while for the
creditor, it is the income from invested
capital.
What is Simple Interest?
Simple interest (I) is defined as the
interest on a loan or principal that is based
only on the original amount of the loan or
principal. This means that the interest
charges grow in a linear function 'bver a
period of time. It can be calculated using
the formula
I= Pin
where: P = principal
i = interest per period
n number of interest period

!lil

',_p
d
- 1-

'I

360

Exact simple interest is based on the


exact number of days in a given year. A
normal year has 365 days while a leap
year (which occurs once every 4 years)
has 366 days. Unlike the ordinary simple
interest where each month has 30 days, in
the type of simple interest, the number of
days in a month is based'on the actual
number of days each month contains in
our Gregorian calendar.

.iII
' II
,il
I I

:ji
]':

:i:!

:1111~11111

II;,
,,
,.1
']''
:;

li

''I
To determine the year whether leap year
or not, one has to divide the year by 4. If it
is exactly divisible by 4, the yearis said to
be leap year otherwise it will be
considered just a normal year with 365
days. However, if the year is a century
year (ending with two zeros, e.g. 1700,
1800 ... ), the ye;3r must be divided by 400
instead of 4 to determine the year whether
or not a leap year. Hence, year 1600 and
year 2000 are leap years.
Under this method of computation of
interest, it must be noted that under
normal year, the month of February has 28
days while during leap years it has 29
days. Again, the values of n to be used in
the preceding formulas are as follows:

il

'II
!I
,,

:'1'
I

.II

For ordinary or normal years:


There are two types of simple interest,
namely ordinary simple interest and exact
~;irnple interest.

d
n = 365

II I

"li
,,

,,,

:,y.

,,

,)

Day 22- Engineering_~~nomics (Simple and CompoundInterest) 537

536 , 100 l Solved Problems i~ E!!,qineering Mathematics (2nd Edition). by Tiong &: Rojas

'"'"' "

'.......' t'l

d
n= 366

"'~:
"""

lllllll~li'l

:,

-1""'
i'ltiiiiH

l~i

...J.J
_.,.rJ''

What is a Discount?
Consider the following case:; wnere
discount is involved:

Compound interest is defined as the


interest of loan or principal which is based
not only on the original amount of the loan
or principal but the amount of loan or
principal plus the previous accumulated
interest. This means that aside from the
principal, the interest now earns interest as
well. Thus, the interest charges grow
exponentially over a period of time.

Using the formula for nth term of a G.P.


an= atfn-1
an = P(1 + i)(1 + i)n-1

CASE 2:

Discount "" Future worth~ present worth

'
The rate of discount is the discount on
one unit of principal per unit time.

Principal

Interest

Pi

P(1 + i)

P(1 + i)i

-2

P(1 + i) 2

P(1 + i)2 i

,,,,, ''.

Total
Amount
P+ Pi=
Pl1 + i)
P(1 + i)(1
+ i) = p
(1 + i)2
P(1 +
i)2 (1 + i)
=P(} +

let x =..!!!..

NR

p( +

F=

"

1 .;rNR)N

I
tt,l''''
,,

F=P[(1+.;JrR)N

but

Lim (1 +
x~oo

.!)x
=e
x

therefore,

B.

The tabulation above shows that the future


amount (total amount} is just the value P(1
+ i) with an exponent which is numerically
equal to the period.

PRESENT WORTH, P

.... I
0

It is also observed that compound interest


is based on the principles of geometric
progression and using such method, the
total amount after each period are as
follows:
a,= P(1 + i)
First term,
2
Second term, a2 P(1 + i)
3
Third term,
a3
P(1 + i)

.......................................... :

. F ,.,p~<"~
:~;~;/,f

f:t:r

=principal
NR =nominal rate

where: P

e =2.71828 ...

= number of years
= ccintinuous compounding

N
e<NRJN

compound amount factor

P-

- (1 + i)n

a ,

NR)mN

F=P ( 1+m

''~

where: P = Principal
i = interest per period (in
decimal)
n = number of interest periods
(1 + i)n = single payment compound
amount factor

J2

r = -1.
a,

but for !Tl periods per year

w~ere:

Solving for the common ratio,

d
'1=:::-'1-:-d

..,,, ,, ,

F = P(1 +i)"

F =P(1+it

and so on.

Cash Flow

j..

P(1~

t:;i~,:~:J ....................................... ~'1f};~:

Let d = rate of discount

1
d=1-- thus
1+ i
'

I... .
0

The future amount of the principal may be


derived by the following tabulation:

In a bank loan, a person borrows P 100 for


1 year with an interest of 10%. The interest
as computed is P 10. The bank deducts
the interest, which is P10 from P 100 and
gives the borrower only P 90. The
borrower then agreed to repay P100 at the
end of the year. The P1 0 that was
deducted represent the interest paid in
'advance. In this case, discount also
represent the difference between the
present worth (i.e. P90} and the future
worth (i.e. P 100}.

FUTURE AMOUNT, F:.

Compound interest is frequently used in


commercial practice than simple interest,
more especially if it is a longer period
which spans for more than a year.

Period

The basic equatioh for future worth of


compound interest is

an =P(1+i)n

A.

that is not due yet but payable in some


future date, desires to exchange it into an
immediate cash. In the process, he will
accept an amount in cash smaller that the
face value of the bond. The difference
between the amount he receives in cash
(present worth) and the face value of the
bond or financial security (future worth) is
known as discount. The process of
converting a claim on a future amount of
money in the present is called
discounting.

The concept of continuous compounding rs


based on the assumption that cash
payments occur once per year but
compounding is continuous throughout the
year.

r = 1+i

CASE 1:

A person has a bond or financial security

What is Continuous Compounding?

P(1 + i)2
r=-P(1 + i)

What is Compound Interest~

For leap years:

( : i)" = single payment present


1
worth tactor

The present worth of continuous


compounding is

P'iii~~~(Nl

il'll
ill'

l'"li''!

!wtl

4:"1 "'

,.,\1::

1
:j
538 1001 Solved Problems ii). Engineering Mathematics (2nd Edition) by Tiong & Rojas

,,,,,,,,,

What is the Difference Between


Nominal and Effective Rate of Interest?.

'~
"' "'

Rate of interest is the cost of borrowing


money. It also refers to the amount -earned.
by a unit principal per unit time.

nominal rate are equal if the mode of


compounding is per annum or annually.
I

,,,

,,

'Ill''"' '

~II

'

1rn:1

~II

,,._~1~~~

-,~.~~~~.'

-~

1~1

_JJ

_fj

There are two types of rates of interest,


namely the nominal rate of interest and the
effective rate of interest.
Nominal rate of interest is defined as the
basic annual rate of interest while
effective rate of interest is defined as the
actual or the exact rate of interest earned
on the principal during a one-year period.
For example: A principal is invested at 5%
compounded quarterly,
In this statement, the nominal rate is 5%
while the effective is greater than 5%
because of the compounding which occurs
four times a year. The following formula is
used to determine the e.ffective rate of
interest:

ER = [1+it-1
where: m = number of interest period per
year
i = interest per period
. NR
1=-

Iii'

~.;,

..,
..
'',,,
...

.. i

Proceed to the next page for your 22nd


test. Detach and use the answer sheet
provided at the last pa.rt of this book. Use
pencil number 2 in shading your answer.
GOOD LUCK I

Mon

'Urribia:

Theory

ER=[1+NmRr-1

Substituting the values of m and i:

ER =[1+

ER =0.0509
ER =5.09%
So, the actual interest rate is not just 5%
but 5.09%. However the effective rate and

0 I
0
Sat

'I'

0
0

Fri

II

Ii
'I

II!
l

ME Board April 1.995


P 4,000 is borrowed for 75 days at 16%

C}U:

per annum simple interest How much will


be due at the end of 75 days?

P4,133.33
P 4,333.33
p 4,166.67
P 4,150.00

9I::t: CE Board May 1.997


fl deposit of P 110,000 was made for 31
days. The net interest after deducting 20%
withholding tax is P 890.36. Find the rate
of return <:mnually.

0 5

~ J -1

.1il

D.
Note: i = NR if the mode of
compounding is annually

Consumer & Producer Goods


and Services
Necessity and Luxury
Different Market Situations
Demand
Supply
Law of Supply and Demand
Simple Interest
- Ordinary Simple Interest
-Exact Simple Interest
Discount
Compound
Interest
1
Continuous Compounding
. Nominal and Effective Rate of
I Interest

~
Thu

Notes

-Goethe

I Engineering Econ~mics

,lli

Problems

Solutions

~uote:
"Mathematicians are like Frenchmen;
whatever you say to them they translate it
into their own language and forthwith, it is
something entirely different."

Wed

,_

A.
B.

or

0
0
0

Tue

Did you know that ... the Gregorean


Calendar we are using was named after a
former teacher of law at the University of
Bologna, Ugo Buoncompagni who became
Pope Gregory XIII in 1572! In February 24,
1582, he issued a Papal edict directing the
former Julian Calendar be allowed to catch
up with the Lord's Time and that aside
from leap year every four years, leap year
be once in every four centennial years, i.e.
every 400 years.

!1111

Topics

11.95%
12.75%
11.75 o;,,

IJ.

1/.?5 'X,

fl.
U.

913: ME Board Aprill993


Agnes Abanilla was granted a loan of P
20,000 by her employer CPM Industrial
Fabricator and Construction Corporation
With an interest of 6 % for 180 days on the
principal collected in advance. The
corporation would accept a promissory
note for P 20,000 non-interest for 180
days. If discounted at once, find the
proceeds of the note.
A.
B.
C.
D.

P 18,600
P 18,800
P 19,000
19.200

II

IIIII

,,,:I

91.4: ECE Board November 1.998


What will be the.future worth of money
after 12 months, if the sum of P 25,000 is
invested today at simple interest rate of
1% per month?
A.

p 30,000

11:

540 100 1 Solved Problems in Engineering Mathematics (2nd Edition) by Tiong & Rojas

,,,,,,./

B.

C.
D.

"'~"'

P 29,000
P 28,000
P 27,859

money was borrowed and the loan is


payable at the end of one year. How much
is the actual rate of interest.

~
'.,' '...

II''"'

'

9S5: ECE Board November S999

A.

If you borrowed money from your


friend with simple interest of 12%, find the
present worth of P 50,000, which is due at
the end of 7 months.

B.

C.
D.

~.~~~:,.

. ;li.

12%
14%
10%
19%

C.
D.

P46,200
P 44,893
P 46,730
P 45,789

9Sft: Annie buys a television set from a


merchant who ask P 1,250 at the end of
60 days. Annie wishes to pay immediately
and the merchant offers to compute the
cash price on the assumption that money
is worth 8% simple interest. What is the
cash price?
A.
B.

c.
D.

P 1,233.55
P 1,244.66
p 1,323.66
P 1,392.67

917: ME Board April1998


It is the practice of almost all banks in the
Philippines that when they grant a loan,
the interest for one year is automatically
deducted from the principal amount upon
release of money to a borrower. Let us
therefore assume that you applied for a
loan with a bank and the P 80,000 was
approved at an interest rate of 14 % of
which P 11 ,200 was deducted and you
were given a check of P 68,800. Since you
have to pay the amount of P 80,000 one
year after, what then will be the effective
interest rate?
A.
B.

c.
D.

15.90%
16.28%
16.30%
16.20%

p 408.00

B.

P415.00
P 551.00
P 450.00

C.
D.

D.

A.
B.

C.
D.

14.49%
12.36%
14.94%
14.88%

c.

D.

The amount of P 50,000 was deposited in


the bank earning at interest of 7.5% per
annum. Determine the total amount at the
end of 5 years, if. the principal and interest
were not withdrawn during the period?
A.
B.

930: ME Board April199~

c.
D.

7.71%
7.22
15.7a %,
21.81 %.

/:

92&: ECE Board April1998

The amount of P 12,800 in 4 years at


5 % compounded quarterly is
A.

19.25%
19.48%
18.46%
18.95%

P 71,781.47
P 72,475.23
P 70,374.90
P 78,536.34

An interest rate is quoted as being 7.5%


compounded quarterly. What is the
effective annu!'ll interest rate?

B.

B.
C.
D.

!II:
I,

P 14,785.34
P 15,614.59
P16,311.26
P 15,847.33

Alexander Michael owes P 25,000.00 due


in 1 year and P 75,000 due in 4 years. He
agrees to pay P 50,000.00 today and the
balance it) 2 years. How much must he
pay at th'ti"end of two years if money is
worth 5% compounded semi-annually?
A.
B.
C.
D.

'I'll
I

illli

,'I

II'

'I

,l'

};
''il

il

P 38,025.28
P 35,021.25
P 30,500.55
P 39,021.28

93S: ECE Board November 1998


At an interest rate of 10% compoundec:l
annually, how much will a deposit of P

1,500 be in 15 years?

927: ECE Board April1999


Find the present worth of a future payment
of P 100,000 tobe made in 10 years with
an interest of 1.2% compounded quarterly.

Mandarin Bank advertises 9.5 %account


that yields 9.84 % annually. Find how
often the, interest is compounded.

B.

P 6,500
P 8,600
P 5,500
P 7,500

D.

A.

3%
13.2%
12%
12.55%

922:ME Board October 1995,


EE Board October 1997

A.

A.
B.
C.
D.

C.

What is the corresponding effective rate of


18% compounded semi-quarterly?

D.

925: ME Board October 199&

Proble~

C.
918: EE Board April 199ft

The effective rate of 14% compounded


semi-annually is

921: ECE Board April!.999

A.
B.

inheritance as of the boy's 6 h birthday, if


the interest is compounded annually?
Assume i 4%.

929: ECE Board April!.999

920: EE Board October 1997

B.
C.
D.

4.06%
1.00%
2.04%
3.36%

924: ECE Board November 1998

A man borrowed P 100,000 at the interest


rate of 12% per annum, compounded
quarterly. What is the effective rate?

A.

A bank pays one percent interest on


savings accounts four times a year. The
effective annual interest rate is

c.

A bank charges 12 % simple interest on a


P 300.00 loan: How much will be repaid if
the loan is paid back in one lump sum after
three years?

A.

923: EE Board October 1993

A.
B.

919: ME Board April1998


A.
B.

Day 22- EnginE!E!ring Economics (Simple and Compound Interest) 541

d~

"I

~lrJ41{

1111

','?
. .. '

A.
B.

C.
D.

Daily
Monthly
Bi-monthly
Quarterly

P 30,444.44

B.
C.

p 33,000.00
P 30,655.68
P 30,546.01

D.

928: EE Board June 1990


On his Gu' birthday a boy is left an

A man borrowed P 20,000 from a local


commercial bank which has a simple
interest of 1So/o but the interest is to be
deducted from the loan at the time that the

11\herit.mce The inheritance will be paid in


51
;t lump swn of P 10,000 on his 21
lllrlhd<~y Wh<tl r~; tlu~ present value of the

~J

P6,100.0U
P 6,234.09
P 6,265.87
P 6,437.90

932: CE Board May !.995


How long (in year's) will it take money to
quadruple if it earns 7 % compounded
semi-annually?

A.
B.

c.
D.

20.15
26.30
33.15
40.30

II

542 l 00 1 Solved Problems in Engineering Mathematics {2"d Edition) by Tkmg & Rojas

Day 22 - Engin~~~!l9. Economics (Simple artd Compound Interest) 543

t,a,,,,.

,,,,,j

t1

l~

'I

~~.:1

9331 ECE Board April :1999

A.

P 1,925.00

In how many ye.ars is required for P 2,000


to increase by P 3,000 if interest at 12%
compounded semi-annually?

B.
C.

P 1,860.00
P 2,345.00

A.

B.

D.

c.

10

D.. P 2,160.00
938: CE Board May 2:996
P 200,000 was deposited on January 1,
1988 at an interest rate of 24 %
compounded semi-annually. How much
would the sum be on January 1, 1993?

934& ME Board April :1.996

Consider a deposit of P 600.00 to be paid


back in one year by P 700:00. What are
the conditions on the rate of interest, i%
per year cOmpounded annually such that
the. net present worth of the investment is
positive? Assume i 2 0.

A.
B.
C.
D.

0 :=; i < 14.3 %


0 :=; i < 16.7%
12.5% :=; i < 14.3%

A.
B.

P 401,170
p 421,170

C.

P 521,170
P 621,170

D.

939: CE Board November :1996


If P 500,000 is deposited at a rate of 11.25
% compounded monthly, determine the
compounded interest after 7 years and 9
months.

B.

P 660,550
P 670,650

A company invests P 10,000 today to be


repaid iq 5 years in one lump sum at 12 %
compounded annually. How much profit in
present day pesos is realized?

C.
D.

P 680,750
P 690,850

p 7,632
P7,236
P 7,326
P 7,362

936a ME Board April :1996


A firm borrows P 2,000 for 6 years at 8 %.
At the end of 6 years, it renews the loan
for the amount due plus P 2,000 more for
2 years at 8 %. What is the lump sum
due?

A.
B.
C.
D.

p
P
P
P

5,355.00
5,892.00
6,035.00
6,135.00

9371 ME Board October :1996


A deposit of P 1, 000 is made in a bank
account that pays 8 % interest
compounded annually. Approximately how
much money wilL be in the account after 10
years?

for eight years, at which time the principal


is withdrawn. The interest has accrued is
left for another eight years. If the effective
annual interest rate is 5 %, what will be the
1
withdrawal amount at the end of the 16 h
year?

at an interest rate of 7% compounded


semi-annually. How much is the sum now?

A.
C.
D.

p 706.00
P 500.00
P 774.00
P 799.00

943: ME Board April :1998


P 1,500.00 was deposited in a bank
account, 20 years ago. Today it is worth P
3,000.00. Interest is paid semi-anr1Ually.
Determine the interest rate paid on this
account.

B.

C.
D.

3%
2.9%
3.5%
4%

944: ME Board April :1998


Fifteen years ago P 1,000.00 was
deposited in a bank account, and today it
is worth P 2,370.00. The bank pays
interest semi-anf1ually. What was the
interest rate paid in this account?

A merchant puts in his P 2,000.00 to a


small business for a period of six years.
With a given interest rate on the
investment of 15 % per year, compounded
annually, how much will he collect at the
end of the sixth year?

A.
B.
C.
D.

A.
B.
C.
D.

9401 ME Beard October :1996

3.8%
4.9%
5.0%
5.8%

94:11 ME Board April :1998


If P 5,000.00 shall accumulate for 10 years
at 8 % compounded quarterly, find the
compounded interest at the end of 10
years.
A.
B.
C.
D.

P 6,005.30
P 6,000.00
P 6,040.20
P 6,010.20

A.
B.
C.
D.

P 4,400.00
P 4,390.15
P 4,200.00
P 4,626.00

945: ECE November 1998


. A man expects to receive P 25,000 in 8
years. How much is that money worth now
considering interest at 8% compounded
quarterly?

A.
B.

c.
D.

p
P
p
P

13,859.12
13,958.33
13,675.23
13,265.83

p 2,000,000
P 2,000,150
P 2,000,300
P 2,000,500

947: ME Board October 1995


In year zero, you invest P 10,000.00 in a
15% security for 5 years: During thattime,
the average annual inflation is.6 %. How
much, in terms of year zero pesos will be
in the account at maturity?
A.
B.

C.
D.

A.

16.7% :=; i < 100%


A.

B.
C.
D.

946: CE Board November 1994


P 500,000 was deposited 20.15 years ago

B.

9351 ME Board October :1995

A.

942: ME Boa&-d April :1998


A sum of P 1,000 is invested now and left

P 12,020
P 13,030
P 14,040
P 15,030

948: ECE Board April1998


By the condition of a will, the sum of P
20,000 is left to a girl to be held in trust
fund,by her guardian until it amounts toP
50,000. When will the girl receive the
money if the fund is invested at 8 % '
compounded quarterly?

A.

B.
C.
D.

7.98 years
10.34 years
11.57 years
10.45 years

949: ME ~ard October 1996


You borrow P 3,500.00 for one year from a
friend at an interest rate of 1.5 % per
month instead of taking a !oan from a bank
at a rate of 18% per year. Compare how
much money you will save or lose on the
transaction.

A.
B.
C.
D.

You will pay P 155.00 more if you


borrowed .from the bank.
You will save P 55.00 by borrowing
from your friend.
You will pay P 85.00 more if you
borrowed from the bank.
You will pay P 55.00 less if you
boHowed from the bank

"
544 lOQl,Solved Problems in Engineering Mathematics (2nd Edition) by Tiong & Rojas

'"' '~
'K""''
"~

' "II

'

l,,!h

'til~
.,.,..

llh'll~ll

-I'""~

~.-t

950: ME Board April %996


What is the present worth of two P 100
payments at the end of the third year and
fourth year? The annual interest rate is
8%.

A.
B.
C.
D.

'1"opics

P 153
P 160
P 162
P 127

0
L.:]

Engineertng Economics
Consumer & Producer Goods
and Services
Necessity and Luxury
Different Market Situations
Demand
Supply
Law of Supply and Demand
Simple Interest
Ordinary Simple Interest
- Exact SimplfJ Interest
Dist::ount
1 Compound Interest
Continuous Compounding
Nominal and Effective Rate of
Interest

Mon

iiUMf

Tue

(,,,!

0 [_j
~01
D lQ
[]
.~
l__; D

-~J

Theory

VIed

Problems

.,

Thu

Solutions

Fri

r~otes

Sat

ANSWER KEY
91 1.
912.
913.
914.
915.
916.
917.
918.
919.
920.

c
B
c
c

A
8
D
A
D

921. 8
922. D
923. B
924.A
925.A
926. 8
927.
928.
929.A
930. D

~'Uk~4&~Ad<t.ii~'

c
c

93'1. c
932.A
933.A
934.8
935.A
936.C
937.0
938. D
939. 0
940. D

941.

942.A
943.
944. D
945. D
946. B
947. D
948.C
949. D
950. A

i't

<I)~

0
c:J
0

RATING
34-40 Topnotcher
26-33 Passer
20-25 Conditional

019 Failed

\11

il

If FAILED, repeat the test.

546

\.;,,.!

,,,,,,

1m
!

11

...,till
~!dij

~-

~~~~~

~ 4,000[1+0.16(;6~)]

I~

i ~ 0.1628
i ~ 16.28%

i=0.1175
i = 11.75%

Interest= 0.06(20,000)
Interest= 1,200

F = P(1 +in)
F = 25,000[ 1+ 0.01(12)]
F = 28,000

a
F=P(1+in)

(1+~r -1

ER = 0.1255
ER= 12.55%

= P[1 +

F = 1,233.55

o.os(

6
)]
360

ER = 0.1948
ER = 19.48%

=(1+ it - 1

ER=(1+~r-1
(
0 0951''
1.0984 = 1+--j
'
n .

ER=(1+ 0 ~ 1 J -1

F =P(1 + i)"
10,000 = P(1 + 0.04)

21

'p = 4,388.336

F1 = 4,388.336(1 + o.o4t

= (1+

F1 = 5,552.645

~ J -1

0 4

Note: From the choices, the nearest


answer is 5,500

ER =0.1449
ER = 14.49%

ER = (1+ir -1
ER = ( 1+

0.~75

F =50,000(1 + 0.075)

50,000

F = 12,800 1+

-1

=o:on1
=7.71%

F = P(1 +i)"

i~llllll i

F = P(1+i)"
F = 71,781.47

~ r4)
5

25,000

75,000

Solving for the effective rate per year:

F = 15,614.59

ER = (1+it -1
F = P(1 + i)"
100,000
p

F, = P(1+ i)"

ER
ER

ER = (1+it -1

ER

F = P(1+in)

ER = (1+it -1

p!

ER = (1+it -1

ER

F = 408

21

F1

ER = (1 + i)m -1

F=P(1+in)

mJ

1m

i = 0.19
i=19%

ER =

ER =0.01
ER =1.00%

ER = (1+iJ -1

p = 46,728.97
Note: From the choices, the
nearest answer is 46,730

1.2so

:. the mode of interest is quarterly

F = 300[ 1+ 0.12(3)]

7
50,000 = p[ 1+ 0.12( 12)]

3,200 = (20,000- 3,200)(i)(1)

Proceeds= 20,000-1,200
Proceeds= 18,800

I=Pin

1,112.95 = 110,000(i{3~~)

By trial and error:


N=4

I~ 0.16(20,000)
I= 3,200

I= Pin

-~J

a
0.801 ~ 890.36
I= 1,112.95

~-l.,,!

Pin

Da! 22 -Engineering Economics @imEle and ComEound Interest} 547

11,200 ~ 68,800(i)(1)

F ~ 4,133.33

-r
=~

F~P(1+in)

~:~

t ."'

i 001 Solved Problems in Engineering: Mathematics ~znct Edition~ by Tiong: & Rojas

=p (

1+

=30,655.68

rO)

'ER=(1+

-~~J -1
1

ER = 0.050625

ll 1: II. ! I
I,,,
11111

lli'llil'll
I

IIli

!,lllll\\r'
I]

548 1001 Solved Problems in Engineering Mathematics (2"d Edition) by Tiong & Rojas

m1

p1

~t.,l

,,,,,I

~
'+.''1

~II

-,<-,

solooo

25,000

75,000

2.5 = (1.06)

F = P(1 + i)"

F = 200,000(1 +0.12t

"

F = 621,170

log2.5 = log(1.06t

= 25,000 + 75,000
50 OOO + _P_
4
2
'
(1+i)
(1+if
(1+i)

p = 39,201.28

DJI

ID

F = P(1 +i)"

ot

Interest= F- P

4P = 3.5oo(1+

0.~7r

4 = (1.035t
Take log on both sides:
log4 = log(1.035)
log4

2
"

Note: From the choices, the nearest


answer is 690,850

=2nlog1.035

n = 20.15 years

F=P{1+i) +P(1+i)

F = 1,000(1 + 0.08)

10

F = 2,158.92
Note: From the choices, the
nearest answer is 2, 160.

.F = P(1 + i)"
F = 5,000( 1 +

F=11,~0

= 1,500 ( 1 + 4

40

NR =3.5%

ml

NR = 5.8%

NR)20(2)

3,000

NR = 0.035

30

0 8

~ r

F= P(1 + i)"
F = 2,'000(1+0.15)

Note: NR = nominal rate


F = P(1 + i)"

F = P(1 + i)"

2 = (1 + 0.5NR)

NR = 0.058

F = 6,034.66

NR)15(2l

2.37 = (1 + 0.5NR)
2

Note: From the choices, the nearest


answer is 706

F = P(1 + i)"

2,370 = 1,000 ( 1 +

F = 2,000(1.08) + 2,000(1.08)

F16 = 705.42

Interest = 690,848.73

F = P(1 + i)"

Profit= 7,623.42

1m

Interest= 1, 190,848.73-500,000

Profit= 17,623.42-10,000

F = P(1 + i)"

F16 = P(1 + i)8


F16 = 477.455{1 + 0.05)

Profit= F -P

F = 6,265.87

Let: F1s = total amount after the end of


161h year

F = 1,190,848.73

F = 17,623.42

F = 3,5oo(1 + o.1

= 1,000{1 + 0.05) 8

Money left= 1,477.455-1000

F =50 ooo(1 + 0.1125)93


'
12

F = 10,000(1 + 0.12}

F = P(1 + i)"

Money left= 477.455

i = 0.1667
i = 16.67%

75,000
(1 + 0.050625t

= P(1 + i)"

F = P(1 + i)"

700 = 600(1 + i)'

&l 000 +p
=
25,000
1
2
(1 + 0.050625)
(1 + 0.050625)
'

After the principal is withdrawn:


n =93

n"" 8 years

rm

Interest= 6,040.20

n = 12(7)+9

log2.5 = 2nlog(1.06)
n = 7.86 years

50,000 + p1 = p2 + p3

Interest= 11,040.20-5,000

F = 1,477.455

ID

. p3

=10

"

Take log on both sides:

r;
................................................................:

Interest = F- P
n = 2(1993 -1988)
n

0 12)
2, 000 + 3, 000 = 2, 000 ( 1+ ~

~p

1m
F = P(1 + i)"

t
t
i' l
i

Day 22_- Engineering Economics (Simple and Compound Interest) 549

F = 4,626

F = P(1 + i)"
25,000

8(4)

= p ( 1 + ~

p = 13,265.83

,J

ii

lijl,

550 1001 Solved Problems in Engineering Mathematics (2"d Edition) by Tiong & Rojas

"''"'~

..,.,t
:~1
'

~
1

lr!~'~lllij

.:J::
-l,,l

-~

F = 500,000(1+

0~7r.tsc2J

F = 200,166

.,:'1\1

r..-lill

F = P(1+i)"

Computing for the amount due after


one year.

a. Borrow money from a friend

F = 3,500(1+ 0.015)

0
0
0

12

P'

=value of the account after 5


years considering there was
no inflation.
value of the account in
today's peso due to inflation

b. Borrow money from a bank

20,113.57 = P'(1 + 0.06)

p = 15,030

o~ar

Thu

Solutions

0
0
Fri

Sat

Notes

"'_5_ + _li_4
(1+it

50,000 = 20,ooo(1+

Problems

P =P1 +P2

=P(1 + i)"

(QJ

0
0

Therefore, you will pay P55 less by


borrowing the money from the bank.

F= P'(1 +i)"

Wed

Theory

F =4,130

F = 20,113.57

F =3,500(1 + 0.018)

F = P(1 + i)"
F =10,000(1 + 0.15)

Tue

F =P(1 +i)"

Engineering Economics
Consumer & Producer Goods
and Services
Necessity and Luxury
Different Market Situations
Demand
Supply
Law of Supply and Demand.
Simple Interest
- Ordinary Simple Interest
-Exact Simple Interest
l;)iscount
Compound Interest
Continuous Compounding
Nominal and Effective Rate of
Interest

Mon

F =4.185

let: F

Topics

F =P(1 + i)"

Note: From the choices, tt)e nearest


answer is 200,150

;I

(1+i)

(1+0.08)

f1JJ

+ ---.!QQ

100
3

(1+o.oat

il ~ II

iU',III1',]'f:

'

P=153

.l

2.5 =(1.04t"
Take log on both sides:
log2.5 = log(1.o2t"
log2.5 = 4nlog1.02
n = 11.57 years

I
.,..
ili

,,.'~'''*'

'II
' ' " ' " ' '"'"';

! ,,

:'II

1111

". '
1[1

I
fill

II,
I

'

~ i

~ ""

;~._.$<;,\,,,,~~~fO'~",~'<c~'>'"'"'

,.,.

"

"'

""

'I;

~ .. .,., ...

" -. .< " "

<"

<:.~

<:o "" ......

~ ~ ~- ~

.,

,, ... i!- ?

.,_

'

'

.,.

'

<

1111(1

, ~ '

'

..

< .. ' ..., "

'

SSZ 100 l Solved Proble!lls in Engineering Mathe~d Edition) by Tiong & Rojas

~"'I

c.n"'
'a:1

Topics.

IIIII

.,:_,,~-r~-

<I

"< <" '

..,;;;~-~

>'- '" ... _.;

)>

--~-,

<' ., , .,., .-, '

.~

....

;:;;<~

<.

< <:

Mon

~ <

"

' ~- *' ' " ' " ~ ~ ' "* ;

Tue

'""'''''"'""'! '"
-r_i_...
~.,

.,,,.,~

, , . , _ .. .,..

... _~--o~-~~-*~

_,O)_.,,..~,,._~,.

~ ~

y'

"

>

"' ,. ., ~ ' ' ,, < "' ' ., ~ ,

l _;t

"' ~

'<

,,..,

~"'~.{+~

""

( "" '

~!l.-\-<f.Y"*1<<t1<.~~~J<>~-,.,
"

/.

;'

' ",;, ., ' ,.

\)

'

~~*~

Problems
~ ., "" ,.

'1- '

~-" ~

o~-.,

tf'<-.<'''<

>.~.'t'
0

'

'

\--c;\1,

'~

O,

'

Thu

D @d
D D

"

VVed

[] D

< ... ' -~ .,.. ~ "' ~ .... ~ .,. ': ~ ,

I'.'>~~~,!-.'~<

Theory

~ ~ }, ~ ;< 'v,,

~ ~. ~

I~

""""'"

, ..,.._,_,,,<;.JI<W< ... ~.>~"'1',..-'<'~{,<-<..tX<>'><~"-"

"'""

'

D
D
D

,i' ) \

Solutions

Fri

Notes

Sat

What is Annuity?
'

. ,<'

"

'

,_""'

' ' ~ ", '

' .- ,, '

. ,, ....... ' '..: ., ..

<

~ ~ ~

,;"..;_,,.;._,,,
,. '

.; e.

Annuity is defined as a series of equal


payments occurring at equal interval of
time. When an annuity has a fixed time
span, it is known as annuity certain.

'

.. "

,. >. ,,., '

'

Types of annuity:

!J

~,

.. ~-,,

'

"

1.
2.
3.
4.

!~

,a
'j"'-"'.'.,..

.'

Ordinary Annuity
Annuity Due
Deferred Annuity
Perpetuity

Annuity
- Ordinary Annuity
- Annuity Due
- Deferred Annuity
- Perpetuity
Capitalized Cost
Annual Cost
Bond
Depreciation
Straight Line Method
-Sinking Fund Method
-Declining Balance Method
- Sum-of--Years Digit Method
Break-Even Analysis
Legal Forms of Business
Organization

Let A be the periodic or uniform


paynent and assuming only four
payments:

t-rll

~ ........,... A(1+ i)

: ................... ->

..

1. Ordinary annuity is a type of annuity


where the payments are made at the
end of each period beginning from the
first period.
Derivation of formula for the sum of
ordinary annuity:

~'

A(1 + i)2

~ ............. ,. ...............,._ A(1+ i) 3

Let: a1 =A

a2 = A(1+i)

a3 = A(1+il
a4

= A{1+i) 3

554 .1001 Solved Problems in Engineering Mathematics (2"d Edition) by Tio~g & Rojas

ta,,U
,,.,,,,.,w

M.

:~l

'"'~I
JH~

Annuity is based on the principles of


compound interest. Hence computation of
the sum of annuity may be done using the
formulas for geometric progression.

. . .I I I I
0

PRESENT WORTH OF ORDINARY


ANNUITY:

3 .. .

A'

.lIII

Solving for common ratio:

r = a2

a,

r = A(1+i)
A
r =1+i

Solving for the sum:

But: F = ---=-----"""
i
Substituting ~he value ofF:

3 ...

p = A[(1+i)"
i(1+ i)"

[<1 + i)" -1]


i(1 + i)"

301

l Il II.

where: i = interest per period


n = number of periods
A = uniform payment

! .. .. ................... ;

where: i = interest per period


A = uniform payment

3.

Capitalized cost of any structure or


property is the sum of its first cost and the
present worth of all costs for replacement,
operation, and maintenance for a long time
or forever, or
'
Capitalized cost

=First cost +
Cost of perpetual
maintenance

Annual cost

=ann1.1al depreciation cost


+ interest of first cost

+ annuaLoperating cost
+ maintenance cost

Deferred annuity is a type of annuity


where the first payment does not
begin until some later date in the cash
flow.

Bond is a financial security note issued by


businesses or corporation and by the
government asa means of borrowing longterm fund. It may also be defined as a
long-term note issued by the lender by the
borrower stipulating the terms of
repayment and other conditions.
Bonds do not represent ownership of a
business or corporation and therefore not
entitled to share of the profits. The
bondholder has no voice in the affair of the
business. However the bondholder has a
more stable and secured investment than
does holder of common or preferred stock
of the business or corporation.
Bonds are issued in certain amounts
known as the face value or par value of
the bond. When the face value has been
repaid, normally a~ maturity, the bond is
said to be redeemed or retired. Bond
rate is the interest rate quoted in the bond.

The following illustrates the normal life


cycle of a bond.

Jtc!>~~

Lender

Cash Flow of Annuity Due

[<1+i)"-1]

amount factor

What is a Capitalized Cost?


Annuity due is a type of annuity
where the payments are made at the
beginning of each period starting from
the first period.

40CJ

worth factor

-.- - = uniform series compound

2.3

=-----""" = uniform series present

. . Af(1+.it,.,-1].

l .

where:

2.

interest of the first cost and the annual


operating and maintenance costs, or

P=~

~ f''

F,;:;:.

-1]

.IIII
A

1'

SUM OF ORDINARY ANNUITY:

!Tlll

A[(1+i)" -1]

S= A[(1+i) -1]

5--n

PRESENT WORTH OF PERPETUITY:

What is a Bond?

P=-F(1 +i)"

1+i-1

4. When an annuity does not have a fixed


time span but continues indefinitely,
then it is referred to as a perpetuity.
The sum of a perpetuity is an infinite
value.

Using compound interest formula:

S= A[(1+it -1]

Cash Flow of Deterred Annuity

- ...............................:

S=a,(r"-1)
r-1

..........................~ F

Day 23 -Engineering Economics (Annuity, Depreciation, Bonds, etc.) 555

Borrower

What is an Annual Cost?


Annual cost of any structure or property is
the sum of the annual dFmr<>,..,i::~tion cost,
i

)II
'

'
II

Day 23- Engineering Economics (Annuity, Depreciation, Bonds, etc.) 557

100 1 Solved Problems in Engineering Mathematics (2nd Edition) by Tiong &Rojas

la . l
.....,1

~.

'Iii

'~I

1111

f~

0~
Borrower

Lender

f ~
Lender

(!)~
(!)

C!J

1~oll1

\,
0

Zr

Zr

Zr

Zr

Types of Depreciation:

Annual depreciation charge, d

:~ ~-: : : : : :..:..:.~: .~::~-~: : : : : ~


Let V n = value of bond n years before
redemption

A.

Physical Depreciation is' due to the


reduction of the physical ability of
equipment or asset to produce
results.

B.

Functional Depreciation is due to the


reduction in the demand for the
function that the equipment or asset
was designed to render. This type of
depreciation is often called
obsolescence.

,,..,..,..i,fi,...-.+,.,

f4=&~

0 =*' l,[\ {:J, \


~

Eq. 1

Using formula for present worth of annuity:

p _ A[(1+i)" -1]
1

i(1+i)"

Borrower
Value of a bond is the present worth of all
the amounts the bondholder will receive
through his possession of the bond. The
two payments that the bondholder will
receive are the following:
A. Periodic payments as interest of the
bond until it is redeemed.
B.. Single payment upon maturity of
bond. This payment is usually equal
to the par value of the bond.
Derivation of the formula for the value of a
bond:

Methods of Computing Depreciation:


1. STRAIGHT LINE METHOD
In this method of computing
depreciation, it .is assumed that the loss
in value is directly proportional to the
age of the equipment or asset.

zr[(1+i)" -1]

P1 =

Cn = cost after "n" years


(salvage/scrap value)

= life of the property

Book value at the end of "m" years of


using, Cm

Cm =Co-Om~
where: Dm

= total depreciation after


"m" years

Dm =

dCu ~en
---~

p =-F2
(1 + i)"

where: Co

p =-2
(1 + i)"

= first cost

Cn = cost after "n" years


(salvage/scrap value)
n

Substituting in Eq. 1

d[(1+it -1]

3. DECLINING BALANCE METHOD


In this method of computing
depreciation, it is assumed that the
annual cost of depreciation is a fixed
percentage of the book value at the
beginning of t~e year. This method is
sometimes known as constant
percentage method or the Matheson
Formula.

= life of the property


Matheson Formula:

_ zr[(1+i)"

Vn -

i(1+ i)n

-1]+~

Book value at the end of "m" years of


using, Cm

(1 + 1)1'1

k=1--t:.

Crn=C0 -Dm
where: Z = par value of the bond
r = rate of interest on the bond
per period
C = redemption price of bond
i = interest rate per period
n = nuf11ber of years before
redemption

where: Dm = total depreciation after


"m" years
Dm d(m)

VCo'

or

i'
'

Annual depreciation charge, d

Using the formula for present worth of


compound interest:

fl

= first cost

where: Co

i(1+ i)"

;;;==-

l !i

an

Borrower

4. Borrower redeems bond after n years,


p~ys principal& gets back

v!.~

! l ! l
Zr

Vn = p1 + p2

lend

Depreciation is the reduction of fall in the


value of an asset or physical property
during the course of its working life and
due to the passage of time.

2. SINKING FUND METHOD


In this method of computing
depreciation, it is assumed that a
sinking fund is established in which
funds will accumulate for replacement
purposes.

What is Depreciation?

Vn

vc;

.k=1--&.m

The value k is the constant percentage.


Hence k must be decimal and a value less
than 1. In this method, the salvage or
scrap value must not be zero.

~r

1
558 .1001 Solved Problems in Engineering .Mathematics (2"d Edition) by Tiong & Roj~

~.,.1

......

K:

'.lll
lg,

~,1,

revenue
Respective depreciation charges:

..

.. ...

,.'n

''

.,,."

''''

'arrtbta:
Did you know that. .. there are three
problems in Geometry, which attracted the
interest of mathematicians in the ancient
times, considered as "impossible
problems". They are the following:

n-1

~~ ~(Co .,. Cn) ~)ears

Second year:

n-2

. daf"T(Co -C")kY~iilrs

Third year:

production

.and so on ...
Book value at the end of ".tJ:~ .. years of
using, Cm

11,

I.

GOOD LUCK!

. o1 .~{C0 ~C"}'!~e~rs

Proceed to the next page for your last test.


Detach and use the answer sheet provided
at the last part of this book. Use pencil
number 2 in shading your answer.

4. SUM-oF-YEARS' DIGIT (SYD)


METHOD

First year:

I
I

,;

Day 23 -Engineering Economics (Annuity, Depreciation, Bonds, etc.) 559

=C0 .i(d1 +d2 -i-,,.:~t.d"') .

Sum of year's digit,

2: years

What are the Legal Forms of Business


Organization?
The legal forms of business organizations
are the following:

1.

.~yea($ =!i(n + 1)

Sole proprietorship.- considered as


t~e simplest type of business
organization wherein the firm is
owned and controlled by a single
person.

2.
What is a Break-Even?
Break-even refers to the situation where
the sales generated (income) is just
enough to cover the fixed and variable
cost (expenses). The level of production
where the total income is equal to the total
expenses is known as break-even point.
Break-even chart is a diagram which
shows relationship between volume and
fixed costs, variable costs, and income.
The following is an example of a breakeven chart.

3.

Partnership- is a firm owned and


controlled by two or more persons
who are bind to a partnership
agreement.

1. Duplication of a cube -: to construct a


cube whose volume shall be twice that
of a given cube .

2. Squaring a circle - to construct a


square whose area shall be equal to
the area of a given circle.

3. Trisection of an arbitrary angle- to


construct an angle that is exactly onethird of a given angle.

<!auote:
"No more fiction; we calculate; but that we
may calculate, we had to make fiction
first."
-Nietzsche

Corporation - is a firm owned by a


group of ordinary shareholders and
the capital of which is divided up to
the number of shares. It is also
defined as a distinct legal entity
separate from the individuals who
owns it and can engage in any
business transaction which a real
person could do. This is sometimes
known as joint-stock company or a
cooperative.
i

.I,]

'I

,,

i!
I 'j I; .

.' - "

______Day 23- Eng-ineering Economics (Jinnuity, Deprecia!~!~.:?o~:!E~,~~l~~~


D.

""'a:a'tn'"'

P 40,544.29

9551 ME Board April :1998

D
D
[] D
D
D ~
D []

_,,.,_

Tue

Wed

Problems

Thu

Solutions

Fri

Notes

Topics

Annuity
-Ordinary Annuity
- Annuity Due
- Deferred Annuity
- Perpetuity
Capitalized Cost
Annual Cost
Bond
Depreciation
-Straight Line Method
- Sinking Fund Method
- Declining Balance Method
-Sum-of-Years Digit Method
Break-Even Analysis
Legal Forms of Business
Organization

Man

Theory

....

Sat

951: EE Board October 1997

953: ME Board Oetober :1.996

A man purchased on monthly installment a


P 100,000 worth of land. The interest rate
is 12 % nominal and payable in 20 years.
What is the monthly amortization?

You need P 4,000 per year for four years


to go to college. Your father invested P
5,000 in 7 % account for your education
when you were born. If you withdraw P
4,000 at the end of your 1ir., 181h, 191h and
201h birthday, how much will be left in the
account at the end of the 21st year?

A.

P 1,101.08

B.
C.
D.

P 1,121.01
P 1,152.15
P 1,128.12

A.
B.

95Z: ECE Board April :1998

C.

Money Qorrowed today is to be paid in 6


equal payments at the end of 6 quarters. If
the interest is 12% compounded quarterly.
How much was initially borrowed if
quarterly payment is P 2000.00?

D.

A.

B.
C.
D.

P 10,834.38
P 10,382.90
P 10,586.99
P 10,200 . 56

P 1,700
P 2,500
P 3,400
P 4,000

954: ECE Board November 1998


What is the accumulated amount of five
year annuity paying P 6,000 at the end of
each year, with interest at 15 %
compounded annually?

A.
B.
C.

P 40,454.29 .
P41,114.29
P 41,454.29

How much must be deposited at 6% each


year beginning on January 1, year 1, in
order to accumulate P 5,000 on the date of
the last deposit, January 1, year 6?

A.
B.

c.
D.

P 751.00
P 717.00
p 715.00
P 725.00

956: ECE Board November 1.~98


A debt of P 10,000 with 10 % interest
compounded semi-annually is to be
amortized by semi-annual payment over
the next 5 years. The first due in 6 months.
Determine the semi-annual payment

A.
B.
C.

P 1,200.00
P t,295.05
P1,193.90

D.

P 1,400.45

9571 EE Board October 997


A young engineer borrowed P 10,000 at
12% interest and paid P 2,000 per annum
for the last 4 years. What does he have to
pay at the end of the fifth year in order to
pay off his loan?

9S9s ME Board O~t(llbell' 1!:.9'94


If you obtain a loan of P 1M at the rate of
12% compounded anrually in order to
build a house, how much must you p.ay
monthly t\) amortize the loan within a
period of ten years?

A.
8.
G.
D.
9~0~

Ecm: ha1d Ap~!i~ l[9'91i


How rnuch mljst you invest today in order
to withdraw P 2,000 annually for 10 years

if the interest rate is 9/c.?

B.
C.
D.

P 6,919.28
P 5,674.00

P 6,074.00
P 3,296.00

9S8s EE Board April 199'7


Mr. Cruz plans to deposit for the education
of his 5 years old son, P 500 a.t the end of
each month for 10 years at 12% annual
interest compounded monthly. The amount
that wili be available in two years is

12,853.~12

B.
C.
o.

P12,f581.37
P 12,::>65.32
P 12,Ba5.32

9fl>:K.f ECE Bom!rd ApwH fi."~

A person buys a piece of !ot for P 'IOC,OOO


dow.1payment and i 0 deferred s~~mi
annual payments of P 8,000 each, startmg
three years from now. What is the 1-m~sent
value of the investment if the rate of
interest is 12% compounded semi-

{1,

A.

P13,994.f7
p
P 15,855.45
P t2,900.25

B.

c.
D.

p 134,866.80
P 143,999.08
p 154,696.80
P 164,969.80

9<&:2: ClE B~ilard l!llay :!998


A man loans P '187,400 from a bank with
interest at 5% compounded annually. He
agrees to pay his obligations by paying 8
equal annual rayments, the first being due
at the end of 10 years. Find the annu<:~i
payments.

A.
B.

P 13,000
p 14,500

A.
B.

P5G,143.03

C.

P 13,500

D.

P 14,000

C.
D.

P 62,334.62
P 38,236.04

P 44,982.04

,.
Day 23- EngineeringEconomics (Annuity, Depreciation, Bond, etc.) 563

562, 100 l Solved Problems in Engineering Mathematic:; (2"d Edition) by Tiong & Rojas

""'K'i
lP

"'~I

I'

~ A housewife bought a brand new


washing machine costing P 12,000 if paid
in cash. However, she can purchase it on
installment bas1s to be paid within 5 years.
If money is worth 8% compounded
annually, what is ner yearly amortization if
all payments are to be made at the
beginning of each year?

A.

B.
C.
D.

p 2,782.85
P 2,872.58
P 2,400.00
P 2,827.58

967: CE Board November 1996

971: EE Board April :1997

97Sz A fund donated by a wealthy person

A man inherited a regular endownment of


P 100,000 every end of 3 months for 10
years. However, he may choose to get a
single lump sum payment at the end of 4
years. How much is this lump sum if the
cost of money is 14% compounded
quarterly?

A small machine has an initial cost of P


20,000, a salvage value of P 2,000 and a
life of 10 years. If your cost of operation
per year is P 3,500 and your revenues per
year is P 9,000, what is the approximate
rate of return (ROR) on the investment?

to I lEE to provide annual scholarships to


deserving EE students. The fund will grant
P 5,000 for each of the first five years, P
8,000 for the next 5 years and p 10,000 for
each year thereafter. The scholarship will
start one year after the fund is established.
If the fund earns 8% interest, what is the
amount of the donation?

A
B.
C.
D.

P
P
P
P

3,802,862
3,702,939
3,502,546
3,602,431

A.
B.
C.

D.

25.0%
22.5%
23.9%
24.8%

97:2.: CE Board November 1996


964: ME Board October 1996
Mr. Ayala borrows P 100,000 at 10%
effective annual interest. He must pay
back the loan over 30 years with uniform
monthly payments due on the first day of
each month. What does Mr. Ayala pay
eoch month?
A.

B.
C.
D.

P 870.00
P 846.00
P 878.00
P 839.00

968: ME Board April 1998


A parent on the day the child is born
wishes to determine what lump sum would
have to be paid into an account bearing
interest at 5 % compounded annually, in
order to withdraw P 20,000 each on the
child's 18th, 19th. 20th and 21t birthdays.
How much is the lump sum amount?
A.
B.

C.

D.

P35,941.73
P33,941.73
P 30,941.73
P 25,941.73

965: ME Board April 1998


A house and lot can be acquired by a
downpayment of P 500,000 and a yearly
payment of P 100,000 at the end of each
year for a period of 10 years, starting at
the end of 5 years from the date of
purchase. If money is worth 14%
compounded annually, what is the cash
price of the property?

969: ME Board April :1998


An instructor plans to retire in exactly one
year and want an account that will pay him
P 25,000 a year for the next 15 years.
Assuming a 6 % annual effective interest
rate, what is the amount he would need to
deposit now? (The fund will be depleted
after 15 years).

A man paid 10% down payment of P


200,000 for a house and lot and agreed to
pay the balance on monthly installments
for "x" years at an interest rate of 15%
compounded monthly. If the monthly
installment wasP 42,821.87, find the value
of)(?
A.
B.
C.
D.

80 employees a holiday bonus. How much


is needed to invest monthly for a year at
12% nominal interest rate compounded
monthly, so that each employee will
receive a P 2,000 bonus?

A.

B.
C.
D

P810,100
P 808,811
P 801,900
P 805,902

'!i!Wo: ME Board April 1998


A piece of machinery can be bought for P
10,000 cash or for P 2,000 down and
payments of P 750 per year for 15 years.
What is the annual interest rate for the
time payments?

A.
B.
C.
D.

4.61%
3.81%
5.71%
11 0 '%

A.

p 249,000

C.

B.
C.

P 242,806
P 248,500
P 250,400

D.

D.

970: EE Board October :1997


An investment of P 350,000 is made to be
followed by payments of P 200,000 each
year for 3 years. What is the annual rate of
return on investment for the project?
A.
B

C.
D.

41.7%
32.7%
51.1%
15%

P
P
P
P

D.

12,608
12,610
12,600
12,300

P 101,605.71
P 10i,505.21
P 100,506.21
P 99,601.71

976: ME Board April 1998


A company issued 50 bonds of P 1,000.00
face value each, redeemable at par at the
enq of 15 years to accumulate the funds
required for redemption. The firm
established a sinking fund consisting of
annual deposits, the interest rate of the
fund being 4 %. What was the principal in
the fund at the eno of the 1'2th year?
A.
B.

97~: ME Board April 1998


A manufacturing firm wishes to give each

B.

A.

11
9
5
7

A.
B.
C.

C.
D.

p 35,983.00
P 38,378.00
P 41,453.00
P 37,519.00

977: ME Board Aprii199:Z


A unit of welding machine cost P 45,000
with an estimated life of 5 years. Its
salvage value is P 2,500. Find its
depreciation rate by straight-line method.
A.

B.
C.
D.

17.75%
19.88%.
18.89%
15.56%

974: CE Board November 1995


Find the present value in pesos, of a
perpetuity of P15,000 payable semiannually if money is worth 8%
compounded quarterly.

A.

P 372,537

P 374,977

C.
D.

P 373,767
P 371,287

978: EE Board A.pril :1997


A machine has an initial cost of P 50,000
and a salvage value of P 10,000 after 10
years. Find the book value after 5 years
using straight-line depreciation.

A.
B.
C.

p 12,500
P 30,000
P 16,400

D.

P 22,300

'

'l.
')

Day 23 - Engineering Economics (Annuity, Depreciation, Bond, etc.) 565

564 1001 Solved Problems in Engineering Mathematics (2nd Edition) by Tiong & Rojas

~ . 1.

,,,,,,

K:'ll'
'

111

~~.1

979: ME Board October 1992

98~: An equipment costs P 10,000 with a

The initial cost of a paint sand mill,


including its installation, is P 800,000. The
BIR approved life of this machine is 10
years for depreciation. The estimat~d
salvage value of the mill is P 50,000 and
the cost of dismantling is estimated to be P
15,000. Using straight-line depreciation,
what is the annual depreciation charge
and what is the book value of the machine
at the end of six years?

salvage value of P 500 at the end of 10


years. Calculate the annual depreciation
cost by sinking fund method.at 4%
interest.

A.

B.
C.
D.

p 74,500
P 76,500
P 76,500
P 77,500

; p 340,250
; P 341,000
; P 342,500
; P 343,250

980: CE Board November :1997


The cost of equipment is P 500,000 and
the cost of installation is P 30,000. If the
salvage value is 10% of the cost of
equipment at the end of 5 years,
determine the book value at the end of the
fourth year. Use straight-line method.
A.

B.

c.
D.

P 155,000
P 140,000
p 146,000
P 132,600

98:1: ME Board Apriil1998


An asset is purchased for P 500,000.00.
The salvage value in 25 years is P
100,000.00. What is the total depreciation
in the first three years using straight-line
method?
A.
B.
C.

D.:

p
P
P
p

48,000
24,000
32,000
i6,000

98:&: ME Board April 1:998


A machine has an initial cost of P 50,000
and a salvage value of P1 0,000.00 after
10 years. What is the book value after 5
years using straight line depreciation?
A.
B.
C.
D.

p
P
P
P

35,000.00
25,000.00
15,500.00
30,000 00

A.

P 791.26

B.

P 950.00
P 971.12
P 845.32

C.
D.

984: CE Board November :1995


A machine costing P 720,000 is estimated
to have a book value of P 40,545.73 when
retired at the end of 10 years. Depreciation
cost is computed using a constant
percentage of the declining book value.
What is the annual rate of depreciation in
%?
A.

B.
C.
D.

28
25
16
30

985: CE Board May :1996


A machine costing P45,000 is estimated
to have a book value of P 4,350 when
retired at the end of 6 years. Depreci~:~tion
cost is computed using a constant
percentage of the declining book value.
What is the annual rate of depreciation in
%?
A.
B.
C.

D.

33.25%
32.25%
35.25%
34.25%

986: ECE Board November ll.998.


ABC Corporation makes it a policy that for
any new equipment purchased, the annual
depreciation cost should not exceed 20%
of the first cost at any time with no salvage
value. Determine the length of service life
necessary if the depreciation used is the
SYD method.
A.

B.
C.
D.

9 years
10 years
12 years
19 years

I
I

987: ME Board April :1998

991.: CE BOard November :1996

A company purchases an asset for P


10,000.00 and plans to keep it for 20
years. If the salvage value is zero at the
end of 20th year, what is the depreciation in
the third year? Use SYD method.

At 6%, find the capitalized cost of a bridge


whose cost is P 250M and life is 20 years,
if the bridge must be partially rebuilt at a
cost of P 1OOM at the end of each 20
years.

A.

A.

P 275.3M

B.
C.
D.

P 265.5M
P 295.3M
P 282.1M

B.
C.

D.

p 1,000.00
P 857.00
P 937.00
P 747.00

988: ECE Board Aprilt.999

992: CE Board May :1997

A Telephone company purchased a


microwave radio equipment for P 6 million,
freight and installation charges amounted
to 4% of the purchased price. If the
equipment will be depreciated over a
period of 10 years with a salvage value of
8%, determine the depreciation cost during
the 5th year using SYD.

A corporation uses a type of motor truck


which costs P 5,000 with life of 2 years
and final salvage value of P 800. How
much could the corporation afford to pay
for another type of truck of the same
purpose whose life is 3 years with a final
salvage value of P 1,000. Money is worth
4%.

A.

B.
C.
D.

P 626,269.10
P 642,786.07
P 638,272.08
P 627,989.90

989: ME Boaad April :1998


An asset is purchased for P 9,000.00. Its
estimated life is 10 years after which it will
be sold for P 1,000.00. Find the book
value during the first year if sum-of-years'
digit (SYD) depreciation is used.
A.
B.

c.
D.

P
P
p
P

8,000.00
6,500.00
7,545.00
6,000.00

990: EE Board April :1997


The maintenance cost for a sewing
machine this year is expected to be P 500.
The cost will increase P 50 each year for
the subsequent 9 years. The interest is 8
% compounded annually. What is the
approximate present worth of maintenance
for the machine over the full 10-year
period?
A.

13.
C.
D.

P 4,700
p 5,300
P 4,300
P 5,500

A.
B.
C.
D.

P 8,450.66
P7,164.37
P 6,398.24
P 9,034.56

99~: ME Board October :1995


A company must relocate one of its

factories in three years. Equipment for the


loading dock is being considered for
purchase. The original cost is P 20,000,
the salvage value of the equipment after
three years is P 8,000. The company's
rate of return on the money is 10%.
Determine the capital recovery rate per
year.
A.
B.
C.
D.

P 5,115
P 4,946
P 5,625
P 4,805

9941 EE Board October 1998


The annual maintenance cost of a
machine shop is P 69,994. If the cost of
making a forging is P 56 per unit and its
selling price is P 135 per forged unit, find
the number of units to be forged to breakeven.
A.
B.

886 units
885 units

Day 23- Engineering Economics (Annuity, Depreciation, Bond, etc.) 567

566 :1001 Solved Problems in Engineering Mathematics (2nd Edition) by Tiong & Rojas

,,,,,

""'a:a
(/l~

C.

D.

688 units
668 units

995: CE Board May 1998


A manufacturer produces certain items at
a labor cost of P 115 each, material cost of
P 76 each and variable cost of P2.32
each. If the item has a unit price of P 600,
how many number of units must be
manufactured each month for the
manufacturer to break even if the monthly
overhead is P 428,000.

998: ME Board October 1990

A company which manufactures electric


motors has a production capacity of 200
motors a month. The variable costs are P
150.00 per motor. The average selling
price of the motors is P 275.00. Fixed
costs of the company amount to P 20,000
per month which includes taxes. The
number of motors that must be sold each
month to break even is closest to:

A.

A.
B.

B.

A.
B.
C.
D.

1053
1138
946
1232

9961 ME Board April1996


Steel drum manufacturer incurs a yearly
fixed operating cost of$ 200,000. Each
drum manufactured cost $160 to produce
and sells $ 200. What is the
manufacturer's break-even sales volume
in drums per year?

A.
B.
C.
D.

1250
2500
5000
1000

9971 JRT Industries manufactures


automatic voltage regulators at a labor
cost of P 85.00 per unit and material cost
of P 350.00 per unit. The fixed charges on
the business are P 15,000 per month and
the variable costs are P 20.00 per unit. If
the automatic voltage regulators are sold
to retailers at P 580.00 each, how many
units must be produced and sold per
month to breakeven?
A.
B.
C.
D.

..~

104
200
120
150

10011 ME Board April1998

Compute for the number of locks that an


ice plant must oe able to sell per month to
break even based on the tbllowing data:
Cost of electricity per block- P 20.00
Tax to be paid per block- P 2.00
Real Estate Tax- P 3,500.00 per month
Salaries and Wages- P 25,000.00/month
Others- P 12,000.00 per month
Selling price of ice - P 55.00 per block

c.
D.

1228
1285
1373
1312

9991 EE Board October 1997


The annual maintenance cost of a
machine is P 70,000. If the cost of making
a forging is P 56 and its selling price is P
125 per forged unit. Find the number of
units to be forged to break even.
A.
B.
C.
D.

1015 units
985 units
1100 units
1000 unit!;

1000: ME Board Aprill998


XYZ Corporation manufactures bookcases
that sells for P 65.00 each. It costs XYZ
Corporation P 35,000 per year to operate
its'l)lant. This sum includes rent,
depreciation charges on equipment, and
salary payments. If the cost to produce
one bookcase is P 50.00, how many cases
must be sold each year "tor XYZ to avoid
taking a loss?
A.
B.
C.
D.

2334
539
750
2333

C.

o.

40
150
80
160

~'

;I

Day 23- Engineering Economics (Annuity, Depreciation, Bonds, etc.) 569.

1&.1

K':ll'
I

ID

4.1

F1 =P1(1 + i)"

012345

e '] rnuity
rlr:'
.f!~

;~A
~~

'

..

:\

t_

!.111on

AAAAA

Ordinary Annuity
Annuity Due
Deferred Annuity
Perpetuity
Capitalized Cost
!! i!._nnual Cost
1 Bond
Dc1preciation
-Straight Line Method
-Sinking Fund Method
- Declining Balance Method
- Sum-of.. Years Digit Method
Break-Even Analysis
Legal Forms of Business
Organization
1

--~
Theory

\Ned

[~

r---~

LJ

[I]

~
Fri

Problems

Solutions

Thu
r~-o

[J (._J I
Sat

Notes

~...1. . 1. . L. . L. .L. . . . . . . . . . . . . . ..i

F2 =

2 -

P= A[(1+i)"-1J

(1 + i)" i
F3

Alr(1 +0.12.) 12(20)_1]

12
.
0.12)12(20)
i
( 1+-12
A=1,101.08

I 951'
A
952. J\
I 953. P..

954.A
955. B
. 956. B
957.A
958.C
959.A
960. D
96'1. B
962. A
963.C

964.
965.
966.
967.
968.
969.

D
B
A
B

c
B

970. B
971. D

9l2.C

973. B

977. c
978. B
979. 8
980.C
981. A
9!32. D
983.A
984. B
985. B
986.A

974. D 987. B
975. D 988.A
976. D

989.

990. A
991. C
992. B
993.C
994.A
995.A
996. c'

997. c
998.A
999.A

1000.

~~

1001.D

c:J43-51 Topnotcher

[~

c:J

33-42 Passer
25-32 Conditional
0-25 failed

If FAILED, repeat the test.

-:-1]
----=-.:..-o.=o1--'---=

F3

=19,002.95

1 0.12)

+4

6000[ (1 + 0.15) 5 -1]


0.15
F = 40,454.29

6
.

p1 ............................................................Jo-jF1

16 17 18 19 20

2~

llll

A A A A

L.. .L.. .L..~Fl

l2
F= A[(1+i)"-1]
i

F = A[(1+i)"-1]
i
F = ----'=-'-------'-------=!

p = 10,834.38

=F3 (1 + i)1

Money left = F1 - F3

111

(1+i)"i

Money left= 20,702.81-19,002.95

P= A[(1+i)"-1]

RATING

Money left= 1,699.86"' 1, 700

-------

F3 =17,759.772(1+0.07)

100,000 =

ANSWER KEY

A((1+i)"-1]

F2 = H, 759.772

P-= 2.ooo[(1+T) -1J


~------

21

F. - 4000[(1+0.07)

0 Il"'l L_J
Tue

F1 = 500(1 + 0.07)
F1 =20,702.81

lllll"'""""""""'' "'""""'""'!
Topics

II

L..,..:F3

F = A((1+i)"-1]
i

5,000 = A[(1 +0.06)


.
0.06
A= 717

-1]

P= A[(1+i)"-1]
(1+i)"i

A((1

+ o.;o)2(5)_ 1
10, 000 = --:-"-';.._..__~-!,...------""
(1+ o.~or5) (0.05)

A= 1,295.05

570 1001 Solved Problems in Engineering Mathematics (2"d Edition) by Tiong & Rojas

k.l
(J.l~

aa
'

ER = {1+it
012345

ER={1-+:i)

l
l
l
l
1
~ ~ ~ ~ ~
~ L....L. .L. .J
...............
1

. !
Pz

..

58,880.69 = p2 (1 + 0.06)

0.12 = {1 + if -1

p2 = 43,999.078

2000((1+0.12) -1]
_

--=-.:.._---,-4_

(1+0.12} (0.12)

Total amount= 100,000+43,999.078

mJ

(1+i)"i

F
+ - - = 10,000
(1+i)"
F
<

_
-10,000

(1+0.12}
F = 6,917.72

Note: From the choices, the nearest


answer is 6,919.28

A({1 + 0.009488)
1,000,000 =

120

-1]

120

{1 + 0.009488)

(0.009488)

A= 13,994.17

10

A[ (1 + i) "-1]

p1 = ----'=-------"8

A[(1 + 0.05) -1]

pl = ----'=---~
8

(1 + 0.05) (0.05)

10 payments

5 6 7 8

100,000

=sao[( + ~)
1

M)_ 1]

0.12
12
F = 13,486.70

P1 = 6.643A

15 16

A A A
P1

6.643A = 187,400(1 + Q.o5l

~.L. . L....L.. . . . . :l... J

2 ~...................... J

P1 = P2 (1 + i)"

A= 44,982 04

Solving for the interest rate per


month,!:

= (1+if 2
2

+ 1 = 1.1 0 = ( 1 + if

i = 0.007974

Borrowed, money = A + P
A[(1+0 007974) 359 -1]
100,000 = A + _ ____e_:.___~__:_---"--359
(1 + 0.007974) (0.0 007974)

A= 839

10 payments

!". . . . . . . . . . lll . . . . .

!l l l l l

500,000

~.. J. . .l. . . l_J.. . !

p2

4 5

p
0

10

pl

AAAAAA

(1 + i)" i
(1 + 0.06) (0.06)

p1 = 58,880.69

1+i

A A

14 15

01234$

_ 8ooo[(1+0.06f -1]

p1-

Solving for the interest rate per month:

ml

~., ..L. .L. . L. . . . . . . . . . . L. .!

0 1

p __
A,_(( 1_+_i)_"-_,1
Note: From the choices, the nearest
answer is 13,500

1718

AA

(1+i)"i

F= A((1+i)"-1]

359 360

Total number of payments = 12(30) = 360

(1 + 0.09} (0.09)

0 1

AAA.A

r. L. J. . ..t. .............L. . ~

-1]

10

r. . . . . . . Tn. . . . . n

A A

Pz ~.................. .

p = 12,835.32

i.= 0.12/12 = 0.01


n 12(2) 24

9101112

P1

(1 + i)" i

3 years

0 1

. . . . . . . . . . lll . . . . . n
(

0 1

P= A((1+i)"-1]

- 2000[(1 + 0.09)

I'll

r rrr . . . . . . . . .

8 payments
~--A

P-

-1]
(1 + 0.08)5 (0.0.08)

ml

Total amount= 143,999.078

p = _A,_(( 1_+_i)_"---"'1

p1 + p2 = 10,000

(1+i)"i

A~1 + 0.08)

A= 2,400

Total amount= 100,000+P2

=12(10) =120

.J

A[(1+i)"-1]

12,0,00 =A+

i =0.009488

__h..;.__:.~

p1 = p2 (1 + i)"

-1

P1

=1

12
2

Day 23- Engineering Economics (Annuity, Depreciation, Bonds, etc.) 571

Cash price = A + 1-'

1
1
12,000 =A+ ---'A[~(_+_:_i)_"-...:!.]
(1 + i)" i

A A

r.L . . .l. ....L. . . . . . l.... J

...................... :

"-1]

_ A[(1 + i)
Pl-----.
(1+i)"i

I.
I

57Z : l 00 l Solved Problems in Engineering Mathematics (2"d Edition) by Tiong & Rojas
- 100,000[(1 + 0.14)

"'
,,,...

p1-

10

-1]

10

(1+0.14)

p,;, A[(1+i)"-1]

(0.14)

(1+i)"i

P1 = 521,611.56

(/'Ill

Q.'l

0 4 24

P=~
2

p=

100,000[(1+
24

(1+i)"

0.~4) (0~4)

(1+

p - 521..611.56
2
4
- (1+0.14)

~ ) -1]

p = 1,605,836.76

p2 = 308,835.90
F = _A[=-(1_+_i)_"--=-1]
Cash price = 500,

ooo + P2

Cash price= 500,000 + 308,835.90


Cash price= 808,835.90
Note: From the choices, the nearest
answer is 808,811

100,000[(1 + 0.035)
.
0.035
F = 2,097,102.97

16

-1]

Day 23 -Engineering Economics (Annuity, Depreciation, Bonds, etc.) 573

p = _A[h.:.(1_+...:...i)_"---=1
(1-+ i)" i

i'l

A= 5,500
1111111111'1!1

Substitute the value of A in Eq. 1:

p = 242,806
0
5500[(1 + if -1]
- - - = - - . .10
:---"'

{1+i) i

350,000

I, IIIII

2000
+-10 = 20' 000
(1+i)
'

By trial and error, i = 24.8%

! l-1

F=

Down payment= 10% of cost of house


and lot

p~ ... L.....L .....l

Lump sum price = P + F

A= 9,000-3,500

15
- 25,000[{1+0.06} -1]
p15
(1 + 0.06) (0.06)

Note:
A = revenue/year- operating co.sVyear

200,000 = 0.10(cost)
cost= 2,000,000

Lump sum price= 1,605,836.76

il

+ 2,097,102.97

Cash price = Down payment


+ Present worth of the
Annuity

.....,.
J

1
1
10 000 = 2 000 + _A.=.:[('--+_i:_)"--....=.
'
'
(1+i)"i
10,000 = 2,000 +

750[ (1 + i)
15

15

(1 + i)

-1]

r. .L ...L...l. ...!

1-

24 payments

p = __,A[:.:_(1_+i)_"----=!1]
(1+i)"i

p2 ~.......................... ......................sv
~

~T

15 16 17 18

A A

(1 + 0.05} (0.05}

39 40

A A

P1 = 70,919

. . .1. . .1_. . . . . . . . . r. . l
A

iii

ii

9 10

20,000

70,919
(1+0.05t

p = 30,941.72

(1.0125)

12

42,821.87[(1 +0.0125) 12-1]


--=~12:;::----'--

(1+0.0125) x(0.0125)
2

-1 = 0.52543(1.0125f x
2
(1.0125f x =2.1072
x

Taking log on both sides:


12xlog1.0125 = log2.1072

il!
I

x = 5 years

P=__!l_
(1 + i)"
p =

~ 800,000

A A A
4

...i

p1

(1 +I")".I
2,000[(1 + 0.05) -1]

p1

= 0.0125

By trial and error, i = 32.7%

p1

i = 0.15/12

(1 + i) (i)

17 18 19 20 21

p~

-n. . . lll r. . . n
LJ.._...Lltl. L_.. J.
A A

. . . . . . . . . . . . l' ! ! !

- 200,000[(1+i) -1]
350,0003

p __
A['=-(1_+;..;..,i}:--n
--=-1

16 payments

Balance= 1,800,000

p1

Ill
0 1 2

liiltiiiill

By trial and error, i = 4.61%

Balance = 2, 000,000- 200,000

(1 + i)" i

10.667 = (1 + if5 -1
15
(1+i} i

Balance= cost- downpayment

p = A[(1+i)"-1]

Lump sum price= 3,702,939.73

p1 + p2 = 20,000
A[(1+i)"

- 1 L_~~

-(1+ i)"i

(1 + 1)

=20,000

~Eq.1

:j

Day 23- Engineering Economics (Annuity, Depreciation, E!onds, etc.) 575

574 1001 Solved Problems in Engineermg Mathematics (2"d Edition) by Tiong & Rojas

u.l
lll
'Ill'

al

. . . . . . . . . . . . !!-!!1

F= A[(1+i)n-1]

i
A[(1 + 0.12)
80(2,000) =
'
12
0.12
12
A = 12,615.80

12

p2

r. .L. .L. .L. .L..J

p3 ....:................... .

= (1 + i)2

A2 [ ( 1 + i)n - 1]

--'~----='

( 1+

i)" i

(1 + 0.08)

J ..~ F

50(1,000)= A[(1+0.04t' -1]


0.04
A= 2,497

l ! !!1..

F~-

Ps ,.;

p - 10,00.Q

-1]

0.08

p4 = 125,000
p 5-

p 5

12

-1]

C 5 = 50,000- 4,000(5)

C5 =.30,000

(Cr,- cost to dismantle)

d = 80,000- (50, 000- 15, 000)


10
d = 76,500

Let:, Ce = book value at the end of 6 years


C6 =C 0 -d(m)

C6 = 341,000

C0 = 500, 000 + 30, 000


C0 = 530,000
en= 0.10(500,000)

d =Co- Cn
n
d = 45,000-2,500
5
d = 8,500

p4 =-:-

5ooo[(1+0.08}
p1 = - - " - - . , .5 - - - (1 + 0.08) (0.08)

1m

(1+i)"i

- 2,497[(1+0.04)
F1.
0.04
F1 = 37,519

I I II

p4

A A A

_ A[(1+i)" -1]

l !!1

C5 = C 0 -d(m)

C6 = 800,000- 76,500(6)
oc

A A A A

1 [(1+i)n -1]
---"'-~--"'

n
d = 50,000-10,000
10
d = 4,000

Let: Cs = book value at the end of 5 years

Let: F1 =value of the given annuity when


n = 12 years

11

012345

P1...:....: ..... .:. ..... :..... .:. .... :

d =Co -Cn

d = C0

F= A[(1+i)n _ 1]

p3 = 21,738.97
0

. .. .
.
. .. .
....

14 15

L .....i ...... i........................L. ..

p3 = 31941.68

p = 15,000
. 0.0404
p = 371.287.13

p1 = 19,963.55

p p2
3 - (1 + i)5

lll .........n
i

P=~

p1 -

Total= 99,601.71

p2 =31,941.68

i = 0.0404

Total= 19,963.55 + 21,738.97 + 57,899.186

8ooo[(1+0.o8t -1]
p2 =
'
(1 + 0.08) 5 (0.08)

0.~8J =(1+i)2

Total= P1 + P3 + P5

( 1+ N4R

7 8 9 10

A A A A A

1}

p2 -

(1+

en= 50,000

d =Co -Cn

n
d = 530, 000 - 50, 000

p4
{1+ifo

depreciation rate =

_i_

d = 96,000

Co
125,000
(1+0.08)

10

P5 =57,899.186

depreciation rate =

8 500

x1 00%
45,000

depreciation rate = 18.89%

. Let: C4 = book value at the end of 4 years

C4

=C 0 -

d(m)

C4 = 53o,ooo- 96,000(4)
C4

...

146,ooo

l,.

576 1001 Solved Problems in Engineering Mathematics (2nd Edition) by Tiong & Rojas

4 .1

u.l
(Jll
a:~

_______ .!_)_~ 23- Eng!~ng Eco!!2._mic~!!\nnuitL.~nreciati<tl.1..l.!1_1?rt'!~!tc.) ~!I.


Using formula for sum of an A.P .

k
d =Co

720;000

-c.

n
25

d 16,000
Let: D = total depreciation after- "m" years

Lyears =.!'_l(n
+ 1)
-----

=0.25

,.._..., years= 55

l~ years "' 3~(20 + 1)

vco-

Substituting:

=16,000(3)

Substituting:

Book value at the end of 6 years, C6:


C1o = 4,350

0=48,000

- "(10
d3
" ' 000--

\
d, "'(9,000 -1,000)1(_!Q).

&iii~!

1!'-R

d=Co-Cn

......

= book value at the end of 5 years


C5

=C0 -d(m)

Solving for C1:

Using_SYD~

the largest charge of


depreciation is the first year.

years "''

~)ears

d1

=(Co -en {

n(~_2-+ 1)

C0

LY!'!ars = n
0

Using formula for sum of an A. P.

=791.26

p == ----~-..._________ _).

....

5n = n(n +1)

Book value at the end of 10 years, C1o:


C1o 40,545.73

Substituting in the formula:

d5

2
n

=9 years

Capitalized cost = C 0 + Co-- Cn

C1 =Co - d1

n-2

d3 =(Co- c.) ) years)

Note: From the choice!i, the nearest


answer is 4,700

= 626,269.10

+50[tf.~"!~i.'rj
________1~-------.! (0.08){1+0.08t
p =4,653.88

d5 = (6,240,000- 499,200)( 5~)

k=1-~:

= 6,240,000

Substituting:

ryears =n(n + 1)

Ill

i(1+ i)"

5oo[(1+o.oat -1l

c, ~' 0.08(6,240,000)
c, = 499,200

d = .!...(1_...;.0,0_0_0-_5_00-d-).!....(0_.04...!..)
(1+0.04f -1

1!

G[-'------------- -- ----.11. ------J

C0 == 6, 000, 000 + 0. 04( 6, 000, 000)


' (1+i)"-1

'1,545.46

'
)10 (008).
(1+fl08

0.20C 0 = Co ( LY:ars)

d =(Co- Cn) ~

,,

lll'(.1-'"-]
ro(1+1)" ---1
_ '"'
IIJ
'
p o' -"-"-. ____..__ +
(1+ i)" i
- f (1+ i)"

= 1Oi:!2_:~ 1}

Lyears = 55

L :ears)

C1

Using uniforrn gradient formula:

C 5 = 30,000

C 1 ~, 9,ooo- 1,454.54

I -.. n_-_---j
d,, =(Co ... '"'"\L:rears
P

Using formula for sum of an AP.

C 5 = 50,000- 4,000(5)

d1 ""t454.54

k =32.25%

llifiM

Let: Cs

\55

18 \1
0) ('--)
210

d3 = 857

Substituting in the formula:

d = 50,000-10,,000
10
d = 4,000

lj

k=1-_rs;

D = d(m)
D

i
I

"'"'
2-L, years= 210

El1

,I

2)ean.; = 10(10 + 1)

k=25%

d"' 500,000-100,000

L years "' !~+


1)
'2--

=1-19/40,545.73

d1 = (Co -

1111

C~ {~):ars

(1 + i)" -1

Capitalized cost = 250 + ---

100

--

(1+0.06)20 -1

Capitalized cost = 295.3 million

u.l
u.J

K~
I

Let AC 1 = annual cost of the old motor


truck
AC 2 = annual cost of the new motor
truck
AC ;=(C )i+ (Co1-Cn1)i
1
01

(1 +

ir -1

135x

= 2,258.82

= 56x + 69,994

79x = 69,994

Let: x

AC ~ (5,000)(0.04) + (5,000- ~00)(0.04)


1
2
(1 + 0.04) -1
AC1

To breakeven:
Income = Expenses

x = 886 units

=number of units to be
manufactured per month

Income

= Expenses

X=

= (C 02 )(0.04) +(Co2

2,258.82

= 0.04C 02

C02

= Expenses

40x = 200, 000


x = 5, 000 units

Income

= Expenses

125x = 15,000

CH = (Co)i+ (Co -Cn)i

(1 + i)" -1
CR

= 20,000(0.10)+ ( 2 0,000-8,000)(0.10)
3

(1 + 0.1 0) -1

" .
1

.R = 5,625

Let: x

x = 120 units

= number of blocks to be sold per

month
Income = Expenses
55x = 20x-t2x +3,500+ 25,000+12,000
33x =40,500
x = 1,227.3 blocks

i f~t: X ::::

10er of units to be forged

:'Ill

Income = Expenses

1il

65x = 50x + 35,000

'I.
:I

x "' 1,228 blocks

'l!lllli,l
i

x "' 2,334 cases

I'

,11,1111
1.11,,11

Let: x = number of motors to be sold each


month

!;

Income = Expenses
125x

580x = 85x + 350 + 20x + 15,000


CR = Annual Cost

Let: x = number of cases to be sold each


year

275x = 150x + 20,000

ml
Let: x = number of units to be produced
per month

Let CR =Capital recovery rate per year

x"' 1,015 units

x = 2, 333.33 cases

-1,000)(0.04)
3
(1+0.04) -1

= 7,164.36

\.

15x = 35,000

200x = 200,000 + 160x

+ 0.32C 02 - 320.35

125x = 56x + 70,000

x"' 1,053 units

Income

AC 1 = AC 2
2,258.82

Income= Expenses

1,052.42

Let: x = number of units to be sold out per


year

Equating:

Let: x = number of units to be forged

x = 1,014.49 units

406.68x = 428,000

AC = (C )(0.04) +(Co2 -1,000)(0.04)


2
02
3
(1 + 0.04) -1

69x = 70,000

600x =c 115x + 76x + 2.32x + 428,000

AC = (C )i +(Co2- Cn2)i
2
02
(1+if-1

Day 23 -Engineering Economics (Annuity, Depreciation, Bonds, etc.) 579

=20,000

x = 160 motors

:I!

582 I Op i Solved Problems. in En!1eermgMathexnatics~2nd Edition)by Tiong& Rojas

Appendix A:

Appendix A - Glossary 583

Glossary

' .r ' " " ~ '

' .;,

"" ">-'

. ~ ~

<'

>! ,., .. """""


........ ~.-

-~.-

-~

' ~' < ' '

_;,

,..

""

abscissa the position of any point on a plane from


the y-axis. Also known as 'the x-coordinate.
The plural form of abscissa is abscissae.

" < .. ~ < :

> i

-~

.( ~,

M"'

..

.<

,0

'f~

....

~ --~

-<

" : < "'

>?:' "'"""'""'

~--. -~-- ~ ~ ~
.:~

~ ... " ' ' ~ ~ "j/>-., ~'

... ~.-_...

'!>~~-"~-~

\ ,...!...-'I'S '>~';?, f'~<'<

.. '

'"-

,.

. ...

.h

<!>'

', , . ' ., ,. <' " .

h" 'f " ... 1" ' ':: '

,.,,,

~-"'

;- ,..,., -~.. 4

-~

.....

,,;.~~~"""""'
~ 'II: .'

'1)"'

' i .....

,, .. "''* .,

....

_,,..~~--

~ ~-~

'

'

)' :'<"'

vrdinate

,.,;-.~;.

-~~

',(.<

.,

N 1 "'

o"O

-~"

~ < -/.:,,>

. .; ,, ~

..

~,

. ..,

<:'

.. ,
,. -;;."' ' .,_ ,

< ~ ~ 1

,,

...

-., "

'" ~ ;.

..

,.~~,.-,_~'-~~

..

"

~ ~ 'J ~ .,

'

,Y

. , . .,,

...., ;

<i'

Y,

<

'

'

:..,

'l, ~ ~

. i ~

" ,

...
"'

...

,.

'

'

'

'

-axis

accuracy a measure of precision of a numerical


value !!f some quantity.

>-I
abscissa

'

'

Rectangular coordinate system showing


abscissa and ordinate. The abscissa of
point Pis x.

the

absolute term (syn. constant term) a term in an


expression which does not have a variable .
absolute value the number written arithmetically
omitting the sign that prefixed it. The absolute
value of the number x is denoted as I x I

<

' '
.., -.'

!Y

,
On'gin(O,O) I<

absolute error the difference between


approximate value and the exact value:

");':'

',x

...

* -~ .... ~ '

'

,.... ,

~ ~

..

''
...

t " .. '

('-"""'"'~""""...;,..,~~

. -~
~

P(x,y)
-~

~."' .... '"""' "; ....... - ....~ <

-~ ~ """" ..... :' ,... " h ( ' .,. : t' "'

\\"''

" v

acceleration the rate of change of velocity per unit


time. Acceleration is a vector quantity. The
standard unit is meter per second per second
(m/s2). The gravitational acceleration on earth
is denoted as g and is equal to 32.2 feet per
second2 or 9.81 meters per second2

v-axis

< r/,;!J'->1.:>

~.., ,.,..""" ,.~ "-"'""-~-~ ~ .,, l<- ~"'."1

~-

.............

~ ~ ~ "'"' .. ' >

abundant number a natural number where the sum


of its distinct factors exceeds the number. For
example, the factors of 12 are 1,2,3,4 and 6.
The sum ol the factors is 16. Since 16 is
greater than 12, hence 12 is an abund.ant
number.

,..,.,_' *!.....,

, ~.:-.. 'I ,,. ' ~-. " , .., fi,<" " ~-" ' _,; :' ._,

"'''

~ " <

",. "" *

..... ,...,,.....
!:' "

t ' '<," \

the number 10 in the hexadecimal number


system

, , . , ..... .,. .

i'i=-~~,~-,,,

<

abstract algebra the part of algebra that deals with


study of groups, semiiJroups, rings, modules,
fields and similar structures.

'

.,_,..

-~

'

, ~ :'. '

acre British unit of area which is equal to 4840


square yards. In metric equivalent, 1 acre =
4046.8 square meters or 0.40468 hectares.
acute refer to something less than 90 It comes
from Latin 'acus" which means "needle"
acute angle an angle less than a right angle or 90
degrees.

~<90
The angle {1 is an acute angle.

absolute value of complex number the distance of


the complex number from the origin when the
complex number is represented as the point
with rectangular coordinates (a,b). Absolute

acute triangle a triangle having all angles acute


angles.

Ja

add to combine numbers of quantities by getting the


toti:il number of units contained in them.

value of a+ bi is

+ b2

absolute value of a vector the numerical value of


the length of a directed line segment
representing the vector. Absolute value of the
vector ai + bj + ck is

Ja

+ b2 + c2

addend the number added to a certain number to


produce a sum. Example: In -the equation 5 +
2 = 7, the number 2 is the addend whi,e the
nu~ber 5 is the augend and 7 is called the
sum.

.584 _1001 Sol~d Pro~el!!!'! in Engi_neering Mathematics (2nd E<,!~tionLey Tiong:_& Rojas.
addition the process of combining numbers or
quantities. The result of addition is called sum.

""'K{
ld

addition formulae formulae which express a


trigonometric functions of the sum or
difference of two angles in terms of the
function of the individual angle. The foilowing
are the addition formulae:
sin (A +B) " sin A cos B +cos A sin 8
sin {A - 8) :: sin A cos B - cos A sin 8
cos (A +B) = cosA cos B -sin A sin B
cos (A - B) = cos A cos B +sin A sin B
tan A +-tan B
tan(A +B)= --------1-tan A tanS
tanA-tanB
lan(A- B)= - - - - - - 1+tan A tan B
additive identity an identity clement of an additive
operation. The additive identity is the integer
zero.
additive inverse the negative of the numbHr. The
additive inverse of 5 is -5.
ad infinitum (latin term) continuing with no end.

algebra the study of operation and relation among


numbers through the use of variables or literal
symbols instead of just constants. The Latin
term "algebra" comes from the Arabic "al-jabr"
which means "to set or consolidate'.

algebraic equation an equation of the form f(x) = 0


where f is a polynomiai of dr;.'gree n with
coefficients in a given base field, usually
rationals, n is the degree of the algebraic
equation. For example, x2 + 3x + 4 = 0 is a
second degree (n = 2) algebraic equation.
algebraic expression any combination of symbols
and numbers related to !he fundamental
operation of algebra.

algebraic number a real number that is a root of


a polynomial equation with integer
coefficient
algorithm a step by step procedure by which
operation can be carried out.

an

aliquot part refers to any divisor of a number


that is not equal to the number itself. This is

adjacent angles two angles that have !he same


point or vertex and a common side between
them.

Aijabr wa'l muqabalah Rough Translations,


Balancing Equations, an Arab texbook written
in the early 800s by AI-Khowzrizmi (from
whose name the word algorithm was coined).
This is where !he name algebra came from
and from this text Europe came out of the dark
ages and learned algebra.

Q{o_
8, and f3 are adjacent angles
agonic another !enn for skew as in agonic line or
skew line.(see skew)

aleph the first letter in the Hebrew alphabet, ~.


This letter was first used in mathematics by
Georg Cantor to denote the various orders,
or sizes, of infinity

also known as proper divisor.

alphanumeric (syn.
alphameric
character)
combination of alphabet, numerical symbols,
punctuation marks and other symbols used in
computer works.
alternate angles either pair of angles contained
between two given lines and a transversal
and lying on opposite sides of l.he transversal.
These angles are equal if the given lines are
parallel.

284, while the numb_er 284 have proper


factors of 1, 2, 4, 71, and 142 which sums up
to 220.
ampere an Sl unit of electric current

algebraic curve a curve !hat describes an algebraic


equation

adjacent lying next to. An adjacent angle (side) is


an angle (side) that lies next to anothei angie
(side).

)p

Appendix A - Glossary 585

(a)

(b)
(}and f3 are alternate angles.
Figure 5b shows equal alternate angles since
the lines are parallel.
alternating series an infinite series whose
successive terms are alternately positive and
negative, such as
1 -1/2 + 1/3- 1/4 + 1/5 .....
altitude a line segment drawn from a vertex
perpendicular to the opposite side (called
base).
altitude of a trapezoid a perpendicular distance
between the bases or parallel sides of a
trapezoid.

amplitude the maximum displacement between in


either positive or negative direction from a
reference level.
amortization as applied to the capitalized cost the
distribution of !he initial cost by periodic
charges to operation as in depreciation or the
reduction of a debt by either periodic or
irregular prearranged program.
anallagmatic curve a curve that is invariant
under inversion. This curve includes the
Cassinian ovals, Lirnacon of Pascal,
Strophoid, Cardioid, etc.
Analytic Geometry the branch of Mathematics
which deals with the properties, behaviors and
solutions of points, lines, curves, angles,
surfaces and solids by means of algebraic
method in relation lo the coordinate system.
This was invented by Rene Descartes.

altitude of a triangle the perpendicular segment


from a vertex of the triangle to the line
containing the opposite side. It also refers to
angle the basic figure formed by two line segments
the perpendicular distance from the apex of a
or rays with a common end point. The symbol
triangle to the base.
for angle is".!". Angles are measured in
different units, namely degrees, radians, grads
ambiguous case the case of a solution of a plane
and mils.
triangle where the given data. lead to two
1 revolution = 360 degrees
solution:;.
c
= 2n radian
= 400 grads
= 6400 mils

A~B
8 - .............:..... .

Figure shows that

if sides a, band angle

A are given, each of the two triangles


ABC and AB'C satisfies the given

angle, of depression or elevation the angle


between the horizontal and the line of sight to
the observed point. If the observed point is
below the horizontal from the observer, it is an
angle of depression. It is an angle of elevation
if the observed point is above the observer.

conditions.
Object

amicable numbers two integers such that each is


equal to the surn of the distinct proper factors
of the other. An example of amicable numbers
are the numbers 220 and 284. The number
220 has a proper factors of 1, 2, 4, 5, 10, 11,
20, 22, 44, 55 and 110 which have the sum of

Lineof~-x
horizontal

Angle of elevation

,,,....,.

honzontal

~\f~"x
f Obiect
Line of sight __

Angle of depression

586 1001 Solved Problems in Engineering Mathematics (2"d Edition) .by Tiong & Rojas

w.

~I

angle of inclination the smallest positive angle that


the straight line makes with the positive x-axis.

apotome a number that has the form of

)(JA- JB).

This

is

number

angular relative to or in terms of angles such as


angular acceleration, angular velocity etc.

categorized by Euclid as one of the


irrational numbers

annulus (syn. circu/auing) a plane figure which


contains an area of a ring-shaped region lying
between two concentric circles. Another term
for annulus is ring. Area of annulus is 1t(R2 r2).

appreciation (ant. depreciation) increase in !tie


value of an asset.
approximation a number which is a close estimate
of another number. The symbol of
approximation is "'
arabic numerals the symbols 0,1 ,2,3,4,5,6, ... that
represent the counting system in the decimal
number system (base 10). (see numerals)

Annulus or Circular ring

annuity a series of equal payments occurring at


equal interval of time. Type of annuity are,
ordinary annuity, annuity due, deferred annuity
and perpetuity.
annuity due a type of annuity where the payment is
made at the beginning of each period starting
from the first period.
antecedent the first term of the ratio. In the
2:3, 2 is the antecedent and 3 is
consequent.
(See
consequent).
antecedent is equal to the product of the
and the consequence.

ratio
the
The
ratio

antilogarithm the inverse function of a logarithm.


antiprism a semi-regular polyhedron from two nsided polygon and 2n triangle.
apex the highest point of a figure with respect to the
base or plane of the base.
0

E~C

arbitrary constant a non-numerical symbol holding


a place for an unspecified constant. For
example, in the general linear equation y = mx
+ b, m and b are arbitrary constants while x
and y are variables.
arc a part of a circle between two points on the
circle. A portion of a curve between two of its
points A and B.
Archimedes Principle Any body immersed in a
fluid is subjected to a buoyant force which is
equal to the Weight of the fluid displaced. This
was discovered by Greek Mathematician,
Archimedes( 287- 212 B.C).
Archimedean solid a solid made from regular
polygonal sides of two or more types that
meet in a uniform pattern around each
corner. It is a convex semi-regular
polyhedron.
Archimedes' spiral (syn. Archimedean spiral) spiral
with the polar equation r = ae, with its graph
as shown in the figure. The radius vector,
increases with polar angle, e, a is the constant
of proportionality.

8
Dis the apex of the polygon
'

apothem the radi~s of the circle inscribed in a


regular polygon .. A line from the center of a
polygon perpendicular to one of its sides.

----1--t~------X

Archimedes' spiral

IIIII

Appendix A ~Ql()f!l!~ 587


are a larger unit of area in the metric system. It is
equivalent to 100 square meters.
area a numerical measure of a two dimensional
geometric figure enclosed within a specific
boundary
Argand diagram (syn. Gaussian Plane) the
rectangular coordinate system used for the
representation of a complex number. The xaxis and the y-axis are known as the axis of
reals and axis of imaginaries, respectively.
Named after the Parisian bookkeeper, Jean
RobertArgand (1768 -1822):
argument (1) the angle between OZ, where Z is
the point representing a complex numbE;r
on an Argand diagram and 0 is the origin,
and the real axis (2) theinput of a fUnction
(3) an informal mathematical proof
arithmetic the branch of mathematics which deals
with calculation of integers using the
fundamental
operations
of
addition,
subtraction, multiplication, division and the
extraction of roots.
arithmetic mean (syn. mean, average) the sum of
'n" numbers divided by n. For example, the
mean of 2,9 and 7 is 6.
arithmetic progression is a sequence in which the
difference between any two successive terms
is a constant and is called the common
difference.
arithmetic series the sum of the terms of an
arithmetic progression.
arm one of the lines forming the a:1gle.
array an arrangement of numbers in rows and
columns (see matrix)
assets refers to everything a company or
corporation owns and has a money value.
Assets are as current assets (cash, bank
account and other items that can be converted
into cash), trade investment (investment in
associated companies), fixed assets (land,
building, etc) and intangible assets (goodwill,
patent, etc.)

astroid (syn. ~tar curve) a hypocycloid with four


cusps and with parametric equation of x =
cos3 t , y = sin3 t. The rolling circle of this
hypocyloid has a diameter one-fourth that of
the fixed circle.

, I

Astroid

astronomical unit (AU) the mean distance between


the earth and the sun. It is about 1.495 x 108
km.
asymmetric not symmetric
asymptote a straight line that approaches the curve
more and more closely but never really
touches it except as a limiting position at
infinity. The word asymptote was coined by
Thomas Hobbes (1588- 1679), using various
latin stems meaning roughly "to fall together
but not touch".
atmospheric pressure the pressure caused by the
weight of air at a given point. Standard value
is 14.7 pounds per square inch or 760 mm of
mercury. It is also equal to the weight of a
column of water about 30 feet high and in
metric system, it is equal to 100 kPa.
Aubel's theorem "Given a quadrilateral and a
square drawn on each side of it, the two
lines connecting the centers of the squares
on opposite sides are perpendicular and of,
equal length."

II,
II,
I

!I

augend the number or quantity to which the addend


is,added. Example: 5 + 2 = 7. The number 5 is
the augend while 2 is the addend.
automorphic number a number n whose square
ends in n. Example: 5 is an automorphic
number because square of 5 is 25 and 25
ends with the number 5.
auxiliary ~ircle a circle with radius equal to half the
major (transverse) axis and its center is at the
center of the ellipse (hyperbola).

'"!!

588 l 001 Solved Problems in Engineering Mathematics (2nd Editi<m) by Tiong & Rojas

w.

Ellipse

Bang's theorem 'If all the faces of a tetrahedron


have the same perimeter, then the faces are
all congruent triangles."

l&J

K~

bar graph a chart or diagram consisting of


horizontal or vertical rectangles or bars, each
of which represents an interval of values of a
variable and has height proportional to the
quantities.

auxiliary_____fi
arde

Auxiliary circle
Avagadro's constant the number of atoms of n
grams in an element with atomic weight n. The
value of this constant is 6.02214199 x 1023
This was named after the Italian phys1cist
Amedio Avagadro (1776 -1856)
average the usual term used for arithmetic mean.
average acceleration the change in velocity of a
body divided by the time interval during which
the change occurs.

barycenter (see center of gravity)


base a 'side of a polygon wh1cn 1s at the bottom of
the orientation.

Bertrand's conjecture 'If n is an integer greater


than 3, then there is at least one prime
number between n and 2n - 2." This postulate
is named after the French mathematician
Joseph Bertrand (1822- 1900)

------------------------=AP:.cP:.:::e::.n:::d=ix A- Glossary 589


possible outcomes of an event (i.e. success and
failure) and the possibilities if the outcomes are
independent and constant
binomial expansion expansion of a binomial in the
form of (x + y)" in accordance with the
binomial theorem.
binomial theorem the theorem that gives the form
of the expansion of any positive integral power
of a binomial (x + y)". Its general equation is
(x+y)"=x"+nxn1y +n(n-:!.)_xn-2Y2+ ...
2!
... +n x yn-1 +yn
bisect to divide a geometric figure into two equal
parts.
bisector (syn.bisectrix) a line or plane that bisects a
given angle or line or any geometric figure.

bicorn any of a collection of quartic curves that has


a rectangular equation of y2( a2 - x2) = (x2 +
2ay- a2)2. This is also known as "cocked-hat".

bit (abbreviation of binary digit) the digits 0 and 1 in


the binary number system.

average velocity the displacement of a body


divided by the time interval during which the
displacement occurs.

bicuspid curve a quartic curve with an equation (x2


- a2)(x- a)2 + (y2- a2)2 = 0

book value the recorded current value of an asset.


The value of an asset that is recorded in the
book of records of the corporation.

axiom a statement of truth which is admitted without


proof.

bilateral having two sides .or relating to the right


side and left side of an object

axiom of induction 'Any property that belongs to


zero, and also to the immediate successor of
any natural number to which it belongs,
belongs to all natural numbers.'

billion refers to one million million (1012) in the


United Kingdom and Germany and one
thousand mi~ion,(109) in the United States and
France. In US and France, 1 million =
1,000,000.

axis the fixed reference line used in a coordinate


system.

binary relating to the binary notation or binary code.

axis of symmetry a line around which a geometric


figure is symmetrical.

binary logarithm logarithm of a number to the base


2

axis of the conic the line through the foc;us and


perpendicular to the directrix.

binary number system (syn. Dyadic number


system) a system of notation for real numbers
that uses the place value method with 2 as the
base. Only two digits are considered, 0 and 1
sometimes called as 'bit' (abbreviation of
binary digits).

B
B the number 11 in the hexadecimal number
system.

binol1Jjal a mathematical expression of two terms.


Example: 5x + 4y
binomial distribution (syn. binomial probability) the
di:;trillrllion of p10hahilil1P:. wlmrn lhnm are two

Boolean Algebra an algebra which deals with the


operation of complementation, union and
intersection. It is devised by the British
mathematician George Boole (1815 - 1864)
who is best known for his innovatory work in
formal logic.
Boyle's Law At constant temperature, the pressure
is inversely proportional to the volume.

bundle a family of lines or planes which all passes


through a single point.
butterfly theorem Let M be !he midpoint of a chord
PQ of a circle, through which two other chords
AB and CD are drawn. If AD intersects PQ at
X and CB intersects PQ at Y, then M is also
the midpoint of XY. The resulting figure of this
theorem forms a butterfly.

,I!! I
,rll

byte a sequence of bits; a unit of information


equivalent to a single character; a unit
capacity of a computer.

c
C the number 12 in the hexadecimal number
system. The number 100 in Roman numerals
calorie the amount of heat required to raise the
temperature of one gram of water 1C.
calculate to determine the value of a given
mathematical procedure; to compute.
Calculus the branch of mathematics created in the
seventeenth century by Isaac Newton(16421727) and German mathematician, Gottfreid
Wilhelm von Leibniz (1646-17'16) which rest
on the basic principles of limits. Calculus is
divided into two subjects, namely, Differential
Calculus and Integral Calculus.
canonical form a form of any given polyhedron
distorted so that every edge is tangent to the
unit sphere and the center of gravity of the
tangent points is the origin

Brianchon's theorem 'Given a conic section, if we


circumscribe a hexagon about it, then the
major diagonal of the hexagon are
concurrent.'

cap the symbol n, which is used to denote the


union between two sets

Briggsian logarithm (see common logarithm)

cardinal numbers numbers used to count objects.


Example: one, two, three, ... twenty...

l:lritish thermal unit (BTU) the amount of heat


required to raise the. temperature of one
pound of water 1" F. 1 BTU = 252 calories

cardioid a heart-shaped curve generated by a fixed


point on a circle as it rolls roung another circle
of equal radius. Its equation is = a(1 - cos<j>).

bulk modulus the ratio of the volume stress to the


volume stress.

II

il
II

"!'
.1

590 100 1 Sblved Problems in Engineering Mathematics (2nd Edition) by Tiong & Rojas
Carmichael number a number n that is a Fermat
pseudoprime to any base, that is, it divides (a"
- a) by any a. This number is also known as
"absolute pseudoprime'
Cartesian
coordinates (syn.
rectangular
coordinates) a method of locating a point by
pair of numbers denoting the distances from
two fixed reference intersecting lines. The first
number is called abscissa which is the
distance from the y-axis while the second is
called ordinate, Which is the distance from the
x-axis. The two intersecting lines are called
coordinates axes.
r.ash flow the flowback of profit plus depreciation
from a given project.
Catalan solid a polyhedron that is a dual of an
Archimedean solid. These solids are named
after the Belgian mathematician Eugene
Catalan (1814- 1894)

ceiling function the function of a number x that is


the smallest integer that is not smaller than x.
center of gravity (syn.
barycenter ) the
resuliant of. the
passes regardless
body in soace.

centroid, center of mass,


point through which the
distributed gravity force
of the orientation of the

center of mass (see center of gravity)


centesimal degree (see gon or grad)
centillion a number 10 raised to a power of 600 or
10600 .
central angle an angle whose vertex is at the
center of a given circle.
central conic a conic with a center. Examples are
ellipse and hyperbola. The only non-central .
conic is the parabola.

Am>enciix A- Glossary 591


center. Its equation is (x - h)2 + (y - k)2 = r2
where center IS at (h, k) and radiU$ is r. The
parametric equation is x = r cos 8 or y = r sin
8. The word circle comes from Latin "circus"
which means 'large round"
circular cone a cone whose base is a circle.
circular cylinder a cylinder with a circular right
section.
circular prime a prime number that remains prime
on any cyclic rotation of its digits.

circumcenter the point of concurrency of the


perpendicular bisectors of the sides of a
triangle.
circumcircle a circle that circumscribed a given
polygon.
circumference the boundary of geometric figure,
especially a circle

centrifugal force a force acting outward on a body


as it traverses a curvilinear path

circumscribe to draw a geometric figure around


another geometric figure in such a way that
they are in contact but does not intersect.

catenoid the surface generated when a catenary is


rotated about its directrix.

centroid (see center of gravity)

cissoid a curve with a rectangular equation of

cathetus a line that is perpendicular to another line.


This usually refers to one of the lines in a right
triangle that is not the hypotenuse

century a period of 100 years. The word century


comes from Latin 'centuria' whfch means 'one
hundred"

caustic the envelope of rays of light reflected or


refracted by a given curve from a given point
source of light

.chain rule a rule of differentiating a function of a


function , f [u(x)]:dj/dx = (dj/du) (du/dx)
characteristic the integer part of the logarithm.

Cavalieri's Principle Given two solids and a plane.


Suppose that every plane parallel to the given
plane, intersecting one of the two solids, also
intersects the other and gives a cross-sections
with the same area, then the two solids have
the same volume. Named after Bonaventura
Cavalieri (1598- 1647).
Cayley's sextic a sinusoidal spiral curve with a
rectangular equation of 4(x 2 + y2 -ax)3 =
27a2(x2 + y2)2. This curve is named after
Arthur Cayley

chord a segment whose end points lie on the circle.


chord of contrast the chord joining the points of
tangency of the two tangent lines from a point
P outside the circle.

cipher an old name for zero. lt is derived from


Ara.bic "sifr", Latin "cephirum" and Italian
'zevero".
circle a close plane figure every point of which is
equidistant from a fixed point called the

coefficient of restitution the ratio of the total


momentum after collision to that of before
collision. It is denoted as 'e'.
coefficient of static friction the ratio of the limiting
frictional force (maximum) to the normal force.
The coefficient of static friction force. The
coefficient of the static friction is always
greater than the coefficient of kinetic friction.
coefficient of volume expansion (see bulk
modulus)

circular ring (syn. annulus) see annulus

central tendency a central value between the


upper and lower limits of a distribution around
which the scores are distributed.

catenary a plane curve described by a heavy


uniform, flexible cable ha'lging freely between
two points. The term 'catenary" comes from
the latin word which means chain. Its equation
is y =a cos h (x/a).

coefficient of kinetic friction the ratio of the


frictional force to the perpendicular force. It is
denoted as mu (J.!).

cofunction (syn. complementary function) In


trigonometry, the function of a pair have equal
values for complementary angles.
Example: Sin 30 = Cos (90 - 30) or Cos
(60). Hence, sine and cosine are
complementary functions.
coincident having all points in common.
collinear points points that lie on the same line.
cologarithm the logarithm of the reciprocal of a
number.
columns the numbers in order which appears
vertically in a matrix.

x3

y2 = - - (2a- x)
clelia the locus of a point P that moves on the
surface of a sphere in such a way that <j>/8 is
constant, where 4> and 8 are the longitude and
colatitude, respectively
cochleoid a spiral curve that has the rectangular
1
2 2
equation of ( x + y ) tan-

U)

ay and a

.
asine
polar equat1on of r = - - . .

coefficient In algebra, it refers to the numerical


factor of a term. For example: In the term 5x, 5
is the coefficient of the term.

combination an arrangement of a set of objects in


no specific order.
The combination of 'n' different things taken
'r' at a time is given as
n!
C(n,r)=~.
\n- rI'11
r,
If taken all, C1n.n1 = 1
combinatorics the study of the ways of choosing
and arranging objects from given collections
and the study of other kinds of problems
relating to counting the number of ways to do
something.
commensurable refers to a ratio of two values that
results to a rational number. Example: If two
distances have a ratio which is a rational

Appendix A- Cl?ssary

59Z 100 l Solved Problems in Engineering Mathematics (Z"d Edition) by Tiong & R~

"-'

~I

number, then the


commensurable.

two

distances

are

common denominator an integer or polynomial


that is exactly divisible by each denol111nator.
common difference the difference of two
successive terms of an arithmetic progression
or sequence.
common factor (syn. common divisor) Of two or
more integers or polynomials, an integer of
polynomial which is a factor of each. For
example: 5 is a common factor of 20 and 25.
common logarithm (syn. Briggs logarithm)
logarithm to the base 10. This was created by
the geometry professor of Gresham College in
Lo11don, Henry BrirJgs (1561-1630) as an
improvement of the natural logarithm.
common multiple an in!eger or polynomial that is
multiple of each in a given set. For example:
90 is a multiple of 15 and 18.
common tangent a line that is tangent to two or
more curves.
commutative law law stating that the sum or
product is unaffected by the order of the
terms. In addition, a + b = b + a. In
multiplication, a times b =b times a.
complementary angles two angles whose sum is
90" or right angle
completing the square the process of modifying a
quadratic polynomial to obtain a perfect
square (trinomial).
complex fraction any fraction which contains one
or more fractions in either numerator or
denominator.
complex number a number of the form a + bi with a
and b real constants and i =the square root of
~.

composite numbers an integer which is the


product of two integers, both different from 1
and -1. The integer 15 is a composite number
since 15 "' 5 limes 3.

conjugate angles (see explemenlary angles)

compound interest the interest charges under !he


condition that interest is charged on any
previous interest earned in any period of time,
as well as the principal. Fomula for future
amount of a principle in compound interest is
F =P(1 + i)", with n as the number of periods
and i is the interest per period.

conjugate arcs two circular arcs together make a


full circle
conic (see conic section)
conic section the locus of a point which moves
such that its distance from a fixed point (focus)
is in constant ratio, e (eccentricity) to its
distance to a fixed straight line (directrix)

compound number a quantity expressed in


different but related units. Example. 5 hours
and 32 minutes.
computable number a real number for which there
is an alqorilhm !hi, given n, calculates the nth
digit
concentric circles circles having the same center
with unequal radii.
conchoid a shell-shaped curve that has a
. o{
rectangular
equation
a(x- a)(x 2 + y2 ) = k2 x2
conclusion a oart of the theorem which is to be
proved.

coordinates the abscissa and ordinate together,


Also known as rectangular coordinates or
Cartesian coordinates.

conoid a surface or solid formed by rotating a conic


section about one of its axes.

corollary a statement of truth which follows with


little of proof trorn the theorem

consequent the SflCOnd term of a ratio. See


antecedent

corporation (type of business mganiza!ion) a

continued fraction a fraction in the form of:


1
x = ao +1
a1 + - -,----1-a2 +-- --1

conditional equation an equation which is satisfied


by some, but not all, of the values of the
variables .for which the members of the
equation are defined.

convex polygon a polygon with no side extended


will pass through the cenler of ihe polygon.
Also it contains no interior angle gr<Jater !han
HJ0. Regular polygons are said to be convex.

coplanar r)oints set of points that lie on the same


plane

constant term a term in a polynomial thai does not


contain a variable

concurrent having a common point.

conclusion and the conclusion becomes the


hypothesis

conical surface a surface generated by a moving


straight line (generator) which always
intersects a fixed plane curve (directrix) and
which always passes through a fixed point
(vertex) not in the plane of the curve

constant of integration an arbitrary constant term


in the expression of indefinite integral of a
function.

concrete number a number that counts a physical


quantity. For example~5 apples, 8 cats

59~

coprime refers lo two or more numbers thai have


no factors in common other than 1

no

distinc;t legal entity separate from the


indi~iduals who own it and which can engage
in any business transaction which a real
person can do.
cosine law a law which relates the sides and angle
of an oblique triangle. I! is used in solving the
parts of an oblique triangle.
cotermina! angles angles of rotation
same initial side and terminal
coterminal angle of 75 is 435 and
difference of coterminal angles

which the
side. The
-285. The
is always

360".

a+--

cone a solid bounded by a conical surface (lateral


surface) whose directrix is a clbsed curve and
a plane (base) which .cuts all the elements.

confocal conics two conics which share the same .


focus. A confocal . ellipse and hyperbola
intersect at right angle.
congruent geometric figures having identical shape
and size.

a4 ...

continuity a property in mathematics that refers to


smoothness of a function or curve

couple a pair of forces equal in magnitude, opposite


in direction and not in !he same line

continuum any set that can be bought into one-toone correspondence with the set of real
numbers

Cramer's rule a method of solving linear equations


of several unknowns simultaneously using
determinants or matrices. II is named after the
S,yiss mathematician and physicist Gabriel
Cramer (1704 -1752).

convergent A convergent sequence has a finite


limit. A convergent series has a finite sum
Opposite of convergent is divergent.

conjecture a mathematical statement which has


neither been proved nor dehied by
counterexamples. Some of the famous
conjectures are Fermat's Last Theorem and
Goldbach Conjecture.

converse of a theorem* another theorem wherein


the hypothesis and conclusion'of the first are
reversed; that is, the hypothesis becomes the

counting numbers (see natural numbers)

critical point (syn. Stationary point) a point at which


a function has a first deril,talive of infinity, thus
havipg a slope which is vertical.
cross product (see vector product)

!I!~

Appendix A- Glossary 595

594 1001 Solved Problems in Engineering Mathematics (2nd Edition) by Tiong & Rojas.

de Moivre's' theorem A theorem that links complex


numbers and trigonometry. It states that for
any real number x and any integer n,

crunode a point where a curve intersects itself so


that two branches of the curve ,have distinct
tangeo1t lines

cyclic number a number of n digits, which, whem


multiplied by 1, 2, 3, 4, ... , n, produces the
same digits in a different order.

cube a polyhedron whose six faces are all squares.


A cube is a regular hexahedron.

cyclic polygon a polygon whose vertices lie on a


circle.

cubic curve an algebraic curve described by a


polynomical equation of the general form ax3 +
bx2y + cxy2 +dy3 + ex2 + fxy + gy2 + hx + iy + j
=0

cycloid the plane curve traces out by a fixed point P


on the circle as the circle rolls along a line, the
base of the cycloid

decahedron a polygon with ten faces. There is no


such thing as a regular decahedron.

cubic equation a polynomial equation of the third


degree.

cylinder a solid bounded by a .closed cylindrical


surface and two parallel planes (see circular
cylinder)

decimal fraction number that consists of an integer


part (which may be zero) and a decimal part
(less the one) that follows the decimal marker,
which may be a point or a comma.

cubit a measure of length which is approximately


equal to the length of a person's forearm (from
the elbow to the fingers). A Roman cubit is
equal to 17.4 inches while the Egyptian cubit
is equal to 20.64 inches

cylindrical surface a surface generated by a


moving straight line (generator) which is
always parallel to a fixed line, and which
always intersects a fixed plane curve
(directrix) not in the plane with the fixed line

cuboctahedron a polygon obtained by cutting the


corners oft a cube or an octahedron. A
cuboctahedron has eight faces. that are
equilateral triangles and six faces that are
squares ..
cuboid hexahedron will all faces rectangles
Cunningham chain a sequence of prime numbers
in which each rnember is twice the previous
one plus one.
cup the symbol v, which is used to denote the
union of two sets
curvature a measure of the amount by which a
curve deviates frorn a straight line
curve (see locus)
cusp a point on the curve where two branches
coming frorn different directions, meet and
have a common tangent. The word cusp
comes from Greek "kera" which means "horn"
and from Latin "cuspis" which means "sharp"
cute number a number such that a square can be
cut into n squares of, at most. two different
sizes.
cyclic quadrilateral a quadrilateral with all vertices
lie on a circle

cylindroid a cylinder with elliptical cross-section.

D
0 the number 13 in the hexadecimal number
system; the number 500 in the roman
numerals ..
Oandelin spheres the two spheres that just fit
inside the cone, one on each side of the plane
and both tangent to it and touching the cone,
when the cone is 'sliced through by a plane

(cosx+isinx)" =cos(nx)+isin(nx)
decagon a polygon of ten sides

decimal number system a system of notation for


real number that uses the place value method
with 10 as the base.
decimal point a dot place between the integral and
fractional parts of a number.
declining balance method a method of computing
depreciation in which the annual charge is a
fixed percentage of the depreciated book
value at the beginning of the year to which the
depreciation applies. This method is also
known as percent on diminishing value.
decrement the negative.of an increment.
defective equation any equation which, because
of some mathematical process, has fewer
roots than its original
defective number (see deficient number)

0' Alemberts principle When a body is subjected


to an acceleration, it is reacted upon be a
reverse effective force opposite in the
direction of the acceleration to balance. The
value of the reverse effective force is equal to
the mass time acceleration. Named after the
French mathematician and physicist, Jean Le
Rond d' Alember (1717 - 1783).
de Malves' theorem "Given a tetrahedron in which
the edges meeting at one vertex, X, form three
right angles (i.e. the tetrahedron is the result
of chopping off the corner of a cuboid), the
square of the face opposite X is equal to the
sum of the squares of the other three faces."

deferred annuity a type of annuity where the


payment of the first amount .is deferred a
certain number of periods after the first.
deflection angle (syn. exterior angle)
deficient number (syn. defective number) any
natura! number the sum of its proper divisors.
All prime numbers are deficient numbers.
definite integral an expression of integrating an
integrand between two limits of integration.
Integrals with limits are definite integrals.
degenerate conic conic obtained when the cutting
plane passes through the vertex of the cone.

II

,,

Example of degenerate conics are the pointellipse, two coincident lines and two
intersecting lines
degree 1. A measure of an angle which is equal to
1/360 of a revolution.1degrce = 60 minutes
and 1 minute = 60 seconds 2. The highest
exponent or sum of exponents in any given
term of a polynomial. The polynomial 7x5y2 +
8x4ys:2x2y is 9th degrees polynomial.
delta curve a curve that can be turned inside an
equilateral triangle while continuously making
contact with all three sides.
deltahedron a polyhedron whose faces are all
equilateral triangles that are all of the same
sizes
deltoid a non-convex quadrilateral with two pairs of
adjacent equal sides.

, I~

. il

denominator the number written below in a given


fraction. The nurnber written above is called
the numerator. For example, 3/4 has a
denominator of 4.
'

ri'

density (syn. specific weight) the mass per unit


volume of a substance.

lli

depletion the lessening of the value of an asset due


to the decrease in the quantity available. This
refers to the natural resources such as coal,
oil and timber in the forest

'!

lj
I

depreciated book value t~e first cost of the


capitalized asset minus the accumulation of
annual depreciation cost charges
depreciation the decline in the value of an asset
due to the passage of time and constant use.
determinant a number which is expressed as a
square matrix (with the number of rows equals
the number of columns)
deviation in statistics, it refers to the differenGP.
between any one of the sequence of observed
value of a variable to some value such as the
mean.

!,II

.~!Ii

J1
596 '1 001 Solved Problems in Engineering Mathematics (2nd Edition) by Tiong & Rojas

'ID"

devil's curve a curve with a rectangular equation of


y4 - a2 y2 = x4 - b2 x2 and the polar equation
r2 ( sin 2 0- cos 2 0) = a2 sin 2 0- b2 cos 2 0
diagonal a segment joining two non consecutive
vertices. It may be calculated using the
formula, n/2 (n-3), where n is the number of
sides of the polygon
digit any specific symbol use to denote a number
whether singly or in combination.

Diophantine approximation the approximation of a


real number by a rational number
Diophantine equations If there exist more
unknowns than the number of equation but
still can be solve bec(luse the values of the
unknown are integers, these equations are
called Diophantine equations (named after
~ Diophantus of Alexandria, a Greek
mathematician" in the 3rd century BC).
Diophantus in his book 'Arithmetica' carried
out his extensive study to the solution to
indeterminate equations.

diameter a chord containing the center of the circle


difference the result of subtraction.
differential calculus a branch of calculus that deals
with the evaluation and use of derivatives and
differernials.
differential eq<~ation an equation to be solved for
an unknown function which involves the first or
higher derivatives of the function ; an iquation
that contains one or more terms involving
derivatives of one variable with respect to
another variable.
Ordinary differential equation - one that
involves function of a Gingle variable and
some of its derivatives.
Partial differential equation - one that
involves functions to two or more variables
and some of !heir partial derivatives
Order ot a differential equation - refers to
the order of the highest derivaiives that is
present in the equation.
Degree of a. differential equation - the
highestyo.wer of the highest-order derivatives.
differentiation
derivative

the operation of finding the

directrix a fixed line opposite the focus of a conic


section which the eccentricity of the conic
section is defined.
Dirichlet's theorem 'For any two positive coprime
integers, a and b, there are infinitely many
prime numbers of the form an + b, where n >
0.'
discontinuity a point at which a function is not
continuous. This is also kn?wn as 'jump"
discriminant' the quanti! that discriminates among
the pvssibilities of a quadratic equation. lt is
expressed as
b2 - 4ac. The discriminant
determines the nature of the roots.
If b2- 4ac =0, the roots are real and equal.
If b2- 4ac > 0, ttie roots are real and unequal
if b2...: 4ac < 0, the roots are imaginary and unequal

displacement
vector quantity which represent
the charge in position of a point. It is equal to
the product of the velocity and lime.
distance the length of the shortest line segment
between two points.
distance formula the formula used lo measure the
distance between two points. This formula
was derived by the. use of the Pythagorean
theorem.

d=~Vc2 -x~J +(y2 -y~J

dihedral angle the angle between two planes


intercepted by a plane perpendicular to the
common edge

divergent not having a finite limit

dimensions of the matrix the number of rows a'nd


columns of a matrix

dividend in the expression a I b, a is iha dividend


and b is the divisor

Appendix ~~-~Qlossary 597

division the process of obtaining the quotient; the


inverse of multiplication
divisor (see dividend) the number that divides the
dividend

eccentricity the ratio of the distance from point to


the focus (focal distance) to the distance from
the point to the directrix. Eccentricity of
parabola, ellipse and hyperbola are 1.0, <1.0
and >1.0, respectively.

dodecagon a polygon with 12 sides

ecenter (see. excenter)

dodecahedron a polyhedron of 12 faces. Each


face of a regular dodecahedron is a regular
pentagon.

ecircle (see escribed circle)

domain the set of all first elements of a relation

,' li
I
'
I

II

economic return the profit derived froni a project or


business enterprise without consideration of
obligations to financial contributors and claims
of others based on profit

hi

dot product (see scalar product)


dozen a term use to denote the number 12. This is
derived from French "douze" which means
twelve. 1 dozen = 12 items.
duodecimal number system a number system
using 12 as a place value. This number
system still survive today as in 1 year = 12
months, a clock dia! has 12 hours, 1 foot= 12
inches, 1 dozen = 12 items and 1 gross = 12
dozens.
dyadic number system (see binary number
system)
dynar:nic (syn. kinetic) branch of mechanics that
deals with the forces that produced a motion.
dyne a unit of force in the cgs metric system and is
equivalent to 1 gram-cm/s2. 1 dynes= 105 N.

economical number a number that has no more


digits than there are digits in its prime
factorization
effective interest the true value of interest rate
computed by the equations for composed
interest for a 1 year period.
Effective interest can be computed using the
formula,
ER = (I + i)m- 1
where: m = number of interest period per year
i = interest per period
II!

Egyptian fraction a fraction in which the numerator


is 1. This is also known as "unit fraction"
Egyptian triangle a right triangle with sides 3,4 and
5 units
eigenvalue a complex number, A, the satisfies the
equation Ax = AX, where A is an n x n matrix
and x is some vector.
eight curve a curve with an appearance of a figure
of eight and has a rectangular equation of
4

E the number 14 in the hexadecimal notation.

x = a (x

e il transcendental number which is approximately


equal to 2.71828 ... 1t is commonly called as
the Euler Number because it was introduced
by Leonhard Euler(1707 - 1783) in the 18th
century. It has the equation

a Lemniscate of Gerono

eccentric geometric figures not l:aving the same


center.

This curve is also known

II
ill

j,
Iii

,,'

,'I

i'

I'' '

'~II'
~

'

I ,

':
II

element 1.member of the set 2. part of a geometric


figure such as point, line or plane).
eliminant (see resultant)
ellipse a locus of a point which moves such that
the sum of distance from two. fixed points (foci)
is constant and is equal to the length of the
major axis (2a).

,I

il,

Appendix A - Glossary 599

598 100 l Solved Problems in Engineering Mathematics (2"" Edition) by Tiong & Rojas
Eccentricity of ellipse is always le$S than 1.
Standard equation of the ellipse is

(x-11)2 + {y-k)2 = 1
a2

y =(a+ b)sin(t) -csin[ ( +

ellipsoid (syn. spheroid) solid of revolution of an


ellipse when rotated about one of its axes.
When rotated about its longer axis, it is called
as prolate ellipsoid. When rotated about its
shorter axis, it is called oblate ellipsoid. The
general equation of an eiHpsoid in rectangular

yz

zz

coordinates is 2 + 2 + 2 = 1
a b c
elliptic curve t[le type of cubic equations whose
solutions takes the form of
y2 +axy+by= x3 +cx 2 +dx+e
emirp a prime number that becomes a different
prime number when its digits are reversed.
The word 'emirp' is 'prime' spelled
backwards. Example is the number 13 which
will become 31 when the digits are reversed
but still a prime number
empirical based only on observations and
experimental evidences
empty set a set the contains no element at all
Engineering Economy the application of
engineering or mathematical analysis and
synthesis to economic decisions
ennea (syn. nona) a prefix which means nine. A
polygon of nine sides is called a nonagon or
enneagon.
envelope an envelope of a family of plane curves is
a curve that is tangent to every member of the
family
epicycloid the plane curve traced by a fixed point
on a circle as it rolls along the outside of a
fixed circle.
epitrochoid a curve traced out by a point that is a
distance c from the center of a circle of radius
b, where c < b, that is rolling around the
outside of another circle of radius a. This
curve is described by the followin!J parametric
equntion~

error the difference between an approximate value


and the true value which it approximates.

-ccos[(~ -1}]
~

b2

xz

x =(a +b)cos(t)

1}]

escribed circle (syn. excircle, ecircle) a circle


tangent to one.side and to the extension of the
two other sides of a triangle.

equal having the same value. The symbol of equal


(=)was introduced in 1557 by Robert Recorda
in his algebra textbook 'The Whetstone of
Witte'.
equally likely a term
equiprobable events.

used

to

describe

equation a mat[lematical statement showing that


two expressions have the same value.
equiangular having all angles equal. An equilateral
triangle is also equiangular.

Euler line a line that connects the centroid aml the


circumcenter of a triangle
Euler number the number e which is equal to
2,71828 ... To obtain this number, get the antinatural logarithm of 1. Or In e = 1. The formula
fore is,

e =Lim
n~a:

(1+ .!)n
n

The symbol 'e' was introduced by the Swiss


Mathematician, Leonard Euler (1707-1783).
even function a function f(x) such that f(x)
for all x

equidistant being of equal distance to any given


points or lines.

even number numbers which are exactly divisible


by two. All even numbers has for its last digit
an even number.

equilateral triangle a triangle having all sides equal


in length. An equilateral triangle is also
equiangular. Each interior angle of an
equilateral triangle is equal to 60.
equilibrium the condition when a body is acted on
by no forces of several forces such that their
vector sum (resultant) is zero. Forces are said
to be in equilibrium if they are-concurrent
(having a common point for the forces' line of
action).
equiprobable having the same probability.
equivalent having some properties (geometric) in
common.
equivalent numbers a number such that the sums
oftheir aliquot parts (proper ciivisors) are the
same
eradius (see exradiusj

'!I
I

extract to find the value of a root.

equichordal point a point inside a closed convex


curve in the ~lane, all the chords through
which have the same length

equilateral hyperbola (see hyperbola)

exterior angle (syn deflectign angle) is the aAgle


formed by the prolongation of one side and
the adjacent side of the polygon. The sum of
all exterior angles in any polygon is always
equal to 360.

=f(-x)

event in probability, it refers to the possible outcome


of a trial.
evolution the operation of root extraction. The
opposite of involution.
exactaccurate, precise
excenter (syn. ecircle) the center of the ascribed
circle.

extrapolate to estimate the value of a quantitr or


measurement beyond the values which are
already known. Opposite of extrapolatiOft is
interpolation.

'"'"""''11.1

I:
'!:1:

extremes the first and the fourth terms in lle


proportion of the four quantities

F
F the number 15 in hexadecimal notation.
face a plane surface of a geometric figure.
factor (syn. divisor) each of two or more numbers
which is multiplied together to form a product.
factorial the product of the n consecutive positive
integers from 1 to n. Example: n! = 1x 2 x 3 x
... x (n- 1) x n. By definition, 0! = 1. The
symbol factorial ( ! ) was introduce by
Christian Kramp (1760 -1826) in 1808.
Recursion formula states that:
(n!)(n + 1) = (n + 1)!
Substituting 0 for n, results to 0! = 1

excircle (see ascribed circle)

factorion a natural number that equals to the sums


of the factorials of its digits in a ~iven base

explementary angles (syn. conjugate angles) two


angle whose sum is 360"

factoring the process of converting expressions as


sums into equivalent expressions as products.

exponent (syn. index) number (usually written


superscript to another number) that is used to
simplify repeated product. Example xJ = XXX

fallacy a contradictory or false proposition; an


invalid argument of form of argument.

exradius (syn .. eradius) the radius of the ascribed


circle.

family ( syn. family of curves) a collection of related


geometric configurations; a set of related
curves or surfaces whose equations vary only
in the values of the constants.
III ~II ill :.

II~

600 1001 Solved Problems in Engineering Mathematics (2nd Edition) by Tiong & Rojas
Fermat's Last Theorem ( syn. Fermats' Great
Theorem) a famous conjecture of
mathema\ics which stales that X'' + yn = zn,
.where n is 3 or greater x, y and z are all
positive integers. This is regarded as the the
least proved theorem in Mathematics.
Fiere de Fermat(1601,1665) claimed !hat he
founded a marvellous root to this theorem,
but the margin is too narrow to contain it. This
is also known as the hardest problem in
Mathematics or the "Mount Everest of
Mathematics'.

Oblong numbers are numbers which can be


drawn as dots and arranged in rectangular
figure.
Example: 2, 6, 12, 20, ...
Pentagonal numbers are numbers which can
be drawn as dots and arranged in pentagonal
'figure.
Example: 1, 5, 12, 22, 35, ...
Gnomon numbers are numbers which can be
drawn as dots on equally long legs of a right
angle. Example: 1, 3, 5, 7, 9, ...

Fermat number a number defined by the formula

F.:=22"+1
Fermat's little theorem "If P is a prime number
!hen for ooy number a, (aP - a) must be
divisible by P."
Fibonacci numbers ( syn. Fibonacci sequence)
the unending sequence
integers formed
according to ihe rule that each integers is the
sum of the preceding two. The Fibonacci
sequence is 1,1 ,2,3,5,8, 13,34,55,89 .......
Name
after
italian
merchant
and
mathematician, Leonardo Fibonacci (1170 -c.
1250).

of

figure 1. any arrangement of points, lines, curves


forming a geometric shape 2. The symbol for
an integer, such as '8' or '69'.
figurate numbers numbers that are represented by
arrangements of dots as geometric figures. In
plane figures, the following are examples of
figurate numbers: Triangular numbers, Square
numbers, Oblong numbers, Pentagonal
numbers, Gnomon numbers. In solid tigues
are the Cubic numbers, Tetrahedral numbers
and Square Pyramidal numbers.
Triangular numbers are numbers which can
be drawn as dots and arranged in triangular
figure. Example: 1, 3, 6, 10, 15, 20, ...
Square numbers are numbers which can be
drawn as dots and arranged in square figure.
Example: 1, 4, 9, 16, 25, ..'.

cu'bic numbers are numbers which can be


drawn as dots and arranged as a cube.
Example: 1, 8, 27, 64, ...
Tetrahedral numbers are numbers which can
be drawn as dots and arranged as a
tetrahedron.
Example: 1, 4, 10, 20, ...
Square pyramidal numbers are numbers
which can be drawn as dots and arranged as
a pyramid with square base. Example: 1, 5,
14, 30, ...
finite can completely be counted off from 1 to the
last whole number.
first cost the initial cost of a capitalized property,
including
transportation,
installation,
preparation for service, taxes, and other
related initial !)xpenditure in order to make the
property functional
first derivative the derivative of a function.
Normally the first derivative of the function is
the slope of the function.
First Proposition of Pappus the area of a surface
generated by rotating.any plane curve about
non intersecting axis in its plane is equal to
the product of the length L of the curve and
the distance traveled by its centroid. Or
. expressed as

Area = length of arc x circumference


described by the centroid of arc

Appendix A- Glossary 601


flat angle (syn. straight angle) an angle whose
measure is 180".
flow chart a sequence of logical computations often
represented with rectangles, parallelograms
and arrows.
focal measurement from the focus to a certain
point.
focal chord a line segment joining two of its points
and passing through a focus of a conic
focal radius is a line SE!!lment from a focus to one
point of the conic.
focus a fixed point on the concave side of a conic
section.
Folium of Descartes a plane curve which forms a
loop on one side and intersect itself at a node.
Its standard equation is
xJ + yJ = 3axy where x + y + a = 0 is the
equation of the line.
formula a symbolic statement of mathematical
expression which is syntactically correct.
Fortune's conjecture 'If q is the smallest prime
greater than P + 1, where P is the product of
the first n primes, then q - P is prime.
fourth proportion In the proportion a:b
the fourth proportion.

=c:d, d is

fractal a geometric shape that can be subdivided at


any scale into parts that are, " at least
approximately, reduced-sized copies of the
whole. The term 'fractal' comes from Latin
"tractus' which means a broken surface

fr3e body diagram the diagram of an isolated body


with the representation of all external forces
acting on it

frame a structure with at least one of its individual


member is a multiforce member.

I
I

1.1'
'
I
i

frequency the number of times an event occurs


within a given period.
frequency polygon
a graph on which the
frequencies of classes are plotted at the class
mark and the class marks are connected by
straig hi lines

,II

friction the limited amount of resistance to sliding


between the surfaces of two bodies in contact
frustum a part of cone or pyramid lying between the
base and a plane parallel to the base.
frustum of a regular pyramid the portion of a
regular pyramid included between the base
and a section parallel to the base
frustum of a right circular cone the portion of a
right circular cone include between the base
and a section parallel to the base.
function a relation in which every ordered pair (x, y)
has one and only one value of y
corresponding to the value of x
fundamental operations of arithmetic referring .to
the four operations - addition, subtraction,
multiplication and division. In algebra,
fundamental operation follows the sequence
of 'My Dear Aunt Sally' which means that
Multiplication and Division must be calculated
first before Addition and Subtraction.

,II

il

,,li
1:'1

I:

fundamental theorem of algebra The result that


any polynomial with real or complex coefficient
has a root in the complex plane.'

fraction a ratio of two integers such as a/b , with a


not a multiple of b and b is not zero or one.
fundamental theorem of arithmetic "Every
The value 'a' is called the numerator and the
positive integer greater than 1 is a prime
value 'b' is called the denominator. When the
number or can be expressed as a unique
numerator is less than the denominator, it is a .
product of primes and powers of primes.'
common or vulgar or proper fraction otherwise
it will be called improper fraction. An improper
future worth the equivalent value at a designated
fraction is a:ways written as a mixed number,
future based on time value of money.
that is an integer and a proper fraction such as

3Y,.
This was formulated by Pappus (c.300-{;.350)
of Alexandria.

\~i:!il il l!l

.,

',!
I
I

602 100 1 Solved Problems in Engineering Mathematics (2nd Edition) by Tiong & RojilS

G
g notation for gravitational constant. g =32.2 feet
per second per second or 9.81 meters per
second per second.
G notation for gravitational constant.
G =6.67 x 10-11 Nm2/kg2.

..!

Gabriel's hom a surface of revolution of y =

generat"r (syn. generatrix) a point. line or plane


whose motion forms a geometric figure.
geometric mean (syn. geometric average) The
geometric mean of n numbers is the nth root of
the product of the numbers. For example, the
geometric mean of a and b is M . The
geometric mean is always less than the
arithmetic mean except if all numbers are
equal.
geometric progression a sequence of numbers
such that the ratio of any term to the
preceding term is constant.

centesimal minutes or centigon and 1


centesimal minute is divided into 100
centesimal seconds or milligan.

Hankel matrix a matrix in which all the elements


are the same along any diagonal that slopes
from northeast to southeast

googol the number represented by 1 followed by a


hundred zeros or 10100. The name was coined
by a 9 year old nephew of the American
mathematician Edward Kasner in 1930s.

harmonic analysis the method of expressing


periodic functions as sums of sines and
cosines

googolplex the number represented by 1 followed


by a google of zeros. 1 googolplex = 10 google =
1010'00
gr'ad (syn. gon or centesimal degree) unit of angle
measurement with one revolution equivalent
to 400 grads.
gradient slope of line or the ratio of vertical change
1o horizontal change.
graph a. plot of x values against y values for a given
function
-Gravitational . acceleration
the
constant
acceleration of a free falling body. Denoted as
'g' and is equal to 9.81 meter per second per
second or 32.2 feet per second per second on
earth's surface. On moon, g =1.62 m/s2 and
on sun, g = 274 m/s2.

geometric series a series whose terms form a


geometric progression.

gravity the force of attracting between two bodies


due to their mass.

Geometry the branch of mathematics which deals


with the properties and relations of
constructible plane and solid figures.

great circle the intersection of the sphere and the


plane through the center of the sphere.

gnomon a geometric figure which is formed by


cutting a parallelogram from one corner of
another but larger parallelogram.

greatest common divisor the largesi integer that


divides each. of a sequence of integers
exactly. This is also known as 'greatest
common factor"

gnomon numbers (see figurate numbers)


Goldbach conjecture the conjecture that every
even number (except 2) equals the sum of two
prime numbers. This was named after the
Prussian-born number theorist and analyst,
Christian Goldbach {1690-1764).
gon {syn. centesimal degree, grad) 1/400 of the full
angle. 1 revolution = 400 gons and 1 right
angle= 100 gons. 1 gon is divided into 100

Appendix A- Glossary 603

harmonic division the division of line segment by


two points such that it is divided externally and
internally in the same ratio
harmonic mean a term in between two harmonic
terms of a harmonic progression. It is the
reciprocal of arithmetic mean. The harmonic
. 2ab
mean of a an db 1s - a+b
harmonic progression a sequence of numbers
whose reciprocals form an arithmetic
progression.
Harshad number a number that is divisible by the
sum of its own digits. This is also known as
Niven number.
height the measure of an altitude of any polygon.
helix a curve in space which lies on a cylinder and
crosses its elements at a constant angle.
hemisphere a portion of sphere cut off by a plane
through its center..A hemisphere is one half of
a sphere.
heptagon a polygon with seven sides

greatest lower bound the largest real number that


is smaller than each of the numbers in a set of
real numbers
gross twelye dozen. This is equivalent to 144 items.

H
Half-angle formulas formulas that express n
trigonometric function of hnlf nn anglo.

Heron's formula the formula for the area qf a


triangle with all sides given. This was named
after the first century Greek mathematician
Heron (c.A.D.75) of Alexandria. The formula is
A= ~s(s-aXs-bXs-c)
where a,b and c are the length of the sides
and s= (P.+b+c)
2
Heronlan triangle a triangle with integer sides and
integer area

hexadecimal number system a number system


which uses a place value of 16. This number
system uses 16 symbols (10 basic numbers
and the letters A, 8, C, D, E. and F).
hexagon a polygon of six sides.

hexahedron a polyhedron with six faces. A cube is


a regular hexahedron.
hippo pede a quartic curve which has a rectangular
equation of
2

(x +y

t +4b(b-a)(x

I
il 1

' 11

+y }-4b 2 x2 =0

where a and b are positive constants


histogram a vertical bar graph that shows the
frequencies of scores or classes of scores by
the height of the bars
holism the idea that the whole is greater than the
sum of its parts
homogeneous For polynomials, it is the one having
all terms the same degree. For example, x3 +
3x2y + y3.
II

Hooke's Law Within the elastic limit, the stress is


proportional to strain. The constant of
proportionality is called the modulus of
elasticity, E or sometimes known as the
Young's modulus. Named after the English
mathematician and scientist, Robert Hooke
(1635-1703).

[I'll,,
I
li:
1.,',

ll':,j.
:1,

II'
lr1

Hydraulics the branch of Physics that deals with


the properties and characteristics of fluids.

1 1

,r1ii

hydrodynamic~

is the branch of hydraulics th!lt


deals with l'fuids in motion.

11!11

hydrostatics is the branch of hydraulics that deals.


with the fluids at rest.
. hyperbola the locus of a point which moves such
that the difference of the distances from two
fixed points (foci) is constant and is equal to
the length of the trilnsverse axis (2a).
Eccentricity of hyperbola . is always greater
than 1.
Hyperbolic spiral a curve with a polar equation of
re =a

11

II!
1

1:

l'i'
il l
1

II1!111h.
1

,,I il

Appendix A- Glossary 605

604 100 1 Solved Problems in Engineering Mathematics (2nd Edition) by Tiong & Rojas
hyperbolic functions functions of angle expressed
in exponential functions such as
sinhx = (ex- e-x)
2

inertia property of a body resisting any change in its


state of rest or of uniform motion in a straight
line.

icosahedron a polyhedron with twenty faces. A


regular icosahedron has faces equilateral
triangles.

inferential statistics the methods used to describe


a population (universe) by studying a random
sample of that population

identity referring to the property of being the same.


idempotent the element x in some algebraic
functions such that x times x =x.

coshx = (ex+ e-x)


2

infinite not finite, having value which are extremely


large amount.

imaginary number number in the form of ~


and denoted as I

tanhx = ex -e-x
ex+e-x
hyperboloid a surface of revolution produced by
rotating a hyperbola in space about one of its
axes, called its axis of revolution

improper fraction a ratio of positive integer in


which the value of the numerator is greater
than.that of the denominator.

hypocycloid the plane curve tr;:~ced by a fixed point


on a circle as it roll along the inside of a lixed
circle. The parametric equations are:

impulse the product of the force and the time during


which it acts

x =(a -b)cos(t)+ bcos[

(~ -1}]

y =(a- b)sin(t) -.bsin[(

~-1}]

infinitesimal a number that is greater than zero yet


smaller than nay positive real number
infinity the concept of being beyond the finite value.
The symbol of infinity (oo) was introduced by
John Wallis (1616-1703) in his Arithmetia in
fini torum in 1655.
inflection a point on a plane curve where the curve
has a stationary tangent, at which the tangent
is changing from rotating in one direction to
rotating in the opposite direction

incenter the point of concurrency of the angle


bisectors of a triangle
incircle the center of the circle inscribed in a
triangle.

inscribe to draw a geometric figure inside another


geometric figure in such a way that the two
figures havillJ common but not intersecting
points.

inclination (see angle of inclination)


hypotenuse the longest side of a right triangle or
the side opposite to the right angle.
hypothesis the part of the theorem which is
assumed to be true.
hypotrochoid a curve formed by the path of a point
attached to a point c, which is not on the
circumference of a circle of radius b that rolls
around the inside of a larger circle or radius a.
The parametric equations are:
x = (a-b)cos(t)+ccos[

(~ -1}]
~

y =(a- b)sin(t)- csin[ ( -1}]

Incomputable number a real number with an


infinite decimal expansion that cannot be
enumerated by any universal computer

inscribed angle an angle whose vertex is a point


on the circle and whose sides are chords

increment small change in the value of the variable.


The increment of x from x = a to x = b is the
difference, .b - a.

instantaneous power the limit of the rate of work


done as time approaches zero.
instantaneous velocity the limit of average
velocity of the body as the . time interval
approaches zero

indeterminate an expression with no direct


meaning \IS a number. Example are 0/0, oo/oo,
0/oo, 00...

Integers are counting numbers (natural numbers).


and th~ negative of the counting numbers and
the number 0.

index (syn. exponent) refers to the number n of a


railical

rr

Integrand the function to be integrated

induction a method of reasoning by which one


infers a generalization from a series of
instances

Integral the result of an integration. The integral


sign

inelastic not elastic; having elasticity tess than one.

i notation for imagi[lary number, ~


I roman numeral for 1.

J by

Leibneiz. The word integral

eomes from a Latin origin which means


'making up a whole".

inequality a statement that one mathematical


expression is 1greater than or less than another

Indefinite integral (syn. Primitive integral or


antidiverivative) an integral with no restrictions
imposed on its .independent variable.

Definite i'ntegral an integral defined by the


limit values of the independent variable.
Double integral an integral in which the
integrand is integrated twice.
Triple integral an integral in which the
integrand is integrated thrice.
Integral Calculus the branch of calculus which
deals with evaluation of integrals and their
applications.
integral part the biggest integer not greater than
the given number. In the number 5.12, the
int!'Qral part is 5.. In the number -5.12, the
integral part is -6.
integration the operation of transforming a function
to its definite or indefinite integrals.
intercept refers to the intersection of two geometric
figures. X and y intercepts are the intersection
of the curve with the x and y axes
respectively.
interest the periodic compensation for the use of
money. Sometimes referred to as the time
value of money.
interest rate the ratio of the interest payment to the
principal for a given unit of time and is usually
expressed as a percentage of the principal.
International System (SI) the metric system,orunit
defined by the General Conference of Weights
and Measurements in 1960. Sl stands for the
French equivalent, Systeme lntemational
interpolation the procedure for estimating
intermediate value that are not listed in a table
of numerical values. The . simplest form of
interpolation is linear interpolation, which
has for its variation, which has for its variation
of the functional described by a straight line. If
the function does not satisfy the condition of
linearity of variation, graphical interpolation
is used.
intersection a point where the. curve crosses the
coordinate axis. This also refers to the set
consisting of the elements that are common to
the original set.

I' IIIII

Appendix A- Glossary 607

606 100 1 Solved Problems in Engineering Mathematics (2"ct Edition) by Tiong & Rojas
intersection of two sets the set of all objects
common to both sets
invariant a value lhat is not changed when a
particular function is applied
involute of a circ)e a curve which is the path of
the end of a taut string as it is unwound from
a circle.
involution the operation of raising to an integral
power, X". This is opposite to evolution.
irrational equation an equation in which a variable
appears under the radical sign
irrational number any number which cannot be
expressed as a quotient of two integers:
Examples are n, e, ../2 , etc.
isochrone a set of points with the property that a
given process or trajectory will take the same
length of time to complete starting from ay of
the points
isodiametric having all diameters of equal length.
.isogon an equilateral polygon.
isomorphism a transformation in geometry tht does
not change the lengths of sides and the
measure of angles of the figure involved
isoperimetric figures
same perimeters

are figures that have the

isosceles having two sides of equal length. An


isosceles triangle has two sides and two
angles equal. The term 'isosceles" comes
from Greek 'iso' (same or equal) and 'skelos"
(legs).
isosceles trapezoid a trapezoid having non
parallel sides equal in length
isosceles triangle a triangle having two side equal
in length

kilowatt a unit of power equivalent to 1000 Watts;

J
notation for imaginary number , ~
engineering and physics applications.

1 kN-m

eqUivalent to - - -

for

Johnson's theorem 'If three congruent circles all


intersect in a single point, then the other three
points of intersection will lie on another circle
of the same radius.
Jordan matrix a matrix whose diagonal elements
are all equal and non-zero and whose
elements above the principal diagonal are
equal to 1 but all other elem~nts are 0
joule (pronounced 'jewel') a unit of work in the Sl
units, named in honor of English physicist,
James Prescott Joule (1835 - 1889). 1 joule =
(1 Newton)(1meter).

kilowatt-hour (kWh) the usual commercial unit of


electrical energy. Kilowatt-hour is a unit of
work or energy, not power. 1 kWh= 3.6 MJ
kinematics the study of motion without reference to
the forces which causes the motion
kinematic viscosity the ratio of viscosity to the
density of the body.
kinetic another name for dynamic.
kinetic energy In Physics, the quantity equivalent to
1/2 mv2, where m is mass and v is the
velocity.
kite a quadrilateral with two pairs of congruent
adjacent sides and forms like that toy kite.

Kepler's Laws the laws which described the motion


of stars, planets and comets, formulated by
Johannes Kepler (1571 - 1630).
Kepler's three laws of planetary motion:
1. All the planets of the solar system
describe elliptical orbits, having the sun
as one of the foci.
2.
A radius vector joining any planet to the
sun sweeps out equal areas in equal
periods oftime.
3.
The squares of the periods of revolution
of the planets about the sun are directly
proportional to the cubes of their mean
distances from the sun (the major semiaxes of the elliptical orbits).
kilogram (kg) mass of a particular cylinder of
platinum-iridium alloy which is now kept at the
International Bureau of Weights and Measures
at Sevres, near Paris. 1kg = 1000 grams.

leading coefficient the coefficient of the term of


highest degree in a polynomial of one
variable. In the polynomial, 4x3 + 3x2 - 8x - 10
, 4 is the leading coefficient.
leg Of a right triangle, any of the two sides other
than the hypotenuse.
lemma a proved proposition which is used mainly
as a preliminary to the proof of a theorem
lemniscate (syn. Lemniscate of Bernoulli) a curve
whose equation is r2 = a2 cos 28, where (r,8)
are polar coordinates. This was conceived by
Jakob Bernoulli (1654- 1705) in 1694.
length measure of a line segment.

K
kappa curve a curve that resembles the Greek
letter, K and has a rectangular equation of
y2 (x2 +y2) = a2x2

Law of Universal Gravitation "Every particle in


the universe attracts every other particle with
a force that is directly proportional to the
masses of the two particles and inversely
proportional to the square of the distance
between the centers of masses".

least common multiple the smallest integer that is


an exact multiple of every number in a set of
integers

L the number 50 in the roman numerals.


Lame curve any family of curves related to the .
ellipse with a general equation of
1

1;1" +ltl" =

lamina a thin sheet of uniform thickness and


density
lateral area area of the surface exclusive of bases.

least upper bound the smallest number that is


larger than every member of a set of numbers
Leibniz harmonic triangle a triangle of fraction
which is closely related to the famous Pascal's
triangle and takes the form of
1/1
1/2 1/2
1/3 1/6 1/3
1/4 1/12 1/12 114

lateral edge the intersection of the lateral faces.


lateral face any side of the polyhedron other than
the base.
lateral surface the union of the lateral faces of a
prism
latus rectum a line through the focus, parallel to
the directrix and intersecting the curve

Lemniscate of Bernoulli a curve which looks like a


bow of a ribbon and has a rectangular
2
equation of ( x2 + y2 ) = a2 ( x2 - y2 )
L'Hospital Rule (pronounced as Lopital's Rule) a
method in calculus in evaluating indeterminate
quantities such as 0/0 and oo/oo. Name in
honor of Johann Bernoull's pupil, Guillaume
Francois de L'Hospital, (1661-1704).

II

.. , I!
.II

608 . 100 l S0lved Problems in Engineering Mathematics (2nd Edition) by Tiong & Roj~s

life the period of after which a machi#le or facility

long radius the distance between a center and a


vertex of a regular polygon.

should be discarded or replaced because of


its excessive costs or reduced profitability.
Also refers to the period of time after which a
machine or facility can no longer be repaired
in ord'er to perform its design function
properly.

Ludolph's number a name used in Germany for


the number pi (rr)

Limacon of Pascal a snail-shaped curve with a


rectangular equation of

June the portion of a sphere lying between two


semi-circles of great circles.

( x2 + y2

2rx

=k

2
(x

lozenge a rhombus with a 6Qo angle

+ y2 ) , where r is

the radius of the rolling circle or the rolled


circle and k is a constant
limping triangle a right triangle with two shorter
sides differ in length by one unit
line the shortest distance between any two points
line segment a, portion of a line bounded by two
points.
linear equation an equation in which the variable or
unknown appears only the first power and only
in the numerator of any fractions
literal equation an equation in which some or all of
the known quantities are represented by
letters.
lituus a plane curve with equation r2 = ~ with (r,e)
as polar coordinates; It resembles a trumpet
shaped spiral and originated with the English
mathematician, Roger Cortes (1682- 1716).
The word lituus, is of Latin origin which means
'a crooked staff. Plural form of lituus is litui.
locus the path of a point which moves according to
a given law or equation. Plural form is loci.
logarithm an exponent when a number, N is
represented as a power of a fixed number
called based.
logarithmic spiral a type of spiral with a polar
equation of r = ab6
logic the branch of mathematics which concern with
how one statement can imply others, or how
set of statements can be connected by chains
of implications

M
M the number 1000 in the roman numerals.
Macclaurin trisectrix an anallagmatic curve that
intersects itself at the origin, It has a
rectangular equation of
y2 (a + x) = x2 (3a- x)
magnitude the absolute value of a vector quantity.
mantissa the decimal part of a logarithm . In the
expression log 25 = 1.39794.. the value
0.39794 is called the mantissa while the value
1 is the characteristic
mass a measure of inertia of a body, which is its
resistance to a change in ve~ocity.
mathematical expectation
expected value.

another term for

mathematics the group of subjects (Algebra,


Trigonometry, Calculus, Geometry, etc.) used
in investigation of numbers, space and the
many generalization of these concepts. It is
also defined as the science of patterns, real or
' imaginary. The word mathematics comes from
Greet 'mathema' which means 'knowledge'
Matheson Formula the formula used for Declining
Balance Method depreciation, k =

_{CO
vcn

where Co and Cn are the firSt and last cost.


respectively. With this method d computing
depreciation, the last cost, Cn should not be
equal to zero.

Appendix A - Glossary _209


matrix a rectangular arrays of numbers forming m
rows and n columns.
Types of matrices:
1. Square matrix - a matrix where the
number f columns equals the number of
rows.
2 Row matrix - a matrix which only one
row.
3. Column matrix - a matrix which has only
one column.
4. Lower triangular matrix - a matrix where
all entries above the main diagonal are
zero
5. Upper triangular matrix - a matrix where
all entries below the main diaqonal are
zero.
6. Scalar matrix - a diagonal matrix where
a11 = an =a33 =.. . = k. where .k is a
constant.
7. Unit matrix or identify matrix- a scalar
matrix where k =1
8. Null matrix - a matrix in which all entries
are zero
9. Complex m&trix - a matrix with at least
one of the entries a complex number.

meter defined in 1960 as 1,650,763,73 wavelength


of !he orange,red light emi!IE)d by atoms of
krypton 86(BSY) in a glow discharge tube.
Redefined in 1983 as the distance !hal light
travels (in vacuum) in 1/299,792,458 second.
The latest definition is more accurate than the
first.

mean (syn. average) 1. The arithmetic average of


all the scores in the distribution 2. the average
of two quantities.

method of exhaustion a method of finding an area


by approximating it by the areas of a
sequence of polygons

mean proportional the second and the third term~


of a proportion with the second term equals
the third term.

mil a unit of angle measure with one revolution


equivalent to 6400 mils

median 1. a line drawn from the vertex of a


triangle to the midpoint of the base 2. (in
stalistic) a point in the distribution of scores at
which 50 percent of the scores falls below and
tile 50 percent of the scores fall above
median of a trapezoid a line joining the midpoints
of the non parallel sides
mediator the perpendicular
segment.

measure a way of determining how large something


in terms of weight, volume, mass, length, etc.
measure theory the part of mathematics which
investigates the conditions under which
integration can be carried out.
mechanics a branch of physical science which
deals with state of rest or motion of bodies
under the action of forces
medial triangle the triangle .whose vertices are
midpoints of the sides of a given triangle

of a line

Mersenne number a positive integer of "the form


2" - 1, where n is a prime number. This was
named after a French mathematician and
Franciscan Priest. Marin Mersenne (15581648).

mile
means the secood and the third terms in the
proportional ,of four quantities

bisecto~

a unit of length.
Statue mile = 5280 feet
Nautical mile " 1 minute angle on a great
circle
= 6280 feet.
The nautical mile is 800 feet longer than the
statue mile. The word mile comes from Latin
'mille passes" which means "one thousand
paces"

milliard In United Kingdom, a thousand million.


Billion (instead of milliard) is used ir1 the
United States of America.
million a thousand thousand. The word comes from
Latin "miile" which means one thousand and
the suffix 'ion' means great; so million iiterally
means a great thousand
minimal prime a prime number that is a substring
of another prime when written in ba~ ~ 10

610 1001 Solved Problems in EngineeriJ;Ig Mathematics (2nd Edition) by Tiong & Rojas
minuend In an expression x- y, x is the minuend
minute a measure of both time and angle
Time: 1 minute = _!_ hour = 60 seconds
.
60

_!_ degree = 60 seconds

monotonic property of a function that is always


strictly decreasing or strictly increasing but
never both
motion any change in the position of a body.

60

multinomial the algebraic expression consisting of


a sum of any number of terms.

Mirifici Logarithmorum Canonis Description a


book published by John Napier in 1614 which
means A Description of the Marvelous Rule
of Logarithm'.

multiple any number of polynomial that is a product


of a given number or polynomial and a
multiplier. For example; 15 is a multiple of 3.

mixed number a number formed of an integer and


a proper fraction. Example 51h

multiplicand the number being multiplied by


another. In an expression 5 x 2 = 10, 5 is the
multiplicand and 2 is the multiplier. Both 5 and
2 are factors of 10.

Angle: 1 minute =

Mobius strip a twisted surface in space formed by


turning one side of a rectangle through 180
(relative to the opposite side) and join it to the
opposite side. This was named after a
German mathematician Agustus Mobius
(1790-1868).
.
mode the most frequently appearing score or group
of scores appearing in the distribution
model theory The study of mathematical structures
that satisfy a particular set of axioms,
especially in the field of logic
modulus of common logarithm in converting
natural logarithms into common logarithms,
the following formula is applied; log N =
0.43429 in N. The number 0.43429 is the
modulus of common logarithm
modulus of elasticity the ratio of the tensile stress
to tensile strain. This is sometimes called
Youg's Modulus, in honor of Thomas Young.
moment of force another name for torque.

multiplicative inverse
the reciprocal of the
number. The multiplicative inverse of 5 is 1/5.
multiplier (see multiplicand)
mutually exclusive a condition where two events
cannot happen at the same time, or when one
occur, the other one will not occur and vice
versa.
myriad very large number. II comes from Greek
'murios' meaning "uncountable'

'N
nadir the point on the celestial sphere directly below
the observer.
Nagel point a point in a triangle where the lines
from the vertices to the points of contact of the
opposite sides with the excirc!es to those
sides meet

moment of inertia the quantity equivalent to the


area times the square of the distance from the
centroid to the axis considered. It has a unit to
the 4th power.

Napierian logarithm logarithm to the base e =


2.718281828 ... This is also known as Natural
logarithm. Denoted as In or log.

momentum the product of !he rnass and velocity of


the body

Napier's rules rules used in solving spherical right


triangles.

monomial an algebraic expression of only one


term

Rule 1 (Taro-ad rule) The sine o/uny middle


part h; equal to the product of llln[lenl of the
adjacent paris.

-------------------------------
Rule 2 (Co-op rule) The sine of any middle
pari is equal to the product of the cosine of the
ppposite paris.
nappe either of the two parts into which a cone is
divided by the vertex.
narcisssictic number an n-digi! number equal to
the sum of its digits raised to the nth power.
Another term for this is "Plus perfect number"
or 'Armstrong number".
natural logarithm (see Napierian Logarithm)
natural number are numbers considered as
counting numbers. Example: 1, 2, 3, ... : Zero
and negative numbers are not considered as
natural numbers.

_____
A~dix A- Glossary 61.1

null equal to zero; empty


null hypothesis a hypothesis that is being tested
for rejection
number theory the study of the whole numbers and
their properties and relationships
numeral symbol or combination of symbols
representing a number such as. Arabic and
Roman numerals. Arabic numerals (e.g. 0, 1,
2, 3, 4, .. ) are the modification of the HinduArabic Numbers. The Roman numerals are
certain letters of the Latin alphabet.
Roman numeral
Arabic equivalent
I
1

X
L

10
50
100
500
1000

negative less than zero.


negative
angle
angle
counterclockwise rotation.

measured

in

newton the amount of force thai gives an


acceleration of one meter per second squared
to a body with mass of one kilogram. Denoted
as N. 1 N =1 kg-m/s2
Newton's First Law Every body continues in its
state of rest. or of uniform motion in a straight
line, unless it is compelled to change that
state by forces impre~sed on it. This is also
knowh as the law of inertia.
Newton's Second Law The rate of change of linear
momentum of a particle is C;Jqual to the total
applied force.

D
M

numerator In the fraction ~ , x is the numerator, y


y
is the denominator.
numerical
integration
(syn.
approximate
integration)
the process of finding an
approximate value of a defmite integral without
carrying out the process of evaluating the
indefinite integral.

Newton's Third Law For every action, there is


always an equal and opposite reaction.

oblate spheroid (syn. oblate ellipsoid) prOdUGed by


rotating an ellipse through a complete
revolution about its minor axis.

nominal interest the number employed loosely to


described the annual interest rate

oblique angle consist of all angles except right and


straight angles

nonagon a polygon of 9 sides

oblique cal'tesian coordinate system a cartesian


COOrdinate system in Which the X and y axes
are not perpendicular

normal another
orthogonal.

name

of

perpendicular

or

normal number a number in which digits


sequences of the same length occur with the
same frequency

oblique circular cone a circular cone whose axis is


not IJerpendicular to the base of the cone.
oblique triangle a triangle with no right angle.

Appendix A - Glossary 613

612 1001 S~lved Problems in Engineering Mathematics (2"d Edition) by Tiong & Rojas
oblong another term for rectangle. This comes from
Latin "ob' meaning excessive and "longus'
meaning long
oblong numbers (see figurate numbers)
obtuse angle an angle that is greater thc'm 90
degrees but less than 180 degrees.
obtuse triangle
angle.

a triangle having one obtuse

octagon a polygon of eight sides


octahedron a polyhedron with eight faces
octal number system a number notation which
uses base 8 as a place value. It uses the
digits 0, 1, 2, 3, 4, 5, 6, and 7.
octants referring to the eight compartments 9f the
rectangular coordinate systems in space
odd not divisible by two.
odd function a function f(x) with the property that
f(-x) = -f(x) for any value of X.
ogive a graph of cumulative frequency distribution
plotted at the class marks and connected by
straight lines
ordinal numbers numbers which state their position
in a sequence. Example: First, second,
third, ...
ordinary annuity an annuity where the payments
are made the end of each period starting from
the first period.
ordinate the position of any point on a plane from
the x-axis. Also known as the y-coordinate.
origin the intersection of the x and y axes of the
cartesian or rectangular coordinate systems.
The origin has coordinates of (0,0).
orthic triangle the triangle whose vertices are lhe
feet of the altitudes of a given triangle
orthocenter the point of intersection of all the
altitudes of a triangle.

orthogonal normal or perpendicular. Its symbol is

pandigital number a number that contains eacli of


the digits from zero to nine exactly once and
whose leading digit is non-zero. Example
1,432,576,098
Pappus theorems theorems that determine tha!
surface area and volume of a figure
generated about an axis. This is named after
Pappus of Alexandria.
First proposition of Pappus:
"If an arc is rotated about an axis, it generates
as surface are which is equal to the length of
arc times the circumference described by the
centroid of the arc.'
Second proposition of Pappus:
"If an area is rotated about an axis, it
generates a solid of revolution, whose volume
is equal to the area times the circumference
described by the centroid of the area'.
"parabola a locus of a point which moves so that it is
always equidistant from a fixed point (focus)
and to a fixed line (directrix). Ecr..entricity of
parabola = 1.
parabolic segment the plane region bounded by a
chord a parabola perpendicular to the al{is and
the arc of the parabola cut off by the chord.
The area of the parabolic segment is 213 of
the circumscribing rectangle.

~bh
3

parallel line or plane that will never intersect no


matter how far they are extended.
parallelepiped a prism
parallelogram region.

Paraboloid

palindronic number a number such as 1,234,321


that reads the same forward and backward

A=

whose

base

is

Rectangular parallelepiped

parallelogram a quadrilateral in which both pairs of


opposite sides are parallel.

The number can be found by adding the two


numbers above it. For example, 6 in the
triangle was obtained by adding 3 an 3, the
numbers above it. Named (;lfter French
mathematician, philosopher and physicist,
Blaised Pascal (1623 -1662).
pearls of Sluze curves that are generated by the
rectangular equation y" = k(a- x)P xm where
n,m and p are integers
pedal curve the locus of the feet of the
perpendiculars from a given point to the
tangents to a given figure
pedal triangle a triangle inscribed in a given
triangle whose vertices are the feet of the
three perpendiculars to the sides from some
point inside the given triangle.
Pelf equation a equation of the form y2 = ax 2 + 1
where a is any positive whole number except
a square number

Parallelogram

parameter generally an arbitrary constant.


partial fraction the parts of an algebraic expression
which contain a polynomial in a single variable
in the denominator, or in the denominator and
numerator, when split
partnership (type of business organization) an
association of two or more persons for the
purpose of engaging in a business for profit.
pascal (Pa) a ~nit of pressure. 1 pascal = 1 newton
per square meter.

percent on diminishing value (see declining


balance method)
pencil a collection of lines that passes
fixed point or a given point.

th~ough

pentagon a polygon. of five sides


pentagonal numbers (see figurate numbers)
pentagram (syn. pentangle, pentacle) a starshaped figure formed by extending the sides
of a regular pertagon and meet at vertices.
pentedecagon a polygon of fifteen sides

paraboloid a solid of a revolution of a parabola.


Volume of the paraboloid is always equal to
one-half of the volume of the circumscribing
cylinder.

Pascal's Law "If any external pressure is applied to


a confined fluid, the pressure will be increased
at every point in the fluid by the amount of the
external pressure.'
Pascal's Triangle a triangular array of numbers
which is made up of the binomial coefficient of
the binomial expansions:

per cent a word of latin origin which means every


hundred. Its symbol is %.
percentage a ratio by which the denominator is
100.

,'1
Appendix A- Glossary 615

614 1001 Sblved Proble~s in Engineering Mathematics~d Edition) by Tiong & Rojas
perfect number a number the sum of whose
factors including one but excluding itself is
exactly equal to the number. Example: The
factors of 6 are 3, 2 and 1. Adding the factors
.will yield the number itself such as

1 + 2 + 3 =6. Hence, 6 is a perfect number.


perfed power an integer of the form m" where m
and n are integers and n > 1
perfect square a number that is the product of two
equal whole numbers. Example 4 =2 x 2.
perigon an angle equal to one revolution (360)
perimeter the sum of the sides of a polygon. This is
known as circumference for a circle.
permutable prime a prime number with at least two
distinct digits, which remains prime on every
rearrangement of the digits.
permutation an arrangement of a set of objects in a
particular order. The permutation of 'n'
different things taken 'r' at a time is given as
n!
P(n,r)=~

1[1- r I'1

Mathematically, A= i+(%)-1, where i is


the number of interior lattice points and b is
the number of boundary lattice points

polyiamond a shape made from identical


equilateral triangles that have been joined at
their edges

place-value system a number system in which the


value of a number symbols depends not only
on the symbols itself but also on the position
where it occurs

polynomial an expression of several terms. It may


include any number of terms.

Planck time the shortest meaning period of time in


quantum m.echanics

planimetry the measurement of plane areas.

postulate in Geometry, the construction or drawing


of lines and figures the possibility of which is
admitted without proof.

point a dimensionless geometric object having no


properties except location and place.

pound a unit of force in the English (British) system.


tf is equivalent to 1 slug-ft/s2. Also 1 pound

point of inflection !he point on the graph where the


curve changes from concave up to concave
down and vice versa

perpetuity an annuity where the payment periods


extends forever or in which the periodic
payments continue indefinitely.

Poisson ratio The ratio of the unit deformations or


strains in a transverse directions is constant
for stresses within the proportional limit.

circumference to the diameter of a circle. Its


value is 3.14159 ... and has for its symbol 'Jt.
This symbol for pi {n) was introduced in 1706
by William Jones (1675 -1749).
Pick's theorem 'The area of a polygon can be
found simply by cotlnting the lattice points on
the interior and hoiJndary of the polygon.'

population (syn. universe) in statistics, it refers to


all the members of a particular group of items
or individuals.
positive having values greater than zero.

plane a surtace such that a straight line joining any


two points in it lies wholly in the surface

point of tangency is the point of contact of the


tangent and the curve

pi an irrational number which the ratio of ttie

polyhedron a solid bounded by planes

pie chart a circular diagram divided into sectors of


which the area are in proportion to the
magnitude of the represented values.

perpendicular (syn. normal, orthogonal) forming a


right angle.

Philosoph late Naturalis Principia Mathematica


('Mathematical Principle of Natural Science')a
book publish by Sir Isaac Newton in 1686.
This book clearly slates the fundamental law~
of nature which is now referred to as the
Newton's Law, the cornerstone of mechanics.

polygonal region is the plane figure formed by


fitting together a finite number of triangular
regions

polar angle (syn. vectorial angle, the argument, the


amplitude of the azimuth of the point) the
angle the vector makes with the polar axis.
polar coordinates coordinates in the form of (r,6)
used to locate a point .in the rectangular
coordinates system.
To convert polar to rectangular, use the following
r'!lations: X ::: r COS 6, y = r sin 6

=4.448221615260 N.
power 1. the rate at which work is done or energy is
transferred 2. (syn.. exponent) the number of
times the number is multiplied by itself.
power law a type of mathematical pattern in which
the frequency of an occurrence of a given size
is inversely proportional to some power of its
size.
power series an infinite series in which successive
terms are of the form of constants times
successive integral power of the variable. It
takes the form of ao+a1x+a2x2 +a:JX3 x ..... .
power set the set of all subsets of. a given set,
containing the original set as well as the
empty set.
powerful number a positive whole number n such
that for every prime number p dividing n, p2
also divides n.

pole the origin of the polar coordinates system


polygon a closed figure bounded by line segments.

practical number a number n such that every


positive integer less than n is either a divisor
or a sum of distinct divisors of n

precision the accuracy in which a calculation is


pertormed.
present worth the equivalent value at the present ,
base on time value of money
pressure the force per unit area. It has a unit of
pascals in the metric system. Since pascal is a
small unit, the unit bar or MPa are used
instead. 1 pascal = 1 newton per square
meter. 1 bar = 105 Pascals.
The gauge pressure can be calculated using
the formula: p = roh, where ro = density
(specific weight) of the fluid and h is the
pressure head.
The absolute pressure is the sum of the gauge
pressure and the atmospheric pressure.
Standard atmospheric pressure = 14.7 psi =
1.01 x 105 Pa = 760 mm of Hg
primitive integral (see integral)
prime number an integer which has no other
factors except 1 and itself
principal In economics, it is the amount invested.
prism a polyhedron of which two faces are equal
polygons in parallel planes, and the order
faces are parallelograms
prismatoid
a polyhedron having bases two
polygons in parallel planes and for lateral
faces triangles or trapezoids with one side
lying one base and the opposite vertex or side
lying on the other base of !h9 polyhedron.
prismoid a prismatoid in which the two bases are
polygons of equal number of sides and the
lateral faces are quadrilaterals.
prismoidal formula formula used in finding the
volume of a prismatoid such as follows,

V=~(A1 +4Am +A2).


where, L is the distance betw.een end areas,
A, and A2 are end areas and Am is the area at
the mid-section.

616 1001 Solved Problems in Engineering Mathematics (2"d Edition) by Tiong & Rojas
probability the ratio of the successful outcome of
an event to the total possible outcome of an
even!. The value of the probability is always
less than 1.
product the result of multiplication.
progression a sequence of numbers in which one
is designated as first. another as second ,
another as third and so on. Types of
progression are Arithmetic progression,
Geometric
progression,
Harmonic
progression, Infinite Geometric progression,
etc.
projectile initial velocity of a body and then follows
a path determined entirely by the effects of
gravitational acceleration and air resistance.
prolate spheroid
{syn. prolate ellipsoid) an
ellipsoid produced by rotating an ellipse
through a coMplete revolution about its major
axis

1
ll

App~!lciixA-

Glossary 617

rational expression any algebraic expression that


is a quotient . of two other algebraic
expressions

QED latin word quod erat demonstrandum, which


.means that a proposition has been proven.
The Greek equivalent was used by Euclid in
the 3rct century B.C.

radian the angle between two radii with an


intercepted arc equal to the radius of the
circle. 1 revolution is equal to 27t radians.

rational number any number which can be


expressed as a .quotient of two integers
(division by zero excluded).

quadrantal spherical triangle a spherical triangle


with at least one side a quarter of a great
circle.
quadrants referring to the four divisions of the
rectangular coordinates system.
quadratic equation an equation in which the
maximum power of the unknown or variable is
2. Standard quadratic equation is in the form
ax2 + bx + c = 0.

radical the symbol . [


This , symbol was
introduced by Christoff Rudolff in 1525.

rationalizing the denominator the process of


removing the radicals (or fractional. exponents)
from the denominator

radical axis the locus of points of equal power with


respect to two circles

ray a straight path of points that begins at one point


and continues in one direction

radicand the quantity inside the radical (square


root sign)

real number a non imaginary number. It includes


the .rational numbers as well as the imaginary
numbers all integers and natural numbers.

radius a segment from the center to a point of the


circle

reciprocal multiplicative inverse of a number. For


example, the reciprocal of 5 is 115.

radius of gyration the distance from a given axis


that a particle of the same mass as a rigid
body must be placed in order to have the
same moment of inertia.

rectangle a parallelogram all of wthose angles are


right angle

radius vector the distance of any point P from the


origin in the polar coordinate system

rectangular hyperbola a hyperbola with length of


semi-transverse axis, "a' equals the length of
the semi-conjugate axis, 'b'. Eccentricity of
this hyperbola is square root of 2.

a polygon offcur sides

radix the base of a number system. For example, 2


is the radix of a binary number system and 10
is the radix of the. decimal number system

rectangular parallelepiped a polyhedron whose six


faces are all rectangles

quantity something with a magnitude or numerical


value.

random variable a numeric quantity which can be


measured in a random experiment.

quarternary having four variables.

range the set of all second elements of a relation

reduction the process of converting a fraction into a


decimal form.

pyramid a polyhedron of which one face, called the


base, is a polygon of any riumber of sides and
the other faces are. triangles which have a
common vertex

quartic a polynomial or polynomial equation that


contains the fourth power of the variable, but
no higher power

rate of return the interest rate at which the present


work of the cash flow on a project is zero, or
the interest earned by an investment

redundant equation any equation which, because


of some maihematical process, has acquired
an extra root

pyramidal numbers (see figurate numbers)

quinary number system which pertains to place


value notation of base 5.

ratio the quotient of two numerical measure of two


magnitudes of the same kind. The word ratio
comes from the Latin verb 'ratus' which
means 'to estimate'.

reentrant angle an inward-pointing angle of a


concave polygon

proper fraction a ratio of positive integers in which


the value of the numerator is less than that of
the denominator
.Ptolemy's Theorem In cyclic qua9rilateral, the sum
of the product of two opposite Sides is equal to
the product of the diagonals. Named after
Ptolemy of Alexandria or Claudius Ptolemaeus
(c.100- c. 168).
puro quadratic a quadratic equation of the form ax2
+ c =0, that is , the coefficient of the first
degree term, b is equal to zero

Pythagorean theorem The sum of the squares of


the sides of a right triangle is equal to the
square of the hypotenuse; in equation, a2 + b2
= c2 with a and b are legs while c is the
hypotenuse. This is named after the Greek
philosopher and mathematician, Pythagoras
(c. 580- c. 500 B.C.) of Samos.

Quadratrix of Hippias the first curve recorded in .


history that is not a part of a line or a circle.
This curve has a rectangular equation of

y = xcot(;:)
quadrature formulas- refers to the formulas used
in numerical intregration.
quad~ilateral

quintic a polynomial ~r polynomial equation that


contains the..fifth power of the variable, but no
higher power

ratio of similitude the common ratio of the


corresponding sides of two similar polygons

quotient the result of division.


'

rational equation an equation which is satisfied by


all value of the variables for which the
members of the equation are defined

rectilinear pertaining to straight line.

reflex angle any angle greater than 180 degrees


but less than 360 degrees
regular polygon a polygon with all sides equal and
all angles equal. A regular polygon is
equiangular and equilateral. Also, a regular
polygon is convex.

r
618 I 00 I Solved Problems in Engineering Mathematics (2nd Edition) by Tiong & Rojas
regular pyramid a pyramid whose base is a regular
polygon and whose center coincides with the
root of the perpendicular dropped from the
vertex to the base
relation is any set of ordered pairs (x,y)
relative ~ensity (see specific grav1ty)
relative error a measure of the difference between
a number and an estimate.
Relativity theory a theory formulated by Albert
Einstein the revolutionized of man's
understanding about time, space and gravity.
Mathematically, it is expressed as E = mc2,
where E, m and c are energy, mass and
speed of light, respectively
remainder the amount left when a quantity cannot
be divided exactly by a divisor.
resultant the single vetor quantity which is the sum
of two or more vector quantities.
rhombus (syn. diamond, lozenge) a parallelogram
all of whose sides are congruent
right angle angle equivalent to 90 degrees
right triangle a triangle having one right angle
rigid body a body which will not be affected or
deformed when an extremely large or
extremely small load or temperature is
applied.
Rolle's theorem "Suppose a continuous function
crosses the x-axis at two points a and b and is
differentiable at all points between a and b;
that is, it has a tangent at all points on the
curve between a and b. Then there is at least
one point between a and b where the
derivative is 0 and the tangent is parallel to the
x-axis
Roman numerals (see numerals)
root value that satisfy a given equation.
rose curve a curve that has the shape of a flower
with petals had has a polar equation of
r asin(rrO)

A~endi:~tl\_~_Giossary

rounding (of numbers) replacing it with another


number to produce fewer significant decimal
digit. For an integer, fewer value carrying
(non-zero) digit. Example: 3.14159 is rounded
off to four decimal places as 3.1416. For
numbers ... 5, the rule states that it should be
rounded off to the .nearest even rounding
boundary to minimize the systematic rounding
error.

score another term used for the number twenty (20)

rows the numbers in order which appears


horizontally in a matrix

second originally defined in 1889 as the fraction


1/86400 of the mean solar day, and redefined
in 1967 as the duration of 9,192,631,770
periods of the radiation of a certain state of the
cesium-133 atom

s
sample any subset of a population
sample space the set of all possible outcomes of
an experiment.
salient angle an outward-pointing angle of a
polygon
salient point the point where two branches of a
curve meet and stop and have different
tangents
.salvage value the cost recovered or which could be
recovered form a used property
when
removed, sold or scrapped. It is sometimes
referred to as second hand value
scalar quantity a physical quantity that is described
by a single number only, the magnitude. It
does not have a direction in space.

scrap value the value of an equipment if disposed


as junk. This is sometimes referred to as junk
value.

619

shear stress a stress that is caused by forces


acting along or parallel to the area resisting
the force
,Ill

short radius the shortest distance from the center


of a regular polygon to any of its sides.

!i:

secant a line which intersects the circle in two


points. The reciprocal of the trigonometric
function, tangent.

Second proposition of Pappus The volume of any


solid of revolution is equal to the generating
area times the circumference of the circle
described by the centroid of the area.
section of a solid the plane figure cut from the solid
rv passing aplane through it
sector a part of a circle bounded by the radii and an
arc.

111:

Figure shows R, the short radius

Siegel's paradox "If a fixed fraction x of a given


amount of money P is lost, then the same
fraction x of the remaining ~ nount is gained,
the result is less than t!:w v11ytnal and equal to
the final amount if a fraction x is first gained,
then lost.'
Sierpinski number a positive, odd integer k such
that k times 2n + 1 is never a prime number for
any value of n

1
1

'1'111'11:: 11 1'

.,:11,,

segment part of a circle bounded by an arc and a


chord.

significant figures fdigits the meaningful digits in a


number. A number is considered significant
unless it is used to the place a decimal point.

septagon a polygon of seven sides


sequence (syn. progression) a succession of
numbers in which one number id designed as
first, another a~ second, another as third and
soon

similar having the same shape but not necessarily


. the same size
simple interest the interest charges under the
condition that interest in any time is only
charged on the principal

series sum of a finite of infinite sequence.

scalar product of A and B is denoted as AB.


Because of this notation, scalar product is also
called as dot product

serpentine a curve which nas a rectangular


abx
.
( )
equat1on y x = -()

scalene triangle a triangle having all sides of


unequal lengths

set (syn. class) a collection of objects.

x2 -a2

scattergram the relation between two variables is


shown by a series of dots plotted on a graph

sexagesimal pertaining to the number 60.

scientific notation a number represent using


powers-of-1 0 notations used to described a
very large small numbers.

sexagesimal number system a number system


.using a place value of 60. This was used by
the Babylonians or Mesopotamians and is
considered be the oldest number system
which dates back to 2 millenium B.C.

sine curve (syn. sine wave) a curve with equation y


=sin x.
sinking fund method a method of depreciation
where a fixed sum of money is regularly
deposited at compound interest in a real or
imaginary fund in order to accumulate an
amount equal to the total depreciation of an
asset at the end of the asset's estimated life.
skew lines two lines that are not coplanar. This is
also known as "crossing lineS"
slant height (syn. element) the length of a
generator of a circular cone.

"
620 1001 Solved Problems in Engineering Mathematics (2nd Edition) by Tiong & Rojas

slope ratio of the vertical distance to horizontal


distance. It also refers to the tangent function
of the angle of inclination. Also refers to rise
over run. In Analytic
Geometry, slope of line is denoted as m.
m = Y2 -Y1
x2 -x1

Smith number a composite number, the sum of


whose digits equal to the sum of the digits of
its prime factors.
snowball prime a prime number whose digits can
be chopped off, one by one, from the righthand side, yet still leave a prime number.

spherical excess the sum of the angles of a


polygon over (n-2) 1so with n as the number.
of sides of the polygon
spherical polygon the portion of a spherical
surface bounded by three or more great circle
arcs
spherical pyramid the portion of a sphere bounded
by a spherical polygon and the plane of the
sides
spherical sector a solid generated by rotating a
sector of a circle about an axis which passes
through the center of the circle but which
contains no point inside the sector

spherical segment asolid bounded by a zone and


Soddy's formula a formula used for four circles of
the planes of the zone's bases
radii, r1, r2, rJ and r4 drawn not overlapping but
each touches the other three. The following is
the Soddy's formula:
spherical triangle a triangle extracted from the
2
(b1 +b2 +b3 +b4) = 2(b/ +b/+bl+b/)
surface of a sphere.
where b1 = 11r1 and so on ...
solid is any limited portion of space, bounded by
surfaces

spherical wedge the portion bounded by a lune and


the planes of two great circles
spheroid another term for ellipsoid.

Appendix A- Glossary 621

statically indeterminate the condition exists in


structures where the reactive forces or the
internal resisting forces over the estimated life
of the asset in terms of the periods or units of
output
stationary point a point on the graph of a function
where the tangent to the graph is parallel to
the x-axis or, equivalently, where the
derivative of the function is zero.
statistics the study of ways that lots of data can be
represented using a few numbers and the
study how such numbers can be chosen and
used to draw reasonable conclusion about the
data
stellation the process of constructing new
polyhedron by extending the face planes of a
given polyhedron past their edges
steradian ( sr ) a unit of measure of solid angle.
Then maximum value for a solid angle is a full
sphere which is equal to 4n steradians.
strain the change of relative positions of points due
to stress. Unit strain is equal to the ratio of the
deformation to the total length.

solid angle an angle formed by three or more


planes intersecting at a common point. Solid
angles are measured in steradians

spherometer an instrument for measuring the


curvature of a surface.

stress a force per unit area

solidus the slant line in a fraction such as alb


dividing the numerator from the denominator

square a rectangular all of whose sides are


congruent

strobogrammatic prime a prime number that


remain the same when rotated through 180
degrees. Example is 619,' when rotated 180
remain the same- 619

space a set of all points

square free an integer that is not divisible by a


perfect square, n2, for n > 1

specific gravity the ratio of the density of the


substance to the density of water. Specific
gravity of water at densed condition (4.C) =
1.0

square matrix (syn. determinant) a matrix with the


same number of rows columns

sphere a solid bounded by a closed surface every


point of which is equidistant from a fixed point
called the center
spherical angle the opening between two great
circle arcs. A spherical angle is measured by
the plane angle formed by the tangents to the
arcs at their point of intersection

square numbers (see figurate numbers)

subset a set that contains some of the elements of


a given set
subtrahend the number to be subtracted. Example:
7 - 5 =2, 5 is the subtrahend.
sum the result of addition.

square pyramidal numbers (see figurate numbers)


standard deviation a quantitative measure defining
the extent to which scores are dispersed
throughout in relation to the arithmetic mean.
This is also equal to the square root of the
variance.

supplementary chords two chords which join a


point on a circle to the end points of a
diameter.

sum-of-years' digits method (syn. SYD method ) a


method of computing depreciation in which the
amount for any year is based on !he ratio:
(years of remaining life/(1 + 2 + 3 + .. + n), with
n being the total anticipated life of the
equipment

Angle 0 is the supplemental angle

supplementary angle two angle whose sum is


equal to 180
surd an irrational number which is a root of a
positive integers or fraction or it is a radical
expressing an irrational number.
Types of surds: Example
Quadratic

J2

Cubic

~
(2

Quartic
The type of surd is named after the index of
the radical.

Pure surd is a surd that contains no rational


number (i.e. all its factors or terms are surds).
Example:

J2 , J3 + J2

.Mixed surd is a surd that contains at least


one rational term. Example: 2 + ..J 3
symmedian reflection of a median of a triangle
about the corresponding angle bisector

T
table c!Wlpilation of values such as trigonometric
table, logarithmic table, etc.
tangent a line (in the same plane) which intersect
the curve in one and only one point. In
trigonometry, it is the ratio of the side opposite
to side adjacent in a right triangle.
tangent plane of a sphere a plane which intersects
the sphere in exactly one point

622 1001 Solved Problems in En'gineering Mathematics (2nd Edition) by Tiong & Rojas
tension force in longitudinal direction.

terminal speed the final speed Vt attained by the


falling body. The principle is that when a. body
first. start to move, v =0, the resisting force is
zero and the initial acceleration is a = g. As it
speed increases, the resisting force also
increase until finally it equals the weight in
magnitude. AI this time, the acceleration
becomes zero and there is no more increase
in its speed.
ternary a number system using a place value
noiation with 3 as the base.

transverse axis the axis of the hyperbola which


passes through the foci, vertices and center

truss a framework composed of members joined at


their ends to form a rigid structure.

trapezium (syn. trapezoid) commonly used term in


United Kingdom rather than trapezoid. In
United States of America, the tem1 trapezoid
is used.

Tschirnhaus's cubic a curve that has a rectangular

theorem a statement of truth of which must be


established by proof

trapezoid a quadrilateral in which one and only one


pair of opposite sides are parallel. The parallel
sides of the trapezoids are called bases.

time value of money the cumulative effect of


elapsed time on the money value of an event,
based on the earning power of equivalent
invested funds capital should or will earn

tree a graph which the property that there is a


unique path from any vertex to any other
vertex traveling along the edges
trefoil curve a plane curve that has a rectangular

torque (syn. moment of force) a force times a


moment ami.
torr a unit of pressure which is equivalent to mm of
mercury (Hg). 1 torr = 1 mm of Hg. 1 torr is
equivalent to 133 Pascals.
torus (syn. anchor ring or doughnut) a solid formed
by revolving a circle about a line not
intersecting it
trace the sum of the terms along the main diagonal
of a matrix
trajectory the path followed by a projectile. It is a
graph of a parabola.
translation a parallel displacement of the original
system along one or more of its axes.
transpose to transfer to the other side of the
equation. When a term is transpose, the sign
must be changed.
transversal the intersecting line of two parallel or
non parallel lines

Appendix A- Glossary 623


truncated value the value of number when written
with the further digits have been suppressed
and replaced with three dots. For example, the
number 1t = 3.141592653... can also be
written by truncation as 3.14159 ... The
truncated value is not a rounded off value and
therefore always smaller than the exact value.

tetrahedral numbers (see figurate numbers)

ton a mass of 1000 kilograms.

equation of x4 + x2 y2 + y4 = x( x2

y2 )

triangular numbers (see figurate numbers)


triangular region is the union of a triangle and its
interior
trident of Newton 'a curve that has a rectangular
equation of xy = cx 3 + dx 2 +ex+ f
trigonometry branch of mathematics which deals
with triangles and trigonometric functions.

equatior. of 3ay 2 = x (x- a)

twin primes prime numbers that appear in pair and


differ by 2. Examples are 3 and 5, 11 and

13, ...

u
undecagon polygon of eleven sides.
undulating number an integer whose digits are
alternate. Example 343,434
unimodal sequence a sequence that first increases
and then decreases
unimodular matrix a square matrix whose
determinant is 1
union a set consisting of all elements that appear at
least once in the original set.
union of two sets a set of all objects that belong to
one or both sets

trillion a million million or 1012

universal set the set that contains all elements


untouchable number a number that is not the sum
of the aliquot part of any other number.

v
variable an expression than is assigned a certain
set of values.
variance a measure of the dispersion of scort:ti tn a
distribution away from the arithmetic mean.
The mean of the squared deviations about the
mean.
Varignon's Theorem The moment of the resultant
or two concurrent forces with respect to the
center in their planes is equal to the algebraic
sum of the moments of the components with
respect to the same center. Named after the
French mathematician, Pierre Varignon (1654-

1722).
vector quantity a physical quantity the described a
magnitude ('row much' or "how big") and the
direction in space
vector product of A and B is denoted as A x B.
Because of this notation, vector product is
also called the cross product.
velocity rate of change of displacement.
Venn diagram a pictorial description of the
probability concepts of independent and
dependent events. This was named after
English logician, John Venn (1834 -1923).

unit circle a circle of radius one unit and is used to


determine the sign of all trigonometric
functions in all quadrants

vertex point of intersection of two sides of a


polygon.

trivial considering the values of all the variables as


zero.

unit fraction a fraction whose numerator is 1

vertex figure the polygon. that appears if a


polyhedron is truncated at a vertex

Truncatable prime a prime number, n that remains


a prime when digits are deleted from it one at
a time.

unit vector a vector having a magnitude of unity,


with no units and is used only to described a
direction in space

vertical angles angles that are opposite to each


other and formed by two intersecting straight
lines. Vertical angles are equal.

truncated prism ihe portion of a prism included


between the base and a plane not parallel to
the base cutting all the edges or elements

unity referring to one.

trinomial having three terms.

universe (statistics, see population)

:Jl

,f

l:\

624 1001 Solved Problems in Engineering Mathematics (2"d Edition) by Tiong & Rojas
Weight =mass x gravitational acceleration

AppendixB:

whole number another term for natural number

Vertical angles (a = 8)

.vigesimal pertaining to the number 20.


vigesimal number system a number system using
the base 20
vinculum the bar that is placed over repeatinp
decimal .fractions to indicate the portion of the
pattern that repeats.
Vlnogradov's theorem 'Every sufficiently large odd
number can be expressed as the sum of three
prime numbers."
Viviani's curve a space curve the marks the
intersection of the cylinder (x - a)2 + yz =a2
and the sphere xz + y2 + z2 = a2. The
parametric equations of Viviani's curve are:
x=a(1+cost)
y =asint

Wilson's theorem 'Any number p is a prime


number if, and only if, (p - 1)! + 1 is divisible
byp.'
word problems (worded problems) real problems
that are usually given orally or written in words
work the force time a distance.

X
x-axis the horizontal axis of the ~ rectangular
coordinate system.
x-intercept the value of the abscissa of the point
where the curve crosses the x-axis

y
yard a unit of distance equivalent to 3 feet

z=2asin(4J
volume space occupied by a solid. Volume is
expressed in cubic units.

w
Waring's conjecture 'For every number k, there is
another number s such that every natural
number can be represented as the sum of s
kth powers.'

watt the Sl unit of power, This is equivalent to joules


per second.
weak inequality an inequality that permits the
equalitt:Case.
weight the force of the earth's gravitational
attraction for the body. It is .. 1ownward force
acting at the centroid or center of gravity of the
Wdy.

y-axis the vertical axis of the rectangular


coordinates system.

y-intercept the value of the ordinate of the point


where the curve crosses the y-axis
Young's modulus (see modulus of elasticity)

z
zenith a point in the celestial sphere directly above
the observer.

zero (syn. cipher) void, emptiness or nothing. Zero


is derived from Hindu word sunya"

Appendix 13 - Conv~sion_ .25

of~eigh~l

The English System


and Measures

Units

A. !.!near Measure,.t!:J!nm.!:!l
1 inch = 1,000 mils .

Definitions:

1 foot= 12 inches
1 yard "' 3 feet

Inch - the length of three barley grains


placed end to end

1 rod= 5.5_yards
1 chain = 4 rods

Digit- the breadth of a finger, about


0.75 inch

1 furlong "' 10 chains


1 furlong :::: 40 rods

Palm ... the breadth of a hand, about 4


inches

1 mile = 8 furlongs
1 mile

Hand - the length from the wrist to the


end of the middle finger, about 8
inches
Cubit- the length of the forearm from
the point of the elbow to the end
of the middle finger, about 18
inches

= 5280 feet

1 mile= 1,760 yards

1 league = 3 miles
B.Naut~

1 fathom :::: 6 feet

Foot- the length of man's foot, about


12 inches_

1 cable = 120 fathoms

Fathom -the length of rope when


pulled between a man's two
outstretch~d arms, about 6 feet

1 nautical mile per hour = 1 knot

Furlong - the length of a short race,


about 1/8 of a mile
Mile - the distance of a thousand
paces, as marked off by a length
5 feet between lifts of the same
foot. The mile is 5280 feet.

league -the distance a person can


see across a flat field, about 3
miles

1 nautical mile

C.

Surve~~!l

Measure

1 link= 7.92 inches

= 100 links
1 chain = 66 feet
1 furlong = 10 chains
1 chain

1 mile

==

80 chains

p.' Square Measure (Area}


1 square foot= 144 square inches
1 square yard

zone a portion of the surface of a sphere included


between two parallel planes

= 6,080 feet

==

9 square feet

1 square rod = 30.25 square yards


1 acre = t60 square rods

Appendix B - Conversion 62!

626 IOO,f Solved Problems in Engineering Mathematics (2nd Edition) by Tiong & Rojas
1 acre = 4,840 square yards
1 square mile = 640 acres

E. Cubic Measure (Volume)

I, Weifiht !Avoirdupois)
The avoirdupois system is used for
general weighing

M. Angular or Circular Measure


1 minute = 60 seconds
1 degree = 60 minutes

1 dram= 27.3438 grains

1 zodiac sign = 30 degrees

1 cubic foot= 1,728 cubic inches

1 ounce = 16 drams

1 radian= 57.2958 degrees

1 cubic yarc.t= 27 cubic feet

1 pound = 16 ounces

1 quadrant I right angle = 90 degrees

1 gallon (US)= 231 cubic inches

1 stone= 14 pounds

1 circle :::: 360 degrees

1 gallon (UK)::: 277.27 cubic inches

1 long hundredweight (UK)

1 bushel (US) = 2,150.42 cubic inches


1 bushel (UK)= 2,219.36 cubic inches

= 4 quarters
1 short hundredweight (US)
= 100 pounds

F. Liquid Measure (Capacity)


1 tablespoon = 3 teaspoon

1 long ton (UK) = 2,240 pounds

Diamond= 10
Corundum= 9

J. Weight (Troy)

Topaz= 8
Quartz= 7

1 cup = 8 fluid ounces

The troy system is used for weighing


precious metals or gems

1 pint = 2 cups

Labradorite = 6
Smithsonite = 5

1 pint= 4 gills

1 carat= 3.086 grains

1 quart = 2 pints

1 pennyweight = 24 grains

1 gallon

The scale for hardness of precious


metals or gems runs from 10 to 1, with
10 being the hardest.

1 short ton (US) = 2,000 pounds

1 fluid ounce = 2 tablespoons


1 gill = 4 fluid ounces

N. Hardness

= 4 quarts

1 ounce = 20 pennyweights

Fluorite= 4
Calcite= 3
Alabaster = 2

1 pound= 12 ounces
G. Dry Measure (Capacity)
1 quart

=2 pints

K. Weight (Apothecaries)

1 peck = 8 quarts
1 bushel

The apothecaries system was formerly


used by pharmacists

= 4 pecks

1 UK dry quart = 1.032

us dry quarts

1 scruple

=20 grains

1 dram = 3 scruples
H. Dry Measure (Cooking)
1 pinch = 1/8 teaspoon

1 ounce = 8 drams

Definitions:
The following are the basic units:
Meter - the distance traveled by light in
vacuum in 1/299.792.458
second

1 pound= 12 ounces

1 dash= 1/16 teaspoon


1 sprinkle= 1/32 teaspoon

The Metric. System of Weights


and Measures

L. Wood Measure
1 board foot = 144 cubic feet
1 cord foot= 16 cubic feet
1 cord = 8 cord feet

Kilogram ..;. the unit of mass equal to


the mass of the platinum-iridium
cylinder kept by the International
Bureau of Weights and
Measures in France.

Second.,.. the duration of 9,192,631,770


periods of radiation
corresponding to the specifiC
transition of the cesium-133
atom
Ampere - the constant current that,
flowing in two parallel
conductors 1 meter apart in a
vacuum, will produce a force\ of
2 x 10"7 newtons per meter of
length.
Kelvin - base on the triple point of
water, which is the point at
which water can exist in three
states: liquid, vapor and ice. It is
defined as 1/273.16 of the
temperatl)re of the triple point of
water.
Candela - intensity of the black-body
radiation from a surface of
1/600,000 square meters at the
temperature of freezing platinum
and at a pressure of 101,325
pascals.
Mole - an amount of substance in a
system that contains as many
elementary entities as there are
atoms in 0.012 kilogram of
carbon 12.
The following are supplementary units:
Radian - a unit of angular measure
equal to the central angle whose
sides are two radii of a circle
that cut off an arc whose length
is equal to the radius of the
circle.
Steradlans - a unit of measure equal to
the solid angle whos.e vertex is
in the center of a sphere, which
cuts off an area equal to the
rad.ius squared on the surface of
the sphere.

Appendix B - Conversion 629


628 1001. Solved Problems in Engineering Mathematics (2"ct Edition) by Tiong & Rojas
The following are derived units:
Newton - the unit of force equal to the
force needed to accelerate 1
kilogram by 1 meter per second
squared.
Joule- the unit of energy and work
equal to the work done when the
point of application 'of a force on
1 newton moves 1 meter in the
direction of the force.

Poundal - the force needed to


accelerate a 1-pound mass 1
foot per second squared.
1 poundaf = 13,825.5 dynes
1 poundal = 0. 138255 newtons
Newton - the force needed to
accelerate 1-kilogram mass 1
meter per second squared
5

1 newton = 10 dynes
Hertz -the unit of frequency in the field
of electromagnetism defined as
1 cycle per second
Watt- the unit of power defined as the
power of 1 joule per second
Volt - the u!'lit of voltage defined as the
difference of electrical potential
between two points of a
conductor carrying a constant
current of 1 ampere when the
power used between them
equals 1 watt,
Ohm - the unit of electrical .resistance
equal to a resistance that
passes a current of 1 ampere
when there is an electrical
potential difference of 1 volt
across it.

[ nne~$~-;e-oTFo~cea~dP~essu;e]
Force and pressure are closely related.
Force is anything that changes the motion
or state of rest in a body while pressure is
a force acting on a surface per unit area ..
Dyne - the force needed to
accelerate a 1-gram mass 1
centimeter per second squared.
1 dyne=: 0.0000723 poundal
1 dyne = 1o- newtons
5

1 newton= 7.23300 pouridals


Pascal -.unit of pressure which is
equal to 1 newton per square
meter.
1 pascal= 0.020855 pounditf
Atmosphere- unit of pressure at sea
level.
1 atm = 14.6952 pounds/in

Daylight Saving Time -In the US,


standard time plus one hour.
Daylight Saving Time seems tc:>
make the day last longer by ari
hour.
Equivalent values:
1 nanosecond

= 1 billionth of a second

1 microsecond = 1 millionth of a second


1 millisecond = 1 thousandth of a second
1 minute = 60 seconds

1 atm = 2,166.102 poiJnds/W

1 hour= 60 minutes

1 atm = 1.0332 kg/cm2

1 day = 24 hours

1 atm = 101,325 newtons/m

c--~-----J

Standard time- Introduced in 1883 by


international agreement,
Standard Time divided the Earth
into 24 time zones. Calculated
on solar time, the base of the
system is the zero meridian that
passes through the Royal
Greenwich Observatory at
Greenwich, England. Time is
measured east or west of this
Prime Meridian according to the
time zones.

Measure of Time _

Solar time - A measurement of time in


which noon occurs when the
sun is at is highest point over a
given location.
Atomic time -A measurement of time
using atomic clocks. Atomic
clocks calculate time with
extreme accuracy based on !he
natural resonance frequency of
!he cesium 133 atom

oc

oF

-273.15
-11
-5
0
4

-459.67
12
23
32
39.2

7
10
20

45
50
68

35
37

95
98.6

38
40
45

101
104
113

58

137

60
66
80
100
190
327

140
151
176
212
374
621

[Y~rthq~~k~-l~t~~sit!J

1 week = 7 days
1 month = 4 weeks

The Richter Scale, developed by


seismologist Charles Richter, is used to
express the amount of energy released at
the focus of an earthquake. The scale is
logarithmic and based on a numerical
system of exponents. For example, the
difference between 6.0 and 7.0 on the
Richter scale is not one but a factor of ten ..
Thus, an earthquake that measures 6.0 on
the Richter scale is a hundred times more
powerful than an earthquake that
measures 4.0. An earthquake of 8.0 is 10
million times greater than a 1.0
earthquake.

1 calendar month = 30 days


1 year = 52 weeks

Sidereal time -Also known as


astronomical time. This method
of measurement uses the.
movement of stars to calculate
time. An average sidereal day is
23 hours, 56 minutes and 4.09
.seconds long.

Reference point
Absolute zero
Frozen yogurt
A snowy day
Water freezes
Water at densed
condition
Cold water
A cool fall day
Room temperature on
winter day
A hot day
Normal body
temperature
A warm bath
A high fever
A hot bath
Highest air
temperature recorded
on Earth
Broiled steak
Hot faucet water
Hot soup
Water boils
Hot oven
Lead melts

1 year = 12 months
1 common year = 365 days
1 leap year = 366 days
1 decade

= 10 years

1 century = 100 years


1 millennium = 1000 years

Temp-erature

The two most common scales used for


measunng temperature are the Celsius
and Fahrenheit scales.

The great San Frar1cisco earthquake in


1906 has a magnitude of 8 ..3 which
destroyed the city. The earthquake that
causes the sinking of the island of
Krakatoa in .1883 has a magnitude of 9.9.

630 100 l Solved Problems in Engineering Mathematics (2nd Edition) by Tiong & Rojas

Comment
Richter
Number
. Near total devastation
9
A disaster, few buildings left
8
standina
Many buildinas.destroyed
7
Buildings shakes; roads and walls
6
crack
Strong rumbli11g; china and dishes
5
break .
Weak; much like a passing truck
4
Very weak; less than 3.5
3
Detectable only bv seismograph
2

M;~~ry

Computer

Onionskin paper = 9 pounds


Standard cc;>pier paper = 20 pounds
Typewriting paper = 20 pounds
Letterhead paper = 24 pounds
For printinv on both sides = 60 pounds
Typical business card = 65 pounds
Poster .paper = 120 pounds

Metric prefix
yotta
zetta
exa
peta
tera
giga
mega
kilo
hecto
deka
deci
centi
milli
micro
nano
pico
femto
alto
zepto

8 bits = 1 character = 1 byte

= 1,024J>ytes = 210

1 megabyte

= 1,048,576 bytes or2 20

1 gigabyte= 1,073,741,832 bytes or 230

Paper Weight~-n-]

Standard Amounts:
1 quire

= 24 sheets

1 printer's quire = 25 sheets


1 ream = 20 quires
1 printer's ream= 21.5 quires
1 bundle= 2 reams.
1 case = 4 bundles
1 printer's bundle

= 4 reams

1 bale = 10 reams
1 short ream = 480 sheets
1 long ream

= 500 sheets

Metric Prefixes --] .

Computer memory is based on bits and


bytes:

1 kilobyte

The following is a table for conversion of units from English System to Metric
System and from Metric System to English System.

Guidelines for Commercial Paper


Weights:

yoct~_

Symbol
y

z
E
p
T
G
M
k
h
dk

d
c
m
ll

n
p

f
a
z
y

Value
1024
102 .
10'"
1015
10

Appendix B - Conversion . 631

12

10~

1()6
10J
102
10
10'
10'2
10'~

10'6
10'"
1012

to_,,

101ll

1o:z
1024

632 1001 Solved Problems in Eng_ineering Mathematics (2nd Edition) by 'I'iong & Rojas

Appendix C- Physical Constants 633

Appendix C:

Physical Constants
+!!!!!!!!!!!!!!!!!!!!!!!!!!!!!!!!!!!!!!!!!!!!!!!!!!!!!!!!!!!!!!!!!!!!!!!!

Appel\dix F- Greek Alphabets 635

634 IOO 1 So~ved Problems in Engineering Mathematics (2nd Edition) by Tiong & Rojas

Appendix D:

Numeration

Appendix F:-
!!!!!!!!!!!!!!!!!!!!!!!!!!!!!!!!!!!!!!!!!!!!!!!!!!!!!!!!!!!!!!!!!!!

Appendix E:

n!! ! !

!!!!!!!!!!!!!!!M!!!!!!.!!!!!!at!!!!!!h!!!!!!N!!!!!!o!!!!!!t!!!!!!a!!!!!!ti!!!!!!o

Greek Alphabets

. .

636 100 1 Solved Problems in Engineering Mathematics (2nd Edition) by Tiong & Rojas

Appendix G:
!!!!!!!!!!!!!!!!!!!!!!!!!!!!!!!!!!!!!!!!!!!!!!!!!!!!!!!!!!!!!!!!!!!!!!!!!!!!!!!!!

+Divisibility Rules

ei

!!!!!!!!!!!!!!!!!!!!!!!!!!!!!!!!!!!!!!!!!!!!!!!!!!!!!!!!!!!!!!!!!!!!!!!!!!!!!!!!!!!

"

PHIUPI?lNE'

& MARKETrNG ~
ft
EXCELLENCE('!;
~~

AWARDS

~~-~

An integer is evenly divisible by


2

- if it is even or it ends with an even number

- if the sum of its digits is divisible by 3

- if the number formed by the last two digits is divisible by 4

- if it ends with either 0 or 5

- if it is divisible by 2 and 3

- if the number formed after cancellation of the units digit and


subtraction of twice the value of the units digit is divisible by 7

- if the number formed by the last three digits is divisible by 8

- if the sum of its digit is divisible by 9

10

- if it ends with zero

11

- if the difference between the cross sums of alternate digits is


divisible by 11

12

- if it divisible by 3 and 4

~~

~-~-~-~.~(~SIQ

THE PHILIPPINE MARKETlNG


EXCELLENCE AWARD
is given to

EXCEL Review center


Most Outstanding Engineering Review Center
In recognition of its outstanding marketing performance leading
towards market dominance, goodwill, high-level customer
confidence, and market acceptability.
Awarded by the Philippine Marketing Excellence Awards Institute,
Sales and Marketing Magazine and the Asian Institute of Marketing
and Entrep}:eneurship this 7th day of July, 2005 at the Westin

Philippine Plaza, City of Pasay, Philippines.

FELIX ~LAO, JR., Ph.D.


Chairman, Awards Committee
National President, Philippine Marketing
Association (1987)

Potrebbero piacerti anche